You are on page 1of 392

Hình học của đường tròn

Nguyễn Văn Linh

Mục lục
1 Phương tích, trục đẳng phương của đường tròn điểm 5
1.1 Lời mở đầu . . . . . . . . . . . . . . . . . . . . . . . . . . . . . . . . . . . . . . . . . . . 5
1.2 Ví dụ . . . . . . . . . . . . . . . . . . . . . . . . . . . . . . . . . . . . . . . . . . . . . . 5
1.3 Bài tập áp dụng . . . . . . . . . . . . . . . . . . . . . . . . . . . . . . . . . . . . . . . . 12

2 Tỉ số phương tích và ứng dụng 13


2.1 Giới thiệu . . . . . . . . . . . . . . . . . . . . . . . . . . . . . . . . . . . . . . . . . . . . 13
2.2 Ứng dụng . . . . . . . . . . . . . . . . . . . . . . . . . . . . . . . . . . . . . . . . . . . . 14
2.3 Đường tròn đẳng phương . . . . . . . . . . . . . . . . . . . . . . . . . . . . . . . . . . . 23
2.4 Bài tập áp dụng . . . . . . . . . . . . . . . . . . . . . . . . . . . . . . . . . . . . . . . . 25

3 Phép vị tự quay 26
3.1 Giới thiệu . . . . . . . . . . . . . . . . . . . . . . . . . . . . . . . . . . . . . . . . . . . . 26
3.2 Tính chất . . . . . . . . . . . . . . . . . . . . . . . . . . . . . . . . . . . . . . . . . . . . 26
3.3 Ví dụ . . . . . . . . . . . . . . . . . . . . . . . . . . . . . . . . . . . . . . . . . . . . . . 28
3.4 Bài tập áp dụng . . . . . . . . . . . . . . . . . . . . . . . . . . . . . . . . . . . . . . . . 35

4 Đường thẳng Euler, đường tròn Euler 37


4.1 Giới thiệu . . . . . . . . . . . . . . . . . . . . . . . . . . . . . . . . . . . . . . . . . . . . 37
4.2 Các định lý, bài toán liên quan . . . . . . . . . . . . . . . . . . . . . . . . . . . . . . . . 38
4.3 Điểm Euler-Poncelet . . . . . . . . . . . . . . . . . . . . . . . . . . . . . . . . . . . . . . 45
4.4 Bài tập áp dụng . . . . . . . . . . . . . . . . . . . . . . . . . . . . . . . . . . . . . . . . 45

5 Đường tròn Apollonius 45


5.1 Giới thiệu . . . . . . . . . . . . . . . . . . . . . . . . . . . . . . . . . . . . . . . . . . . . 45
5.2 Tính chất . . . . . . . . . . . . . . . . . . . . . . . . . . . . . . . . . . . . . . . . . . . . 46
5.3 Ứng dụng . . . . . . . . . . . . . . . . . . . . . . . . . . . . . . . . . . . . . . . . . . . . 50
5.4 Bài tập áp dụng . . . . . . . . . . . . . . . . . . . . . . . . . . . . . . . . . . . . . . . . 55

6 Đường thẳng Simson, đường thẳng Steiner của tam giác 56


6.1 Đường thẳng Simson . . . . . . . . . . . . . . . . . . . . . . . . . . . . . . . . . . . . . . 56
6.2 Đường thẳng Steiner của tam giác . . . . . . . . . . . . . . . . . . . . . . . . . . . . . . 56
6.3 Tính chất và ứng dụng . . . . . . . . . . . . . . . . . . . . . . . . . . . . . . . . . . . . . 58
6.4 Bài tập áp dụng . . . . . . . . . . . . . . . . . . . . . . . . . . . . . . . . . . . . . . . . 65

7 Một số vấn đề về tứ giác toàn phần 65


7.1 Giới thiệu . . . . . . . . . . . . . . . . . . . . . . . . . . . . . . . . . . . . . . . . . . . . 65
7.2 Tính chất . . . . . . . . . . . . . . . . . . . . . . . . . . . . . . . . . . . . . . . . . . . . 66
7.3 Bài tập áp dụng . . . . . . . . . . . . . . . . . . . . . . . . . . . . . . . . . . . . . . . . 72

8 Định lý ERIQ 74
8.1 Giới thiệu . . . . . . . . . . . . . . . . . . . . . . . . . . . . . . . . . . . . . . . . . . . . 74
8.2 Chứng minh . . . . . . . . . . . . . . . . . . . . . . . . . . . . . . . . . . . . . . . . . . . 74
8.3 Ứng dụng . . . . . . . . . . . . . . . . . . . . . . . . . . . . . . . . . . . . . . . . . . . . 75

1
9 Đường thẳng Newton mở rộng 85
9.1 Giới thiệu . . . . . . . . . . . . . . . . . . . . . . . . . . . . . . . . . . . . . . . . . . . . 85
9.2 Ứng dụng . . . . . . . . . . . . . . . . . . . . . . . . . . . . . . . . . . . . . . . . . . . . 86

10 Định lý Pascal 89
10.1 Giới thiệu . . . . . . . . . . . . . . . . . . . . . . . . . . . . . . . . . . . . . . . . . . . . 89
10.2 Chứng minh . . . . . . . . . . . . . . . . . . . . . . . . . . . . . . . . . . . . . . . . . . . 89
10.3 Ứng dụng . . . . . . . . . . . . . . . . . . . . . . . . . . . . . . . . . . . . . . . . . . . . 90
10.4 Bài tập áp dụng . . . . . . . . . . . . . . . . . . . . . . . . . . . . . . . . . . . . . . . . 95

11 Định lý Brianchon 96
11.1 Giới thiệu . . . . . . . . . . . . . . . . . . . . . . . . . . . . . . . . . . . . . . . . . . . . 96
11.2 Chứng minh . . . . . . . . . . . . . . . . . . . . . . . . . . . . . . . . . . . . . . . . . . . 97
11.3 Ứng dụng . . . . . . . . . . . . . . . . . . . . . . . . . . . . . . . . . . . . . . . . . . . . 99
11.4 Bài tập áp dụng . . . . . . . . . . . . . . . . . . . . . . . . . . . . . . . . . . . . . . . . 103

12 Đường tròn mixtilinear 103


12.1 Giới thiệu . . . . . . . . . . . . . . . . . . . . . . . . . . . . . . . . . . . . . . . . . . . . 103
12.2 Tính chất . . . . . . . . . . . . . . . . . . . . . . . . . . . . . . . . . . . . . . . . . . . . 104
12.3 Đường tròn Thebault . . . . . . . . . . . . . . . . . . . . . . . . . . . . . . . . . . . . . . 116
12.4 Ứng dụng của bổ đề Sawayama-Thebault . . . . . . . . . . . . . . . . . . . . . . . . . . 122

13 Đường tròn Hagge 125


13.1 Giới thiệu . . . . . . . . . . . . . . . . . . . . . . . . . . . . . . . . . . . . . . . . . . . . 125
13.2 Chứng minh . . . . . . . . . . . . . . . . . . . . . . . . . . . . . . . . . . . . . . . . . . . 126
13.3 Tính chất . . . . . . . . . . . . . . . . . . . . . . . . . . . . . . . . . . . . . . . . . . . . 127
13.4 Từ các kết quả về ba đường tròn bằng nhau . . . . . . . . . . . . . . . . . . . . . . . . . 133

14 Định lý Fontené 135


14.1 Giới thiệu . . . . . . . . . . . . . . . . . . . . . . . . . . . . . . . . . . . . . . . . . . . . 135
14.2 Hệ quả . . . . . . . . . . . . . . . . . . . . . . . . . . . . . . . . . . . . . . . . . . . . . . 136

15 Định lý Monge-D’Alembert 142


15.1 Giới thiệu . . . . . . . . . . . . . . . . . . . . . . . . . . . . . . . . . . . . . . . . . . . . 142
15.2 Ứng dụng . . . . . . . . . . . . . . . . . . . . . . . . . . . . . . . . . . . . . . . . . . . . 143
15.3 Bài tập tự luyện . . . . . . . . . . . . . . . . . . . . . . . . . . . . . . . . . . . . . . . . 151

16 Định lý Casey 152


16.1 Giới thiệu . . . . . . . . . . . . . . . . . . . . . . . . . . . . . . . . . . . . . . . . . . . . 152
16.2 Chứng minh định lý . . . . . . . . . . . . . . . . . . . . . . . . . . . . . . . . . . . . . . 153
16.3 Ứng dụng . . . . . . . . . . . . . . . . . . . . . . . . . . . . . . . . . . . . . . . . . . . . 156
16.4 Bài tập tự luyện . . . . . . . . . . . . . . . . . . . . . . . . . . . . . . . . . . . . . . . . 163

17 Tam giác paralogic 164


17.1 Lời mở đầu . . . . . . . . . . . . . . . . . . . . . . . . . . . . . . . . . . . . . . . . . . . 164
17.2 Một số tính chất . . . . . . . . . . . . . . . . . . . . . . . . . . . . . . . . . . . . . . . . 165

18 Một số vấn đề về đa giác lưỡng tâm 170


18.1 Giới thiệu . . . . . . . . . . . . . . . . . . . . . . . . . . . . . . . . . . . . . . . . . . . . 170
18.2 Tính chất . . . . . . . . . . . . . . . . . . . . . . . . . . . . . . . . . . . . . . . . . . . . 171
18.2.1 Tam giác . . . . . . . . . . . . . . . . . . . . . . . . . . . . . . . . . . . . . . . . 171
18.2.2 Tứ giác . . . . . . . . . . . . . . . . . . . . . . . . . . . . . . . . . . . . . . . . . 172

2
18.2.3 Đa giác nhiều hơn 4 đỉnh . . . . . . . . . . . . . . . . . . . . . . . . . . . . . . . 182

19 Hình sao năm cánh 191


19.1 Giới thiệu . . . . . . . . . . . . . . . . . . . . . . . . . . . . . . . . . . . . . . . . . . . . 191
19.2 Đường tròn Miquel trong hình sao . . . . . . . . . . . . . . . . . . . . . . . . . . . . . . 191
19.3 Hình sao Morley . . . . . . . . . . . . . . . . . . . . . . . . . . . . . . . . . . . . . . . . 194
19.3.1 Chứng minh . . . . . . . . . . . . . . . . . . . . . . . . . . . . . . . . . . . . . . . 194
19.3.2 Khai thác . . . . . . . . . . . . . . . . . . . . . . . . . . . . . . . . . . . . . . . . 195

20 Chuỗi đường tròn 199


20.1 Chuỗi đường tròn Clifford . . . . . . . . . . . . . . . . . . . . . . . . . . . . . . . . . . . 199
20.1.1 Giới thiệu . . . . . . . . . . . . . . . . . . . . . . . . . . . . . . . . . . . . . . . . 199
20.1.2 Khai thác một dạng phát biểu khác của chuỗi đường tròn Clifford . . . . . . . . 201
20.2 Chuỗi đường tròn Morley và điểm de Longchamps . . . . . . . . . . . . . . . . . . . . . 203
20.3 Chuỗi đường tròn Steiner và Pappus . . . . . . . . . . . . . . . . . . . . . . . . . . . . . 205
20.3.1 Chuỗi Steiner . . . . . . . . . . . . . . . . . . . . . . . . . . . . . . . . . . . . . . 205
20.3.2 Chuỗi Pappus . . . . . . . . . . . . . . . . . . . . . . . . . . . . . . . . . . . . . . 206
20.4 Chuỗi đường tròn Apollonius . . . . . . . . . . . . . . . . . . . . . . . . . . . . . . . . . 207

21 Các tam giác có dạng AB + AC = kBC 209


21.1 Mở đầu . . . . . . . . . . . . . . . . . . . . . . . . . . . . . . . . . . . . . . . . . . . . . 209
21.2 Các tam giác có dạng AB + AC = 2BC . . . . . . . . . . . . . . . . . . . . . . . . . . . 209
21.3 Các tam giác có dạng AB + AC = 3BC . . . . . . . . . . . . . . . . . . . . . . . . . . . 217
21.4 Trường hợp tổng quát AB + AC = kBC . . . . . . . . . . . . . . . . . . . . . . . . . . . 224

22 Định lý Đào về đường thẳng Simson mở rộng 226

23 Một số vấn đề khác 232

24 Xung quanh bài 3 VMO 2012 233


24.1 Giới thiệu . . . . . . . . . . . . . . . . . . . . . . . . . . . . . . . . . . . . . . . . . . . . 233
24.2 Chứng minh bài toán . . . . . . . . . . . . . . . . . . . . . . . . . . . . . . . . . . . . . . 233
24.3 Khai thác bài toán . . . . . . . . . . . . . . . . . . . . . . . . . . . . . . . . . . . . . . . 234

25 Bài toán 6 trong kì thi chọn đội tuyển quốc gia Iran năm 2013 237
25.1 Giới thiệu . . . . . . . . . . . . . . . . . . . . . . . . . . . . . . . . . . . . . . . . . . . . 237
25.2 Chứng minh . . . . . . . . . . . . . . . . . . . . . . . . . . . . . . . . . . . . . . . . . . . 237
25.3 Mở rộng . . . . . . . . . . . . . . . . . . . . . . . . . . . . . . . . . . . . . . . . . . . . . 242

26 Bài toán số 5 trong kì thi chọn đội tuyển toán Quốc tế của Việt Nam năm 2015 245
26.1 Mở đầu . . . . . . . . . . . . . . . . . . . . . . . . . . . . . . . . . . . . . . . . . . . . . 245
26.2 Chứng minh . . . . . . . . . . . . . . . . . . . . . . . . . . . . . . . . . . . . . . . . . . . 246
26.3 Khai thác . . . . . . . . . . . . . . . . . . . . . . . . . . . . . . . . . . . . . . . . . . . . 247

27 Bài toán số 3 trong kì thi chọn HSG Quốc gia của Việt Nam năm 2016 251
27.1 Giới thiệu . . . . . . . . . . . . . . . . . . . . . . . . . . . . . . . . . . . . . . . . . . . . 251
27.2 Khai thác . . . . . . . . . . . . . . . . . . . . . . . . . . . . . . . . . . . . . . . . . . . . 252

28 Hai bài hình trong kì thi Việt Nam TST 2016 265

29 Từ một bổ đề về đường thẳng Euler 270

3
30 Đường tròn tiếp xúc trong tứ giác ngoại tiếp 279
30.1 Giới thiệu. . . . . . . . . . . . . . . . . . . . . . . . . . . . . . . . . . . . . . . . . . . . . 279
30.2 Chứng minh. . . . . . . . . . . . . . . . . . . . . . . . . . . . . . . . . . . . . . . . . . . 279
30.3 Khai thác. . . . . . . . . . . . . . . . . . . . . . . . . . . . . . . . . . . . . . . . . . . . . 281

31 Chuỗi bài toán về họ đường tròn đi qua điểm cố định 285

32 Tứ giác ngoại tiếp thần kì 293

33 Chứng minh định lý Sondat dựa theo ý tưởng của Jean-Louis Ayme 297
33.1 Giới thiệu . . . . . . . . . . . . . . . . . . . . . . . . . . . . . . . . . . . . . . . . . . . . 297
33.2 Chứng minh . . . . . . . . . . . . . . . . . . . . . . . . . . . . . . . . . . . . . . . . . . . 297

34 Sáng tạo trong hình học 300


34.1 Mở đầu . . . . . . . . . . . . . . . . . . . . . . . . . . . . . . . . . . . . . . . . . . . . . 300
34.2 Sáng tạo ra bài toán mới như thế nào? . . . . . . . . . . . . . . . . . . . . . . . . . . . . 300
34.2.1 Lấy cảm hứng từ một mô hình đã biết . . . . . . . . . . . . . . . . . . . . . . . . 300
34.2.2 Đặc biệt hóa . . . . . . . . . . . . . . . . . . . . . . . . . . . . . . . . . . . . . . 304
34.2.3 Tổng quát hóa . . . . . . . . . . . . . . . . . . . . . . . . . . . . . . . . . . . . . 306
34.2.4 Nghịch đảo . . . . . . . . . . . . . . . . . . . . . . . . . . . . . . . . . . . . . . . 309
34.3 Thế nào là một bài toán đẹp? . . . . . . . . . . . . . . . . . . . . . . . . . . . . . . . . . 310

35 Bài tập tổng hợp 311

4
PHẦN I: KIẾN THỨC CƠ BẢN, CÁC PHƯƠNG PHÁP GIẢI

1 Phương tích, trục đẳng phương của đường tròn điểm


1.1 Lời mở đầu
Trước tiên ta nhớ lại định nghĩa trục đẳng phương của hai đường tròn không đồng tâm là tập
hợp các điểm có cùng phương tích với hai đường tròn đó. Từ đó cho điểm A không trùng tâm O của
đường tròn (O, R). Ta xây dựng trục đẳng phương của điểm A và đường tròn (O) là tập hợp các
điểm có cùng phương tích với (A, 0) và (O, R). Nghĩa là nếu M nằm trên trục đẳng phương trên thì
M A2 = M O 2 − R 2 .
Có hai trường hợp xảy ra. Nếu A nằm ngoài (O), dễ thấy trục đẳng phương của A và (O) là đường
thẳng nối trung điểm các tiếp tuyến kẻ từ A tới (O). Nếu A nằm trong (O), kẻ đường thẳng qua A
vuông góc với OA, cắt (O) tại hai điểm M, N . Tiếp tuyến của (O) tại M, N cắt nhau tại B. Khi đó
trục đẳng phương của A và (O) là đường thẳng nối trung điểm BM và BN .
Ta cũng có thể mở rộng định nghĩa trục đẳng phương cho hai điểm A và B. Trong trường hợp này
thực chất nó là đường trung trực của đoạn thẳng AB. Trục đẳng phương của một điểm và một đường
tròn là công cụ rất hữu hiệu cho nhiều bài toán khó. Chúng ta hãy thử áp dụng định nghĩa này qua
một số ví dụ sau.

1.2 Ví dụ
Bài 1. Cho tam giác ABC nội tiếp đường tròn (O), ngoại tiếp đường tròn (I). Qua I lần lượt kẻ
đường thẳng vuông góc với IA, IB, IC cắt BC, CA, AB tại X, Y, Z. Chứng minh rằng X, Y, Z thẳng
hàng.

O
I

X
B C

1
Chứng minh. Ta có ∠IBC = ∠ABC = ∠AIC − 90o = ∠CIX.
2
Suy ra IX 2 = XB.XC hay PX /(I, 0) = PX /(O).
Tương tự, PY /(I, 0) = PY /(O), PZ /(I, 0) = PZ /(O).
Do đó X, Y, Z cùng nằm trên trục đẳng phương của I và (O), suy ra đpcm.

Nhận xét. Ta có thể tổng quát bài toán 1 như sau.

Bài 2. Cho tam giác ABC. P là một điểm bất kì nằm trên mặt phẳng. Tiếp tuyến tại P của đường
tròn ngoại tiếp tam giác BP C giao BC tại A1 , tương tự xác định B1 , C1 . Chứng minh rằng A1 , B1 , C1
thẳng hàng.

5
Cách giải tương tự bài toán 1.
Hoặc có thể tổng quát theo một hướng khác như sau.

Bài 3. Cho tam giác ABC. P là điểm bất kì trên mặt phẳng. Qua P kẻ đường thẳng vuông góc với
P A cắt BC tại A1 , tương tự xác định B1 , C1 . Chứng minh rằng A1 , B1 , C1 thẳng hàng.

A3

C2 B2

C3
B3

A1 B A2 C

A4

Chứng minh. Gọi A2 B2 C2 là tam giác vết của P ứng với tam giác ABC. A3 , B3 , C3 lần lượt là điểm
đối xứng với P qua 3 cạnh tam giác A2 B2 C2 , A4 là điểm đối xứng với P qua BC.
Dễ thấy A2 P = A2 B3 = A2 C3 = A2 A4 nên P, B3 , C3 , A4 cùng nằm trên đường tròn tâm A2 .
Theo tính chất hàng điều hoà cơ bản ta có A2 (ABB2 C2 ) = −1, mà P A1 ⊥ AA2 , P A4 ⊥ A2 B,
P C3 ⊥ A2 B2 , P B3 ⊥ A2 C2 nên P (A1 A4 C3 B3 ) = −1.
Do A1 P ⊥ P A2 nên A1 P, A1 A4 là tiếp tuyến của (A2 , A2 P ). Từ đó P B3 A4 C3 là tứ giác điều hoà,
tức là A1 , B3 , C3 thẳng hàng.
Ta thu được A1 B3 .A1 C3 = A1 P 2 . Tức là A1 nằm trên trục đẳng phương của (P, 0) và đường tròn
(A3 B3 C3 ).
Chứng minh tương tự suy ra A1 , B1 , C1 thẳng hàng.

Bài 4. Cho đường tròn (O) và một điểm A nằm ngoài đường tròn. Từ A kẻ tiếp tuyến AB, AC tới
(O). E, F lần lượt là trung điểm AB, AC. D là một điểm bất kì trên EF . Từ D kẻ tiếp tuyến DP, DQ
tới (O). P Q giao EF tại M . Chứng minh rằng ∠DAM = 90o .

E F
D
M

B C

6
Chứng minh. Ta có D nằm trên EF là trục đẳng phương của A và (O) nên DA2 = DP 2 = DQ2 . Suy
ra D là tâm đường tròn ngoại tiếp tam giác AP Q.
Lại có M nằm trên trục đẳng phương của A và (O) nên M A2 = M P .M Q. Nghĩa là M A là tiếp
tuyến của (D, DA).
Vậy ∠DAM = 90o .

Bài 5. Cho ba đường tròn (O1 ), (O2 ), (O3 ) đôi một ngoài nhau. A là một điểm trên mặt phẳng và nằm
ngoài các đường tròn trên. Gọi d1 , d2 , d3 lần lượt là đường thẳng nối hai tiếp điểm kẻ từ A đến đường
tròn (O1 ), (O2 ), (O3 ). Giả sử điểm A chuyển động sao cho d1 , d2 , d3 đồng quy tại P . Chứng minh rằng
P thuộc một đường tròn cố định.

P O3

M
F
O1

O2
A

Chứng minh. Gọi E, F là tiếp điểm của các tiếp tuyến kẻ từ A đến (O1 ); M là trung điểm AP .
Ta có M thuộc đường trung bình ứng với đỉnh A của tam giác AEF nên M thuộc trục đẳng phương
của A và (O1 ).
Suy ra PM /(O1 ) = M A2 .
Tương tự ta thu được M là tâm đẳng phương của ba đường tròn (O1 ), (O2 ), (O3 ). Suy ra M cố
định.
Ta có M P 2 = M A2 nên P thuộc đường tròn tâm M bán kính bằng căn bậc hai của phương tích
từ M tới ba đường tròn.

Bài 6. Cho (O1 , R1 ) tiếp xúc ngoài với (O2 , R2 ) tại M (R2 > R1 ). Xét điểm A di động trên (O2 ) sao
cho A, O1 , O2 không thẳng hàng. Từ A kẻ tiếp tuyến AB, AC tới (O1 ). Các đường thẳng M B, M C cắt
lại (O2 ) tại E, F . D là giao điểm của EF với tiếp tuyến tại A của (O2 ). Chứng minh rằng D di động
trên một đường thẳng cố định (Việt Nam TST 2003 ).

7
A D
x
B
F

O2
M O1

y
E

Chứng minh. Qua M kẻ tiếp tuyến chung xy của (O1 ) và (O2 ). Ta có ∠M CA = ∠CM y = ∠F M x =
∠F AM.
Do đó 4F AM ∼ 4F CA(g.g). Ta thu được F A2 = F M .F C = F O12 − R12 .
Tương tự EA2 = EO12 − R12 . Suy ra EF là trục đẳng phương của điểm A và (O1 ).
Do D ∈ EF nên DA2 = DO12 − R12 . Từ đó PD /(O1 ) = PD /(O2 ).
Vậy D nằm trên trục đẳng phương của hai đường tròn cố định (O1 ) và (O2 ).

Bài 7. Cho tam giác ABC ngoại tiếp đường tròn (I), nội tiếp đường tròn (O). (I) tiếp xúc với
BC, CA, AB lần lượt tại D, E, F . Gọi XY Z là tam giác tạo bởi các đường thẳng nối trung điểm
AE, AF ; BF, BD; CD, CE. Chứng minh rằng tâm của đường tròn ngoại tiếp các tam giác ABC và
XY Z trùng nhau (Ba Lan MO 2011 )

Y
A
A1
A2
E
Z
F

O C2
B1 I

B B2 D C1 C

Chứng minh. Gọi A1 , A2 , B1 , B2 , C1 , C2 lần lượt là trung điểm AE, AF, BF, BD, CD, CE.
Ta nhận thấy A1 A2 , B1 B2 , C1 C2 lần lượt là trục đẳng phương của A và (I), B và (I), C và (I).
Theo định lý về tâm đẳng phương, A1 A2 giao B1 B2 tại tâm đẳng phương của A, B, (I). Tức là
Z thuộc trục đẳng phương của A và B. Suy ra Z nằm trên trung trực của đoạn AB hay ZO ⊥ AB.
Tương tự, XO ⊥ BC.
Lại có BI ⊥ XZ nên ta có ∠OZX = ∠ABI = ∠IBC = ∠OXZ. Suy ra OX = OZ.
Chứng minh tương tự ta thu được OX = OY = OZ hay O là tâm đường tròn ngoại tiếp của tam
giác XY Z.

Bài 8. Cho tam giác ABC nội tiếp đường tròn (O), ngoại tiếp đường tròn (I). Gọi A1 , B1 , C1 lần lượt
là tiếp điểm của đường tròn (I) với BC, CA, AB. A2 là tiếp điểm của đường tròn ωa qua B, C và tiếp
xúc với (I), tương tự xác định B2 , C2 . Chứng minh rằng A1 A2 , B1 B2 , C1 C2 đồng quy tại một điểm
nằm trên OI.

8
B3

B1
A2
C1 I O
X
B2 C2
A1

B C

A3

Chứng minh. Gọi A3 là giao điểm thứ hai của A1 A2 với ωa . Tương tự với B3 , C3 . Dễ thấy A3 là điểm
chính giữa cung BC của ωa và A3 C 2 = A3 A1 .A3 A2 .
Điều này nghĩa là A3 nằm trên trục đẳng phương của (I) và (C, 0).
Chứng minh tương tự B3 cũng nằm trên trục đẳng phương của (I) và (C, 0). Suy ra A3 B3 ⊥ CI.
Mà A1 B1 ⊥ CI nên A3 B3 k A1 B1 .
Tương tự suy ra hai tam giác A1 B1 C1 và A3 B3 C3 có cạnh tương ứng song song. Từ đó A1 A3 , B1 B3 , C1 C3
đồng quy tại tâm vị tự X của hai tam giác.
Mặt khác, IA1 k OA3 , IB1 k OB3 nên O và I là ảnh của nhau qua phép vị tự tâm X tỉ số bằng tỉ
số đồng dạng của hai tam giác trên. Suy ra X ∈ OI.

Nhận xét. Dễ thấy X là tâm đẳng phương của ωa , ωb , ωc .

Bài 9. Cho tam giác ABC có ∠C = 2∠A, phân giác CD. Gọi S là tâm của đường tròn nằm cùng
phía với B bờ AC sao cho tiếp xúc với AC và tiếp xúc ngoài với đường tròn ngoại tiếp 4ACD và
4BCD,. Chứng minh rằng AB ⊥ CS. (Romania TST 2009)

Q K
A
I R
D

C J B

Chứng minh. Gọi P, Q, R lần lượt là tiếp điểm của ω với AC, (ACD), (BCD). Gọi I, J lần lượt là tâm
của (ACD), (BCD).
Ta có ∠CAD = ∠DCA nên D là điểm chính giữa cung AC. Suy ra ID ⊥ AC và ID k SP . Do Q
là tâm vị tự trong của (S) và (I) nên P, Q, D thẳng hàng.

9
Gọi K là giao điểm thứ hai của AQ với (S). Ta có P K k AD nên ∠DAQ = ∠QKP = ∠AP D, suy
ra DP.DQ = DA2 .
Mặt khác, gọi E là giao điểm thứ hai của AC với (J). Dễ thấy B là điểm chính giữa cung AC nên
chứng minh tương tự suy ra B, R, P thẳng hàng và BP.BR = BC 2 .
Điều này có nghĩa là B, D cùng nằm trên trục đẳng phương của (S) và (C, 0). Vậy CS ⊥ AB.

Bài 10. Cho tứ giác ABCD ngoại tiếp đường tròn (I), ∠A = 90◦ . BI giao AD tại M , DI giao AB
tại N . Chứng minh rằng AC vuông góc với M N.

A B N
Y
X

O C

Chứng minh. Gọi X, Y, Z lần lượt là giao điểm của AC và BD, AB và CD, AD và BC. O là trung
điểm M N .
1
Ta có ∠AN I = 90◦ − ∠ADI = 90◦ − ∠ADY = 180◦ − ∠AIY.
2
Suy ra IY là tiếp tuyến của (AIN ) hay Y I 2 = Y N.Y A, tức là Y nằm trên trục đẳng phương của
(I, 0) và (M N ). Chứng minh tương tự suy ra Y Z là trục đẳng phương của (I, 0) và (M N ). Từ đó
Y Z ⊥ OI.
Mà Y Z là đường đối cực của X ứng với (I) nên IX ⊥ Y Z. Suy ra I, O, X thẳng hàng.
Gọi P là giao của BD và Y Z thì AC là đường đối cực của P ứng với (I). Suy ra IP ⊥ AC.
Mặt khác, dễ thấy (DBXP ) = −1 suy ra I(DBXP ) = −1.
Từ đó I(N M OP ) = −1. Mà O là trung điểm M N nên IP k M N hay M N ⊥ AC.

Bài 11. Cho tam giác ABC ngoại tiếp đường tròn (I). Gọi D là tiếp điểm của (I) với BC, đường
thẳng qua I song song với BC cắt AB, AC lần lượt tại M, N . DM, DN lần lượt cắt AC, AB tại E, F.
AD cắt EF tại J. Chứng minh rằng JB = JC.

10
A

M I N

B C
D

J
F

Chứng minh. Gọi K là giao của AD với M N . Ta có (ADKJ) = −1 nên B(ADKJ) = −1, mà
M I k BD suy ra M J đi qua trung điểm BD. Chú ý rằng ∠M BI = ∠IBD = ∠BIM nên M B = M I.
Suy ra M thuộc trục đẳng phương của đường tròn (B, 0) và đường tròn ω đường kính ID. Từ đó M J
là trục đẳng phương của (B, 0) và ω.
Tương tự N J là trục đẳng phương của (C, 0) và ω. Suy ra J là tâm đẳng phương của (B, 0), (C, 0)
và ω hay JB = JC.
Bài 12. Gọi P là một trong hai giao điểm của hai đường tròn (O1 ), (O2 ). AB là tiếp tuyến chung của
hai đường tròn. Đường thẳng qua A vuông góc với BP cắt O1 O2 tại C. Chứng minh rằng AP ⊥ P C.

A
P

O1 C O2

Chứng minh. Gọi H là giao của AC và BP.


Ta có O1 A2 = O1 P 2 nên O1 nằm trên trục đẳng phương của đường tròn (P, 0) và đường tròn đường
kính AB. Tương tự với O2 .
Do đó O1 O2 là trục đẳng phương của (P, 0) và (AB).
Mà C nằm trên O1 O2 nên CH.CA = CP 2 , theo hệ thức lượng trong tam giác vuông suy ra
∠AP C = 90◦ .
Bài 13. Đường tròn nội tiếp (I) của tam giác ABC tiếp xúc với AB, AC lần lượt tại Z và Y . Gọi G
là giao điểm của BY và CZ. Dựng các hình bình hành BCY R, BCSZ. Chứng minh rằng GR = GS
(IMO Shortlist 2009 )

11
A

R Y
Z S

L
B C
X

Ia

Chứng minh. Gọi X là tiếp điểm của (I) với BC, L, K là tiếp điểm của đường tròn (Ia , ra ) bàng tiếp
góc A với BC, AC.
Do BCY R là hình bình hành nên BR = CY = CX = BL. Suy ra BR2 = BL2 .
Mặt khác, Y R = BC = BL + LC = CX + CK = CY + CK = Y K. Suy ra Y R2 = Y K 2 .
Từ đó BY là trục đẳng phương của R và (Ia ). Mà G ∈ BY nên GR2 = GIa2 − ra2 .
Tương tự, GS 2 = GIa2 − ra2 . Ta có đpcm.

1.3 Bài tập áp dụng


Bài 14. Cho tam giác nhọn ABC. M là trung điểm BC. Kẻ các đường cao CD, BE. K, L lần lượt
là trung điểm M E and M D. KL cắt đường thẳng qua A song song với BC tại T . Chứng minh rằng
TA = TM.

Bài 15. Cho đường tròn (O) và một điểm A nằm ngoài đường tròn. Kẻ các tiếp tuyến AB, AC tới
(O). E di chuyển trên cung nhỏ BC. Tiếp tuyến của (O) qua E giao trung trực đoạn AE tại H. Chứng
minh rằng H thuộc một đường thẳng cố định.

Bài 16. Cho hình thang ABCD (AB k CD) thoả mãn ∠CDB = ∠CAD. Gọi I là giao điểm của AC
và BD. O là tâm đường tròn ngoại tiếp tam giác AIB. M là một điểm nằm trên AD. Chứng minh
rằng DO ⊥ CM khi và chỉ khi M là trung điểm AD.

Bài 17. Cho (O) và một điểm A nằm ngoài đường tròn. Từ A kẻ hai tiếp tuyến AB, AC tới (O). Gọi
M, N lần lượt là trung điểm AB, AC. Một đường tròn ω chuyển động sao cho luôn đi qua A và tiếp
xúc ngoài với (O). Gọi E, F là giao điểm của M N với ω. Chứng minh rằng ∠EAF luôn không đổi.

Bài 18. (Nguyễn Văn Linh). Cho tam giác ABC. Các điểm B1 , B2 nằm trên AC, C1 , C2 nằm trên
AB sao cho các tam giác BB1 B2 , CC1 C2 lần lượt cân tại B và C. Gọi ωb , ωc lần lượt là các đường
tròn tiếp xúc với BB1 , BB2 tại B1 , B2 và tiếp xúc với CC1 , CC2 tại C1 , C2 . Gọi M là trung điểm BC.
Chứng minh rằng M nằm trên trục đẳng phương của ωb và ωc .

Bài 19. (Nguyễn Văn Linh). Cho tam giác ABC. P là điểm bất kì trong mặt phẳng. Trên BC, CA, AB
lần lượt lấy hai điểm A1 , A2 ; B1 , B2 ; C1 , C2 sao cho các tam giác P A1 A2 , P B1 B2 , P C1 C2 đều cân tại
P và đồng dạng với nhau. Dựng đường tròn (Oa ) tiếp xúc với P A1 , P A2 tại A1 , A2 . Tương tự có
(Ob ), (Oc ). Chứng minh rằng tâm đẳng phương của (Oa ), (Ob ), (Oc ) là tâm đường tròn pedal của P
ứng với tam giác ABC.

Bài 20. Cho tứ giác ABCD nội tiếp đường tròn (O). AB giao CD tại E, AD giao BC tại F . Chứng
minh rằng đường tròn đường kính EF trực giao với (O).

12
Bài 21. Cho tứ giác ABCD nội tiếp đường tròn (O). AB giao CD tại E, AD giao BC tại F . Gọi
M, N lần lượt là trung điểm AC, BD. Đường tròn đường kính EF giao (O) tại X, Y . Chứng minh
rằng:
a) EF tiếp xúc với (EM N ) và (F M N ).
b) (M N X), (M N Y ) tiếp xúc với (O).
Bài 22. Cho tứ giác ABCD nội tiếp đường tròn (O). AB giao CD tại E, AD giao BC tại F . L
là trung điểm EF . LR giao (O) tại P, Q. Chứng minh rằng (EP F ), (EQF ) tiếp xúc với (O). (RMM
2013)

2 Tỉ số phương tích và ứng dụng


2.1 Giới thiệu
Chúng ta bắt đầu từ công thức hiệu số phương tích của một điểm đối với hai đường tròn.
Cho hai đường tròn không đồng tâm (O1 , R1 ) và (O2 , R2 ) có trục đẳng phương d. Xét một điểm
M bất kì, gọi K là hình chiếu của M trên d, H là giao điểm của O1 O2 với d.
Khi đó PM/(O1 ) − PM/(O2 ) = 2O1 O2 · KM . (1)

M
K

O1 H I O2

Chứng minh. Gọi I là trung điểm O1 O2 .


Ta có PM/(O1 ) − PM/(O2 ) = (M O12 − R12 ) − (M O22 − R22 ).
= M O12 − M O22 + R22 − R12 = (M O12 − M O22 ) + (HO22 − HO12 )
−−−→ −−−→ −−−→ −−−→ −−→ −−→ −−→ −−→
= (M O1 − M O2 )(M O1 + M O2 ) − (HO1 − HO2 )(HO1 + HO2 ).
−−−→ −−→ −−−→ −→
= O2 O1 · 2M I − O2 O1 · 2HI
−−−→ −−→ −→ −−−→ −−→
= 2O2 O1 · (M I − HI) = 2O2 O1 · M H
−−−→ −−→
= 2O2 O1 · M K = 2O1 O2 · KM

Nhận xét. Nếu điểm M nằm trên (O2 ) ta có PM/(O2 ) = 0, công thức hiệu số phương tích trở
thành:
PM/(O1 ) = 2O1 O2 · KM . (2)
Từ đó chúng ta có hệ quả sau:
Hệ quả 1. Cho ba đường tròn (O1 ), (O2 ), (O3 ) đồng trục và một điểm M bất kì nằm trên (O3 ).
PM/(O1 ) O3 O1
Khi đó = .
PM/(O2 ) O3 O2

Chứng minh. Gọi K là hình chiếu của M trên trục đẳng phương d của 3 đường tròn.
PM/(O1 ) O1 O3 · KM O3 O1
Theo nhận xét trên ta có = = .
PM/(O2 ) O2 O3 · KM O3 O2

13
Ngược lại, ta cũng có:
PM/(O1 )
Hệ quả 2. Quỹ tích các điểm M thỏa mãn = k không đổi là một đường tròn đồng trục
PM/(O2 )
với (O1 ) và (O2 ).

Chứng minh. Dựng đường tròn (O3 ) qua M sao cho (O3 ), (O1 ), (O2 ) đồng trục, suy ra O1 , O2 , O3 thẳng
hàng.
PM/(O1 ) O3 O1
Theo hệ quả trên = = k.
PM/(O2 ) O3 O2
Do đó O3 cố định. Với mỗi vị trí của tâm O3 chỉ có duy nhất một đường tròn đồng trục với (O1 )
và (O2 ). Như vậy (O3 ) không phụ thuộc vào vị trí của M , tức là M chuyển động trên đường tròn (O3 )
đồng trục với (O1 ) và (O2 ). p
R22 k 2 + R12 + (O1 O22 − R12 − R22 )k
Ngoài ra ta có thể chứng minh bán kính của (O3 ) bằng .
|1 − k|

Nhận xét.
1. Khi k = 1 thì (O3 ) suy biến thành trục đẳng phương d. Ta có bài toán quỹ tích quen thuộc: tập
hợp các điểm M có cùng phương tích với hai đường tròn (O1 ) và (O2 ) là trục đẳng phương d.
2. Khi k = −1 thì O3 là trung điểm của O1 O2 . Đường tròn (O3 ) được gọi là đường tròn đẳng
phương của hai đường tròn (O1 ) và (O2 ). Một số tính chất của đường tròn đẳng phương sẽ được xem
xét ở mục sau.
3. Khi (O1 ) và (O2 ) cùng suy biến thành đường tròn điểm, (O3 ) trở thành đường tròn Apollonius
của đoạn thẳng O1 O2 ứng với tỉ số k.

2.2 Ứng dụng


Bài 23. Cho 3 đường tròn (O1 ), (O2 ), (O3 ) có tâm cùng nằm trên đường thẳng d. Kí hiệu dij là trục
đẳng phương của cặp đường tròn (Oi ) và (Oj ) (i, j = 1, 3, i 6= j). Gọi A1 , A2 , A3 lần lượt là giao điểm
A1 A2 A2 A3 A1 A3
của d23 , d13 , d12 với d. Chứng minh rằng = = .
O1 O2 O2 O3 O1 O3
Chứng minh. Do A1 nằm trên trục đẳng phương của (O2 ) và (O3 ) nên PA1 /(O2 ) = PA1 /(O3 ) .
Ta thu được PA1 /(O1 ) − PA1 /(O2 ) = PA1 /(O1 ) − PA1 /(O3 ) .
Theo công thức (1) suy ra 2O1 O2 · A3 A1 = 2O1 O3 · A2 A1 .
A1 A3 A1 A2
Từ đó = . Chứng minh tương tự suy ra đpcm.
O1 O3 O1 O2
Bài 24. Cho đường tròn (O, R) và một điểm A cố định. Gọi M là điểm chuyển động trên (O). Chứng
minh rằng trục đẳng phương d của (O, R) và (M, M A) luôn tiếp xúc với một đường tròn cố định.

Chứng minh. Gọi H là hình chiếu của A trên d. Do A ∈ (M, M A) nên theo công thức (2) ta có
PA/(O) = 2OM .HA.
PA/(O)
Suy ra HA = .
2OM
|AO2 − R2 |
Vậy AH = =const. Tức là d tiếp xúc với đường tròn cố định có tâm A, bán kính
2R
2
|AO − R |2
.
2R
Bài 25. Cho 3 đường tròn (O1 ), (O2 ), (O3 ) đồng trục. A là điểm bất kì trên (O3 ) sao cho A nằm ngoài
(O1 ) và (O2 ). Lần lượt kẻ tiếp tuyến AB, AC tới (O1 ), (O2 ). BC giao (O1 ), (O2 ) lần thứ hai lần lượt
BD
tại D, E. Chứng minh rằng tỉ số không đổi.
CE

14
A B

D
E C

O3 O2 O1

PA/(O1 ) AB 2 O3 O1
Chứng minh. Do (O1 ), (O2 ), (O3 ) đồng trục nên theo hệ quả 1, = 2
= .
PA/(O2 ) AC O3 O2
Áp dụng định lý hàm số sin cho tam giác ABC ta có
AB sin ∠ACB sin 21 ∠EO2 C EC 2O1 B EC R1
= = = · = · .
AC sin ∠ABC sin 12 ∠DO1 B 2O2 C DB DB R2
r
BD R1 AC R1 O2 O3
Vậy = · = · =const.
CE R2 AB R2 O1 O3
Bài 26. Cho hai đường tròn (O1 ) và (O2 ) có P, Q lần lượt là tâm vị tự ngoài và trong. Chứng minh
rằng đường tròn đường kính P Q đồng trục với (O1 ) và (O2 ).
2 2
P O1 R1 P O1 R12 PP/(O1 ) P O1 − R12 R12
Chứng minh. Ta có = do đó 2 = , suy ra = = .
P O2 R2 P O2 R22 PP/(O2 ) 2
P O2 − R22 R22
PP/(O1 ) R2 PQ/(O1 )
Tương tự với Q suy ra = 12 = . Theo hệ quả 2, đường tròn đường kính P Q đồng
PP/(O2 ) R2 PQ/(O2 )
trục với (O1 ) và (O2 ).

Bài 27. (Greek MO 2003). Cho đường tròn (O), một điểm A cố định trên (O) và một điểm R cố định
nằm trong (O). Một đường thẳng d chuyển động qua R cắt (O) tại B, C. Gọi H là trực tâm tam giác
ABC. Chứng minh rằng tồn tại duy nhất một điểm T sao cho HA2 + HT 2 =const.

T
H

B
R A1
C

A2

Chứng minh. Gọi A1 là hình chiếu của A trên BC. AA1 cắt (O) lần thứ hai tại A2 .
PH/(AR) HA · HA1 HA1 1
Ta có = = = =const.
PH/(O) HA · HA2 HA2 2
Suy ra H chuyển động trên một đường tròn ω cố định và đồng trục với (AR) và (O). Hiển nhiên
A ∈ ω.

15
Gọi T là điểm đối xứng với A qua tâm của ω. Suy ra điểm T xác định duy nhất và HA2 + HT 2 =
AT 2=const.

Bài 28. Cho tứ giác lồi ABCD, AD giao BC tại E, AB giao CD tại F . Chứng minh rằng các đường
tròn đường kính AC, BD, EF đồng trục.

G
H
A
I
J

D C
F

Chứng minh. Kẻ CG vuông góc với AD; AH, F I, DJ cùng vuông góc với BC. Đặt các góc của tứ giác
ABCD lần lượt là A, B, C, D.
PE/(AC) EG · EA
Ta có = .
PE/(BD) EB · EJ
EG EC PE/(AC) EC EA
Do EG · ED = EJ · EC nên = . Suy ra = ·
EJ ED PE/(BD) ED EB
PE/(AC) sin D sin B
Theo định lý hàm số sin ta thu được = .
PE/(BD) sin C sin A
PF/(AC) sin D sin B
Chứng minh tương tự, = .
PF/(BD) sin C sin A
PI/(AC) IH · IC FA FC PF/(AC) PE/(AC) PF/(AC) PI/(AC)
Lại có = = · = . Như vậy = = .
PI/(BD) IJ · IB FB FD PF/(BD) PE/(BD) PF/(BD) PI/(BD)
Theo hệ quả 2 thì (EF I) đồng trục với (AC) và (BD) hay 3 đường tròn (AC), (BD), (EF ) đồng
trục.

Nhận xét. Từ bài toán 5 ta thu được hai định lý quen thuộc:
1. Trung điểm của AC, BD, EF thẳng hàng (đường thẳng Gauss-Newton).
2. Các trực tâm của các tam giác EAB, ECD, F AD, F BC thẳng hàng (đường thẳng Steiner của
tứ giác toàn phần).

Bài 29. (IMO Shortlist 2012). Cho tam giác ABC nội tiếp đường tròn tâm O. Gọi d là đường thẳng
bất kì cắt BC, CA, AB lần lượt tại X, Y, Z; P là hình chiếu của O trên d. Chứng minh rằng các đường
tròn (AXP ), (BY P ), (CZP ) đồng trục.

16
A

Y
P F

B
X C

PA/(BP Y ) PX/(BP Y )
Chứng minh. Đường tròn (AP X) đồng trục với (BP Y ) và (CP Z) khi và chỉ khi =
PA/(CP Z) PX/(CP Z)
AB · AE XP · XY XY
Hay = = . (1)
AC · AF XP · XZ XZ
XY BZ CA
Áp dụng định lý Menelaus cho tam giác AY Z với đường thẳng (X, B, C) ta có · · = 1.
XZ BA CY
XY BA CY
Suy ra = · .
XZ BZ CA
AE CY
Như vậy (1) tương đương = .
AF BZ
Ta có ZA · ZB − Y A · Y C = PZ/(O) − PY /(O)
= ZO2 − Y O2 = ZP 2 − Y P 2 = ZY · ZP − Y Z · Y P = ZB · ZE − Y F · Y C.
AE CY
Do đó ZB · (ZA − ZE) = Y C · (F Y + Y A) hay ZB · EA = Y C · F A hay = .
AF BZ
Ta có đpcm.

Bài 30. Cho tam giác ABC và điểm P nằm trong tam giác. Một đường tròn ω qua P cắt (BP C),
(CP A), (AP B) lần thứ hai lần lượt tại X, Y, Z. Các đường thẳng AP, BP, CP cắt ω lần thứ hai tại
M, N, L. Chứng minh rằng XM, Y N, ZL đồng quy.

A'
N'

Y'
U M'
L'
Z' P

C'
X' B'

17
Chứng minh. (Luis González).
Xét phép nghịch đảo cực P phương tích bất kì. Khi đó ω biến thành đường thẳng l không qua P ,
(P AB), (P BC), (P CA) lần lượt biến thành các đường thẳng A0 B 0 , B 0 C 0 , C 0 A0 . X, Y, Z biến thành giao
điểm X 0 , Y 0 , Z 0 của l với B 0 C 0 , C 0 A0 , A0 B 0 . M, N, L biến thành giao điểm M 0 , N 0 , L0 của P A0 , P B 0 , P C 0
với l. Như vậy XM, Y N, ZL đồng quy khi và chỉ khi ω1 = (P X 0 M 0 ), ω2 = (P Y 0 N 0 ), ω3 = (P Z 0 L0 )
đồng trục.
Gọi U là giao của CP 0 với A0 B 0 . Suy ra P (A0 B 0 U Z 0 ) = P (M 0 N 0 L0 Z 0 ), C 0 (A0 B 0 U Z 0 ) = C 0 (Y 0 X 0 L0 Z 0 ).
Z 0 N 0 L0 M 0 L0 Y 0 Z 0 X 0
Từ đó 0 0 · 0 0 = 0 0 · 0 0
ZM LN LX ZY
L0 X 0 · L0 M 0 Z 0M 0 · Z 0X 0 PL0 /ω1 PZ 0 /ω1
Suy ra 0 0 0 0 = 0 0 Hay =
LY ·LN Z N ·Z Y 0 0 PL0 /ω2 PZ 0 /ω2
0 0
Điều này nghĩa là (P Z L ) đồng trục với ω1 và ω2 . Từ đó có đpcm.

Bài 31. Một đường thẳng d nào đó cắt hai đường tròn (O) và (I) theo thứ tự tại các cặp điểm A, A0
và B, B 0 . Khi đó các giao điểm của các tiếp tuyến với đường tròn thứ nhất tại A và A0 và các tiếp
tuyến với đường tròn thứ hai tại B và B 0 cùng nằm trên một đường tròn có tâm thẳng hàng với các
tâm của hai đường tròn đã cho.

Q
P A'
B'
N

A I
O

Chứng minh. Giả sử các tiếp tuyến cắt nhau tạo thành tứ giác M N P Q như hình vẽ.
Đặt ∠N AB = ∠P A0 B 0 = α, ∠P BA0 = ∠N B 0 A = β.
PM/(O) PN/(O) PP/(O) PQ/(O)
Ta cần chứng minh = = = .
PM/(I) PN/(I) PP/(I) PQ/(I)
M A2 N A2 P A02 QA02
Khi và chỉ khi = = = .
M B2 N B 02 P B2 QB 02
MA sin ∠ABM sin β
Theo định lý hàm số sin, = = . Hoàn toàn tương tự với các điểm N, P, Q ta
MB sin ∠M AB sin α
M A2 N A2 P A02 QA02 sin2 β
thu được = = = = .
M B2 N B 02 P B2 QB 02 sin2 α
Vậy M, N, P, Q cùng thuộc một đường tròn đồng trục với (O) và (I), tức là có tâm nằm trên
OI.

Bài 32. Cho hai tam giác ABC và A0 B 0 C 0 cùng nội tiếp đường tròn (O, R) và có cùng trọng tâm G.
Gọi X, Y, Z lần lượt là giao điểm của BB 0 và CC 0 , AA0 và CC 0 , AA0 và BB 0 . Chứng minh rằng tâm
đường tròn (XY Z) nằm trên OG.

18
B'1

Y
B'2

A C'

C'1 B2

A' C1 G B1
O A
2
Z

B C2 A1 C A'2

B'
C'2

A'1

B3

Chứng minh. (dựa theo Nguyễn Minh Hà).


Do hai tam giác ABC và A0 B 0 C 0 có cùng trọng tâm và tâm đường tròn ngoại tiếp nên hiển nhiên
chúng có chung đường tròn Euler.
Gọi A1 , B1 , C1 ; A2 , B2 , C2 lần lượt là trung điểm BC, CA, AB; B 0 C 0 , C 0 A0 , A0 B 0 . Suy ra A1 , B1 ,C1 ,
A2 , B2 , C2 cùng thuộc một đường tròn.
Phép nghịch đảo cực O phương tích R2 : IO R2 : A 7→ A0 , A 7→ A0 , B 7→ B 0 , B 7→ B 0 , C 7→
1 1 2 2 1 1 2 2 1
C10 , C2 7→ C20 .
Suy ra A01 , B10 , C10 , A02 , B20 , C20 cùng thuộc một đường tròn ω.
Dễ thấy hai tam giác A01 B10 C10 và A02 B20 C20 có chung đường tròn nội tiếp (O).
Gọi B 0 C 0 giao BC tại X 0 suy ra A01 , A02 , X cùng nằm trên đường đối cực của X 0 với (O), tức là
A1 , A02 , X thẳng hàng.
0

Gọi B3 là giao của C10 C20 và A01 A02 . Ta có ∠B3 XZ = ∠B 0 XA02 + ∠B 0 A02 X = ∠C10 BZ + ∠ZC10 B =
∠B3 ZX, suy ra tam giác B3 ZX cân tại B3 .
Chứng minh tương tự suy ra đường tròn (XY Z) là đường tròn nội tiếp tam giác tạo bởi giao điểm
các đường thẳng A01 A02 , B10 B20 , C10 C20 .
Áp dụng bài toán 9 cho hai đường tròn (O) và (XY Z) với đường thẳng (Z, B, B 0 , X) suy ra đường
tròn ω đồng trục với (O) và (XY Z).
Mặt khác do ω là ảnh của đường tròn Euler qua phép nghịch đảo IO R2 nên tâm của ω nằm trên

OG.
Vậy tâm của (XY Z) nằm trên OG.

Bài 33. (Đường thẳng Droz-Farny). Cho tam giác ABC với trực tâm H. Gọi d1 , d2 là hai đường thẳng
qua H sao cho d1 ⊥ d2 . Gọi A1 , B1 , C1 lần lượt là giao của d1 với BC, CA, AB; A2 , B2 , C2 lần lượt là
giao của d2 với BC, CA, AB. Chứng minh rằng trung điểm của A1 A2 , B1 B2 , C1 C2 cùng nằm trên một
đường thẳng gọi là đường thẳng Droz-Farny của tam giác ABC ứng với d1 và d2 .

19
B2

C2 Y

Z B1

H
C1
A1

B X A2 C

Chứng minh. Trung điểm của A1 A2 , B1 B2 , C1 C2 thẳng hàng khi và chỉ khi các đường tròn đường kính
A1 A2 , B1 B2 , C1 C2 đồng trục.
PA1 /(B1 B2 ) PA2 /(B1 B2 ) A1 C 1 · A1 H A2 C 2 · A2 H
Khi và chỉ khi = hay =
PA1 /(C1 C2 ) PA2 /(C1 C2 ) A1 B 1 · A1 H A2 B 2 · A2 H
A1 C 1 A2 C2 A1 C 1 A1 B 1
Như vậy ta cần chứng minh = hay = .
A1 B 1 A2 B 2 A C
2 2 A 2 B2
A1 C1 sin B A2 C 2 sin B A1 C 1 A1 B sin ∠HC2 C1 A1 B
Ta có = , = . Do đó = · = ·
A1 B sin ∠HC1 C2 BA2 sin ∠HC2 C1 A2 C 2 A2 B sin ∠HC1 C2 A2 B
HC1
.
HC2
A1 B sin ∠A1 HB HA1 sin ∠HB2 B1 HA1 HB1 HA1
Lại có = · = · = · .
A2 B sin ∠A2 HB HA2 sin ∠HB1 B2 HA2 HB2 HA2
A1 C 1 HA1 HB1 HC1
Vậy = · · .
A2 C 2 HA2 HB2 HC2
A1 C 1 A1 B 1
Tương tự suy ra = . Từ đó có đpcm.
A2 C 2 A2 B 2
Bài 34. Cho tam giác ABC nội tiếp đường tròn (O). Các đường cao BB1 , CC1 giao nhau tại H. Gọi
M là trung điểm BC. M C1 giao AC tại I, M B1 giao AB tại J. Gọi D là hình chiếu của A trên OH.
Chứng minh rằng A, D, I, J cùng thuộc một đường tròn.

20
I

A
D B1

C1
H E
F

O
B M C

Chứng minh. Gọi E, F lần lượt là trung điểm AC, AB.


Dễ thấy AD là trục đẳng phương của đường tròn đường kính AH và đường tròn đường kính AO.
Ta có B1 C1 là đường đối song với BC, mà EF k BC nên B1 C1 là đường đối song với EF hay tứ
giác C1 B1 EF nội tiếp.
Suy ra ∠JC1 B1 = ∠B1 EF . Lại có JM là tiếp tuyến của (AH) nên ∠JB1 C1 = 180◦ − ∠BAC =
∠F B1 E (do tam giác F AB1 cân tại F ).
JB1 JC1 EF
Suy ra 4C1 B1 J ∼ 4EB1 F . Ta thu được = = .
JA JB1 F B1
Như vậy tồn tại một phép vị tự quay tâm B1 biến F 7→ J, E 7→ C1 nên 4F B1 J ∼ 4EB1 C1 .
JB1 JF JB1 B 1 C1
Suy ra = hay = .
B1 C1 C1 E JF C1 E
JB12 EF.B1 C1 a/2.B1 C1 PJ/(AH) 2a.B1 C1
Vậy = = hay = .
JA.JF F B1 .C1 E c/2.b/2 PJ/(AO) bc
PJ/(AH) PI/(AH) 2a.B1 C1
Chứng minh tương tự suy ra = = .
PJ/(AO) PI/(AO) bc
Tức là I, J nằm trên một đường tròn đồng trục với (AH) và (AO). Vậy A, D, I, J cùng thuộc một
đường tròn.

Bài 35. (Trần Quang Hùng). Cho tam giác ABC. M, N lần lượt là các điểm trên AB, AC sao cho
M N song song với đường đối trung ứng với đỉnh A. Một đường tròn (O1 ) qua B, M giao một đường
tròn (O2 ) qua C, N tại hai điểm X, Y . Chứng minh rằng tâm đường tròn ngoại tiếp tam giác AXY là
trung điểm O1 O2 .

21
M

N X

O1
O
O2

B K J C

Chứng minh. Gọi K là giao của đường đối trung ứng với đỉnh A với BC; J là giao của M N với BC.
AN KJ AM KJ
Ta có = , = .
AC KC AB KB
2 2 2
PA/(O1 ) AB · AM AB AM AC AB KJ KC AB KC
Suy ra = = 2 · . = 2 · · = 2 · = −1.
PA/(O2 ) AC · AN AC AB AN AC KB KJ AC KB
Do đó A nằm trên đường tròn đẳng phương của (O1 ) và (O2 ), tức là tâm của (AXY ) là trung điểm
của O1 O2 .

Bài 36. (Mathley 7). Cho tứ giác ABCD nội tiếp đường tròn (O). AB giao CD tại E, AD giao BC
tại F , AC giao BD tại I. Gọi K, L lần lượt là giao điểm thứ hai của (F AB) và (F CD) với F I. Chứng
minh rằng EK = EL.

A K

I
M N
E
H O
D

L
C

Chứng minh. Gọi H là giao của EO với F I. M, N lần lượt là trung điểm của AD, BC. Theo định lý
Brocard suy ra EO ⊥ F I. Do đó A, M, N, H, O cùng nằm trên đường tròn đường kính AO.
PM/(F AB) MA · MF
Xét = = −1.
PM/(F CD) MD · MF
PM/(F AB) PN/(F AB)
Tương tự suy ra = = −1.
PM/(F CD) PN/(F CD)
PH/(F AB)
Từ đó (F M N ) đồng trục với (F AB) và (F CD). Mà H ∈ (F M N ) nên = −1.
PH/(F CD)

22
HK · HF
Hay = −1. Suy ra H là trung điểm KL.
HL · HF
Tam giác EKL có H vừa là chân đường cao vừa là trung điểm KL nên EK = EL.

2.3 Đường tròn đẳng phương


Đường tròn đẳng phương đang xem xét được phân biệt với đường tròn đẳng phương của 3 đường
tròn (xem [4]). Ở phần 1 chúng ta đã biết định nghĩa đường tròn đẳng phương ω của hai đường tròn
(O1 , R1 ) và (O2 , R2 ) là quỹ tích các điểm có tỉ số phương tích đến hai đường tròn bằng −1. ω đồng
trục với (O1 ) và (O2 ) đồng thời có tâm O là trung điểm O1 O2 .
Với mỗi điểm M nằm trên ω, ta có PM/(O1 ) + PM/(O2 ) = 0.
Suy ra M O12 − R12 + M O22 − R22 = 0 hay M O12 + M O22 = R12 + R22 .
1
Theo công thức tính đường trung tuyến suy ra 2M O2 + O1 O22 = R12 + R22 .
2
1 1
Như vậy bình phương bán kính R của ω bằng (R12 + R22 ) − O1 O22 . Điều kiện bán kính phải không
2 4
âm do đó ta cần xem xét các trường hợp sau.
Nếu (O1 ) và (O2 ) cắt nhau thì ω luôn tồn tại, nhưng trong trường hợp (O1 ) và (O2 ) không cắt
nhau, có hai trường hợp xảy ra. Nếu (O1 ) và (O2 ) chứa nhau thì ω tồn tại. Nếu (O1 ) và (O2 ) ngoài
nhau, vị trí của O1 và O2 phải thỏa mãn trung điểm O của O1 O2 nằm ngoài hai điểm tới hạn của bộ
đường tròn đồng trục d là trục đẳng phương của (O1 ) và (O2 ).
Trong trường hợp hai đường tròn (O1 ) và (O2 ) trực giao, ω trở thành đường tròn đường kính O1 O2 .
Tính chất 1. Nếu hai đường tròn (O1 ) và (O2 ) cắt nhau tại A, B. Một đường thẳng bất kì qua A cắt
(O1 ), (O2 ) lần lượt tại C, D. Khi đó quỹ tích trung điểm của CD là đường tròn đẳng phương của (O1 )
và (O2 ).
PM/(O1 ) MA · MC
Chứng minh. Gọi M là trung điểm CD. Ta có = = −1. Suy ra M ∈ ω.
PM/(O2 ) MA · MD

Tính chất 2. Một cát tuyến cắt (O1 ), (O2 ) lần lượt tại các điểm A, B và C, D. Khi đó (ABCD) = −1
khi và chỉ khi trung điểm của AB hoặc CD nằm trên đường tròn đẳng phương của (O1 ) và (O2 ).

M
C
O1 O2
B
N

Chứng minh. Gọi M, N lần lượt là trung điểm của AB, CD. Ta có (ABCD) = −1 khi và chỉ khi
M B 2 = M C · M D.
PM/(O1 ) M A. · M B
Tương đương = = −1 hay M nằm trên ω. Chứng minh tương tự với điểm
PM/(O2 ) M B2
N.

Hệ quả. Nếu hai đường tròn (O1 ) và (O2 ) trực giao, cát tuyến d cắt hai đường tròn thành một
hàng điểm điều hòa khi và chỉ khi d đi qua một trong hai tâm của (O1 ) và (O2 ).

23
1
Tính chất 3. P là một điểm bất kì trên mặt phẳng. Khi đó PP/ω = (PP/(O1 ) + PP/(O2 ) ).
2
Chứng minh. Theo công thức tính bán kính đường tròn đẳng phương ta có
1 1 1 1 1 1
PP/ω = P O2 −R2 = (P O12 +P O22 − O1 O22 )− (R12 +R22 )+ O1 O22 = (P O12 −R12 )+ (P O22 −R22 ) =
2 2 2 4 2 2
1
(P + PP/(O2 ) ).
2 P/(O1 )
Tính chất 4. Cho 3 đường tròn (O1 ), (O2 ), (O3 ) sao cho mỗi cặp hai trong ba đường tròn đều có đường
tròn đẳng phương. Gọi ωij là đường tròn đẳng phương của hai đường tròn (Oi ), (Oj ) (i, j = 1, 3, i 6= j).
P là một điểm bất kì trên mặt phẳng. Khi đó
PP/ω12 + PP/ω23 + PP/ω13 = PP/(O1 ) + PP/(O2 ) + PP/(O3 )

Chứng minh. Dễ dàng chứng minh tính chất 4 theo tính chất 3.

Tính chất 5. Với giả thiết như tính chất 4. Ta có đường tròn đẳng phương của ωij và (Ok ) trùng với
đường tròn đẳng phương của ωik và ωjk (i, j, k = 1, 3, i 6= j 6= k).

O2
O1
O12

I
O13 O23

O3

Chứng minh. Kí hiệu Oij là tâm của ωij .


Gọi P là tâm đẳng phương của (O1 ), (O2 ), (O3 ) và phương tích từ P đến 3 đường tròn bằng T .
1 1
Theo tính chất 3, PP/ωij = (PP/(Oi ) + PP/(Oj ) ) = .2T = T .
2 2
Như vậy P là tâm đẳng phương của 6 đường tròn (O1 ), (O2 ), (O3 ), ω12 , ω23 , ω13 .
Gọi (I) là đường tròn đẳng phương của ω12 và ω23 . Chứng minh tương tự ta cũng suy ra PP/(I) = T .
Ta có I là trung điểm O12 O23 ; O12 , O23 , O13 lần lượt là trung điểm của O1 O2 , O2 O3 , O1 O3 nên I
là trung điểm của O2 O13 .
Như vậy 3 đường tròn (O2 ), ω13 , (I) có trục đẳng phương song song. Mà 3 đường tròn có tâm đẳng
phương P nên chúng đồng trục. Nghĩa là (I) là đường tròn đẳng phương của (O2 ) và ω13 . Chứng minh
tương tự cho các cặp còn lại.

Từ lời giải trên ta thu được tính chất tổng quát sau.
Tính chất 6. Từ 3 đường tròn (O1 ), (O2 ), (O3 ), ta xây dựng một chuỗi đường tròn (O1 ), (O2 ), ..., (On )
sao cho (Ok ) là đường tròn đẳng phương của 2 trong k − 1 đường tròn (O1 ), (O2 ), ..., (Ok−1 ) (k = 4, n).
Khi đó n đường tròn (O1 ), (O2 ), ..., (On ) có chung tâm đẳng phương.

24
Tính chất 7. Cho tam giác ABC và hai điểm P, Q liên hợp đẳng giác. Một đường tròn ωP tâm P
cắt 3 cạnh BC, CA, AB lần lượt tại các cặp điểm A1 , A2 ; B1 , B2 ; C1 , C2 . Khi đó tồn tại đường tròn
ωQ tâm Q trực giao với các đường tròn đường kính A1 A2 , B1 B2 , C1 C2 và đường tròn pedal của tam
giác ABC ứng với hai điểm P, Q là đường tròn đẳng phương của ωP và ωQ .

A
C1
B1

Pb

B2
Pc

P O Q

C2

B C

Chứng minh. Gọi Pb , Pc , Pa lần lượt là hình chiếu của P trên AC, AB, BC. Suy ra Pb , Pc lần lượt là
tâm của (B1 B2 ) và (C1 C2 ).
Ta có AQ ⊥ Pb Pc và AC1 · AC2 = AB1 · AB2 nên AQ là trục đẳng phương của (B1 B2 ) và (C1 C2 ).
Chứng minh tương tự suy ra Q là tâm đẳng phương của (A1 A2 ), (B1 B2 ), (C1 C2 ). Như vậy đường tròn
tâm Q bán kính bằng căn của phương tích từ Q đến 3 đường tròn (A1 A2 ), (B1 B2 ), (C1 C2 ) sẽ trực giao
với 3 đường tròn đó.
PPb /ωP Pb B1 · Pb B2
Ta có = = −1.
PPb /ωQ Pb B22
Suy ra Pb nằm trên đường tròn đẳng phương của ωP và ωQ .
Chứng minh tương tự suy ra đường tròn ngoại tiếp tam giác Pa Pb Pc là đường tròn đẳng phương
của ωP và ωQ . Ta có đpcm.

2.4 Bài tập áp dụng


Bài 37. Cho tứ giác ABCD. AD giao BC tại K. Đường tròn ngoại tiếp các tam giác KAC và KDB
cắt nhau lần thứ hai tại T . Gọi M, N lần lượt là trung điểm AD, BC. Chứng minh rằng tứ giác
KM T N nội tiếp.
Bài 38. Cho tam giác ABC với phân giác AD. Gọi (I) và (Ia ) lần lượt là đường tròn nội tiếp và bàng
tiếp góc A. Chứng minh rằng đường tròn đường kính AD đồng trục với (I) và (Ia ).
Bài 39. Cho tam giác ABC có trực tâm H, trọng tâm G. Chứng minh rằng đường tròn đường kính
HG đồng trục với đường tròn ngoại tiếp và đường tròn Euler của tam giác ABC.
Bài 40. Cho tam giác ABC nội tiếp đường tròn (O), với trực tâm H và tâm đường tròn Euler E.
Gọi B 0 , C 0 lần lượt là hai điểm trên AB, AC sao cho E là trung điểm của B 0 C 0 . Chứng minh rằng các
đường tròn (AB 0 C 0 ), (AH), (O) đồng trục.
Bài 41. Cho tam giác ABC nhọn nội tiếp đường tròn (O). Hai đường cao BB 0 và CC 0 giao nhau
tại trực tâm H. Đường thẳng qua H song song với AB cắt AC tại X, đường thẳng qua H song song
với AC cắt AB tại Y . (AH) giao (O) lần thứ hai tại Z. Chứng minh rằng A, X, Y, Z cùng thuộc một
đường tròn.
Bài 42. Cho hai đường tròn (O1 ) và (O2 ) giao nhau tại hai điểm. Gọi AB là một dây cung chuyển
động trên (O2 ) sao cho đường tròn tâm P đường kính AB trực giao với đường tròn (O1 ). Tìm quỹ tích
điểm P.

25
Bài 43. (Nguyễn Văn Linh). Cho hai đường tròn (O1 ) và (O2 ) giao nhau tại A, B. Gọi C, D lần lượt
là các điểm trên (O1 ), (O2 ) sao cho ∠CAB = ∠DAB. BC, BD giao (O2 ), (O1 ) lần thứ hai tại E, F .
Đường thẳng qua A vuông góc với AB cắt (O1 ), (O2 ) lần lượt tại X, Y . Gọi M là trung điểm XY .
Chứng minh rằng tứ giác CDEF nội tiếp đường tròn tâm M.

Bài 44. Cho tam giác ABC có trực tâm H. Phân giác ngoài góc A cắt các đường tròn (ABH), (ACH)
lần lượt tại D, E. Chứng minh rằng trung điểm của BC, AH, DE thẳng hàng.

Bài 45. (IMO Shortlist 1987). Cho tam giác ABC. Một tam giác đều A0 B 0 C 0 chuyển động sao cho
A0 B 0 C 0 ngoại tiếp tam giác ABC (các đỉnh của tam giác ABC tương ứng nằm trên cạnh tam giác
A0 B 0 C 0 ). Tìm quỹ tích trọng tâm của tam giác A0 B 0 C 0 .

3 Phép vị tự quay
3.1 Giới thiệu
Phép vị tự và phép quay là những phép biến hình quen thuộc. Tuy nhiên phép vị tự quay còn ít
được đề cập tới. Vì vậy trong bài viết này xin giới thiệu với bạn đọc các tính chất và ứng dụng của
phép vị tự quay.
Trước tiên chúng ta cần hiểu định nghĩa của phép vị tự quay.

Định nghĩa. Phép vị tự quay là hợp của một phép vị tự và một phép quay có chung tâm.
k,α
Nếu kí hiệu phép vị tự quay có tâm O, tỉ số k và góc quay α là SO thì
k,α OB − → −−

SO : A 7→ B khi và chỉ khi = k và (OA, OB) = α.
OA

A'

O 
A

Ở đây ta hiểu α là góc có hướng theo chiều ngược kim đồng hồ, −α là góc có chiều xuôi kim đồng
hồ. Tuy nhiên trong bài viết này không quan tâm tới hướng của góc α.

3.2 Tính chất


Tính chất 8. SO : A 7→ A0 , B 7→ B 0 thì (AB) 7→ (A0 B 0 ), [AB] 7→ [A0 B 0 ].

M
A

A' N B'

Chứng minh. Thật vậy, gọi M là điểm bất kì trên đường thẳng AB, N là điểm trên A0 B 0 sao cho
MA N A0
= .
MB N B0

26
MA N A0
Từ giả thiết suy ra 4OAA0 ∼ 4OBB 0 , từ đó 4OAB ∼ 4OA0 B 0 . Mà = nên 4OAM ∼
MB N B0
4OA0 N . Từ đó 4OAA0 ∼ 4OM N hay SO : M 7→ N.

Từ lời giải tính chất 1 chúng ta có tính chất 2.

MA N A0
Tính chất 9. SO : [AB] 7→ [A0 B 0 ], M ∈ AB, N ∈ A0 B 0 sao cho = thì SO : M 7→ N .
MB N B0
Tính chất 10. SO : [AB] 7→ [A0 B 0 ] thì tồn tại một phép vị tự quay khác có tâm O, S 0 O : [AA0 ] 7→
[B 0 B 0 ].

Chứng minh. Ta có 4OAA0 ∼ 4OBB 0 nên 4OAB ∼ 4OA0 B 0 . Do đó tồn tại một phép vị tự quay
tâm O biến [AA0 ] thành [BB 0 ].

Tính chất 11. Cho hai đoạn thẳng không song song [AB] và [A0 B 0 ]. Khi đó luôn tồn tại một phép vị
tự quay tâm O biến [AB] thành [A0 B 0 ]. Nếu gọi P là giao của hai đường thẳng AB và A0 B 0 thì O là
giao điểm thứ hai của đường tròn ngoại tiếp các tam giác P AA0 và P BB 0 .

A'

B B'

Chứng minh. Do O là giao điểm thứ hai của (P AA0 ) và (P BB 0 ) nên ∠OAP = ∠OA0 P , ∠OBP =
∠OB 0 P . Từ đó 4OAA0 ∼ 4OBB 0 . Như vậy O là tâm của phép vị tự quay biến [AB] thành [A0 B 0 ].

Từ tính chất 3 và 4 ta có hệ quả quen thuộc: điểm Miquel của tứ giác toàn phần.
Cho tứ giác ABCD. AD giao BC tại E, AB giao CD tại F . Khi đó đường tròn ngoại tiếp của
các tam giác EAB, ECD, F BC, F AD đồng quy tại một điểm.

M
B

F
D C

27
Như vậy điểm Miquel của tứ giác toàn phần là tâm của phép vị tự quay của các cặp cạnh đối diện
của tứ giác.

Tính chất 12. Cho hai tam giác ABC và A0 B 0 C 0 đồng dạng cùng hướng. Khi đó luôn tồn tại một
phép vị tự quay biến tam giác ABC thành tam giác A0 B 0 C 0 .

A
O

A'
B'

C'

Chứng minh. Xét phép vị tự quay SO : [AB] 7→ [A0 B 0 ]. Khi đó 4OAB ∼ 4OA0 B 0 . Mà 4ABC ∼
4A0 B 0 C 0 nên dễ chứng minh 4OAA0 ∼ 4OCC 0 .
Do đó SO : C 7→ C 0 , suy ra SO : 4ABC 7→ 4A0 B 0 C 0 .

Tính chất 13. Cho hai đường tròn (O1 ) và (O2 ) giao nhau tại A và B. Khi đó A và B là hai tâm vị
tự quay biến (O1 ) thành (O2 ).

O1 O2 Q

B
P

Chứng minh. Gọi P là một điểm bất kì trên (O1 ). Đường thẳng P B giao (O2 ) lần thứ hai tại Q. Dễ
thấy 4AP Q ∼ 4AO1 O2 . Do đó xét phép vị tự tâm A biến O1 thành O2 :
R2
,(AO1 ,AO2 )
SAR1 : P 7→ Q. Như vậy ứng với mỗi điểm P , ảnh của P qua phép vị tự này luôn nằm
trên (O2 ). Do đó phép vị tự tâm A biến (O1 ) thành (O2 ). Chứng minh tương tự với tâm B.

3.3 Ví dụ
Bài 46. (Bổ đề ERIQ) Cho hai đường thẳng d1 và d2 . Trên d1 lấy các điểm A1 , B1 , C1 , trên d2 lấy các
A1 B 1 A2 B2
điểm A2 , B2 , C2 sao cho = = k. Trên A1 A2 , B1 B2 , C1 C2 lần lượt lấy các điểm A3 , B3 , C3
B1 C1 B 2 C2
A3 A1 B3 B1 C3 C1 A3 B 3
sao cho = = . Chứng minh rằng A3 , B3 , C3 thẳng hàng và = k.
A3 A2 B3 B2 C3 C2 B 3 C3

28
A3 A2
A1

B1 B3 B2

C1
C3 C2

Chứng minh. Xét phép vị tự quay tâm O, SO : [A1 C1 ] 7→ [A2 C2 ].


A1 B1 A2 B 2
Do = nên SO : B1 7→ B2 . Suy ra SO : [A1 B1 ] 7→ [A2 B2 ]. Theo tính chất 3, tồn tại một
B 1 C1 B2 C2
A3 A1 B3 B1
phép vị tự quay S 0 O : [A1 A2 ] 7→ [B1 B2 ]. Lại có = nên S 0 O : A3 7→ B3 .
A3 A2 B3 B2
Suy ra tồn tại một phép vị tự quay S 00 O : [A3 B3 ] 7→ [A2 B2 ].
Tương tự, tồn tại phép vị tự quay S 000 O : [A3 C3 ] 7→ [A2 C2 ].
Do S 00 O và S 000 O cùng biến A3 thành A2 nên S 00 O ≡ S 000 O .
Vậy S 00 O : A3 7→ A2 , B3 7→ B2 , C3 7→ C2 .
A2 B2 A3 B3
Mà A2 , B2 , C2 thẳng hàng đồng thời = k nên A3 , B3 , C3 thẳng hàng và = k.
B 2 C2 B3 C3
Bài 47. Cho hai đường thẳng d và d0 . Trên d lấy ba điểm A, B, C, trên d0 lấy ba điểm A0 , B 0 , C 0 sao
AB A0 B 0
cho = 0 0 . Gọi P là giao của d và d0 . Chứng minh rằng các đường tròn ngoại tiếp các tam giác
BC BC
P AA0 , P BB 0 , P CC 0 đồng trục.

A'
A

B B'

C
C'

AB A0 B 0
Chứng minh. Xét phép vị tự quay SO : [AB] 7→ [A0 B 0 ] thì P ∈ (P AA0 ), (P BB 0 ). Do = 0 0 nên
BC BC
SO : C 7→ C 0 . Suy ra O ∈ (P CC 0 ). Vậy 3 đường tròn (P AA0 ), (P BB 0 ), (P CC 0 ) có chung trục đẳng
phương OP.

Bài 48. Cho tam giác ABC nội tiếp đường tròn (O), BC cố định, A chuyển động trên một trong hai
CP
cung BC. P là điểm thuộc đoạn AC sao cho = k không đổi. Chứng minh rằng P chuyển động
AB
trên một đường tròn cố định.

29
A

B C

Chứng minh. Xét phép vị tự quay SQ : [AB] 7→ [P C]. Theo tính chất 3, tồn tại S 0 Q : [AP ] 7→ [BC].
Do AP giao BC tại C nên theo tính chất 4, Q ∈ (ABC) và Q ∈ (P CC) (đường tròn qua P và tiếp
QC
xúc với BC tại C). Do SQ : B 7→ C nên = k. Suy ra Q cố định. Do đó (QCC) cố định. Vậy P
QB
luôn chuyển động trên đường tròn qua Q và tiếp xúc với BC tại C.

Bài 49. (IMO Shortlist 2014). Cho 3 điểm A, B, C cố định trên đường tròn (O). Gọi λ là một hằng
số và λ ∈ (0, 1). Với mỗi điểm P 6= A, B, C trên (O), gọi M là điểm trên đoạn thẳng CP sao cho
CM = λ · CP . Q là giao điểm thứ hai của (AM P ) và (BM C). Chứng minh rằng khi P chuyển động,
Q luôn nằm trên một đường tròn cố định.

G P

K
Q M
X

B
C

Chứng minh. Gọi G là giao của BQ với (O), K là giao của GC và QM . Do tứ giác BQM C nội tiếp
KC MC
nên theo định lý Reim, P G k QM . Do đó = = λ.
GC PC
◦ ◦
Ta có ∠AGK = 180 − ∠AP M = 180 − ∠AGC nên A, G, M, Q cùng thuộc một đường tròn.
Do (GQK) giao (GBC) tại A nên tồn tại một phép vị tự quay SA : [BG] 7→ [CK]. Suy ra
BQ AB BQ AB
= . Từ đó = · λ. Áp dụng bài toán 3 suy ra Q chuyển động trên một đường tròn đi
CK AC CG AC
XB AB
qua điểm X trên (O) sao cho = · λ và tiếp xúc với BC tại B.
XC AC
Bài 50. (IMO 2007) Cho hình bình hành ABCD. Một đường thẳng d đi qua D cắt AB, BC lần lượt
tại M, N . Gọi O là tâm ngoại tiếp tam giác BM N . Chứng minh rằng nếu O nằm trên (ABC) thì d là
phân giác ∠ADC.

30
O N

Y
A M X
B

D C

MA AD BC
Chứng minh. Xét phép vị tự quay SO0 : [AB] 7→ [CN ]. Ta có = = nên SO0 : M 7→ B.
MB BN BN
Gọi X, Y lần lượt là trung điểm của M B, N B suy ra SO0 : X 7→ Y.
Do đó SO0 : [AX] 7→ [CY ]. Mà AX giao CY tại B nên theo tính chất 4, O0 ∈ (BXY ), (BAC). Suy
ra O0 ≡ O. Từ đó 4OAC ∼ 4OM B, suy ra OA = OC hay BO là phân giác ngoài của ∠ABC. Suy
ra tam giác BM N cân tại B.
Ta thu được ∠M DC = ∠BM N = ∠BN M = ∠ADN.

Bài 51. Cho tứ giác nội tiếp ABCD. AC giao BD tại O. (OAB) giao (OCD) tại K nằm trong
ABCD. Dựng điểm L thuộc nửa mặt phẳng bờ BC không chứa O sao cho tam giác BCL đồng dạng
với tam giác ADK. Chứng minh rằng tứ giác BKCL ngoại tiếp.

B L
A

O
I
K

Chứng minh. Dựng điểm I sao cho hai tam giác BKD và BIC đồng dạng thuận. Suy ra 4BKI ∼
4BDC. Từ đó ∠BKI = ∠BDC.
Do 4KAC ∼ 4KBD nên 4CIB ∼ 4CKA, từ đó 4CKI ∼ 4CAB.
Ta thu được ∠CKI = ∠CAB. Lại có ∠BDC = ∠CAB nên KI là phân giác ∠BKC.
Mặt khác, ta có ∠KBI = ∠DBC, ∠KBL = ∠KBC + ∠CBL = ∠KBC + ∠DAK = ∠DAC +
∠OAK + ∠KBC = ∠DAC + ∠OBK + ∠KBC = ∠DAC + ∠DBC = 2∠KBI.
Suy ra BI là phân giác ∠KBL. Tương tự CI là phân giác ∠KCL. Vậy tứ giác BKCL ngoại tiếp
đường tròn tâm I.

31
Bài 52. (Lym) Cho tam giác ABC nội tiếp (O), ngoại tiếp (I). (I) tiếp xúc với BC tại D. J là trung
điểm cung BC không chứa A. E thuộc ID, F thuộc BC sao cho EF đi qua J và IE = IF. EF cắt
(O) tại G. IG cắt (O) tại L. Chứng minh rằng đường thẳng Simson của L ứng với tam giác ABC tiếp
xúc với (I).

L
P O
I

H
B J
C
D

G
K

Chứng minh. Kẻ đường kính LQ của đường tròn ngoại tiếp tam giác LIG.
Do GI 2 = GJ.GH nên ∠GIH = ∠IJG = ∠IKJ = 90◦ − ∠GJB = 90◦ − ∠GLH.
Mà ∠GIH + ∠GIQ = 90◦ nên ∠QIG = ∠ILG = ∠IQG, suy ra GI = GQ hay Q ∈ (BIC).
Kẻ LM ⊥ AC, LP ⊥ AB. Ta có ∠LQI = ∠LGI = ∠LBP = ∠LCM nên phép vị tự quay
LP
,(LB,LP )
SLLB : B 7→ P, Q 7→ I, C 7→ M, suy ra 4BQC 7→ 4P IM . Như vậy 4BQC ∼ 4P IM . Suy ra
1
∠P IM = ∠BQC = 180◦ − ∠BIC = 90◦ − ∠M AP . Vậy (I) là đường tròn bàng tiếp góc A của tam
2
giác AP M hay P M tiếp xúc với (I). Ta có đpcm.

Bài 53. Cho tứ giác ABCD. Hai đường chéo AC và BD giao nhau tại P . Gọi O1 , O2 lần lượt là tâm
đường tròn ngoại tiếp các tam giác AP D và BP C. Gọi M, N, O lần lượt là trung điểm AC, BD, O1 O2 .
Chứng minh rằng O là tâm ngoại tiếp tam giác M P N.

P
N
O1 O2
O
M
Q
D

32
Chứng minh. Gọi Q là giao điểm thứ hai của (O1 ) và (O2 ). Do Q là tâm vị tự quay của (O1 ) và (O2 )
nên SQ : A 7→ C, D 7→ B. Do đó tồn tại phép vị tự quay S 0 Q : [AC] 7→ [DB].
Lại có M, N lần lượt là trung điểm của AC, BD nên S 0 Q : M 7→ N.
Từ đó (P AD), (P BC), (P M N ) cùng đi qua Q. Gọi O0 là tâm của (P M N ).
Ta có 4O1 QO0 ∼ 4DQN , 4O1 QO2 ∼ 4DQB nên O0 là trung điểm O1 O2 hay O0 ≡ O. Ta có
đpcm.

Nhận xét. Theo cách giải trên ta hoàn toàn có thể tổng quát bài toán bằng cách chọn các điểm
MA ND OO1
M, N, O sao cho = = = k.
MC NB OO2
Cách giải khác cho bài toán này, xem [2].

Bài 54. Cho n đường tròn C1 , C2 , ..., Cn cùng đi qua O. Gọi Aij là giao điểm của Ci và Cj . B1 là điểm
bất kì trên C1 . B1 A12 cắt C2 lần thứ hai tại B2 . Tương tự ta được B3 , B4 , ..., Bn , Bn+1 . Chứng minh
rằng Bn+1 ≡ B1 .

O1
O2
A2

A12
A1

Chứng minh. Gọi O1 , O2 , ..., On lần lượt là tâm của C1 , C2 , ..., Cn .


Dễ chứng minh hai tam giác O1 OO2 và A1 OA2 đồng dạng cùng hướng.
Do đó (OA1 , OA2 ) ≡ (OO1 , OO2 ) (mod π).
Chứng minh tương tự suy ra (OA1 , OAn+1 ) ≡ (OA1 , OA2 ) + (OA2 , OA3 ) + ... + (OAn , OAn+1 )
≡ (OO1 , OO2 ) + (OO2 , OO3 ) + ... + (OOn , OO1 ) ≡ 0 (mod π).
Vậy An+1 ≡ A1 .

Nhận xét. -Có thể chứng minh bài toán bằng phép quy nạp dựa trên điểm Miquel của tam giác.
-Dễ dàng chứng minh tích của n phép vị tự quay chung đỉnh là một phép vị tự quay. Kí hiệu
SOxy là phép vị tự quay tâm O biến (Ox ) thành (Oy ). Ta có thể chứng minh bài toán bằng cách xét
SO1n = SO(n−1)n ◦ ... ◦ SO34 ◦ SO23 ◦ SO12 . Suy ra B1 , An1 , Bn thẳng hàng.

Bài 55. Cho tam giác ABC. P là điểm bất kì trong mặt phẳng. Gọi XY Z là tam giác pedal của P
ứng với tam giác ABC. A1 B1 C1 là tam giác nội tiếp tam giác ABC sao cho A1 B1 C1 đồng dạng thuận
với XY Z. Chứng minh P là điểm Miquel của tam giác ABC ứng với bộ (A1 , B1 , C1 ).

33
A

C1

Y
Z
B1
P

B A1 X C

Chứng minh. Do hai tam giác A1 B1 C1 và XY Z đồng dạng thuận nên tồn tại phép vị tự quay SP0 :
4A1 B1 C1 7→ 4XY Z.
Suy ra tồn tại S 0 0P : [A1 X] 7→ [B1 Y ]. Do A1 X giao B1 Y tại C nên P 0 ∈ (A1 B1 C), (XY C).
Chứng minh tương tự suy ra P 0 là giao của (AY Z), (BXZ), (CXY ) hay P 0 ≡ P . Mà P là giao của
(AB1 C1 ), (BA1 C1 ), (CA1 B1 ) nên P là điểm Miquel của tam giác ABC ứng với bộ (A1 , B1 , C1 ).

Bài 56. (USA TST 2012) Cho tam giác ABC. P là một điểm chuyển động trên BC. Gọi Y, Z lần
lượt là các điểm trên AC, AB sao cho P Y = P C, P Z = P B. Chứng minh rằng (AY Z) luôn đi qua
trực tâm của tam giác ABC.

Chứng minh. Cách 1.

Z
H

B Q P C

Dựa theo ý tưởng bài 8, ta sẽ tìm một điểm Q trên BC sao cho 4QY Z đồng dạng cùng hướng với
tam giác hình chiếu của trực tâm.
Gọi Q là giao điểm thứ hai của (P Y Z) với BC.
Ta có ∠Y QZ = ∠Y P Z = 180◦ −2A, ∠ZY Q = ∠ZP B = 180◦ −2B, ∠Y ZQ = ∠Y P C = 180◦ −2C.
Ta biết rằng tam giác hình chiếu DEF của trực tâm có 3 góc lần lượt bằng 180◦ − 2∠A, 180◦ − 2∠B,
180◦ − 2∠C, do đó 4QY Z ∼ 4DEF . Suy ra trực tâm H là điểm Miquel của tam giác ABC ứng với
bộ điểm (Q, Y, Z). Vậy (AY Z) luôn đi qua H.
Cách 2.

34
A

Z T
H
S

B G P C

Gọi T, S lần lượt là hình chiếu của P trên AC, AB. Kẻ đường cao AG cắt (AEF ) tại H, cắt (ABC)
tại L.
Ta thấy S, T, G lần lượt là trung điểm của BZ, CY, HL.
Theo bài toán 2, (AHY ), (AGT ), (ALC) đồng trục và (AHZ), (AGS), (ALB) đồng trục.
Do (ALC) ≡ (ALB), (AGT ) ≡ (AGS) nên (AHY ) ≡ (AHZ). Vậy H ∈ (AY Z).

Nhận xét. Một số cách giải khác, xem [3].

3.4 Bài tập áp dụng


Bài 57. a) Cho tam giác ABC đồng dạng thuận với tam giác A0 B 0 C 0 , X, Y, Z lần lượt thuộc
AX BY CZ
AA0 , BB 0 , CC 0 sao cho 0
= 0
= . Chứng minh rằng 4XY Z ∼ 4ABC ∼ 4A0 B 0 C 0 .
XA YB ZC 0
b) Chứng minh bài toán với các điểm X, Y, Z thỏa mãn các tam giác AXA0 , BY B 0 , CZC 0 đồng
dạng thuận.

Bài 58. (Vietnam TST 2013). Cho tứ giác ABCD nội tiếp (O). AC giao BD tại I. phân giác góc
AIB cắt AB, BC, CD, DA lần lượt tại X, Y, Z, T . Chứng minh rằng (AXT ), (BXY ), (CY Z), (DZT )
cùng đi qua một điểm.

Bài 59. Cho tam giác ABC nội tiếp (O), ngoại tiếp (I). Đường tròn đường kính AI cắt (O) tại X.
(I) tiếp xúc với BC tại D. Chứng minh rằng XD đi qua trung điểm cung BC không chứa A.

Bài 60. Cho hình thang ABCD vuông (∠A = ∠B = 90◦ ) có AD = 2AB = 2BC. M là một điểm
chuyển động trên cạnh BC. Đường thẳng qua M và vuông góc với AM cắt CD tại N . Chứng minh
rằng trung điểm của M N chuyển động trên một đường thẳng cố định. thẳng hàng.
AM CN
Bài 61. Cho tứ giác ABCD. AC giao BD tại P . M, N lần lượt thuộc AD, BC sao cho = .
MD NB
M N cắt AC, BD tại E, F .
a) Chứng minh (P EF ) luôn đi qua điểm cố định khác P .
b) Nếu tứ giác ABCD nội tiếp. Gọi Q là giao của AD và BC. Chứng minh rằng (QM N ) và (P EF )
tiếp xúc nhau.

Bài 62. (Malaysia Junior Olympiad 2014). Cho tam giác ABC nội tiếp (O). X là một điểm bất kì
trên (O). Kẻ XC1 ⊥ AB, XB1 ⊥ AC. Gọi ωC là đường tròn có tâm là trung điểm AB và đi qua C1 .
Tương tự xác định ωB . Chứng minh rằng ωB và ωC có chung một điểm trên OX.

35
Bài 63. Cho tam giác ABC. Trung tuyến AM . O1 , O2 lần lượt là tâm ngoại tiếp các tam giác
AM B, AM C, O là tâm ngoại tiếp tam giác ABC. Chứng minh AO là đường đối trung của tam giác
AO1 O2 .

Bài 64. Cho tứ giác ABCD nội tiếp (O). AD giao BC tại P . AC giao BD tại I, AB giao CD tại Q.
P I cắt (P AB), (P CD) lần thứ hai tại M, N . Chứng minh rằng QM = QN.

Bài 65. (Iran 1997). Cho tam giác ABC nội tiếp (O). P chuyển động trên cung BC không chứa A.
I1 , I2 lần lượt là tâm nội tiếp các tam giác AP B, AP C. Chứng minh rằng (P I1 I2 ) luôn đi qua một
điểm cố định.

Bài 66. (IMO Shortlist 2002) Hai đường tròn S1 và S2 giao nhau tại P và Q. Chọn hai điểm A1 và
B1 bất kì trên S1 . A1 P, B1 P giao S2 lần thứ hai tại A2 , B2 , A1 B1 giao A2 B2 tại C. Chứng minh rằng
khi A1 và B1 chuyển động, đường tròn ngoại tiếp tam giác A1 A2 C luôn nằm trên một đường tròn cố
định.

Bài 67. (USA TST 2006) Cho tam giác ABC, các đường cao AD, BE, CF đồng quy tại H. Một
đường tròn tâm O, đi qua A và H cắt các cạnh AB và AC lần lượt tại Q, P . Giả sử đường tròn ngoại
CR ED
tiếp tam giác OP Q tiếp xúc với BC tại R. Chứng minh rằng = .
BR FD
Bài 68. Cho tam giác ABC. P là một điểm chuyển động trên BC. Gọi Y, Z lần lượt là các điểm trên
AC, AB sao cho Y P = Y C, ZP = ZB. Chứng minh rằng (AY Z) luôn đi qua qua một điểm cố định
khác A.

Bài 69. Cho tam giác ABC. P là một điểm chuyển động trên BC. Gọi Y, Z lần lượt là các điểm trên
AC, AB sao cho CP = CY, BP = BZ. Chứng minh rằng (AY Z) luôn đi qua một điểm cố định khác
A.

Bài 70. Cho tam giác ABC. P là một điểm chuyển động trên BC. Gọi Y, Z lần lượt là các điểm trên
AC, AB sao cho P Y k AB, P Z k AC. Chứng minh rằng (AY Z) luôn đi qua một điểm cố định khác A.

Bài 71. Cho tam giác ABC nội tiếp (O). Một đường tròn ω có tâm nằm trên đường cao ứng với đỉnh
A của tam giác ABC, cắt AB, AC lần lượt tại P, Q sao cho BP · CQ = AP · AQ. Chứng minh rằng ω
tiếp xúc với (BOC).

36
PHẦN II: HÌNH HỌC CỦA ĐƯỜNG TRÒN

4 Đường thẳng Euler, đường tròn Euler


4.1 Giới thiệu
Trong mục này xin giới thiệu tới bạn đọc hai định lý mang tên Leonhard Euler, người được xem là
một trong những nhà toán học lừng lẫy nhất mọi thời đại.
Đường thẳng Euler. Cho tam giác ABC nội tiếp đường tròn (O), với trực tâm H, trọng tâm G.
Khi đó H, G, O thẳng hàng và GH : GO = −2.

H
O
G

B M C

Chứng minh. Gọi E là điểm đối xứng với A qua O. M là trung điểm BC. Dễ thấy BHCE là hình
bình hành nên M đồng thời là trung điểm HE. Suy ra M O là đường trung bình của tam giác AHE.
AG AH
Do = = 2 nên theo định lý Thales, G là điểm chia đoạn HO theo tỉ số −2.
GM OM
Đường tròn Euler (đường tròn 9 điểm). Cho tam giác ABC với trực tâm H, tâm đường tròn
ngoại tiếp O. Gọi A1 , B1 , C1 lần lượt là trung điểm BC, CA, AB; A2 , B2 , C2 lần lượt là chân đường
cao hạ từ A, B, C xuống BC, CA, AB; A3 , B3 , C3 lần lượt là trung điểm AH, BH, CH. Khi đó 9 điểm
A1 , B1 , C1 , A2 , B2 , C2 , A3 , B3 , C3 cùng nằm trên đường tròn có tâm là trung điểm OH.

A3 B2
B1
C1

C2
H O
E

B3 C3

B A2 A1 C

37
Chứng minh. Theo phép chứng minh đường thẳng Euler, AH k= 2OA1 nên AA3 k= OA1 , từ đó
AA3 A1 O là hình bình hành. Suy ra A1 A3 đi qua trung điểm E của HO và A1 A3 = AO = R. Do
R
∠A3 A2 A1 = 90◦ nên A1 , A2 , A3 cùng nằm trên đường tròn (E, ). Chứng minh tương tự suy ra 9
2
R
điểm cùng nằm trên (E, ).
2

4.2 Các định lý, bài toán liên quan


Bài 72. Cho tam giác ABC nội tiếp đường tròn (O), ngoại tiếp đường tròn (I). Gọi D, E, F lần lượt
là tiếp điểm của (I) với BC, CA, AB. Khi đó OI là đường thẳng Euler của tam giác DEF .

Ib

Ic

F I
O

B D C

Ia

Chứng minh. Gọi Ia , Ib , Ic là tâm đường tròn bàng tiếp góc A, B, C của tam giác ABC. Dễ thấy I là
trực tâm tam giác Ia Ib Ic , O là tâm đường tròn Euler của tam giác Ia Ib Ic nên OI là đường thẳng Euler
của tam giác Ia Ib Ic . Do EF ⊥ AI, Ib Ic ⊥ AI nên Ib Ic k EF . Chứng minh tương tự suy ra hai tam giác
Ia Ib Ic và DEF có cạnh tương ứng song song, do đó đường thẳng Euler của hai tam giác song song với
nhau. Mặt khác, I là tâm đường tròn ngoại tiếp tam giác DEF nên I nằm trên đường thẳng Euler
của hai tam giác DEF và Ia Ib Ic . Do đó hai tam giác DEF và Ia Ib Ic có chung đường thẳng Euler. Ta
có đpcm.

Bài 73. Cho tam giác ABC, P là điểm bất kì nằm trên mặt phẳng. Kí hiệu Oa , Ob , Oc lần lượt
là tâm ngoại tiếp các tam giác P BC, P CA, P AB. Khi đó đường thẳng Euler của các tam giác
P BC, P AC, P AB đồng quy khi và chỉ khi AOa , BOb , COc đồng quy.

38
A Ob

Oc
A'' Gc Gb
P

A'
B C

Oa

Chứng minh. Gọi Ga , Gb , Gc lần lượt là trọng tâm các tam giác BP C, CP A, AP B.
Gọi A0 , B 0 , C 0 là giao điểm của Ob Oc và BC, Oa Oc và AC, Oa Ob và AB; A00 , B 00 , C 00 là giao điểm
của Gb Gc và Ob Oc , Gc Ga và Oc Oa , Ga Gb và Oa Ob .
Theo định lý Desargues, Oa Ga , Ob Gb , Oc Gc đồng quy khi và chỉ khi (Gb Gc ∩ Ob Oc ), (Gc Ga ∩
Oc Oa ), (Ga Gb ∩ Oa Ob ) hay A00 , B 00 , C 00 thẳng hàng.
A00 Gb B 00 Gc C 00 Ga
Khi và chỉ khi 00 · · = 1(1).
A Gc B 00 Ga C 00 Gb
A00 Gb A0 B A0 B B 0 C C 0 A
Do Gb Gc k BC nên 00 = 0 , tương tự suy ra (1) tương đương 0 · 0 · 0 = 1 hay
A Gc AC AC BA CB
A0 , B 0 , C 0 thẳng hàng.
Lại theo định lý Desargues, điều này tương đương AOa , BOb , COc đồng quy.

Bài 74. Trên mặt phẳng cho 4 điểm A, B, C, D. Khi đó nếu đường thẳng Euler của 3 trong 4 tam giác
ABC, BCD, CDA, DAB đồng quy thì cả 4 đường thẳng Euler của 4 tam giác trên đồng quy.

Chứng minh. Trước tiên ta chứng minh một bổ đề.


Bổ đề 1. (Định lý Sondat). Cho hai tam giác trực giao ABC và A1 B1 C1 có tâm trực giao là P, Q;
đồng thời thấu xạ theo tâm O. Khi đó O, P, Q thẳng hàng.
Chứng minh. (Chỉnh sửa chứng minh của Jean-Louis Ayme)
Ta cần 3 bổ đề mới.
Bổ đề 2. (Định lý Dergiades). Cho tam giác ABC. 3 đường tròn ωa , ωb , ωc lần lượt đi qua các cặp
đỉnh B, C; C, A; A, B. Gọi D, E, F là giao điểm thứ hai của 3 đường tròn này. Đường thẳng Qua D
vuông góc với AD cắt BC tại X. Tương tự xác định Y, Z. Khi đó X, Y, Z thẳng hàng.
Chứng minh.

39
A

E
F J

D C
X B

Đặt ∠BEC = ∠BF C = α, ∠ADC = ∠AF C = β, ∠AEB = ∠ADB = γ, bán kính của ωa , ωb , ωc
lần lượt tại Ra , Rb , Rc .
XB BD · sin ∠XDB BD · (− cos ∠ADB) BD · cos γ
Ta có = = = .
XC CD · sin ∠XDC CD · (− cos ∠ADC) CD · cos β
XB Y C ZA BD CE AF
Chứng minh tương tự suy ra · · = · · .
XC Y A ZB CD AE BF
BD 2Rc sin ∠BAD
Ta lại có = . Tương tự và áp dụng định lý Céva sin cho tam giác ABC với các
CD 2Rb sin ∠CAD
BD CE AF
đường AD, BE, CF đồng quy tại tâm đẳng phương của ωa , ωb , ωc ta thu được · · = 1.
CD AE BF
Vậy X, Y, Z thẳng hàng.
Bổ đề 3. Cho hai tam giác ABC và XY Z thỏa mãn các đường vuông góc kẻ từ A, B, C tới
Y Z, ZX, XY và các đường vuông góc kẻ từ X, Y, Z tới BC, CA, AB cùng đồng quy tại O. Khi đó
hai tam giác ABC và XY Z thấu xạ.
Chứng minh.

40
F

X
A

E
Y'
Z'

O
P

t Z C
B D

Y
K

Gọi X 0 , Y 0 , Z 0 lần lượt là hình chiếu của X, Y, Z trên BC, CA, AB. D, E, F lần lượt là giao của BC
và Y Z, AC và XZ, AB và XY . Gọi H, K lần lượt là giao của AB và OY 0 , AC và OZ 0 .
Do O là trực tâm của tam giác AHK nên AO ⊥ HK. Mà AO ⊥ Y Z nên Y Z k HK. Lại có
HZ 0 Y 0 K là tứ giác nội tiếp nên áp dụng định lý Reim suy ra Y, Z, Y 0 , Z 0 cùng nằm trên đường tròn
ωx . Tương tự có ωy , ωz .
Áp dụng định lý Dergiades cho tam giác XY Z và 3 đường tròn ωx , ωy , ωz suy ra D, E, F thẳng
hàng. Theo định lý Desargues ta có hai tam giác ABC và XY Z thấu xạ.
Trở lại định lý Sondat.

A A1
A2

Q P

P' C1
C'2
C2

B
C
B1 B'2
B2

Gọi A2 là điểm nằm trên AA1 sao cho P A2 ⊥ BC, B2 , C2 là hai điểm trên BB1 , CC1 sao cho
A2 B2 k A1 B1 , A2 C2 k A1 C1 .
Do A1 A2 , B1 B2 , C1 C2 đồng quy tại O nên hai tam giác A1 B1 C1 và A2 B2 C2 vị tự theo tâm O. Suy
ra B1 C1 k B2 C2 . P là tâm trực giao của tam giác A2 B2 C2 ứng với tam giác ABC.
Gọi D, E, F là giao điểm của B2 C2 với BC, A2 C2 với AC, A2 B2 với AB. Do hai tam giác A2 B2 C2
và ABC thấu xạ nên theo định lý Desargues, D, E, F thẳng hàng.

41
Qua P kẻ đường thẳng vuông góc với AC, AB cắt A2 B2 , A2 C2 lần lượt tại B20 , C20 . Do các đường
vuông góc kẻ từ B2 tới AC, C2 tới AB cắt nhau tại một điểm P 0 trên A2 P nên hai tam giác P B20 C20 và
P 0 B2 C2 vị tự theo tâm A2 . Suy ra B20 C20 k B2 C2 và do đó P là tâm trực giao của hai tam giác A2 B20 C20
và ABC. Theo bổ đề trên suy ra A2 B20 C20 và ABC thấu xạ. Theo định lý Desargues, giao điểm D0 của
B20 C20 với BC nằm trên EF . Mà D và D0 cùng nằm trên BC nên D ≡ D0 hay hai tam giác ABC và
A2 B2 C2 có chung tâm trực giao P.
Ta có hai tam giác A2 P B2 và A1 QB1 có cạnh tương ứng song song nên vị tự theo tâm O. Suy ra
O, P, Q thẳng hàng.
Trở lại bài toán.

A Ob

Oc

Od
P D

B
C

Oa

Theo bài toán trên ta đưa bài toán về dạng sau: nếu 3 trong số 4 đường nối điểm này với tâm
ngoại tiếp tam giác còn lại đồng quy thì cả 4 đường đồng quy. Gọi Oa , Ob , Oc , Od lần lượt là tâm đường
tròn ngoại tiếp các tam giác BCD, CDA, DAB, ABC và AOa , BOb , COc đồng quy tại P . Ta có D là
tâm trực giao của tam giác Oa Ob Oc ứng với A, B, C; Od là tâm trực giao của tam giác ABC ứng với
Oa , Ob , Oc nên theo định lý Sondat, P, D, Od thẳng hàng. Từ đó suy ra đpcm.

Bài 75. (Kostas Vittas) Cho tứ giác ABCD nội tiếp đường tròn (O). AC giao BD tại P . Khi đó
đường thẳng Euler của các tam giác AP B, BP C, CP D, DP A đồng quy.

42
B
O1

A H3

Q H2
H4
O2
Y O4 P
H1 O
M N

D Z

C
O3

Chứng minh. Gọi H1 , H2 , H3 , H4 lần lượt là trực tâm, O1 , O2 , O3 , O4 lần lượt là tâm đường tròn ngoại
tiếp các tam giác AP B, BP C, CP D, DP A.
Do tứ giác ABCD nội tiếp nên 4AP B ∼ 4DP C, suy ra P O1 và P O3 đẳng giác trong ∠AP B.
Suy ra P O3 đi qua H1 . Tương tự ta thu được O1 H3 , O3 H1 , O2 H4 , O4 H2 đồng quy tại P .
Ta có O1 O2 ⊥ DB, O3 O4 ⊥ DB nên O1 O2 k O3 O4 . Tương tự suy ra O1 O2 O3 O4 là hình bình hành.
Mặt khác, H1 H4 cùng nằm trên đường vuông góc kẻ từ A tới BD nên H1 H4 ⊥ BD, tương tự
H2 H3 ⊥ BD. Suy ra H1 H4 k H2 H3 . Chứng minh tương tự ta thu được H1 H2 H3 H4 là hình bình hành.
Xét hai tam giác H1 H2 H4 và O3 O4 O2 thấu xạ tâm P nên theo định lý Desargues, H1 H2 ∩ O3 O4 =
{X}, H2 H4 ∩ O4 O2 = {Y }, H1 H4 ∩ O3 O2 = {Z} thẳng hàng.
Gọi H3 H4 ∩ O3 O4 = {M }, H3 H2 ∩ O3 O2 = {N }, M N ∩ XZ = {Y1 }, H2 H4 ∩ M N = {Y2 }.
Y1 M ZO3 XM
Áp dụng định lý Menelaus cho tam giác O3 M N và 3 điểm Y1 , X, Z ta có = · .
Y1 N ZN XO3
Lại áp dụng định lý Menelaus cho tam giác H3 M N và 3 điểm Y2 , H4 , H2 ta có
Y2 M H2 H3 H4 M M X O3 Z Y1 M
= · = · = .
Y2 N H2 N H4 H3 XO3 ZN Y1 N
Suy ra Y1 ≡ Y2 ≡ Y . Áp dụng định lý Desargues ta thu được hai tam giác H3 H2 H4 và O3 O2 O4
thấu xạ hay O2 H2 , O3 H3 , O4 H4 đồng quy. Chứng minh tương tự suy ra đpcm.

Bài 76. (Định lý Gossard). Cho tam giác ABC. Đường thẳng Euler của tam giác ABC cắt các cạnh
BC, CA, AB lần lượt tại A0 , B 0 , C 0 . Khi đó tam giác tạo bởi đường thẳng Euler của các tam giác
AB 0 C 0 , BA0 C 0 , CA0 B 0 là ảnh của tam giác ABC qua phép vị tự có tâm nằm trên đường thẳng Euler
của tam giác ABC, tỉ số −1.

Chứng minh.

Bài 77.

Chứng minh.

43
Bài 78.

Chứng minh.

Bài 79.

Chứng minh.

Bài 80.

Chứng minh.

Bài 81.

Chứng minh.

Bài 82.

Chứng minh.

Bài 83.

Chứng minh.

Bài 84.

Chứng minh.

Bài 85.

Chứng minh.

Bài 86.

Chứng minh.

Bài 87.

Chứng minh.

Bài 88.

Chứng minh.

Bài 89.

Chứng minh.

Bài 90.

Chứng minh.

Bài 91.

Chứng minh.

44
4.3 Điểm Euler-Poncelet
4.4 Bài tập áp dụng
Bài 92. Cho tam giác ABC nội tiếp đường tròn (O), các đường cao AHa , BHb , CHc . Gọi ωa là đường
tròn đi qua A và Ha với OA. Tương tự xác định ωb , ωc . Chứng minh rằng 3 đường tròn ωa , ωb , ωc cắt
nhau tại hai điểm.
Bài 93. Cho tam giác ABC, các đường cao AD, BE, CF . EF cắt BC tại X, DF cắt AC tại Y .
Chứng minh rằng XY vuông góc với đường thẳng Euler của tam giác ABC.
Bài 94. Cho tam giác ABC, đường cao BHb , CHc . Gọi Mb , Mc lần lượt là trung điểm AC, AB. Hb Mc
giao Hc Mb tại T . Chứng minh rằng T nằm trên đường thằng Euler của tam giác ABC.
Bài 95. Gọi I là tâm nội tiếp tam giác ABC. Chứng minh rằng đường thẳng Euler của các tam giác
AIB, BIC, CIA đồng quy.
Bài 96. Gọi H là trực tâm của tam giác ABC. Chứng minh rằng đường thẳng Euler của các tam giác
AHB, BHC, CHA đồng quy.
Bài 97. Cho tam giác đều ABC. P là điểm bất kì trên mặt phẳng, chứng minh rằng đường thẳng
Euler của các tam giác AP B, BP C, CP A đồng quy.
Bài 98. Cho tam giác nhọn ABC. P là điểm nằm trong tam giác sao cho ∠AP B = ∠BP C = ∠CP A =
120◦ . Chứng minh rằng đường thẳng Euler của các tam giác AP B, BP C, CP A đồng quy.
Bài 99. Cho tam giác ABC với trực tâm H, tâm ngoại tiếp O. Gọi H 0 là điểm đối xứng với H
qua O. Gọi Ga , Gb , Gc lần lượt là trọng tâm các tam giác BH 0 C, CH 0 A, AH 0 B. Chứng minh rằng
AGa , BGb , CGc đồng quy.
Bài 100. Cho tam giác ABC với trực tâm H, các đường cao AD, BE, CF . Gọi M là giao của DE và
F C, N là giao của DF và BE. O là tâm ngoại tiếp tam giác BHC. Chứng minh rằng M N ⊥ AO.
Bài 101. Cho tam giác ABC, trực tâm H. Đường tròn đường kính AB giao (AHC) tại X, đường
tròn đường kính AC giao (AHB) tại Y . Chứng minh rằng tâm đường tròn ngoại tiếp tam giác AXY
nằm trên đường thẳng Euler của tam giác BHC.

5 Đường tròn Apollonius


5.1 Giới thiệu
Apollonius là một nhà hình học người Hy Lạp lỗi lạc, người khai sinh ra các tên gọi ellipse,
hyperbola, parabola mà chúng ta vẫn sử dụng ngày nay. Tên tuổi của ông gắn liền với một số bài toán
nổi tiếng, đặc biệt là bài toán về đường tròn Apollonius được phát biểu như sau:
PA
Cho hai điểm A, B. Tập hợp các điểm P sao cho tỉ số = k không đổi (k > 0) là một đường
PB
tròn, được gọi là đường tròn Apollonius của đoạn thẳng AB ứng với tỉ số k.
P

D A C B

45
CA DA
Chứng minh. Gọi C, D là hai điểm nằm trong và ngoài đoạn thẳng AB sao cho = = k.
CB DB
PA CA DA
Khi đó = = nên C, D lần lượt là chân đường phân giác trong và ngoài của góc AP B.
PB CB DB

Suy ra ∠CP D = 90 . Vậy P nằm trên đường tròn đường kính CD.
Ngược lại giả sử P là điểm bất kì nằm trên đường tròn đường kính CD. Khi đó ∠CP D = 90◦ . Mà
(ABCD) = −1 nên theo tính chất hàng phân giác, suy ra C, D lần lượt là chân đường phân giác trong
PA
và ngoài của góc AP B. Từ đó = k.
PB
Như vậy tập hợp các điểm P là đường tròn đường kính CD.

Chú ý rằng khi k = 1, đường tròn Apollonius suy biến thành đường trung trực của đoạn thẳng
AB.
Từ định nghĩa đường tròn Apollonius của đoạn thẳng chúng ta xây dựng đường tròn Apollonius
của một tam giác như sau.
Đường tròn Apollonius của tam giác ABC ứng với đỉnh A là đường tròn có đường kính là đoạn
thẳng nối hai chân đường phân giác trong và ngoài góc BAC.
Như vậy trong một tam giác, có ba đường tròn Apollonius ứng với ba đỉnh của tam giác. Rõ ràng
các đỉnh của tam giác đều nằm trên đường tròn Apollonius tương ứng. Sau đây chúng ta tìm hiểu một
số tính chất của các đường tròn này.

5.2 Tính chất


Tính chất 14. Đường tròn Apollonius trực giao với đường tròn ngoại tiếp.

E J B D C

Chứng minh. Gọi D, E lần lượt là chân đường phân giác trong và ngoài góc A của tam giác ABC. J
là trung điểm DE. (O) là đường tròn ngoại tiếp tam giác ABC.
Do (BCDE) = −1 nên theo hệ thức Newton JA2 = JD2 = JB.JC, hay JA là tiếp tuyến của (O).
Điều đó có nghĩa là (J) và (O) trực giao.

Nhận xét. Từ tính chất trên ta thấy tâm của đường tròn Apollonius là giao của tiếp tuyến của
đường tròn ngoại tiếp kẻ từ một đỉnh tới cạnh đối diện.

Tính chất 15. Ba đường tròn Apollonius của một tam giác thì đồng trục với trục đẳng phương là
đường thẳng nối tâm ngoại tiếp với điểm Lemoine.

46
A

O
L
Oa
B C

Chứng minh. Gọi (Oa ), (Ob ), (Oc ) lần lượt là các đường tròn Apollonius ứng với các đỉnh A, B, C; X
là giao điểm thứ hai của (Oa ) với (O).
Do hai đường tròn (Oa ) và (O) trực giao nên Oa A, Oa X là hai tiếp tuyến của (O). Suy ra tứ giác
ABXC điều hoà hay AX là đường đối trung ứng với đỉnh A. Như vậy AX đi qua điểm Lemoine L
của tam giác ABC.
Nghĩa là L nằm trên trục đẳng phương của (Oa ) và (O), tương tự L nằm trên trục đẳng phương
của (Ob ) và (O), (Oc ) và (O) hay L có cùng phương tích tới ba đường tròn (Oa ), (Ob ), (Oc ).
Mặt khác, phương tích từ O đến ba đường tròn Apollonius đều bằng nhau và bằng R2 . Vậy OL là
trục đẳng phương của (Oa ), (Ob ), (Oc ). Ta có đpcm.

Tính chất 16. Ba đường tròn Apollonius giao nhau tại hai điểm isodynamic, hai điểm này là hai
điểm nghịch đảo ứng với đường tròn (O).

Chứng minh. Theo tính chất 2 thì ba đường tròn Apollonius đồng trục. Đồng thời phương tích từ L
đến ba đường tròn là âm, phương tích từ O đến ba đường tròn là dương nên ba đường tròn trên phải
giao nhau tại hai điểm P1 và P2 . Ta có OP1 .OP2 = R2 nên P1 và P2 là hai điểm nghịch đảo ứng với
(O).

Ta có thể mở rộng tính chất của ba đường tròn Apollonius của một tam giác đồng trục như sau.

Tính chất 17. Cho tam giác ABC. Gọi ωa , ωb , ωc lần lượt là đường tròn Apollonius của các đoạn
BC, CA, AB theo tỉ số x, y, z sao cho xyz = 1. Khi đó ωa , ωb , ωc đồng trục.
PC PA PC PB
Chứng minh. Gọi P là giao của ωb và ωc , khi đó = y, = z nên = yz hay = x, điều
PA PB PB PC
đó nghĩa là P nằm trên ωa . Tương tự với giao điểm thứ hai của ωb và ωc . Ta có đpcm.

Tính chất 18. Cho tam giác ABC. P là điểm bất kì trong mặt phẳng. Gọi XY Z là tam giác pedal của
P ứng với tam giác ABC (X ∈ BC, Y ∈ CA, Z ∈ AB). Khi đó P nằm trên đường tròn A-Apollonius
khi và chỉ khi tam giác XY Z cân tại X.

47
A
Y
P
Z

B X C

Chứng minh. Ta có XY = XZ khi và chỉ khi P C. sin ∠ACB = P B. sin ∠ABC.


PC sin ∠ABC AC
Hay = = .
PB sin ∠ACB BC
Điều kiện này tương đương P nằm trên đường tròn A-Apollonius.

Tính chất 19. (Tournament of the Towns 1995). Trên mặt phẳng chỉ tồn tại hai điểm sao cho tam
giác pedal của hai điểm đó ứng với tam giác ABC là tam giác đều.

Y2

Y1

Z1 P1
P2
Oa
B
X2
X1 C
Z2

Chứng minh. Gọi P là điểm bất kì trong mặt phẳng, XY Z là tam giác pedal của P ứng với tam giác
ABC.
Theo tính chất 5 thì tam giác XY Z cân tại X khi và chỉ khi P nằm trên A-Apollonius. Tương tự
tam giác XY Z cân tại Y khi và chỉ khi P nằm trên B-Apollonius. Như vậy tam giác XY Z đều khi và
chỉ khi P là giao điểm của ba đường tròn Apollonius.

Tính chất 20. Điểm isodynamic là điểm liên hợp đẳng giác của điểm Fermat.

48
A

Y
Z P
F

B X C

Chứng minh. Gọi P là điểm isodynamic thứ nhất, F là điểm liên hợp đẳng giác của P , XY Z là tam
giác pedal của P . Do XY Z là tam giác đều nên ∠ZBP + ∠Y CP = ∠ZXP + ∠Y XP = 60◦ .
Suy ra ∠F BC + ∠F CB = 60◦ hay ∠BF C = 120◦ .
Tương tự suy ra điểm F nhìn ba cạnh của tam giác ABC dưới góc 120◦ hay F là điểm Fermat thứ
nhất.

Tính chất 21. Cho tam giác ABC với P là điểm isodynamic. Khi đó một đỉnh bất kì của tam giác
ABC là điểm isodynamic của tam giác tạo bởi hai đỉnh còn lại và điểm P .

Chứng minh. Theo định nghĩa và do P là giao của ba đường tròn Apollonius, ta có P A.BC =
P B.CA = P C.AB.
Điều này nghĩa là vai trò của các điểm P, A, B, C là như nhau trong các đẳng thức trên. Như vậy ta
có thể đảo lại vị trí của các đỉnh hay A là điểm isodynamic của tam giác P BC, tương tự với B, C.

Nhận xét. Ta gọi bộ bốn điểm A, B, C, P là bộ điểm isodynamic.

Tính chất 22. Phép nghịch đảo tâm là điểm isodynamic, phương tích bất kì biến tam giác ABC
thành một tam giác đều, đồng thời biến điểm isodynamic còn lại thành tâm của tam giác đều đó.

A'

P'1 P

C'

P1
B C

B'

Chứng minh. Xét phép nghịch đảo IPk : A 7→ A0 , B 7→ B 0 , C 7→ C 0 .


k.AB k.AC AB AC
Suy ra A0 B 0 = , A0 C 0 = . Mà = nên A0 B 0 = A0 C 0 , chứng minh tương tự
P A.P B P A.P C PB PC
suy ra tam giác A0 B 0 C 0 đều.
Mặt khác, gọi P1 là điểm isodynamic thứ hai, P10 là ảnh của P1 qua IPk .
k.P1 A k.P1 B
Ta có P10 A0 = , P 0 B0 = .
P P1 .P A 1 P P1 .P B

49
P1 A P1 B P1 A PA
Do đó P10 A0 = P10 B 0 khi và chỉ khi = hay = , luôn đúng.
PA PB P1 B PB
Chứng minh tương tự suy ra P10 là tâm của tam giác A0 B 0 C 0 .

Tính chất 23. Cho tam giác đều ABC và điểm P bất kì trong mặt phẳng. Phép nghịch đảo tâm P
phương tích k bất kì biến A, B, C lần lượt thành A0 , B 0 , C 0 . Khi đó P, A0 , B 0 , C 0 lập thành một bộ điểm
isodynamic.
k.AB k.AC
Chứng minh. Ta có tam giác ABC đều và A0 B 0 = , A0 C 0 = nên A0 B 0 .P C 0 = A0 C 0 .P B 0
P A.P B P A.P C
P C0 P B0
khi và chỉ khi = . Điều này tương đương P C 0 .P C = P B 0 .P B, luôn đúng do bằng k.
PB PC
Chứng minh tương tự suy ra A0 B 0 .P C 0 = A0 C 0 .P B 0 = B 0 C 0 .P A0 . Ta có đpcm.

Tính chất 24. Cho bộ điểm isodynamic A, B, C, D. Gọi A1 là điểm isodynamic thứ hai của tam giác
BCD, tương tự xác định B1 , C1 , D1 . Khi đó A1 , B1 , C1 , D1 cũng lập thành một bộ điểm isodynamic.

C'1 A' B'1

D'1

B' C'

A'1

Chứng minh. Xét phép nghịch đảo tâm D phương tích k biến điểm P bất kì thành điểm P 0 .
Theo tính chất 8 ta thu được tam giác A0 B 0 C 0 đều. Do C1 là điểm isodynamic thứ hai của tam
giác ABD nên A, B, C1 , D là một bộ điểm isodynamic. Suy ra A0 B 0 C10 là tam giác đều. Do C 0 và C10
phân biệt nên C 0 và C10 đối xứng nhau qua A0 B 0 .
Tương tự A01 và A0 đối xứng nhau qua B 0 C 0 , B10 và B 0 đối xứng nhau quaA0 C 0 . Suy ra tam giác
A01 B10 C10 là tam giác đều.
Hơn nữa cũng theo tính chất 8, D10 là tâm của tam giác A0 B 0 C 0 nên D10 cũng là tâm của tam
giác A01 B10 C10 . Theo tính chất 9 suy ra D1 là điểm isodynamic thứ hai của tam giác A1 B1 C1 . Vậy
A1 , B1 , C1 , D1 lập thành một bộ điểm isodynamic.

5.3 Ứng dụng


Bài 102. Cho tam giác ABC, dựng tam giác đều XY Z nội tiếp tam giác ABC sao cho diện tích của
tam giác XY Z nhỏ nhất.

Y
Y1
Z1
P
Z

B X1 X C

50
Chứng minh. Gọi P là điểm Miquel của tam giác ABC ứng với bộ điểm X, Y, Z. X1 Y1 Z1 là tam giác
pedal của P ứng với tam giác ABC.
Ta có ∠ZP X = 180◦ − ∠ABC = ∠Z1 P X1 , tương tự suy ra ∠X1 P X = ∠Y1 P Y = ∠Z1 P Z = α.
Do đó các tam giác X1 P X, Y1 P Y, Z1 P Z đồng dạng.
P X1
Phép vị tự quay tâm P tỉ số < 1, góc quay α lần lượt biến X thành X1 , Y thành Y1 , Z thành
PX
Z1 .
Vậy tam giác X1 Y1 Z1 là tam giác đều nội tiếp tam giác ABC có diện tích nhỏ nhất, đó là tam giác
pedal của điểm isodynamic P .

Bài 103. Cho tam giác ABC và một điểm P bất kì trong mặt phẳng. Khi đó các đường tròn Apollonius
ứng với đỉnh P của các tam giác BP C, CP A, AP B đồng trục.

Chứng minh. Gọi Q là giao điểm thứ hai của đường tròn P -Apollonius của các tam giác BP C và
CP A.
QB P B QC PC QB QC PB PC QB PB
Ta có = , = . Từ đó · = · hay = . Có nghĩa là Q nằm
QC P C QA PA QC QA PC PA QA PA
trên đường tròn P -Apollonius của tam giác AP B.

Bài 104. Cho tứ giác toàn phần ABCDEF (AB ∩ CD = {E}, AD ∩ BC = {F }). Chứng minh rằng
các đường tròn đường kính AC, BD, EF đồng trục.

Y
B
A

F
D C

Chứng minh. Gọi X là giao của AC và BD, Y là giao của AC và EF , Z là giao của BD và EF .
Theo tính chất của tứ giác toàn phần ta có (EF Y Z) = (ACXY ) = (BDXZ) = −1. Do đó
(AC), (BD), (EF ) lần lượt là đường tròn Apollonius của các đoạn thẳng XY, ZX, Y Z theo tỉ số
AX DZ EY
, , .
AY DX EZ
AX DZ EY
Theo định lý Menelaus, · · = 1 nên theo tính chất 4 ta có đpcm.
AY DX EZ
Bài 105. Cho tam giác ABC có I, J lần lượt là tâm đường tròn nội tiếp và tâm đường tròn bàng tiếp
IB JB
góc A. Gọi ω1 , ω2 lần lượt là đường tròn Apollonius của đoạn thẳng BC ứng với các tỉ số , .
IC JC
Chứng minh rằng IJ là tiếp tuyến chung của ω1 và ω2 .

51
A

I
E C F
N B M

IB
Chứng minh. Gọi M, N lần lượt là điểm chia trong và chia ngoài BC theo tỉ số , E là trung điểm
IC
M N.
Do (BCM N ) = −1 nên theo hệ thức Newton, EI 2 = EM 2 = EB.EC hay EI tiếp xúc với đường
tròn (BIC). Do tâm của (BIC) là trung điểm cung BC nên ∠EIJ = 90◦ . Vậy IJ tiếp xúc với ω1 ,
tương tự IJ tiếp xúc với ω2 .

Bài 106. Cho tam giác ABC có F là điểm Fermat. Chứng minh các đường thẳng đối xứng với F A
qua BC, F B qua AC, F C qua AB đồng quy.

P
F

B C

Pa

Chứng minh. Gọi P là điểm liên hợp đẳng giác của F trong tam giác ABC. Pa đối xứng với P qua
BC.
Ta có ∠Pa BC = ∠P BC = ∠ABF, ∠Pa CB = ∠P CB = ∠ACF .
Do đó Pa và A liên hợp đẳng giác trong tam giác BF C. Mà A nằm trên đường phân giác của
∠BF C nên A, F, Pa thẳng hàng.

52
Như vậy điểm đối xứng của P qua BC nằm trên AF , điều đó nghĩa là điểm đường thẳng đối xứng
với AF qua BC đi qua P . Chứng minh tương tự suy ra các đường đối xứng này đồng quy tại P .

Bài 107. Cho tam giác ABC. P là điểm bất kì nằm trên đường tròn Apollonius ứng với đỉnh A. Gọi
I1 , I2 lần lượt là tâm đường tròn nội tiếp các tam giác AP B, AP C, Q là chân đường phân giác ngoài
đỉnh A. Chứng minh rằng Q, I1 , I2 thẳng hàng.

T
I2

I1 P

Q B C

AT BA CA
Chứng minh. Gọi T là giao của BI1 với AP . Ta có = = , suy ra T, I2 , C thẳng hàng.
PT BP CP
Q0 B I2 C I1 T
Gọi Q0 là giao của I1 I2 với BC. Theo định lý Menelaus ta có 0 · · = 1.
Q C I2 T I1 B
Q0 B AC AT Q0 B AB
Hay 0 · · = 1, điều này nghĩa là 0 = .
Q C AT AB QC AC
Vậy Q0 ≡ Q. Ta có đpcm.

Bài 108. (All Russian MO 2011) Cho tam giác ABC, M, N lần lượt là trung điểm BC và điểm chính
giữa cung BAC. I1 , I2 lần lượt là tâm bàng tiếp góc A của các tam giác M AB, M AC. Chứng minh
rằng A, I1 , I2 , N cùng thuộc một đường tròn.

53
N'

N
A

I'2
C'
I'1

M'
B'

M C
B

I2
I1

Chứng minh. Xét phép nghịch đảo tâm A phương tích k biến điểm P bất kì thành điểm P 0 .
Ta có IA k : BC 7→ (AB 0 C 0 ), (ABC) 7→ B 0 C 0 . Do M ∈ BC nên M 0 ∈ (AB 0 C 0 ).

Hai đường thẳng B 0 C 0 và BC đối song trong ∠BAC và AM là đường trung tuyến của tam giác
ABC nên AM 0 là đường đối trung trong tam giác AB 0 C 0 .
Gọi I10 , I20 lần lượt là tâm đường tròn nội tiếp các tam giác AB 0 M 0 , AC 0 M 0 . Ta có ∠AI1 M =
1 1
∠ABM = ∠AM 0 B 0 = ∠I10 M 0 A. Do đó tứ giác I1 I10 M 0 M nội tiếp. Vậy I10 là ảnh của I1 qua IA k.
2 2
Tương tự với I20 .
Do N là điểm chính giữa cung BAC nên N 0 là chân phân giác ngoài góc A của tam giác AB 0 C 0 .
Như vậy A, I1 , I2 , N cùng thuộc một đường tròn khi và chỉ khi I10 , I20 , N 0 thẳng hàng. Điều này hiển
nhiên đúng theo bài toán trên do M 0 nằm trên đường tròn A-Apollonius của tam giác AB 0 C 0 .

Bài 109. (Romanian Master in Mathematics 2009). Cho 4 điểm A1 , A2 , A3 , A4 trên mặt phẳng sao
cho không có ba điểm nào thẳng hàng và thỏa mãn A1 A2 .A3 A4 = A1 A3 .A2 A4 = A1 A4 .A2 A3 . Kí hiệu
Oi là tâm đường tròn ngoại tiếp tam giác Aj Ak Al ({i, j, k, l} = {1, 2, 3, 4}). Chứng minh rằng 4 đường
thẳng Ai Oi đồng quy hoặc song song.

Chứng minh. Điều kiện của đề bài cho thấy 4 điểm A1 , A2 , A3 , A4 lập thành một bộ điểm isodynamic.
Gọi ωij là đường tròn Apollonius của đoạn thẳng Ai Aj và đi qua Ak , Al .
Xét tam giác A1 A2 A3 . Theo tính chất của đường tròn Apollonius ta có ω12 , ω23 , ω13 cùng đi qua
A4 và có A4 O4 là trục đẳng phương.
Chứng minh tương tự ta có:
ω12 , ω14 có trục đẳng phương là A3 O3 , ω12 , ω13 có trục đẳng phương là A4 O4 , ω13 , ω14 có trục đẳng
phương là A2 O2 . Suy ra A2 O2 , A3 O3 , A4 O4 đồng quy hoặc song song. Chứng minh tương tự ta có
đpcm.

Bài 110. (ELMO 2013). Cho tam giác ABC (AB < AC). Gọi D, P lần lượt là chân phân giác trong
và ngoài góc A. M là trung điểm BC, ω là đường tròn ngoại tiếp tam giác AP D. Q là điểm nằm trên
cung nhỏ AD sao cho M Q tiếp xúc với ω. QB giao ω lần thứ hai tại R. Đường thẳng qua R vuông
góc với BC giao P Q tại S. Chứng minh rằng SD tiếp xúc với đường tròn ngoại tiếp tam giác QDM .

54
A

P B D M C

Chứng minh. SD tiếp xúc với (QDM ) khi và chỉ khi ∠SDP = ∠DQM = ∠SP D, khi và chỉ khi R là
điểm chính giữa cung P D.
Do (P DBC) = −1 nên theo hệ thức Newton, M Q2 = M D.M P = M B 2 = M C 2 . Do đó tam giác
BQC vuông tại Q.
Mặt khác ∠P QD = 90◦ nên theo tính chất hàng phân giác ta có QD là phân giác ∠BQC. Suy ra
∠RQD = 45◦ . Từ đó R là điểm chính giữa cung P D.

5.4 Bài tập áp dụng


Bài 111. Cho tam giác ABC nội tiếp đường tròn (O), M là trung điểm BC. AM giao (O) lần thứ
hai tại X. Dựng hình bình hành BXCP . Tiếp tuyến của (O) tại A giao BC tại K. Chứng minh rằng
KA = KP .
Bài 112. Cho tam giác ABC nội tiếp đường tròn (O). P là điểm bất kì trong mặt phẳng. AP, BP, CP
giao (O) lần lượt tại X, Y, Z. Chứng minh rằng chỉ tồn tại hai điểm trong mặt phẳng sao cho tam giác
XY Z đều.
Bài 113. Cho tam giác ABC và hai điểm M, N thỏa mãn AM : BM : CM = AN : BN : CN . Chứng
minh rằng M N đi qua tâm ngoại tiếp tam giác ABC.
Bài 114. (ELMO Shortlist 2014). Cho đa giác nội tiếp 2013 đỉnh A1 A2 ...A2013 . Chứng minh rằng với
mọi điểm P không nằm trên đường tròn ngoại tiếp đa giác và không trùng với tâm đường tròn, luôn
Ai P
tồn tại điểm Q sao cho = k không đổi (i ∈ {1, 2, ..., 2013})
Ai Q
Bài 115. (St. Petersburg MO 2008). Cho tam giác nhọn ABC. K là điểm thỏa mãn ∠AKC = 2∠ABC
AB 2
 
AK
và = . Gọi A1 , C1 lần lượt là trung điểm BC, AB. Chứng minh rằng K nằm trên đường
KC BC
tròn ngoại tiếp tam giác A1 BC1 .
Bài 116. (IMO 1996). Cho P là điểm nằm trong tam giác ABC thỏa mãn ∠AP B − ∠ACB =
∠AP C − ∠ABC. Gọi D, E lần lượt là tâm nội tiếp các tam giác AP B, AP C. Chứng minh rằng
AP, BD, CE đồng quy.
Bài 117. Cho tam giác ABC với P là điểm isodynamic thứ nhất. Gọi I1 , I2 , I3 lần lượt là tâm đường
tròn nội tiếp các tam giác BP C, CP A, AP B. Chứng minh rằng AIa , BIb , CIc đồng quy.
Bài 118. (Sharygin Geometry Olympiad 2008). Cho tứ giác ABCD. Biết rằng đường đẳng giác với
BD trong góc B cắt đường đẳng giác với DB trong góc D tại trung điểm AC, chứng minh rằng đường
đẳng giác với AC trong góc A cắt đường đẳng giác với CA trong góc C tại trung điểm BD.

55
Bài 119. Cho tam giác nhọn ABC. M là trung điểm BC. Các đường cao AD, BE, CF giao nhau tại
H. Gọi S là trung điểm AH, G là giao điểm của EF và AH, N là giao của đoạn thẳng AM với đường
tròn ngoại tiếp tam giác BCH. Chứng minh rằng ∠HM A = ∠GN S.

Bài 120. Cho tam giác ABC nội tiếp đường tròn (O). AO giao BC tại E, P là giao điểm của đường
đối trung ứng với đỉnh A với (O). D là hình chiếu vuông góc của A trên BC. Chứng minh rằng tâm
ngoại tiếp tam giác P DE nằm trên OP .

6 Đường thẳng Simson, đường thẳng Steiner của tam giác


6.1 Đường thẳng Simson
Đường thẳng Simson lần đầu tiên được đặt tên bởi Poncelet, tuy nhiên một số nhà hình học cho
rằng nó không thực sự nằm trong công trình nào của Robert Simson. Vì vậy ngày nay nó thường được
biết đến với tên gọi đường thẳng Wallace-Simson. Bài toán phát biểu như sau.
Cho tam giác ABC nội tiếp đường tròn (O). P là điểm bất kì trong mặt phẳng. Khi đó hình chiếu
vuông góc của P trên các cạnh của tam giác ABC thẳng hàng khi và chỉ khi P nằm trên (O).

B1

A1
B
C
C1

Chứng minh. Gọi A1 , B1 , C1 lần lượt là hình chiếu của P trên BC, CA, AB.
Ta có A1 , B1 , C1 thẳng hàng khi và chỉ khi ∠BA1 C1 = ∠B1 A1 C.
Do các tứ giác P C1 BA1 , P A1 B1 C nội tiếp nên ∠BA1 C1 = ∠BP C1 , ∠B1 A1 C = ∠B1 P C.
Vậy ∠BA1 C1 = ∠B1 A1 C khi và chỉ khi ∠BP C1 = ∠B1 P C hay ∠BP C = ∠B1 P C1 = 180◦ −
∠BAC. Điều này tương đương P nằm trên (O).

6.2 Đường thẳng Steiner của tam giác


Đường thẳng Steiner của tam giác được phát biểu như sau.
Cho tam giác ABC nội tiếp đường tròn (O). P là điểm bất kì nằm trên (O). Khi đó các điểm đối
xứng với P qua ba cạnh của tam giác ABC cùng nằm trên một đường thẳng, đồng thời đường thẳng
đó đi qua trực tâm của tam giác ABC.

56
A B3

B2

C3
H O
A2 B1

C2
B
A1 C

C1

Chứng minh. Gọi A2 , B2 , C2 lần lượt là điểm đối xứng với P qua BC, CA, AB; H là trực tâm tam
giác ABC; B3 , C3 lần lượt là giao điểm thứ hai của BH, CH với (O).
Dễ thấy B3 , C3 lần lượt là điểm đối xứng với H qua AC, AB. Do đó HC3 C2 P, HB3 B2 P là các hình
thang cân.
Ta có ∠C3 HC2 + ∠C3 HB3 + ∠B3 HB2 = ∠HC3 P + 180◦ − ∠BAC + ∠HB3 P = ∠P AC + ∠P AB +
180 − ∠BAC = 180◦ .

Vậy C2 , H, B2 thẳng hàng. Chứng minh tương tự ta có đpcm.

Dựa theo chứng minh đường thẳng Simson, ta nhận thấy chiều đảo của đường thẳng Steiner cũng
đúng:
Nếu các điểm đối xứng của P qua BC, CA, AB thẳng hàng thì P nằm trên đường tròn ngoại tiếp
tam giác ABC.
Chúng ta đến với một định lý quan trọng khác.
(Định lý Collings). Cho tam giác ABC có trực tâm H. Gọi d là đường thẳng bất kì đi qua H. Khi
đó các đường thẳng đối xứng với d qua BC, CA, AB đồng quy tại một điểm nằm trên đường tròn ngoại
tiếp tam giác ABC.

A B2

C2 B1
H

C1 O

B C

Chứng minh. Gọi A1 , B1 , C1 lần lượt là giao của d với BC, CA, AB; A2 , B2 , C2 là giao điểm thứ hai
của AH, BH, CH với (O); B1 B2 giao C1 C2 tại P.
Chú ý rằng B2 , C2 lần lượt đối xứng với H qua AC, AB.
Ta có ∠C1 P B1 = 180◦ − ∠P C1 B1 − ∠P B1 C1 = 180◦ − (360◦ − 2∠AC1 B1 − 2∠AB1 C1 ) = 180◦ −
2∠BAC = ∠ABB2 + ∠ACC2 .
Suy ra P nằm trên (O). Chứng minh tương tự ta thu được A1 A2 , B1 B2 , C1 C2 đồng quy.

57
Nhận xét. Điểm P được gọi là điểm Anti-Steiner của đường thẳng d.

6.3 Tính chất và ứng dụng


Trong mục này chúng ta tìm hiểu một số tính chất và các bài toán sử dụng đến hai đường thẳng
Simson và Steiner. Một số ứng dụng khác sẽ được đề cập ở mục Tứ giác toàn phần.

Tính chất 25. Đường thẳng Simson của P đi qua trung điểm đoạn nối P với trực tâm H.

Chứng minh. Tính chất này hiển nhiên dựa theo đường thẳng Steiner của tam giác.

Tính chất 26. Đường thẳng qua P vuông góc với BC cắt (O) lần thứ hai tại A0 . Chứng minh rằng
AA0 song song với đường thẳng Simson của P .

A'
A

B1

A1

B C

Chứng minh. Do tứ giác P A1 B1 C nội tiếp nên ∠A0 A1 B1 = ∠ACP = ∠AA0 P . Từ đó AA0 k A1 B1 .

Tính chất 27. Cho tam giác ABC nội tiếp đường tròn (O). Khi đó góc giữa hai đường thẳng Simson
của hai điểm P và P 0 trên (O) ứng với tam giác ABC bằng một nửa số đo cung P P 0 .

Chứng minh. Gọi A00 là giao của đường thẳng qua P 0 vuông góc với BC với (O). Theo tính chất 2 thì
AA00 song song với đường thẳng Simson của P 0 .
Như vậy góc giữa hai đường thẳng Simson của P và P 0 bằng ∠A0 AA00 . Do A0 A00 P 0 P là hình thang
cân nên hai cung A0 A00 và P P 0 bằng nhau. Từ đó suy ra đpcm.

Nhận xét. Có thể nói ba bài toán trên là ba tính chất cơ bản nhất của đường thẳng Simson. Rất
nhiều ứng dụng tập trung xoay quanh ba tính chất này.

Bài 121. Cho tứ giác ABCD nội tiếp đường tròn (O). Chứng minh rằng các đường thẳng Simson của
A, B, C, D ứng với tam giác BCD, CDA, DAB, ABC đồng quy.

58
B

Ha

Hb
O

D
M
C

Chứng minh. Gọi Ha , Hb , Hc , Hd lần lượt là trực tâm các tam giác BCD, DAC, ABD, ABC. M là
trung điểm DC.
Ta có AHa k= 2OM k= BHb nên AHa Hb B là hình bình hành. Như vậy AHa , BHb cắt nhau tại
trung điểm mỗi đường.
Chứng minh tương tự suy ra AHa , BHb , CHc , DHd cắt nhau tại trung điểm mỗi đường. Theo tính
chất 1 thì các đường thẳng Simson của A, B, C, D ứng với tam giác đối diện lần lượt đi qua trung điểm
của đoạn nối điểm đó với trực tâm. Vậy bốn đường thẳng Simson phải đồng quy.

Bài 122. Cho hai tam giác ABC và A0 B 0 C 0 cùng nội tiếp đường tròn (O). P là một điểm chuyển
động trên (O). Chứng minh rằng góc tạo bởi đường thẳng Simson của P ứng với hai tam giác ABC
và A0 B 0 C 0 không phụ thuộc vào vị trí của P .

L A'
A
K

C
B'

B C'

Chứng minh. Qua P kẻ đường thẳng vuông góc với BC và B 0 C 0 , cắt (O) lần thứ hai lần lượt tại K, L.
Theo tính chất 2 thì KA, LA0 lần lượt song song với đường thẳng Simson của P ứng với hai tam giác
ABC và A0 B 0 C 0 .
Do đó ta cần tính (KA, LA0 ).
1 −→ −−→
Ta có (KA, LA0 ) ≡ (KA, KA0 ) + (KA0 , LA0 ) ≡ (OA, OA0 ) + (KP, LP )
2
1 −→ −−→0 0 0
≡ (OA, OA ) + (BC, B C ) (mod π).
2
Như vậy (KA, LA0 ) không đổi.

59
Bài 123. Cho tam giác ABC nội tiếp đường tròn (O). P và P 0 là hai điểm nằm trên (O) và đối xứng
với nhau qua O. Chứng minh rằng giao điểm của hai đường thẳng Simson của P và P 0 ứng với tam
giác ABC nằm trên một đường tròn cố định khi P và P 0 chuyển động.

A P'

E O
H

B C

Chứng minh. Gọi L là giao điểm của hai đường thẳng Simson, H là trực tâm tam giác ABC, E là
tâm đường tròn Euler; J, K lần lượt là trung điểm của HP, HP 0 .
Theo tính chất 1, J nằm trên đường thẳng Simson của P, K nằm trên đường thẳng Simson của P 0 .
Do P và P 0 đối xứng nhau qua O nên theo tính chất 3, ∠KLJ = 90◦ .
1
Xét phép vị tự HH2
: P 7→ J, P 0 7→ K, O 7→ E nên JK là đường kính của đường tròn Euler. Vậy L
nằm trên đường tròn Euler.

Bài 124. Chứng minh rằng khoảng cách từ P tới đường thẳng Simson của P ứng với tam giác ABC
P A.P B.P C
bằng , với R là bán kính của (ABC).
4R2

B'

A'
B C
H
C'

Chứng minh. Gọi A0 , B 0 , C 0 là hình chiếu của P trên BC, CA, AB; H là hình chiếu của P trên đường
thẳng Simson.
PA
Ta có ∠HA0 P = ∠ACP nên P H = P A0 . sin ACP = P A0 . .
2R
P B.P C. sin ∠BP C P B.P C
Mặt khác, P A0 .BC = P B.P C. sin ∠BP C nên P A0 = = .
BC 2R
P A.P B.P C
Vậy P H = .
4R2

60
Bài 125. (IMO 2003). Cho tứ giác nội tiếp ABCD. Gọi P, Q, R lần lượt là hình chiếu của D trên
BC, CA, AB. Chứng minh rằng P Q = QR khi và chỉ khi phân giác các góc ABC và ADC cắt nhau
trên AC.

R
A

D
Q
O

DA AB BC DC
Chứng minh. Dễ thấy 4DAB ∼ 4DQP, 4DBC ∼ 4DRQ. Do đó = , = .
DQ P Q RQ DQ
DA DC
Suy ra P Q = QR khi và chỉ khi = , điều này tương đương phân giác các góc ABC và
AB CB
ADC cắt nhau trên AC.

Bài 126. Cho tam giác ABC nội tiếp đường tròn (O). M là điểm bất kì trên (O). Gọi X, Y, Z lần
lượt là hình chiếu của M trên AO, BO, CO. Chứng minh rằng đường thẳng Simson của M ứng với
tam giác ABC đi qua tâm nội tiếp tam giác XY Z.

Hc Y
P

O Q

T I
X Z
B C Ha

Chứng minh. Gọi Ha , Hc lần lượt là hình chiếu của M trên BC, AB; P, Q lần lượt là giao của
M Hc , M Ha và đường tròn đường kính OM.
Chú ý rằng X, Y, Z, O, M đồng viên. Do ∠Y M P = ∠ABO = ∠BAO = ∠P M X nên P là trung
điểm của cung Y X. Tương tự, Q là trung điểm của cung Y Z.
Gọi T là giao của Hc X và Ha Z. Ta có:
(T X, T Z) ≡ (T X, Hc M ) + (Hc M, Ha M ) + (Ha M, T Z) ≡ (AO, AM ) + (BA, BC) + (CM, CO)
≡ (BA, BC) + (AO, CO) + (CM, AM ) ≡ (BA, BC) + (XO, ZO) + (BC, BA) ≡ (XO, ZO) (mod π)
Suy ra T ∈ (OM ). Áp dụng định lý Pascal cho 6 điểm P, M, Q, Z, X, T ta thu được Hc , I, Ha thẳng
hàng.

61
Bài 127. Chứng minh rằng tồn tại 3 điểm trên đường tròn ngoại tiếp tam giác ABC sao cho đường
thẳng Simson của 3 điểm này tiếp xúc với đường tròn Euler của tam giác ABC, đồng thời 3 điểm này
là các đỉnh của một tam giác đều.

Chứng minh. (Luis González). Kí hiệu G(A, ~v ) là phép biến hình biến tam giác ABC thành tam giác
AB 0 C 0 cùng nội tiếp đường tròn (O) sao cho đường thẳng B 0 C 0 là ảnh của BC qua phép tịnh tiến theo
vector ~v .
Ta có một số nhận xét sau.
1) đường thẳng Simson d0 của P ứng với tam giác AB 0 C 0 là ảnh của đường thẳng Simson d của P
ứng với tam giác ABC qua phép tịnh tiến ~v .
Chứng minh. Qua P kẻ đường thẳng vuông góc với BC cắt BC, B 0 C 0 , (O) lần lượt tại A1 , A2 , A3 .
Suy ra AA3 song song với đường thẳng Simson của P ứng với hai tam giác ABC và AB 0 C 0 . Lại có
A1~A2 = ~v nên d0 là ảnh của d qua phép tịnh tiến ~v .
2) Đường tròn Euler của tam giác AB 0 C 0 là ảnh của đường tròn Euler của tam giác ABC qua phép
tịnh tiến ~v .
Chứng minh. Gọi M, M 0 lần lượt là trung điểm BC, B 0 C 0 ; H, H 0 lần lượt là chân đường cao kẻ từ
−−−→ −−→
A tới BC, B 0 C 0 . Ta có M M 0 = HH 0 = ~v . Do bán kính của hai đường tròn Euler đều bằng 1/2R và
cùng nằm trên trung trực của M H và M 0 H 0 suy ra khẳng định 2 đúng.
3) Tam giác ABC có thể biến thành một tam giác đều A0 B 0 C 0 qua 2 phép G.
Thật vậy, tồn tại ~v sao cho G(A, ~v ) biến tam giác ABC thành tam giác AB 0 C 0 cân tại C 0 . Dễ dàng
suy ra G(C, ~u) : AB 0 C 0 7→ A0 B 00 C 0 sao cho tam giác A0 B 00 C 0 đều.
Như vậy qua 2 phép biến hình G, ta biến tam giác ABC thành tam giác đều A0 B 0 C 0 . Khi đó đường
tròn nội tiếp (E) (đồng thời là đường tròn Euler) tam giác A0 B 0 C 0 là ảnh của đường tròn Euler (E)
của tam giác ABC qua 2 phép vị tự ~v và ~u, đường thẳng Simson d của P ứng với tam giác ABC biến
thành đường thẳng Simson d0 của P ứng với tam giác A0 B 0 C 0 . Như vậy d tiếp xúc với (E) khi và chỉ
khi d0 tiếp xúc với (E 0 ). Dễ chứng minh d’ tiếp xúc với (E’) khi và chỉ khi P là điểm chính giữa của
một trong các cung B’C’,C’A’,A’B’. Ba vị trí này lập thành một tam giác đều.
Như vậy tồn tại 3 điểm là đỉnh của một tam giác đều trên đường tròn ngoại tiếp tam giác ABC
sao cho đường thẳng Simson của 3 điểm này tiếp xúc với đường tròn Euler của tam giác ABC.

Bài 128. Cho tam giác nhọn ABC nội tiếp (O). Kí hiệu lA , lB , lC lần lượt là tiếp tuyến tại A, B, C
của (O). Một đường thẳng l qua trực tâm H sao cho l ⊥ OH. A1 = l ∩ lA . A2 là điểm đối xứng của
A1 qua A, Tương tự xác định được B2 , C2 . Chứng minh rằng A2 , B2 , C2 thẳng hàng.

A2
A
A1 L

H' O
C2
H

B C

C1

62
Chứng minh. Gọi L là điểm Anti-Steiner của l ứng với tam giác ABC. Kí hiệu H 0 là điểm đối xứng
với H qua AB.
Ta sẽ chứng minh C2 L là tiếp tuyến của (O).
Điều này tương đương OC2 ⊥ LC
1
⇔ ∠COC2 = ∠COL = ∠CH 0 L(1)
2
Do C1 HOC là tứ giác nội tiếp nên ∠COC2 = ∠C1 OC = ∠C1 HC = ∠H 0 HA1 = ∠LH 0 C.
Vậy (1) đúng. Tương tự suy ra A2 , B2 , C2 nằm trên tiếp tuyến kẻ từ L của (O).
Bài 129. Cho hình bình hành ABCD có góc A nhọn. Trên cạnh BC lấy điểm T sao cho tam giác
AT D nhọn. Gọi O1 , O2 , O3 lần lượt là tâm đường tròn ngoại tiếp của các tam giác ABT, ADT, CDT .
Chứng minh rằng trực tâm của tam giác O1 O2 O3 nằm trên AD.

A B

T
O2

O1
H

O3
C
D

Chứng minh. Gọi P là giao của (O1 ) và AD. Ta có ∠DP T = 180◦ − ∠ABT = ∠DCT nên P ∈ (O3 ),
suy ra P đối xứng với T qua O1 O3 .
Mặt khác A, D lần lượt là điểm đối xứng với T qua O1 O2 , O1 O3 và A, D, P thẳng hàng nên
T ∈ (O1 O2 O3 ) và đường thẳng qua A, D, P là đường thẳng Steiner của T ứng với tam giác O1 O2 O3 ,
hay trực tâm của tam giác O1 O2 O3 nằm trên AD.
Bài 130. (IMO Shortlist 2009). Cho tứ giác ngoại tiếp ABCD. Một đường thẳng qua A cắt đoạn
thẳng BC tại M và đường thẳng CD tại N . Gọi I1 , I2 , I3 lần lượt là tâm nội tiếp các tam giác
ABM, M N C, N DA. Chứng minh rằng trực tâm tam giác I1 I2 I3 nằm trên M N .
B

A
I1

I
I3 M

I2
D
N
C

63
1 1
Chứng minh. Ta có ∠I1 I2 I3 = (∠M N C + ∠CM N ) = ∠BCD.
2 2
Gọi K là giao điểm thứ hai của tiếp tuyến kẻ từ C tới (I3 ) với AM . Do tứ giác AKCD ngoại tiếp
nên AK + DC = AD + KC hay AK − KC = AD − DC = AB − CB.
Suy ra AB + KC = BC + AK hay tứ giác ABCK ngoại tiếp.
1 1
Vậy ∠I1 CI3 = ∠I1 CK + ∠I3 CK = (∠BCK + ∠DCK) = ∠BCD.
2 2
Ta thu được ∠I1 I2 I3 = ∠I1 CI3 hay I1 , I2 , I3 , C nằm trên một đường tròn.
Dễ thấy điểm đối xứng với C qua I1 I2 , I3 I2 nằm trên M N nên M N là đường thẳng Steiner của C
ứng với tam giác I1 I2 I3 . Điều đó nghĩa là trực tâm tam giác I1 I2 I3 nằm trên M N .

Bài 131. Cho tam giác ABC nội tiếp đường tròn (O), trực tâm H. P là điểm bất kì trên (O) và một
đường thẳng l bất kì qua P . Gọi A0 B 0 C 0 là tam giác tạo bởi giao điểm của các đường thẳng đối xứng
với l qua BC, CA, AB. Chứng minh rằng điểm Anti-Steiner L của HP ứng với tam giác ABC nằm
trên (A0 B 0 C 0 ).

B1
A

L
B'
C2

H O
A1 Y
X
B C
C1
C'
P
Z A2

A'

Chứng minh. Gọi X, Y, Z lần lượt là giao của l với BC, CA, AB; A1 , B1 , C1 là các điểm đối xứng với
P qua BC, CA, AB.
Theo phép đối xứng ta có A1 , B1 , C1 lần lượt nằm trên các cạnh của tam giác A0 B 0 C 0 .
Do XB, ZB lần lượt là phân giác ∠B 0 XZ, ∠B 0 ZX nên B là tâm đường tròn nội tiếp tam giác
B 0 ZX. Suy ra ∠ZB 0 X = 2∠ZBX − 180◦ = 180◦ − 2∠ABC.
Mặt khác, gọi A2 , C2 lần lượt là giao của AH, CH với (O). L là điểm Anti-Steiner của HP . Suy
ra L, C1 , C2 thẳng hàng, L, A1 , A2 thẳng hàng.
Từ đó ∠C1 LA1 = ∠C2 LA2 = ∠HCB + ∠HAB = 180◦ − 2∠ABC = ∠ZB 0 X.
Suy ra L nằm trên (A1 C1 B 0 ). Chứng minh tương tự, L nằm trên (A1 B1 C 0 ) hay L là điểm Miquel
của tứ giác toàn phần A0 B 0 C 0 A1 B1 C1 . Ta thu được L nằm trên (A0 B 0 C 0 ).

Nhận xét. Khi l tiếp xúc với (O) ta thu được bài toán 6 IMO 2011.

64
6.4 Bài tập áp dụng
Bài 132. Cho tứ giác ABCD nội tiếp đường tròn (O). P là một điểm thuộc (O). Gọi X, Y, Z, T, U, V
lần lượt là hình chiếu của P trên AB, BC, CD, DA, AC, BD. Chứng minh rằng trung điểm của
XZ, Y T, U V thẳng hàng.

Bài 133. Cho tam giác ABC với trung tuyến AM , phân giác AN . Gọi Q, P là giao của đường thẳng
qua N và vuông góc N A với M A, BA. O là giao của đường thẳng qua P vuông góc BA với AN .
Chứng minh rằng QO vuông góc với BC.

Bài 134. Cho tam giác ABC nội tiếp đường tròn (O), trực tâm H. Một đường thẳng bất kì qua H
giao (O) tại hai điểm D, E. Gọi F là điểm đối xứng với A qua O. OD, OE cắt BC tại hai điểm M, N.
Chứng minh rằng O, M, N, F cùng thuộc một đường tròn.

Bài 135. (Nguyễn Văn Linh). Cho tam giác ABC nội tiếp đường tròn (O), trực tâm H. Một đường
thẳng bất kì qua H giao (O) tại hai điểm M, N. Gọi A1 , B1 , C1 lần lượt là điểm đối xứng với
A, B, C qua O. M A1 , N A1 giao BC tại A2 , A3 . Tương tự xác định B2 , B3 , C2 , C3 . Chứng minh rằng
(A1 A2 A3 ), (B1 B2 B3 ), (C1 C2 C3 ) đồng trục.

Bài 136. (IMO Shortlist 1991). Cho lục giác ABCDEF nội tiếp. Chứng mịnh rằng các đường thẳng
Simson của A, B, D, E lần lượt ứng với các tam giác BDF, ACE, ABF, ABC đồng quy khi và chỉ khi
CDEF là hình chữ nhật.

Bài 137. Cho tam giác ABC nội tiếp đường tròn (O). P là điểm bất kì trên mặt phẳng. AP, BP, CP
cắt (O) lần thứ hai tại A1 , B1 , C1 . Gọi A2 , B2 , C2 lần lượt là điểm đối xứng với P qua BC, CA, AB.
Chứng minh rằng (P A1 A2 ), (P B1 B2 ), (P C1 C2 ) đồng quy tại một điểm nằm trên (O).

Bài 138. Cho tứ giác ngoại tiếp ABCD. E là điểm bất kì nằm trên CD. Gọi (I1 ), (I2 ), (I3 ) lần lượt là
đường tròn nội tiếp các tam giác AED, BEC, ABE. Chứng minh rằng trực tâm tam giác I1 I2 I3 nằm
trên tiếp tuyến chung ngoài khác CD của (I1 ) và (I2 ).

7 Một số vấn đề về tứ giác toàn phần


7.1 Giới thiệu
Một hình tạo bởi giao điểm của bốn đường thẳng sao cho không có ba đường thẳng nào đồng quy,
được gọi là một tứ giác toàn phần. Ví dụ trong hình vẽ ở dưới, ABCDEF được gọi là một tứ giác
toàn phần với ba đường chéo AC, BD, EF.

D C E

Sau đây chúng ta sẽ tìm hiểu các tính chất liên quan đến tứ giác toàn phần.

65
7.2 Tính chất
Nếu không có giải thích gì thêm, tứ giác toàn phần đang xem xét là ABCDEF.

Tính chất 28. Đường tròn ngoại tiếp của các tam giác ABF, DCF, BCE, ADE đồng quy. Điểm đồng
quy đó được gọi là điểm Miquel của tứ giác toàn phần.

D C E

Chứng minh. Gọi M là giao của (AF B) và (BEC). Ta có ∠AM E = ∠AM B + ∠EM B = ∠AF B +
∠BCD = 180◦ − ∠ADE.
Từ đó M ∈ (ADE), tương tự M ∈ (F DC).

Tính chất 29. Điểm Miquel và tâm của các đường tròn ngoại tiếp tam giác ABF, DCF, BCE, ADE
cùng nằm trên một đường tròn - đường tròn Miquel của tứ giác toàn phần.

M
O1

A
O4
O2
B

O3

D C E

Chứng minh. Gọi O1 , O2 , O3 , O4 lần lượt là tâm của (F AB), (F CD), (EAD), (EBC).
Dễ thấy điểm đối xứng với M qua ba cạnh tam giác O1 O2 O4 lần lượt là F, B, C. Do F, B, C thẳng
hàng nên M nằm trên (O1 O2 O4 ) và BC là đường thẳng Steiner của M ứng với tam giác O1 O2 O4 .
Chứng minh tương tự suy ra O1 , O2 , O3 , O4 , M cùng thuộc một đường tròn.

66
Tính chất 30. Điểm Miquel có chung đường thẳng Simson ứng với các tam giác ABF, DCF, BCE,
ADE.

Chứng minh. Đây là hệ quả trực tiếp của tính chất 1. Hiển nhiên hình chiếu của M trên AB, BC, CD, DA
thẳng hàng.

Tính chất 31. Trực tâm của các tam giác ABF, DCF, BCE, ADE cùng nằm trên một đường thẳng,
được gọi là đường thẳng Steiner của tứ giác toàn phần.

F M

A
H1
B
H3

H2

D C E

H4

Chứng minh. Đây là hệ quả của tính chất 3, các trực tâm của tam giác ABF, DCF, BCE, ADE lần
lượt nằm trên đường thẳng Steiner của M ứng với các tam giác ABF, DCF, BCE, ADE. Mà M có
chung đường thẳng Simson nên cũng có chung đường thẳng Steiner với 4 tam giác này. Suy ra trực
tâm của 4 tam giác thẳng hàng.

Tính chất 32. Trung điểm các đường chéo của một tứ giác toàn phần cùng nằm trên một đường
thẳng, gọi là đường thẳng Gauss-Newton của tứ giác toàn phần.

K
L
A
H1
P
Q T

N B

M R
H2

D S C E

67
Chứng minh. Gọi H1 , H2 lần lượt là trực tâm các tam giác AF B, DF C. Kẻ các đường cao AK, BL, F T
của tam giác F AB, DR, CQ, F S của tam giác F DC.
Ta có H1 A.H1 K = H1 B.H1 L, H2 D.H2 R = H2 C.H2 Q nên H1 H2 là trục đẳng phương của (AC) và
(BD).
H1 F .H1 T = H1 B.H1 L, H2 F .H2 S = H2 D.H2 R nên H1 H2 là trục đẳng phương của (BD) và (EF ).
Như vậy ba đường tròn (AC), (BD), (EF ) đồng trục và có trục đẳng phương là H1 H2 . Do đó trung
điểm của AC, BD, EF thẳng hàng.

Từ đó ta có tính chất sau.

Tính chất 33. Các đường tròn đường kính là ba đường chéo của tứ giác toàn phần đồng trục và có
trục đẳng phương là đường thẳng Steiner của tứ giác toàn phần đó.

Tính chất 34. Gọi XY Z là tam giác tạo bởi ba đường chéo AC, BD, EF . Khi đó (XY Z) trực giao
với ba đường tròn đường kính AC, BD, EF , đồng thời tâm của (XY Z) nằm trên đường thẳng Steiner.

O Y

A
B

D C

Chứng minh. (XY Z) trực giao với (BD) hiển nhiên do (DBXY ) = −1, tương tự với (AC), (EF ).
2
Gọi O là tâm của (XY Z). Khi đó PO/(BD) = PO/(AC) = PO/(EF ) = R(XY Z) . Vậy O nằm trên trục
đẳng phương của (AC), (BD), (EF ) hay O nằm trên đường thẳng Steiner của tứ giác toàn phần
ABCDEF.

Tính chất 35. Các phân giác trong và ngoài của tứ giác toàn phần giao nhau tại 16 tâm nội tiếp và
bàng tiếp của bốn tam giác. 16 điểm này nằm trên hai bộ đường tròn đồng trục với giao điểm của hai
trục đẳng phương là điểm Miquel của tứ giác toàn phần.

Chứng minh. Xem phần Xung quanh bài 3 VMO 2012.

Tính chất 36. Điểm Miquel M là tâm vị tự quay của tứ giác ABCD. Nghĩa là tồn tại các phép vị
tự quay tâm M biến AB thành CD, AD thành BC.

Chứng minh. Bằng cộng góc dễ thấy 4M AD ∼ 4M BC, 4M AB ∼ 4M DC, từ đó có đpcm.

Tính chất 37. (Định lý Emelyanov). Gọi XY Z là tam giác tạo bởi các đường chéo AC, BD, EF ; M
là điểm Miquel của tứ giác toàn phần ABCDEF . Khi đó M nằm trên đường tròn Euler của tam giác
XY Z.

68
Z

X'

M
Y' F

H'
G Y
H
A
Z'
B
X

E
D C

Chứng minh. (Eisso J. Atzema). Gọi X, Y, Z là giao điểm của AC, BD, EF . X 0 , Y 0 , Z 0 lần lượt là trung
điểm Y Z, ZX, XY . Qua X kẻ các đường thẳng song song với AD, BC và cắt BC, AD lần lượt tại H, G.
Kéo dài XH cắt Y Z tại H 0 .
Ta có F (DCXE) = −1 và XH k F D nên H là trung điểm XH 0 . Như vậy H nằm trên đường
trung bình ứng với đỉnh X của tam giác XY Z. Tương tự với G. Điều đó có nghĩa là G, H nằm trên
Y 0Z 0.
AG BH
Mặt khác, = X(A∞GD) = X(CH∞B) = .
AD BC
Theo tính chất 9, M là tâm vị tự quay biến AD thành BC nên biến GD thành HC. Nghĩa là
M ∈ (F GH).
Ta thu được hình chiếu của M trên GH nằm trên đường thẳng Simson của M ứng với tam giác
DF C. Theo tính chất 3 suy ra hình chiếu của M trên Y 0 Z 0 nằm trên đường thẳng Simson của M ứng
với 4 tam giác F AB, F CD, EAD, EBC. Tương tự hình chiếu của M trên X 0 Y 0 , X 0 Z 0 cũng nằm trên
đường thẳng này. Theo định lý Simson đảo suy ra M, X 0 , Y 0 , Z 0 cùng thuộc một đường tròn.

Tính chất 38. Gọi E1 , E2 , E3 , E4 lần lượt là tâm đường tròn Euler của các tam giác F AB, F CD,
EAD, EBC. Khi đó các đường thẳng lần lượt qua E1 , E2 , E3 , E4 và vuông góc với CD, AB, BC, AD
đồng quy tại một điểm nằm trên đường thẳng Steiner của tứ giác toàn phần ABCDEF , gọi là điểm
Morley của tứ giác toàn phần.

Q
I
L O1 E1 H1 B

R
K
O2
J
A E2

H2

E D T M C

69
Chứng minh. Gọi H1 , H2 lần lượt là trực tâm, O1 , O2 lần lượt là tâm ngoại tiếp của tam giác F AB, F CD,
M, J lần lượt là trung điểm CD, AB; K, T lần lượt là hình chiếu của F trên AB, CD. Đường thẳng
qua E1 và vuông góc với CD giao F K tại I, đường thẳng qua E2 và vuông góc với AB giao F T tại
L. JE1 ∩ F K = {Q}, M E2 ∩ F T = {R}.
Do F K k E2 H nên ∠E2 LH2 = ∠T F K. Tương tự, ∠E1 IH1 = ∠T F K. Suy ra ∠E2 LH2 = ∠E1 IH1 .
Ta sẽ chứng minh ∠RE2 M = ∠JE1 Q, khi và chỉ khi ∠T LE2 +∠LRE2 = 270◦ −∠E1 JK −∠E1 IH1 .
Tương đương 2∠T F K = 90◦ − ∠E1 JK + ∠T RM = ∠JQK + ∠T RM .
Hay 2∠E = ∠O1 P H1 + ∠O2 P H2 = |∠A − ∠B| + |∠C − ∠D| (đúng)
Như vậy hai tam giác E2 LR và E1 QI có ∠RE2 L = ∠QE1 I và ∠RLE2 + ∠QIE1 = 180◦ .
QI RL QI RL O1 J R1 cos F R1
Áp dụng định lý hàm số sin suy ra = hay 1 = 1 . Nhưng = =
E1 Q E2 L 2 R1 2 R2
O2 M R 2 cos F R2
QI O1 J F H1 IH1 FL
nên = = . Vậy = . Áp dụng định lý Thales suy ra đường thẳng qua E1
RL O2 M F H2 FI LH2
vuông góc với CD giao đường thẳng qua E2 vuông góc với AB tại một điểm trên H1 H2 . Chứng minh
tương tự suy ra đpcm.

Nhận xét. Bài toán tương tự được phát biểu trong IMO Shortlist 2009, G6.

Tính chất 39. Các đường trung trực của O1 H1 , O2 H2 , O3 H3 , O4 H4 đồng quy tại một điểm, gọi là
điểm Hervey của tứ giác toàn phần.

Chứng minh. Cách 1.


Ta phát biểu một bổ đề.
Bổ đề 4. Cho hai đường tròn (O1 ) và (O2 ) giao nhau tại F và G. Đường thẳng qua F và vuông góc
với F G giao (O1 ) và (O2 ) lần lượt tại B và C. Gọi J, I là hai điểm bất kì trên (O1 ) và (O2 ). Đường
thẳng qua B song song với F I giao đường thẳng qua C song song với F J tại A. Kẻ AH ⊥ BC. Khi
đó I, J, F, H cùng thuộc một đường tròn.
Chứng minh.
A

B H F C

Q J
O2
O1 R
I
T

M X S Y N

Gọi Q, R là giao điểm thứ hai của AB với (O1 ), AC với (O2 ). F I giao GR tại X, F J giao GQ tại
Y . Do F X k AB và GQ ⊥ AB nên F X ⊥ GQ. Tương tự, F Y ⊥ GR. Suy ra G là trực tâm tam giác
XF Y . Ta thu được F G ⊥ XY hay XY k BC.

70
Dựng hai hình bình hành BM XF và F CN Y . Gọi S là hình chiếu của F trên XY . Ta có Y S.Y M =
Y G.Y Q = Y J.Y F suy ra tứ giác M SJF nội tiếp hay ∠M JF = 90◦ . Tương tự, ∠N IF = 90◦ .
Từ đó AH, M J, N I là ba đường cao của tam giác AM N và đồng quy tại trực tâm T .
Vậy H, I, J nằm trên đường tròn đường kính F T .
Trở lại bài toán.

N
M

H3
Q
H1 S

D
E3
E1 Y

R
B O3 O1
X
C
V
U

Theo tính chất 11, gọi S là giao của các đường thẳng lần lượt qua E1 , E2 , E3 , E4 và vuông góc với
CD, AB, BC, AD.
Kẻ AQ ⊥ H1 H3 , M, N lần lượt là trung điểm AH1 , AH3 , U, V lần lượt là trung điểm BF, DE.
MQ AH1 2O1 U MQ NQ
Ta có = = = 2 cos ∠BAD. Tương tự suy ra = .
M E1 R(ABF ) R(ABF ) M E1 N E3
Mặt khác, ta có ∠H1 M Q + ∠H3 N Q = 2∠H1 AH3 = ∠H1 AO1 + ∠H3 AO3 = ∠H1 M E1 + ∠H3 N E3 .
Suy ra ∠H1 M Q − ∠H1 M E1 = ∠H3 N E3 − ∠QN H3 hay ∠QM E1 = ∠QN E3 . Do đó hai tam giác
M QE1 và N QE3 đồng dạng. Suy ra ∠E3 QE1 = ∠M QN = ∠M AN = ∠E3 SE1 hay E3 , S, Q, E1 cùng
thuộc một đường tròn. (1)
Gọi R là giao của trung trực đoạn O1 H1 với đường thẳng qua S vuông góc với H1 H3 . Đường thẳng
qua S song song với AH3 giao (H1 R) tại E10 . Theo bổ đề 1 ta có E3 , S, Q, E10 cùng thuộc một đường
tròn. Kết hợp với (1) suy ra E10 ≡ E1 hay E1 R là trung trực của H1 O1 . Vậy trung trực của H1 O1 , H3 O3
giao nhau tại R nằm trên đường thẳng qua S vuông góc với đường thẳng Steiner của tứ giác toàn
phần ABCDEF . Chứng minh tương tự suy ra trung trực của H1 O1 , H2 O2 , H3 O3 , H4 O4 đồng quy tại
R.
Cách 2.

71
C

H1
H'2 A
H'4
O3 R O1
D
C T
H'3

H'1 O

O4 O2 F

Gọi H10 , H20 , H30 , H40 lần lượt là trực tâm các tam giác O2 O3 O4 , O1 O3 O4 , O1 O2 O4 , O1 O2 O3 .
Dễ thấy O1 H30 k= O3 H10 nên O1 H10 giao O3 H30 tại trung điểm T của mỗi đường. Tương tự suy ra
O1 H10 , O2 H20 , O3 H30 , O4 H40 đồng quy tại T . Phép vị tự HT−1 : O1 7→ H10 , O2 7→ H20 , O3 7→ H30 , O4 7→ H40
nên H10 , H20 , H30 , H40 cùng nằm trên đường tròn tâm R là đối xứng của O qua T.
Ta sẽ chứng minh trung trực của O1 H1 , O2 H2 , O3 H3 , O4 H4 đồng quy tại R.
Dễ thấy 4O2 O3 O4 ∼ 4F CD ∼ 4H20 H30 H40 , đồng thời O1 là trực tâm tam giác H20 H30 H40 .
Ta có RO1 và H1 O1 là đường thẳng Euler của hai tam giác H20 H30 H40 và F CD. Do hai tam giác
này đồng dạng nên ∠RO1 H1 = ∠(H20 H40 , DF ) = ∠(O2 O4 , DF ) = 90◦ − ∠M CF = 90◦ − ∠M BC =
90◦ − ∠M O3 O1 = ∠OO1 M.
H1 O1 R(CDF ) R(CDF ) O1 M
Mặt khác, = = = .
RO1 R(H20 H30 H40 ) R(O2 O3 O4 ) O1 O
Từ đó hai tam giác RO1 H1 và OO1 M đồng dạng, suy ra tam giác RO1 H1 cân tại R. Chứng minh
tương tự suy ra trung trực của H1 O1 , H2 O2 , H3 O3 , H4 O4 đồng quy tại R.

Trên đây là một số tính chất cơ bản của tứ giác toàn phần. Xuyên suốt cuốn sách này chúng ta
cũng bắt gặp rất nhiều định lý hay bài toán liên quan đến nó. Phải kể đến các tính chất của điểm
Euler-Poncelet (xem chương ), định lý Brocard trong trường hợp tứ giác ABCD nội tiếp (xem chương
), các tâm nghịch đảo trong trường hợp tứ giác ABCD ngoại tiếp (xem chương ) và các tính chất của
tứ giác lưỡng tâm (xem chương ).

7.3 Bài tập áp dụng


Bài 139. Cho tam giác ABC và một đường thẳng ∆ cắt các đường thẳng BC, CA, AB tại D, E, F
theo thứ tự. Gọi M là điểm Miquel của tứ giác toàn phần ABCDEF ; MA là giao điểm khác M của
đường tròn (AEF ) với đường tròn Miquel của tứ giác toàn phần ABCDEF . Các điểm MB , MC được
xác định tương tự. Chứng minh rằng các đường thẳng AMA , BMB , CMC đồng quy.

Bài 140. Cho tứ giác ABCD có DA, BC không song song. P là giao điểm của các đường chéo AC, BD.
DM BN
Các điểm M, N theo thứ tự chạy trên các đoạn DA, BC sao cho = . M N theo thứ tự cắt
DA BC
AC, BD tại Q, R. Chứng minh rằng đường tròn (P QR) luôn đi qua một điểm cố định khác P .

72
Bài 141. Cho tam giác ABC nội tiếp đường tròn (O). Tiếp tuyến của (O) kẻ từ B và C cắt nhau
tại T . S là điểm nằm trên tia CB sao cho AS ⊥ AT . B1 và C1 nằm trên tia ST ( T nằm giữa B1 và
C1 , B1 nằm giữa S và T ) sao cho B1 T = BT = C1 T . Chứng minh rằng hai tam giác ABC và AB1 C1
đồng dạng.
Bài 142. Cho tứ giác ABCD, AC cắt BD tại G. Gọi tâm ngoại tiếp của các tam giác ABG, BCG,
CDG, ADG lần lượt là O1 , O2 , O3 , O4 . O1 O3 cắt O2 O4 tại M . Đường thẳng d1 qua G cắt các đường
tròn (O2 ), (O4 ) lần lượt tại J, K; đường thẳng d2 qua G cắt các đường tròn (O1 ), (O3 ) lần lượt tại S, T .
Gọi I, U là trung điểm của JK, ST , CMR M I = M U .
Bài 143. (IMO shortlist 2008). Cho tứ giác lồi ABCD. Chứng minh rằng tồn tại điểm P nằm trong
tứ giác thỏa mãn:

∠P AB + ∠P DC = ∠P BC + ∠P AD = ∠P CD + ∠P BA = ∠P DA + ∠P CB = 90◦
khi và chỉ khi hai đường chéo AC và BD vuông góc.
Bài 144. Cho tam giác ABC nội tiếp (O) với trực tâm H. Hai đường thẳng d1 và d2 bất kì vuông
góc với nhau và đi qua H. d1 cắt BC, CA, AB lần lượt tại X1 , Y1 , Z1 . Tương tự ta xác định X2 , Y2 , Z2 .
Khi đó hai tứ giác toàn phần ABCX1 Y1 Z1 và ABCX2 Y2 Z2 có chung điểm Miquel.
Bài 145. (Đường thẳng Droz-Farny). Cho tam giác ABC nội tiếp (O) với trực tâm H. Hai đường
thẳng d1 và d2 bất kì vuông góc với nhau và đi qua H. d1 cắt BC, CA, AB lần lượt tại X1 , Y1 , Z1 .
Tương tự ta xác định X2 , Y2 , Z2 . Chứng minh rằng trung điểm các đoạn thẳng X1 X2 , Y1 Y2 , Z1 Z2 thẳng
hàng.
Bài 146. Cho tam giác ABC, đường thẳng l lần lượt cắt các cạnh BC, CA, AB tại D, E, F . Gọi
O1 , O2 , O3 lần lượt là tâm ngoại tiếp các tam giác AEF, BF D, CDE. Chứng minh rằng trực tâm tam
giác O1 O2 O3 nằm trên l.
Bài 147. Một đường tròn tâm O đi qua hai đỉnh A và C của tam giác ABC và cắt AB, BC lần thứ
hai tại K, N . Gọi M là giao điểm thứ hai của đường tròn ngoại tiếp các tam giác ABC và KBN .
Chứng minh rằng ∠OM B = 90◦ .
Bài 148. Cho 4ABC nội tiếp đường tròn tâm O. Gọi D là giao của tiếp tuyến kẻ từ A của (O) với
BC. Tương tự ta xác định được E, F . Gọi M, N, P lần lượt là trung điểm AD, BE, CF . Chứng minh
rằng M, N, P thẳng hàng.
Bài 149. Cho tam giác ABC. H là điểm nằm trong tam giác sao cho ∠HBA = ∠HCA. Gọi hình
chiếu của H trên phân giác trong và ngoài góc A lần lượt là P và Q. M là trung điểm BC. Chứng
minh rằng P, Q, M thẳng hàng.
Bài 150. Cho tam giác ABC. D, E, F là các điểm nằm trên BC, CA, AB sao cho AD, BE, CF đồng
quy. Gọi A1 , B1 , C1 lần lượt là trung điểm các cạnh BC, CA, AB và D1 , E1 , F1 lần lượt là trung điểm
các cạnh EF, F D, DE. Chứng minh rằng A1 D1 , B1 E1 , C1 F1 đồng quy.
Bài 151. (China TST 2004 Quiz). Cho hai đường tròn bằng nhau (O1 ) và (O2 ) cắt nhau tại P và Q.
Gọi O là trung điểm P Q. Kẻ 2 đường thẳng AD và BC qua P sao cho A, C ∈ (O1 ) và B, D ∈ (O2 ).
Gọi M và N lần lượt là trung điểm AB và CD . Biết rằng O1 và O2 không nằm trong phần mặt phẳng
giao của hai đường tròn , M , N không trùng O. Chứng minh rằng M , N , O thẳng hàng.
Bài 152. Cho tứ giác ABCD có AD = BC, AC ∩ BD = {O}, phân giác các góc ∠DAB và ∠CBA
cắt nhau tại I. Chứng minh rằng trung điểm các đoạn AB, CD, OI thẳng hàng.
Bài 153. Cho tam giác ABC và một điểm P bất kì nằm trong mặt phẳng tam giác. (P AB) cắt AC
tại E; (P CA) cắt AB tại F . M, N lần lượt là trung điểm BC, EF . Q là điểm liên hợp đẳng giác của
P ứng với tam giác ABC. Chứng minh rằng M N k AQ.

73
Bài 154. Cho tứ giác lồi ABCD có hai đường chéo cắt nhau tại M . Gọi trọng tâm hai tam giác
AM D, CM B lần lượt là P, Q; trực tâm hai tam giác DM C, M AB lần lượt là R, S. Chứng minh rằng
P Q ⊥ RS.
Bài 155. (IMO Shortlist 2009). Cho tứ giác nội tiếp ABCD. AC giao BD tại E, AD giao BC tại F .
Gọi G, H lần lượt là trung điểm AB và CD. Chứng minh rằng EF tiếp xúc với đường tròn ngoại tiếp
tam giác EGH.
Bài 156. Cho tứ giác ABCD. P là một điểm nằm trong tứ giác ABCD. Chứng minh rằng SAP B +
SCP D = SBP C + SAP D khi và chỉ khi P nằm trên đường thẳng Gauss của tứ giác ABCD.
Bài 157. Cho tứ giác ABCD ngoại tiếp đường tròn (I). Chứng minh rằng I nằm trên đường thẳng
Gauss-Newton của tứ giác ABCD.
Bài 158. Cho tứ giác nội tiếp ABCD nội tiếp (O). Chứng minh rằng đường thẳng Steiner của tứ giác
ABCD đi qua giao điểm P của hai đường chéo AC, BD.
Bài 159. Cho tứ giác nội tiếp ABCD. AD cắt BC tại E, AC cắt BD tại G. Gọi H1 , H2 , H3 , H4 lần
lượt là trực tâm các tam giác ECD, EAB, GCD, GAB. Chứng minh tứ giác H1 H3 H2 H4 là hình bình
hành.
Bài 160. Cho tứ giác nội tiếp ABCD, AB cắt CD tại P , AD cắt BC tại Q. Chứng minh rằng khoảng
cách giữa trực tâm hai tam giác AP D và AQB bằng khoảng cách giữa trực tâm hai tam giác CQD
và BP C.

8 Định lý ERIQ
8.1 Giới thiệu
Định lý ERIQ lần đầu tiên được đặt tên bởi tác giả Kostas Vittas trên diễn đàn toán học AoPS,
là các chữ viết tắt của cụm từ Equal Ratios In Quadrilateral. Định lý được phát biểu như sau:
Cho hai đường thẳng d1 và d2 . Trên d1 lấy các điểm A1 , B1 , C1 , trên d2 lấy các điểm A2 , B2 , C2
A1 B1 A2 B 2 A3 A1
sao cho = = k. Trên A1 A2 , B1 B2 , C1 C2 lần lượt lấy các điểm A3 , B3 , C3 sao cho =
B 1 C1 B 2 C2 A3 A2
B3 B1 C3 C1 A3 B3
= . Khi đó A3 , B3 , C3 thẳng hàng và = k.
B3 B2 C3 C2 B 3 C3
Trong phần này chúng ta sẽ chứng minh định lý trên và tìm hiểu một số ứng dụng trong các bài
toán về thẳng hàng. Các ví dụ được chỉnh sửa và bổ sung từ bài viết Bổ đề ERIQ và ứng dụng của
Trần Minh Ngọc, sinh viên ĐH Sư phạm TPHCM.

8.2 Chứng minh

A3 A2
A1

B1 M B3 B2

P
C1
C3 C2
Q

74
Dựng các hình bình hành A1 A3 M B1 , A1 A3 P C1 , A3 A2 B2 N, A3 A2 C2 Q.
B1 M A1 A3 B1 B3 B3 M B3 B1
Ta có = = nên theo định lý Thales, B3 , M, N thẳng hàng và = .
B2 N A2 A3 B2 B3 B3 N B3 B2
C3 P B3 M
Chứng minh tương tự suy ra C3 , P, Q thẳng hàng và = .(1)
C3 Q B3 N
A3 M A1 B1 A2 B 2 A3 N
Mặt khác ta có = = = . Suy ra M N k P Q.(2)
A3 P B1 C1 B 2 C2 A3 Q
A3 B3 A3 M
Từ (1) và (2) suy ra A3 , B3 , C3 thẳng hàng và = = k.
B 3 C3 A3 P
Nhận xét. Ta cũng có thể sử dụng phép cộng vector để chứng minh định lý trên.

8.3 Ứng dụng


Bài 161. Cho tam giác ABC. Các điểm D, E nằm trên AB, F, G nằm trên AC sao cho AD =
BE, AF = CG. Gọi M, I, J lần lượt là trung điểm BC, DF, EG. Chứng minh rằng AIM J là hình
bình hành.

F
I
D

P K N

E
J G

B M C

Chứng minh. Gọi N, P, K lần lượt là trung điểm của AC, AB, N P.
Từ giả thiết suy ra P D = P E, N F = N G. Áp dụng định lý ERIQ cho hai bộ ba D, P, E và F, N, G
suy ra I, K, J thẳng hàng đồng thời K là trung điểm IJ.
Mặt khác, AP M N là hình bình hành nên K là trung điểm AM . Suy ra AIM J là hình bình
hành.

Bài 162. Cho tam giác ABC. Trên các cạnh BC, CA, AB lần lượt lấy các điểm A1 , B1 , C1 . Gọi
Ga , Gb , Gc lần lượt là trọng tâm các tam giác AB1 C1 , BA1 C1 , CA1 B1 . Chứng minh rằng trọng tâm
các tam giác ABC, A1 B1 C1 , Ga Gb Gc thẳng hàng.

Ga
B1
M1
C1
G1 G
N1 M3 G2
Gb P1 Gc
M2
B A1 M C

75
Chứng minh. Gọi M, N, P lần lượt là trung điểm BC, CA, AB; M1 , N1 , P1 lần lượt là trung điểm
B1 C1 , C1 A1 , A1 B1 , M2 là trung điểm Gb Gc , M3 là trung điểm N1 P1 .
BGb CGc
Do = = 2 nên theo định lý ERIQ, trung điểm của BC, Gb Gc , N1 P1 hay M, M2 , M3
Gb N1 Gc P 1
M M2
thẳng hàng và = 2.
M 2 M3
Mặt khác gọi G, G1 , G2 lần lượt là trọng tâm tam giác ABC, A1 B1 C1 , Ga Gb Gc .
Dễ thấy G1 cũng là trọng tâm tam giác M1 N1 P1 . Lại áp dụng định lý ERIQ cho hai bộ ba A, Ga , M1
M M2 AGa GA G 2 Ga G1 M 1
và M, M2 , M3 ta có = = 2, đồng thời = = = −2 nên G, G1 , G2
M2 M 3 G a M1 GM G 2 M2 G1 M 3
thẳng hàng.

Bài 163. (Đường thẳng Gauss). Cho tứ giác toàn phần ABCDEF (AB∩CD = {E}, AD∩BC = {F }).
Gọi M, N, P lần lượt là trung điểm của AC, BD, EF . Chứng minh rằng M, N, P thẳng hàng.

M
K B

D L C E

Chứng minh. Qua B kẻ BK k DC, BL k AD(K ∈ AD, L ∈ CD). Ta có DKBL là hình bình hành
nên N là trung điểm KL.
Bài toán đưa về chứng minh trung điểm của KL, AC, F E thẳng hàng.
CL AK
Dễ thấy B(KCLE) = B(KF LA) nên = . Áp dụng định lý ERIQ cho hai bộ ba (L, C, E)
CE AF
và (K, A, F ) ta thu được N, M, P thẳng hàng.

Bài 164. Cho tứ giác ABCD nội tiếp đường tròn (O). P là điểm bất kì trong mặt phẳng. Gọi
X, Y, Z, T, U, V lần lượt là hình chiếu của P trên AB, BC, CD, DA, AC, BD. Chứng minh rằng trung
điểm của XZ, Y T, U V thẳng hàng.

Chứng minh. Xét 3 trường hợp.


-Trường hợp 1: P ≡ O. Bài toán hiển nhiên do XZ, Y T, U V đồng quy tại trung điểm mỗi đường
và điểm đồng quy là trọng tâm tứ giác ABCD.
-Trường hợp 2: P ∈ (O). Theo định lý về đường thẳng Simson dễ thấy X, Y, Z, T, U, V là các đỉnh
của một tứ giác toàn phần nên theo định lý về đường thẳng Gauss, trung điểm của XZ, Y T, U V thẳng
hàng.
-Trường hợp 3: P 6≡ O và P 6∈ (O).

76
B
M
X' A
X

R R'
G
P P'
O
Z
Z'
D

Gọi P 0 là giao của tia OP với (O). Kẻ X 0 , Y 0 , Z 0 , T 0 , U 0 , V 0 là hình chiếu của P 0 trên AB, BC, CD,
DA, AC, BD. G là trọng tâm tứ giác ABCD. Kẻ OM ⊥ AB, ON ⊥ CD. Gọi R, S, W lần lượt là trung
điểm của XZ, Y T, U V ; R0 , S 0 , W 0 lần lượt là trung điểm của X 0 Z 0 , Y 0 T 0 , U 0 V 0 .
XX 0 PP0 ZZ 0
Ta có P X k P 0 X 0 k OM, P Z k P 0 Z 0 k ON nên 0 = 0 = 0 .
XM PO ZN
RR 0 P P 0
Theo định lý ERIQ thì R, R0 , G thẳng hàng và 0 = 0 .
RG PO
PO
0
Xét phép vị tự HGP O : R0 7→ R, S 0 7→ S, W 0 7→ W . Theo trường hợp 2 thì R0 , S 0 , W 0 thẳng hàng
nên R, S, W thẳng hàng.

Bài 165. (Đường thẳng Newton). Cho tứ giác ABCD ngoại tiếp đường tròn (I). Chứng minh rằng I
nằm trên đường thẳng Gauss của tứ giác ABCD.

Chứng minh. Cách 1 (Trần Minh Ngọc).

A
B

T
N

I M
Y
D
O

Qua A, B, C, D lần lượt kẻ các đường thẳng vuông góc với IA, IB, IC, ID, cắt nhau tạo thành tứ
giác XY ZT.
Rõ ràng ∠AXB = 180◦ − ∠AIB = ∠DIC = 180◦ − ∠DZC.

77
Suy ra tứ giác XY ZT nội tiếp.
Đồng thời ∠DIC+∠DIT +∠CIY = ∠AXB+∠T AD+∠CBY = ∠AXB+∠XAB+∠XBA = 180◦ .
Suy ra T, I, Y thẳng hàng. Tương tự X, I, Z thẳng hàng.
Áp dụng bài toán 2 khi P ≡ I ta thu được trung điểm của AC, BD, I thẳng hàng.
Cách 2.
B

J Z'
X

A
Y

T M

I N

E X'

D Z K C

Gọi X, Y, Z, T lần lượt là tiếp điểm của (I) với AB, BC, CD, DA; AB giao CD tại E.
Qua I kẻ đường thẳng vuông góc với EI, cắt AB, CD lần lượt tại J, K. Do EI là phân giác ∠XEZ
nên IJ = IK.
Như vậy bài toán đưa về chứng minh trung điểm của AC, BD, JK thẳng hàng. Theo định lý ERIQ
JA KC
ta cần chứng minh = .
JB KD
Gọi X 0 , Z 0 lần lượt đối xứng với X, Z qua I.
JA
Ta có = I(∞JAB).
JB
Do EI đi qua trung điểm XZ nên EI là đường trung bình của tam giác XZZ 0 , suy ra XZ 0 k IE ⊥
IJ.
Đồng thời XT ⊥ IA, XY ⊥ IB, XX 0 ⊥ AB nên I(∞JAB) = X(X 0 Z 0 T Y ).
Chứng minh tương tự, I(∞KCD) = Z(X 0 Z 0 T Y ). Vậy I(∞JAB) = X(X 0 Z 0 T Y ) = Z(X 0 Z 0 T Y ) =
I(∞KCD).
Từ đó suy ra đpcm.

Bài 166. Cho tứ giác ABCD. P là điểm bất kì nằm trong góc tạo bởi hai đường thẳng AB và CD.
1
Chứng minh rằng P nằm trên đường thẳng Gauss khi và chỉ khi SAP B + SCP D = SABCD .
2

E
B

P
P'
L

F F' D
C

78
Chứng minh. Bỏ qua trường hợp đơn giản AB k CD.
Qua P kẻ đường song song với AB, cắt đường trung bình ứng với đỉnh A của tam giác ADC tại
L. AL cắt CD tại F , F P cắt AB tại E.
Ta có LA = LF, LP k AE nên P là trung điểm EF .
-Nếu P nằm trên đường thẳng Gauss.
AB CD
Theo định lý ERIQ, trung điểm EF, AC, BD thẳng hàng khi và chỉ khi = = k.
AE CF
Khi đó
k k k 1
SAP B + SCP D = k(SAP E + SCP F ) = SAECF = (SACE + SACF ) = · (SACB + SADC ) =
2 2 2 k
1
SABCD .
2
1
-Nếu SAP B + SCP D = SABCD .
2
0 AB CD
Trên DC lấy F sao cho = . Gọi P 0 là trung điểm EF 0 .
AE CF 0
1
Theo chiều thuận SAP 0 B + SCP 0 D = SABCD = SAP B + SCP D .
2
Do P P 0 k CD nên SCP D = SCP 0 D , do đó SAP 0 B = SAP B , khi và chỉ khi P P 0 k AB, điều này nghĩa
là P ≡ P 0 . Vậy P nằm trên đường thẳng Gauss.

Bài 167. (China TST 2004). Cho hai đường tròn (O1 ) và (O2 ) có cùng bán kính và cắt nhau tại A và
B. Hai đường thẳng d và d0 đi qua A cắt (O) và (O0 ) lần lượt tại M, N, P, Q (M, P ∈ (O1 ), N, Q ∈ (O2 )).
Chứng minh rằng trung điểm của AB, M Q, N P thẳng hàng.

Y
X Q
M
A

N
P
O2
O1

Chứng minh. (Phạm Minh Khoa) Gọi X là giao của M N và P B, Y là giao của P Q và BN . Do
AP BN XY là tứ giác toàn phần nên trung điểm AB, P N, XY thẳng hàng.
XN YP
Vậy ta chỉ cần chứng minh trung điểm của XY, M Q, N P thẳng hàng. Khi và chỉ khi = .
MN QP
Do (O1 ) và (O2 ) bằng nhau nên ∠AP B = ∠AM B = ∠AQB = ∠AN B. Suy ra 4M BN ∼ 4P BQ.
MN BN BN
Từ đó = = .
PQ BQ BP
XN BN BN
Dễ thấy 4XBN ∼ 4Y BP nên = = . Từ đó suy ra đpcm.
YP BP BQ

Bài 168. (Nguyễn Văn Linh). Cho tam giác ABC. M là điểm bất kì nằm trên đường cao kẻ từ A.
BM, CM lần lượt giao đường tròn đường kính BC tại K, L. BM giao (AM C) tại X, CM giao (AM B)
tại Y . Chứng minh rằng trung điểm của BC, XY, KL thẳng hàng.

79
A

Y
K
L
M

B H C

Chứng minh. Áp dụng định lý hàm số sin ta có:


BK sin ∠BCK sin ∠AM X
= = .
CL sin ∠CBL sin ∠AM Y
BX BA sin ∠BM A sin ∠AM X
Mặt khác dễ thấy 4Y AC ∼ 4BAX nên = = = .
YC YA sin ∠Y M A sin ∠AM Y
BK BX
Vậy = , từ đó theo định lý ERIQ ta thu được trung điểm của BC, XY, KL thẳng hàng.
CL CY
Bài 169. (Việt Nam TST 2009). Cho đoạn thẳng AB cố định. Với mỗi điểm M không nằm trên AB,
tia phân giác trong của ∠AM B cắt đường tròn đường kính AB tại N , phân giác ngoài của ∠AM B
cắt N A, N B lần lượt tại P, Q. Đường tròn đường kính N Q giao M A lần thứ hai tại R, đường tròn
đường kính N P giao M B lần thứ hai tại S. Chứng minh rằng trung tuyến ứng với đỉnh N của tam
giác N RS luôn đi qua một điểm cố định khi M thay đổi.
R

S Q
M
P

B
A

N
L

Chứng minh. Gọi K, L lần lượt đối xứng với M qua N A, N B.


Hiên nhiên K, L lần lượt nằm trên (N P ), (N Q).
Ta có ∠AKN = ∠AM N = ∠BM N = ∠SKN nên K, A, S thẳng hàng. Tương tự L, B, R thẳng
hàng.

80
Do N A, N B lần lượt là phân giác các góc M N K, M N L và ∠AN B = 90◦ nên K, N, L thẳng hàng.
Đồng thời N K = N M = N L nên N là trung điểm LK.
Ta sẽ chứng minh trung điểm của KL, AB, SR thẳng hàng. Theo định lý ERIQ ta cần chứng minh
SA RB SA RB
= hay = . (1)
SK BL MA MB
sin ∠SM A sin ∠BM R
Áp dụng định lý hàm số sin ta thu được (1) tương đương = . Điều này hiển
sin ∠ASM sin ∠M RB
nhiên đúng do ∠SM A = ∠BM R và ∠ASM + ∠M RB = ∠M N K + ∠M N L = 180◦ .
Vậy trung tuyến ứng với đỉnh N của tam giác N RS luôn đi qua trung điểm đoạn thẳng AB.

Nhận xét. Bài toán có thể được tổng quát như sau.

Bài 170. Cho tam giác N P Q, đường cao N M . K là một điểm bất kì trên N M . QK, P K lần lượt
cắt N P, N Q tại A, B. Đường tròn đường kính N P cắt M B tại M, S. Đường tròn đường kính N Q cắt
M A tại M, R. Chứng minh rằng đường thẳng nối trung điểm AB và RS đi qua N .

B
S
A

R K

Q
P M

Chứng minh. Dễ thấy M (ABN P ) = −1 và ∠N M P = 90◦ nên M N là phân giác ∠AM B.


Gọi T là giao của M B và N P thì (N P AT ) = −1 nên S(N P AT ) = −1. Mà ∠N SP = 90◦ nên SN
là phân giác ∠ASB.
Suy ra N là tâm đường tròn bàng tiếp góc M của tam giác AM S.
Tương tự N là tâm đường tròn bàng tiếp góc M của tam giác BM R.
Như vậy tứ giác ASBR ngoại tiếp đường tròn tâm N . Theo định lý về đường thẳng Newton thì N
nằm trên đường thẳng nối trung điểm AB và RS.

Bài 171. (Đường thẳng Droz-Farny). Cho tam giác ABC có trực tâm H. Hai đường thẳng d1 và d2
vuông góc với nhau tại H. d1 cắt BC, CA, AB lần lượt tại X1 , Y1 , Z1 . d2 cắt BC, CA, AB lần lượt tại
X2 , Y2 , Z2 . Chứng minh rằng trung điểm của X1 X2 , Y1 Y2 , Z1 Z2 thẳng hàng.

Chứng minh. Bổ đề. Cho tam giác ABC. Một đường thẳng d qua trực tâm H cắt AB, AC lần lượt
HZ MB
tại Z, Y. Đường thẳng d0 vuông góc với d tại H cắt BC tại M . Khi đó = .
HY MC
Chứng minh.

81
A
l

B M C

Qua A kẻ đường thẳng l song song với d, qua H kẻ đường thẳng l0 song song với BC.
HZ
Ta có = A(ZY Hl).
HY
MB
Do AZ ⊥ HC, AY ⊥ HB, AH ⊥ l0 , l ⊥ HM nên A(ZY Hl) = H(CBl0 M ) = .
MC
HZ MB
Vậy = .
HY MC
Trở lại bài toán.

Y1

Z1
Y2

H
Z2

X2 K L
B X1 C

X1 Y1 X2 Y2
Theo định lý ERIQ, ta sẽ chứng minh = .
X1 Z1 X2 Z2
HY1 Z2 A
−1 −1
X1 Y1 HY1 − HX1 HX1 Z2 B Z 2 A − Z 2 B Y2 C BA Y2 C
Xét = = = = · = · .
X1 Z1 HZ1 − HX1 HZ1 Y2 A Y2 A − Y2 C Z2 B CA Z2 B
−1 −1
HX1 Y2 C
X2 Y2 BA Y1 C
Tương tự, = · .
X2 Z2 CA Z1 B
Y2 C Y1 C Y2 C Z2 B
Như vậy ta cần chứng minh = hay = .
Z2 B Z1 B Y1 C Z1 B
Gọi K, L là hai điểm sao cho HK k AB, HL k AC.
Y2 C Z2 B
Ta có = H(CLY2 Y1 ) = H(KBZ1 Z2 ) = (do HC ⊥ HK, HL ⊥ HB, HY2 ⊥ HZ1 , HY1 ⊥
Y1 C Z1 B
HZ2 ).
Vậy trung điểm của X1 X2 , Y1 Y2 , Z1 Z2 thẳng hàng.

82
Nhận xét. Từ cách giải trên ta có thể tổng quát đường thẳng Droz-Farny cho các điểm chia đoạn
X1 X2 , Y1 Y2 , Z1 Z2 theo cùng một tỉ số.

Bài 172. (Nguyễn Văn Linh) Cho tam giác ABC. M, N nằm trên AB, P, Q nằm trên AC sao cho
MN PQ
= = k. Chứng minh rằng các điểm chia đoạn nối trọng tâm và tâm đường tròn ngoại tiếp
NB QC
của các tam giác AM P, AN Q, ABC theo cùng một tỉ số là các bộ điểm thẳng hàng.

Chứng minh. Gọi X1 , X2 , X3 lần lượt là trung điểm M P, N Q, BC. Theo định lý ERIQ dễ thấy
X1 X2
X1 , X2 , X3 thẳng hàng và = k. Gọi G1 , G2 , G3 lần lượt là trọng tâm các tam giác AM P, AN Q, ABC.
X2 X3
AG1 AG2 AG3 2 G 1 G2
Ta có = = = nên theo định lý Thales, G1 , G2 , G3 thẳng hàng = k.
AX1 AX2 AX3 3 G 2 G3
A

P
G1
G2
X1 Q
M

X2 G3
N

B X3 C

A
X1

Y1 P
O1 X2
Y2
M X3
O2
Y3 Q

N O3

B C

Mặt khác, gọi O1 , O2 , O3 lần lượt là tâm đường tròn ngoại tiếp các tam giác AM P, AN Q, ABC;
X1 , X2 , X3 lần lượt là hình chiếu của O1 , O2 , O3 trên AC, Y1 , Y2 , Y3 lần lượt là hình chiếu của O1 , O2 , O3
trên AB.
1 1 1
Ta có X1 X2 = AX2 − AX1 = (AQ − AP ) = P Q, X2 X3 = QC.
2 2 2
X1 X2 PQ
Vậy = = k.
X2 X3 QC
Y1 Y2
Tương tự suy ra = k.
Y2 Y3
Chú ý rằng O1 X1 k O2 X2 k O3 X3 , O1 Y1 k O2 Y2 k O3 Y3 . Theo định lý Thales suy ra O1 , O2 , O3
O1 O2
thẳng hàng và = k.
O2 O3
Vậy theo định lý ERIQ, các điểm chia đoạn thẳng Oi Gi theo cùng tỉ số thẳng hàng.

Bài 173. Cho tứ giác ABCD. Gọi (Ga , Gb , Gc , Gd ); (Oa , Ob , Oc , Od ); (Ta , Tb , Tc , Td ) lần lượt là trọng
tâm, tâm đường tròn ngoại tiếp, điểm chia đoạn thẳng Gi Oi theo cùng tỉ số của các tam giác

83
BCD, CDA, DAB, ABC. Chứng minh rằng giao điểm hai đường chéo của các tứ giác Ga Gb Gc Gd ,
Oa Ob Oc Od , Ta Tb Tc Td thẳng hàng.

Chứng minh. Gọi P là giao của AC và BD, G là giao của Ga Gc và Gb Gd , M là trung điểm CD. Ta
M Ga M Gb 1 1
có = = nên Ga Gb k= AB.
MA MB 3 3
GGa P A GGb PB
Tương tự suy ra hai tứ giác ABCD và Ga Gb Gc Gd đồng dạng. Từ đó = , = .
GGc P C GGd PD
B

P Gd
Gc

Ga
Gb

D
M C

Od
Oa
O
Ob Oc

D
C

Gọi O là giao của Oa Oc và Ob Od . Hiển nhiên đường nối hai tâm ngoại tiếp bất kì vuông góc với
đường nối hai trong 4 đỉnh của tứ giác ABCD.
Ta có
OOa OOa OOd sin ∠Oa Od Ob sin ∠Oa Oc Od sin ∠ACB sin ∠ABD PB PA PA
= · = · = · = · = .
OOc OOd OOc sin ∠Oc Oa Od sin ∠Ob Od Oc sin ∠DBC sin ∠BAC PC PB PC
OOb PB
Tương tự = .
OOd PD
GGa OOa P A GGb OOb PB
Như vậy = = , = = .
GGc OOc P C GGd OOd PD
Gọi OG giao Ta Tc tại T1 , OG giao Tb Td tại T2 .
Ta Ga Tc Gc GGa OOa
Xét hai bộ ba (Oa , Ga , Ta ) và (Oc , Gc , Tc ) có = = k, đồng thời = nên theo
Ta Oa Tc Oc GGc OOc
T1 G T2 G
định lý ERIQ, = k. Tương tự, = k hay T1 ≡ T2 , tức là giao của Ta Tc và Tb Td nằm trên
T1 O T2 O
OG.

84
9 Đường thẳng Newton mở rộng
9.1 Giới thiệu
Trong chương tứ giác toàn phần và định lý ERIQ chúng ta đã nghiên cứu về đường thẳng Newton
của tứ giác ngoại tiếp. Tiếp theo, chúng ta sẽ tổng quát định lý trên và đưa ra ba hệ quả đặc sắc của
đường thẳng Newton mở rộng.
Định lý. Cho tứ giác ABCD nội tiếp đường tròn (O). P là một điểm bất kì trên mặt phẳng. Các
đường vuông góc kẻ từ A, B, C, D tới P A, P B, P C, P D cắt nhau tạo thành tứ giác XY ZT . Khi đó O
nằm trên đường thẳng nối trung điểm các đường chéo của tứ giác XY ZT .

Chứng minh. Nếu P nằm trên (O), các đường vuông góc kẻ từ A, B, C, D tới P A, P B, P C, P D đồng
quy tại điểm đối xứng với P qua O nên bài toán hiển nhiên đúng.
Ta chứng minh bài toán trong trường hợp P nằm trong (O) (trường hợp P nằm ngoài được chứng
minh tương tự).
Trước hết ta phát biểu lại một bổ đề đã giới thiệu trong hai chương trước.
Bổ đề 5. Cho tứ giác ABCD. Gọi M, N lần lượt là trung điểm AC, BD, Q là điểm trên mặt phẳng
1
sao cho SAQB + SCQD = SBQC + SAQD = SABCD . Khi đó M, N, Q thẳng hàng.
2
Chứng minh.

N Q
E
M
I

D
F
C

Gọi I là giao của AB và CD. Lấy E trên đoạn IB sao cho AB = IE, F trên đoạn IC sao cho
IF = DC.
1
Ta có SABCD = SAQB + SDQC = SIEQ + SIF Q = SIEQF .
2
Mặt khác do N là trung điểm BD nên SIEN D = SIEN +SIDN = SAN B +SDN C = SAN B +SDN C =
1
SABCD
2
Suy ra SIEQF = SIEN D hay SEN F = SEQF . Từ đó N Q k EF . Tương tự ta cũng chứng minh được
M Q k EF .
Vậy M, N, Q thẳng hàng.
Trở lại bài toán.

85
X
A P'
A' B'
T H
B

D P

D' Q O

C
C'
Z

Gọi A0 , B 0 , C 0 , D0 lần lượt là giao điểm thứ hai của (O) với T X, XY, Y Z, ZT . Gọi H là trung điểm
BB 0 suy ra OH ⊥ BB 0 .
Đường thẳng kẻ từ B 0 vuông góc với XY cắt OP tại Q. Ta có OH là đường trung bình của hình
thang QB 0 BP nên O là trung điểm P Q.
Tương tự ta thu được các đường vuông góc kẻ từ A0 , B 0 , C 0 , D0 tới các cạnh T X, XY, Y Z, ZT đồng
quy tại Q.
Do tứ giác QA0 XB 0 nội tiếp nên ∠QXT = ∠A0 B 0 Q = 90◦ − ∠XB 0 A0 .
Tương tự ∠P XY = 90◦ − ∠XAB.
Mà ∠XAB = ∠XB 0 A0 do tứ giác A0 AB 0 B nội tiếp nên ∠QXT = ∠P XY (1)
Tương tự ta cũng có ∠P Y X = ∠QY Z(2), ∠P ZY = ∠QZT, ∠QT Z = ∠P T X.
Theo bổ đề trên thì O thuộc đường nối trung điểm hai đường chéo của tứ giác XY ZT nếu
SXOY + SZOT = SXOT + SY OZ
1 1
Hay (SXP Y + SXQY + ST P Z + ST QZ ) = (ST P X + ST QX + SY P Z + SY QZ )(3)
2 2
Gọi P 0 là điểm đối xứng của P qua B.
Ta có SXQY + SXP Y = SXQY + SXP 0 Y = SXQY P 0 = SQXP 0 + SQY P 0
1
= .(XQ.XP 0 . sin ∠QXP 0 + Y Q.Y P 0 . sin ∠QY P 0 )
2
Từ (1) và (2) suy ra ∠QXP 0 = ∠T XY, ∠QY P 0 = ∠XY Z.
Do đó XQ.XP 0 . sin ∠QXP 0 + Y Q.Y P 0 . sin ∠QY P 0 = XQ.XP. sin ∠T XY + Y Q.Y P. sin ∠XY Z
1
Tương tự ta thu được SXP Y +SXQY +ST P Z +ST QZ = (XQ.XP. sin ∠T XY +Y Q.Y P. sin ∠XY Z+
2
QZ.P Z. sin ∠Y ZT + QT.P T. sin ∠ZT X).
Tương tự với ST P X + ST QX + SY P Z + SY QZ suy ra (3) đúng. Vậy ta có đpcm.

Nhận xét 1. Khi P trùng O ta thu được đường thẳng Newton.


Nhận xét 2. Ta thấy P , Q là hai điểm liên hợp đẳng giác trong tứ giác XY ZT . Như vậy đường
thẳng Newton mở rộng trở thành "Một tứ giác tồn tại hai điểm liên hợp đẳng giác thì trung điểm của
đoạn nối hai điểm đó nằm trên đường thẳng Gauss-Newton".

9.2 Ứng dụng


Sau đây là ba ứng dụng đặc sắc của đường thẳng Newton mở rộng:

Bài 174. Cho tứ giác ABCD nội tiếp (O). Gọi P là điểm bất kì trên mặt phẳng, O1 , O2 , O3 , O4 lần
lượt là tâm đường tròn ngoại tiếp các tam giác AP B, BP C, CP D, DP A. Chứng minh rằng trung điểm
của các đoạn thẳng O1 O3 , O2 O4 , OP thẳng hàng.

86
X
B

O1
T
K' Y
O4

K O
L O2
P
H
D H'

O3

Chứng minh. Gọi X, Y, Z, T lần lượt các điểm đối xứng của P qua O1 , O2 , O3 , O4 ; L, K, H, K 0 , H 0 lần
lượt là trung điểm P O, O2 O4 , O1 O3 , Y T, XZ.
Ta có ∠XAP = ∠T AP = 90◦ nên X, A, T thẳng hàng. Tương tự, X, B, Y thẳng hàng; Y, C, Z
thẳng hàng; Z, D, T thẳng hàng.
Xét phép vị tự HP2 : O1 7→ X, O2 7→ Y, O3 7→ Z, O4 7→ T nên K 7→ K 0 , H 7→ H 0 Mà L 7→ O nên ta
cần chứng minh K 0 , O, H 0 thẳng hàng.
Điều này đúng theo định lý về đường thẳng Newton mở rộng với chú ý rằng P A ⊥ T X, P B ⊥
XY, P C ⊥ Y Z, P D ⊥ ZT.

Bài 175. Cho tứ giác ABCD nội tiếp (O).P là điểm bất kì trên mặt phẳng. Gọi X, Y, Z, T, H, K lần
lượt là chân đường vuông góc kẻ từ P đến AB, BC, CD, DA, AC, BD. Khi đó trung điểm các đoạn
XZ, Y T, HK thẳng hàng.

Chứng minh. Trường hợp 1. P nằm trên (O).

A X

O
K H
Z C
D
Y
T

87
Áp dụng định lý về đường thẳng Simson ta có các bộ ba điểm (X, H, Y ); (X, K, T ); (X, Z, Y ); (T, Z, H)
thẳng hàng. Khi đó trung điểm các đoạn XZ, Y T, HK cùng nằm trên đường thẳng Gauss của tứ giác
toàn phần XKY ZHT nên chúng thẳng hàng.
Trường hợp 2. P không nằm trên (O).
B

X
A

Y
H
K
T

P O

D
Z
C

Ta có ∠T XK = ∠AXK − ∠AXT = ∠KP B − ∠AP T = ∠AP B − ∠T P K = ∠AP B − ∠ADB


Tương tự ∠Y XH = ∠AP B − ∠ACB. Do đó ∠T XK = ∠Y XH. Tương tự ta thu được K và H là
hai điểm liên hợp đẳng giác trong tứ giác XY ZT . Theo nhận xét 2 ta thu được trung điểm của các
đoạn XZ, Y T, KH thẳng hàng.

Bài 176. (Nguyễn Văn Linh). Cho 4 điểm A, B, C, D theo thứ tự nằm trên đường tròn (O). Dựng
các đường tròn (O1 ), (O2 ), (O3 ), (O4 ) lần lượt qua các cặp điểm (A, B); (B, C); (C, D); (D, A), lần
lượt xoay vòng giao nhau lần thứ hai tại A0 , B 0 , C 0 , D0 . Chứng minh rằng A0 , B 0 , C 0 , D0 cùng nằm trên
đường tròn (O0 ) đồng thời trung điểm của O1 O3 , O2 O4 , OO0 thẳng hàng.

O1
A B

A' B'

O4 O2
O'
O
D' C'

D C

O3

Chứng minh. Ta có ∠D0 A0 B 0 = ∠ADD0 + ∠ABB 0 = 180◦ − ∠B 0 BC − ∠D0 DC = ∠B 0 C 0 D0 . Do đó tứ


giác A0 B 0 C 0 D0 nội tiếp đường tròn (O0 ).

88
Mặt khác, ∠O4 O1 O0 = 180◦ − ∠AA0 B 0 = ∠ABB 0 = ∠OO1 O2 nên O1 O0 , O1 O đẳng giác trong góc
O4 O1 O2 . Chứng minh tương tự suy ra O và O0 liên hợp đẳng giác trong tứ giác O1 O2 O3 O4 . Áp dụng
nhận xét 2 suy ra đpcm.

10 Định lý Pascal
10.1 Giới thiệu
Năm 16 tuổi, Pascal công bố một công trình toán học : Về thiết diện của đường cônic, trong đó
ông đã chứng minh một định lí nổi tiếng và gọi là Định lí về lục giác thần kì. Ông rút ra 400 hệ quả từ
định lí này. Nhà toán học và triết học vĩ đại lúc bấy giờ là Descartes cho rằng công trình của Pascal
đã bao hàm được bốn cuốn sách đầu của Apollonius. Ông đánh giá rất cao công trình toán học này
và nói : "Tôi không thể tưởng tượng nổi một người đang ở tuổi thiếu niên mà lại có thể viết được một
tác phẩm lớn như thế".
Định lý Pascal tổng quát phát biểu cho các đường conic tuy nhiên ở đây chúng ta chỉ xem xét
trong trường hợp đường tròn, phát biểu như sau:
Cho sáu điểm bất kì A, B, C, A0 , B 0 , C 0 cùng thuộc một đường tròn. Khi đó giao điểm của các cặp
đường thẳng (AB 0 , BA0 ), (AC 0 , CA0 ), (BC 0 , CB 0 ) thẳng hàng.
Đường thẳng đi qua 3 giao điểm trên được gọi là đường thẳng Pascal. Có thể thấy vai trò của
A, B, C, A0 , B 0 , C 0 như nhau nên chúng ta hoàn toàn có thể hoán vị các điểm này. Từ đó thu được rất
nhiều đường thẳng Pascal khác nhau. Cụ thể là 60 đường thẳng. Một số tính chất của đường thẳng
Pascal xem tại [2].
Để dễ quan sát các cặp đường  thẳng và không  bị ngộ nhận hay nhầm lẫn, đôi khi người ta sử dụng
A B C
cách kí hiệu 6 điểm như sau: .
A0 B 0 C 0
Trong trường hợp có hai điểm trùng nhau, chẳng hạn A ≡ B 0 , ta có thể coi đường thẳng AB 0 là
tiếp tuyến của đường tròn tại A. Từ đó thu được định lý Pascal cho lục giác suy biến thành ngũ giác,
tứ giác, tam giác nội tiếp.

10.2 Chứng minh


Có rất nhiều cách chứng minh định lý Pascal tuy nhiên ở đây xin giới thiệu tới bạn đọc hai cách
chứng minh sau.
Cách 1 (Jan van Yzeren).

A
C
K

Z
Y X

A' J
C'

B'

Gọi X, Y, Z lần lượt là giao điểm của các cặp đường thẳng (BC 0 , CB 0 ), (AC 0 , CA0 ), (AB 0 , BA0 ).

89
A0 C, AC 0 giao (XCC 0 ) lần thứ hai tại K, J.
Ta có ∠A0 KX = ∠XC 0 C = ∠BA0 C nên XK k A0 Z. Chứng minh tương tự ta suy ra hai tam giác
XKJ và ZA0 A có cạnh tương ứng song song. Từ đó XZ, KA0 , JA đồng quy hay X, Y, Z thẳng hàng.
Cách 2.
B

A
C
M
N

Z
Y X

A'
C'

B'

Gọi M, N lần lượt là giao của AC 0 và BA0 , BC 0 và CA0 .


Ta có A(A0 C 0 B 0 B) = C(A0 C 0 B 0 B) nên (A0 M ZB) = (N C 0 XB). Điều đó nghĩa là M C 0 , N A0 , XZ
đồng quy hay X, Y, Z thẳng hàng.

10.3 Ứng dụng


Bài 177. (Bổ đề Sawayama-Thebault). Cho tam giác ABC nội tiếp đường tròn (O), ngoại tiếp đường
tròn (I). Đường tròn ω tiếp xúc trong với (O) và tiếp xúc với AB, AC lần lượt tại Ab , Ac . Chứng minh
rằng I là trung điểm của Ab Ac .

A
E

F
Ac
I
O
Ab

Oa

B C

Chứng minh. Gọi X là tiếp điểm của ω và (O), E, F lần lượt là giao điểm của XAc , XAb với (O). Dễ
thấy E, F lần lượt là điểm chính giữa cung AC, AB. Suy ra BE giao CF tại I.
Áp dụng định lý Pascal cho lục giác AF BXCE ta có (AB ∩ XF ), (AC ∩ XE), (BE ∩ CF ) thẳng
hàng hay Ab , I, Ac thẳng hàng.
Mặt khác tam giác AAb Ac cân tại A có AI là phân giác ∠Ab AAc nên I là trung điểm Ab Ac .

Bài 178. Cho tứ giác ABCD nội tiếp đường tròn (O). AD giao BC tại P , BA giao CD tại Q. Đường
thẳng qua Q vuông góc với AC cắt OP tại X. Chứng minh rằng ∠ABX = 90o .

90
P

X
B
F
K
A

Q O

E
C

Chứng minh. Gọi E, F lần lượt là điểm đối xứng với A, C qua O. DF cắt BE tại X 0 .
Áp dụng định lý Pascal cho 6 điểm A, F, B, E, C, D ta có AE ∩ CF = {O}, DF ∩ BE = {X 0 },
AD ∩ BC = {P } thẳng hàng.
Điều còn lại là chứng minh QX 0 ⊥ AC.
Thật vậy, gọi K là giao của AC và QX 0 . Do tứ giác QX 0 BD nội tiếp nên ∠X 0 QB + ∠KAQ =
∠F DB + ∠BAC = ∠F CB + ∠BF C = 90◦ . Vậy X 0 ≡ X. Ta có đpcm.

Bài 179. (Định lý Collings) Cho tam giác ABC nội tiếp đường tròn (O), trực tâm H. Một đường
thẳng d bất kì đi qua H. Chứng minh rằng các đường thẳng đối xứng với d qua ba cạnh tam giác
ABC đồng quy tại một điểm nằm trên (O). Điểm này gọi là điểm Anti-Steiner của d ứng với tam giác
ABC.

A
B'

C'

F E
H O

B C

Chứng minh. Gọi da , db , dc là các đường thẳng đối xứng với d qua BC, CA, AB. d cắt AC, AB lần lượt
tại E, F . BH, CH giao (O) lần thứ hai tại B 0 , C 0 .
C 0 F giao (O) tại T . T B 0 giao AC tại E 0 .
Áp dụng định lý Pascal cho 6 điểm A, C 0 , B, T, C, B 0 suy ra F, H, E 0 thẳng hàng. Suy ra E 0 ≡ E.
Do C 0 và H đối xứng nhau qua AB nên C 0 F ≡ dc , tương tự B 0 E ≡ db . Vậy db cắt dc tại T nằm
trên (O). Chứng minh tương tự suy ra da , db , dc đồng quy tại T.

91
Bài 180. (Bài toán con bướm trong đường tròn). Cho đường tròn (O) và một dây cung AB. M là
trung điểm AB. Hai dây cung CD và EF bất kì đi qua M . CF, DE lần lượt AB tại I, J. Chứng minh
rằng M I = M J.

F D

A I M J B

C
E
N
K
L

Chứng minh. Gọi L, K lần lượt là điểm đối xứng với F, D qua O. CL giao EK tại N.
Áp dụng định lý Pascal cho 6 điểm F, D, E, L, K, C suy ra M, O, N thẳng hàng.
Do ∠IM N = ∠ICN = 90◦ , ∠N M J = ∠N EJ = 90◦ nên các tứ giác IM N C, JM N E nội tiếp.
Suy ra ∠IN M = ∠ICM = ∠JEM = ∠M N J. Vậy M I = M J.

Bài 181. Cho tam giác ABC nội tiếp đường tròn (O). Một đường thẳng d bất kì đi qua O cắt CA, AB
tại Y, Z. Gọi M, N, P lần lượt là trung điểm BY, CZ, Y Z. Chứng minh rằng O, M, N, P cùng thuộc
một đường tròn.

K J

P O
Z Y

M N

B C

Chứng minh. Gọi J, K lần lượt là điểm đối xứng với B, C qua O. KZ cắt (O) tại L, LJ cắt AC tại
Y 0.
Áp dụng định lý Pascal cho 6 điểm A, K, B, L, C, J suy ra Y 0 , O, Z thẳng hàng hay Y 0 ≡ Y . Ta
thu được đường tròn đường kính BY và CZ giao nhau tại L nằm trên (O).
Suy ra M O ⊥ BL, N O ⊥ CL. Từ đó ∠M ON = 180◦ − ∠BLC = ∠BAC = ∠M P N (do M P k
AB, N P k AC).
Vậy O, M, N, P cùng thuộc một đường tròn.

92
Nhận xét. Nếu gọi X là giao của d với BC thì chúng ta có (AX), (BY ), (CZ) đồng trục.

Bài 182. Cho tam giác ABC nội tiếp (O), ngoại tiếp (I). R là điểm bất kì nằm trên cung BC không
chứa A. Kẻ tiếp tuyến RX, RY tới (ABI) và tiếp tuyến RZ, RT tới (ACI). Chứng minh rằng trung
điểm XY, ZT và I thẳng hàng.

A
K

J Z
Y
I M
L

X T
B C

Chứng minh. Gọi J, K lần lượt là điểm chính giữa cung AB, AC. JR cắt AB tại L, KR cắt AC tại
M.
Ta có JX 2 = JB 2 = JL.JR nên L chính là trung điểm XY . Tương tự M là trung điểm ZT.
Áp dụng định lý Pascal cho 6 điểm A, J, B, R, C, K ta có JR ∩ AB = {L}, JC ∩ KB = {I},
KR ∩ AC = {M } thẳng hàng. Ta có đpcm.

Bài 183. Cho tam giác ABC, trực tâm H. M, N là hai điểm nằm trên AB, AC, P nằm trên nửa mặt
phẳng bờ BC chứa A sao cho các tam giác HM N, BP C là tam giác đều. Chứng minh rằng P M = P N .

P
A

M
L E
G K
N

F
H

B C

Chứng minh. Gọi L là trung điểm M N . E, F lần lượt là hình chiếu của B, C trên AC, AB. LF giao
P B tại G, LE giao CP tại K.

93
Ta có tứ giác M LHF nội tiếp nên ∠GF H = ∠LM H = 60◦ = ∠GBC. Suy ra B, F, G, C cùng
thuộc một đường tròn hay G nằm trên (BC). Tương tự K nằm trên (BC). 
B F K
Áp dụng định lý Pascal cho 6 điểm B, F, G, E, K, C, hay viết lại , ta có BE ∩ CF =
C E G
{H}, BG ∩ CK = {P }, F G ∩ EK = {L} thẳng hàng.
Điều này nghĩa là P nằm trên trung trực của M N hay P M = P N.

Bài 184. Cho tam giác ABC tại A nội tiếp đường tròn (O). Kẻ đường kính AD. S chuyển động trên
(O). SB giao AC tại M , SD giao BC tại N . Chứng minh rằng M N luôn đi qua một điểm cố định.

M
O

N
B
C
D
I

Chứng minh. Gọi I là giao điểm thứ hai của AN với (O). Do IN, SN lần lượt là phân giác các góc
SB IB
BIC, BSC nên = hay BSCI là tứ giác điều hoà. Điều này nghĩa là SI đi qua giao điểm J
SC IC
của tiếp tuyến tại B và C.  
S C A
Áp dụng định lý Pascal cho 6 điểm A, B, I, C, C, S ,hay viết lại , ta có BS ∩ AC =
C I B
{M }, AI ∩ CB = {N }, SI ∩ CC = {J} thẳng hàng. Vậy M N luôn đi qua J cố định.

Bài 185. Cho tứ giác ABCD ngoại tiếp (O). Gọi M, N, P, Q lần lượt là tiếp điểm của (O) với
AB, BC, CD, DA. Chứng minh rằng AC, BD, M P, N Q đồng quy.

I B

M
A
N

Q
J

D P C

Chứng minh. Gọi I là giao của M N và P Q, J là giao của M P và N Q.

94
 
M Q N
Áp dụng định lý Pascal cho 6 điểm Q, Q, M, M, N, P , hay viết lại , suy ra M P ∩
Q M P
N Q = {J}, M M ∩ QQ = {A}, P Q ∩ M N = {I} thẳng hàng hay A ∈ IJ.
Chứng minh tương tự C ∈ IJ hay AC, M P, N Q đồng quy tại J. Tương tự suy ra AC, BD, M P, N Q
đồng quy.

Bài 186. (Sharygin Geometry Olympiad 2012). Cho đường tròn (O), dây cung AB. Gọi (I) là đường
tròn tiếp xúc trong với (O) và tiếp xúc với AB. Gọi M, N lần lượt là điểm chính giữa cung AB chứa
(I) và không chứa (I). Kẻ tiếp tuyến N C, N D tới (I). AC giao BD tại X, AD giao BC tại Y . Chứng
minh rằng X, Y, I, M thẳng hàng.
M
F

Y
I C
D

X B
A
E

Chứng minh. Gọi E, F lần lượt là tiếp điểm của (I) với AB, (O). Suy ra E, F, N thẳng hàng.
Ta có N D2 = N C 2 = N E.N F = N A2 = N B 2 nên A, B, C, D cùng nằm trên đường tròn (N, N A).
Chú ý rằng tiếp tuyến tại C, D của (N ) giao nhau tại I, tiếp tuyến tại A, B của (N ) giao nhau
tại M . Áp dụng định lý Pascal cho 6 điểm A, B, C, C, D, D ta có CC ∩ DD = {I}, BD ∩ AC =
{X}, AD ∩ BC = {Y } thẳng hàng.
Lại áp dụng định lý Pascal cho 6 điểm A, A, B, B, C, D ta có AA ∩ BB = {M }, BD ∩ AC =
{X}, AD ∩ BC = {Y } thẳng hàng.
Vậy X, Y, I, M thẳng hàng.

10.4 Bài tập áp dụng


Bài 187. Cho tam giác ABC nội tiếp đường tròn (O). M là điểm bất kì nằm trong tam giác. Gọi
A0 , B 0 , C 0 là giao của AM, BM, CM với (O), cạnh của tam giác A0 B 0 C 0 cắt cạnh tam giác ABC theo
thứ tự tại X, Y, Z, T, U, V . Chứng minh rằng XT, Y U, ZV đồng quy.

Bài 188. Cho tam giác ABC nội tiếp đường tròn (O). Gọi M, N, P lần lượt là trung điểm BC, CA, AB.
Chứng minh rằng các đường tròn (AOM ), (BON ), (COP ) đồng trục.

Bài 189. Cho đường tròn (O) đường kính AB. Gọi E là điểm nằm ngoài đường tròn, kẻ tiếp tuyến
EC, ED. AD giao BC tại F sao cho F nằm trong (O). Chứng minh rằng EF ⊥ AB.

Bài 190. (APMO 2013). Cho tứ giác ABCD nội tiếp đường tròn (O). P nằm trên tia AC sao cho
P B, P D tiếp xúc với (O). Tiếp tuyến của (O) tại C cắt P D tại Q, AD tại R. E là giao điểm thứ hai
của AQ và (O). Chứng minh rằng B, E, R thẳng hàng.

95
Bài 191. Cho tam giác ABC nội tiếp đường tròn (O). P là điểm bất kì trên mặt phẳng. AP, BP, CP
cắt (O) lần thứ hai tại A1 , B1 , C1 . Đường tròn đường kính AP, BP, CP giao (O) lần thứ hai tại
A2 , B2 , C2 . Chứng minh rằng A1 A2 , B1 B2 , C1 C2 , OP đồng quy.
Bài 192. Cho tam giác ABC, đường cao BD, CE giao nhau tại trực tâm H. Gọi S, R là trung điểm
BH, CH. SC giao BR tại Q, ES giao DR tại G. Chứng minh rằng A, Q, G thẳng hàng.
Bài 193. Cho tam giác ABC nội tiếp đường tròn (O), trực tâm H. M là trung điểm BC. Tia M H
cắt (O) tại E. Đường thẳng qua qua A song song với BC cắt (O) tại D. DE giao OH tại T . Chứng
minh rằng T A tiếp xúc với (O).
Bài 194. Cho tứ giác ABCD nội tiếp đường tròn (O). AC giao BD tại E. P là điểm nằm trong tứ
giác sao cho ∠P AB + ∠P CB = ∠P BC + ∠P DC = 90o . Chứng minh rằng O, P, E thẳng hàng.
Bài 195. (THTT). Cho tứ giác ABCD vừa nội tiếp đường tròn (O) vừa ngoại tiếp đường tròn (I).
AC giao BD tại P . Chứng minh rằng O, I, P thẳng hàng.
Bài 196. (Romania TST 3 2010). Cho tam giác không cân ABC. Phân giác BB0 , CC0 cắt (O) lần
thứ hai tại B1 , C1 . Gọi I là tâm nội tiếp tam giác ABC. B0 C0 giao B1 C1 tại P . Chứng minh rằng
IP k BC.
Bài 197. Cho tam giác ABC nội tiếp đường tròn (O). AO giao (O) lần thứ hai tại A1 . Tiếp tuyến
của (O) tại A1 giao BC tại D. DO giao AB, AC lần lượt tại P, Q. Chứng minh rằng OP = OQ.
Bài 198. (IMO Shortlist 2007). Cho tam giác ABC nội tiếp (O), A1 , B1 , C1 lần lượt là trung điểm
BC, CA, AB. P là điểm chuyển động trên (O). P A1 , P B1 , P C1 cắt (O) lần thứ hai tại A0 , B 0 , C 0 . Chứng
minh rằng diện tích tam giác tạo bởi giao điểm các đường thẳng AA0 , BB 0 , CC 0 không phụ thuộc vào
vị trí của P .
Bài 199. (Bulgaria TST 2003). Cho tứ giác ABCD ngoại tiếp đường tròn (O). Hạ OP ⊥ AC. Chứng
minh rằng ∠AP B = ∠AP D.
Bài 200. (China 2005). Một đường tròn cắt các cạnh của tam giác ABC theo thứ tự tại D1 , D2 ,
E1 , E2 , F1 , F2 . Gọi L, M, N lần lượt là giao điểm của D1 E1 và D2 F2 , E1 F1 và E2 D2 , F1 D1 và F2 E2 .
Chứng minh rằng AL, BM, CN đồng quy.

11 Định lý Brianchon
11.1 Giới thiệu
Định lý Brianchon ,được đặt theo tên nhà toán học Charles Julien Brianchon, là một trong những
định lý nổi tiếng của hình học xạ ảnh. Nó được coi là kết quả liên hợp của định lý Pascal. Trong hình
học Euclide, định lý được phát biểu như sau:
Cho lục giác ABCDEF ngoại tiếp đường tròn (O). Khi đó các đường chéo AD, BE, CF đồng quy.
B
A

E
D

96
Chú ý rằng lục giác ABCDEF không nhất thiết phải là lục giác lồi.

11.2 Chứng minh


Cách 1 (A.S.Smogorzhevski). [1]

O1 Z'
V'

B
X Y
A
C

V
X'
Z

O2 F D Y'
U
T
E O3

T'

U'

Gọi X, Y, Z, T, U, V lần lượt là tiếp điểm của (O) với AB, BC, CD, DE, EF, F A.
Trên tia XA lấy X 0 , tia Y C lấy Y 0 , tia ZC lấy Z 0 , tia T E lấy T 0 , tia U E lấy U 0 , tia V A lấy V 0 sao
cho XX 0 = Y Y 0 = ZZ 0 = T T 0 = U U 0 = V V 0 .
Rõ ràng tồn tại đường tròn (O1 ) tiếp xúc với V V 0 , ZZ 0 tại V 0 , Z 0 ; đường tròn (O2 ) tiếp xúc với
Y Y 0 , U U 0 tại Y 0 , U 0 ; đường tròn (O3 ) tiếp xúc với XX 0 , T T 0 tại X 0 , T 0 .
Lại có AV 0 = V V 0 − V A = XX 0 − XA = AX 0 . Suy ra A nằm trên trục đẳng phương của (O1 ) và
(O3 ), chứng minh tương tự suy ra AD là trục đẳng phương của (O1 ) và (O3 ).
Tương tự ta thu được AD, BE, CF đồng quy tại tâm đẳng phương của (O1 ), (O2 ), (O3 ).
Cách 2.

B
P
X Y
A
Q

V B' C

C'
F A'

U Z

D
T
E

Kí hiệu các tiếp điểm giống cách 1. Gọi XY ∩ V Z = {P }, XZ ∩ Y U = {Q}.


Áp dụng định lý Pascal cho 6 điểm X, X, Y, Z, V, V suy ra P, A, Q thẳng hàng. Tương tự suy ra
P, Q, A, C thẳng hàng.

97
Bằng phương pháp tương tự ta thu được XT, ZV, AD đồng quy tại A0 , XT, Y U, BE đồng quy tại
B0, YU, ZV đồng quy tại C 0 .
Xét hai tam giác ABC và tam giác A0 B 0 C 0 có AB∩A0 B 0 = {X}, AC∩A0 C 0 = {P }, BC∩B 0 C 0 = {Y }
thẳng hàng nên theo định lý Desargues, AA0 , BB 0 , CC 0 đồng quy hay AD, BE, CF đồng quy.
Cách 3.
X B
A
Y
B'

X'
A' Y' C' C

V' O
V
Z
Z'
F'
U'
T' D'
F
E' D
U T
E

Vẫn kí hiệu các tiếp điểm giống cách 1.


Gọi A0 là giao của OA với XV , tương tự xác định B 0 , C 0 , D0 , E 0 , F 0 .
Xét phép nghịch đảo IO R2 : A 7→ A0 , B 7→ B 0 , C 7→ C 0 , D 7→ D 0 , E 7→ E 0 , F 7→ F 0 .
1
Xét phép vị tự HO2 : X 7→ X 0 , Y 7→ Y 0 , Z 7→ Z 0 , T 7→ T 0 , U 7→ U 0 , V 7→ V 0 .
Khi đó AD, BE, CF đồng quy khi và chỉ khi đường tròn ngoại tiếp các tam giác A0 OD0 , B 0 OE 0 , C 0 OF 0
đồng trục. Điều này xảy ra khi và chỉ khi tâm ngoại tiếp của 3 tam giác này thẳng hàng hay
X 0 V 0 ∩ Z 0 T 0 , X 0 Y 0 ∩ T 0 U 0 , Y 0 Z 0 ∩ U 0 V 0 thẳng hàng.
R
Mà X 0 , Y 0 , Z 0 , T 0 , U 0 , V 0 cùng nằm trên (O, ) nên theo định lý Pascal, X 0 V 0 ∩ Z 0 T 0 , X 0 Y 0 ∩
2
T 0 U 0 , Y 0 Z 0 ∩ U 0 V 0 thẳng hàng. Ta có đpcm.
Cách 4.
M

X
B
Y
C
V

Z
N
D
T
F U E

98
Áp dụng định lý Pascal cho 6 điểm X, Y, Z, T, U, V suy ra V X ∩ZT = {M }, XY ∩U T = {N }, Y Z ∩
U V = {P } thẳng hàng.
Xét cực và đối cực ứng với đường tròn (O).
Ta có M nằm trên đường đối cực của A và D nên A, D nằm trên đường đối cực của M . Vậy AD
là đường đối cực của M.
Tương tự BE là đường đối cực của N, CF là đường đối cực của P.
Do M, N, P thẳng hàng nên AD, BE, CF đồng quy tại cực của đường thẳng qua M, N, P.
Nhận xét. Cách 3,4 cho thấy định lý Brianchon và định lý Pascal liên hợp với nhau. Có thể dùng
định lý này để chứng minh định lý kia.

11.3 Ứng dụng


Bài 201. (Czech-Polish-Slovak 2008). Cho lục giác lồi ABCDEF sao cho ∠F AB = ∠BCD = ∠DEF
và AB = BC, CD = DE, EF = F A. Chứng minh rằng AD, BE, CF đồng quy.

A B

C
I
F

D
E

Chứng minh. Gọi I là tâm ngoại tiếp tam giác ACE. Do BA = BC, IA = IC nên BI là phân giác
góc ABC, tương tự DI, F I là phân giác các góc CDE, EFA.
Mặt khác, từ giả thiết ∠F AB = ∠BCD = ∠DEF suy ra ∠IAF = ∠IEF = ∠DEF − ∠IED =
∠BCD − ∠ICD = ∠BCI = ∠BAI, suy ra AI là phân giác ∠BAF.
Chứng minh tương tự suy ra I là tâm đường tròn nội tiếp lục giác ABCDEF . Theo định lý
Brianchon ta thu được AD, BE, CF đồng quy.

Bài 202. Cho đường tròn (O) và một điểm E nằm ngoài đường tròn. Kẻ các tiếp tuyến EA, EC tới
(O). B là điểm bất kì thuộc cung nhỏ AC. Tiếp tuyến của (O) tại B cắt EA, EC lần lượt tại F, D.
Gọi P là điểm bất kì nằm trong (O). DP, EP, F P cắt BC, CA, AB lần lượt tại I, J, K. Chứng minh
rằng AI, BJ, CK đồng quy.

A
F

K
B
E
O

J
P
I
D

Chứng minh. Áp dụng định lý Brianchon cho lục giác ngoại tiếp suy biến AF BDCE suy ra AD, BE, CF
đồng quy. Do A, B, C lần lượt nằm trên các cạnh của tam giác DEF ; I, J, K nằm trên các cạnh của

99
tam giác ABC và DI, EJ, F K đồng quy tại P nên theo định lý Céva-nest ta thu được AI, BJ, CK
đồng quy.

Bài 203. (Iran TST 2007). Cho tam giác ABC ngoại tiếp đường tròn (I). Gọi M, N lần lượt là
tiếp điểm của (I) với AB, AC, M N giao BC tại D. EF là tiếp tuyến của (I) sao cho EF k BC
(E ∈ AB, F ∈ AC). Gọi J là trung điểm EF , chứng minh rằng DJ tiếp xúc với (I).

J F
E
N

M I

D B C

Chứng minh. Gọi J 0 là giao của tiếp tuyến của (I) qua D với EF .
Áp dụng định lý Brianchon cho lục giác suy biến DJ 0 EAN C suy ra DN, AJ 0 , CE đồng quy.
Chứng minh tương tự suy ra AJ 0 , EC, F B, M N đồng quy.
Do EF k BC nên theo bổ đề hình thang, AI đi qua trung điểm EF . Vậy J 0 E = J 0 F hay J 0 ≡ J.
Ta có đpcm.

Bài 204. (Iran 2002). Cho đường tròn (O) và một điểm A nằm ngoài đường tròn. Từ A kẻ hai tiếp
tuyến AB, AC tới (O). Tiếp tuyến l bất kì của (O) cắt AB, AC lần lượt tại P, Q. Đường thẳng qua P
song song với AC cắt BC tại R. Chứng minh rằng khi l chuyển động, QR luôn đi qua một điểm cố
định.

C
B
R
O
T

P Q

Chứng minh. Gọi giao của QR với AB là T . Giả sử tiếp tuyến khác T A kẻ từ T với (O) cắt AC tại J.
Áp dụng định lý Brianchon cho lục giác ngoại tiếp suy biến JCQP BT ta có JP, CB, QT đồng quy
tại R.
Tuy nhiên P R song song với AC nên J là điểm vô cùng, tức là tiếp tuyến khác T A kẻ từ T tới (O)
song song với AC. Suy ra T là điểm cố định.

Bài 205. (Iran 2003). Cho tam giác ABC và một điểm Q nằm trong tam giác. M, N, P lần lượt
là giao của AQ, BQ, CQ với BC, CA, AB. Đường tròn nội tiếp (I) của tam giác ABC tiếp xúc với

100
BC, CA, AB lần lượt tại D, E, F . Tiếp tuyến của (I) tại M, N, P khác các cạnh của tam giác ABC
cắt nhau tạo thành tam giác D0 E 0 F 0 . Chứng minh rằng DD0 , EE 0 , F F 0 đồng quy tại Q.

E
F

P I
N
Q

D F'
B M C

D'

E'

Chứng minh. Áp dụng định lý Brianchon cho lục giác suy biến BP D0 N CD suy ra BN, CP, DD0 đồng
quy tại Q.
Chứng minh tương tự ta có đpcm.

Bài 206. (Poncelet porism). Cho tam giác ABC nội tiếp đường tròn (O), ngoại tiếp đường tròn (I).
Gọi D là điểm bất kì trên (O). Kẻ các tiếp tuyến từ D đến (I), cắt (O) lần thứ hai tại E, F . Chứng
minh rằng EF tiếp xúc với (I).

E
A

N
F
J O
I
M

B C

Chứng minh. Gọi M, N, J lần lượt là giao điểm của DF và AB, DE và AC, BE và CF.
Áp dụng định lý Pascal cho 6 điểm F, A, E, C, D, B suy ra M, J, N thẳng hàng. Hay BE, CF, M N
đồng quy.
Gọi E 0 là giao của tiếp tuyến tại F khác F D của (I) với DN . Áp dụng định lý Brianchon cho
lục giác ngoại tiếp F E 0 N CBM suy ra BE, CF 0 , M N đồng quy. Điều này nghĩa là F 0 ≡ F . Ta có
đpcm.

101
Bài 207. Cho tam giác ABC nội tiếp đường tròn (O), ngoại tiếp đường tròn (I). P, Q là hai điểm
nằm trên (O). Kẻ tiếp tuyến P X, P Y, QM, QN tới (I) (X, Y, M, N ∈ (O)). Gọi T, R lần lượt là giao
của P Y, QN với BC. Chứng minh rằng M T và XR cắt nhau tại một điểm nằm trên (O).

N
A

Y
O
I
M
E
D R
B T C

P K
Q

Chứng minh. Gọi D là giao của P X và QM , K là giao của M T và XR. Theo định lý Poncelet porism
ta thu được M N tiếp xúc với (I).
Từ đó áp dụng định lý Brianchon cho lục giác RN M DP T suy ra RD, N P, M K đồng quy tại E.
Do P X ∩ M Q = {D}, M K ∩ P N = {E}, XK ∩ QN = {R} thẳng hàng nên áp dụng định lý Pascal
dễ dàng suy ra K nằm trên (O). Ta có đpcm.

Bài 208. (Iran TST 2007). Cho tam giác ABC ngoại tiếp đường tròn (I) và l là một tiếp tuyến của
(I). Gọi l0 là đường thẳng bất kì cắt BC, CA, AB lần lượt tại A0 , B 0 , C 0 . Tiếp tuyến của (I) tại A0 khác
BC cắt l tại A1 . Tương tự xác định B1 , C1 . Chứng minh rằng AA1 , BB1 , CC1 đồng quy.

C' l
C
B A' C1
B1

l'
A1 B'

Chứng minh. Áp dụng định lý Brianchon cho lục giác A0 CAC 0 C1 A1 ta thu được A0 C 0 , CC1 , AA1 đồng
quy. Có nghĩa là AA1 cắt CC1 tại một điểm nằm trên l0 . Chứng minh tương tự suy ra AA1 , BB1 , CC1
đồng quy tại một điểm nằm trên l0 .

102
Nhận xét. Trong một số ví dụ trên ta đã áp dụng định lý Brianchon cho các lục giác suy biến,
lục giác không lồi hay gọi là một đường gấp khúc khép kín ngoại tiếp. Có thể xảy ra trường hợp như
hình vẽ dưới đây. Rõ ràng ABCDEF cũng được gọi là đường gấp khúc ngoại tiếp, tuy nhiên định lý
Brianchon không thể áp dụng trong trường hợp này. Chính vì vậy chúng ta cần định nghĩa lại một
đường gấp khúc 6 đỉnh ngoại tiếp một cách chặt chẽ hơn để thoả mãn định lý Brianchon như sau:
Một đường gấp khúc khép kín 6 đỉnh A1 A2 A3 A4 A5 A6 thoả mãn A1 A2 , A2 A3 , ..., A6 A1 đều tiếp xúc với
một đường tròn (O) đồng thời Ai Ai−1 , Ai Ai+1 là hai tiếp tuyến khác nhau kẻ từ Ai đến (O). Khi đó
A1 A4 , A2 A5 , A3 A6 đồng quy.

C
A B

F
E
D

11.4 Bài tập áp dụng


Bài 209. Cho tứ giác ABCD ngoại tiếp đường tròn (O). Gọi X, Y, Z, T lần lượt là tiếp điểm của (O)
với AB, BC, CD, DA. Chứng minh rằng AC, BD, XZ, Y T đồng quy; AZ, CT, BD đồng quy.

Bài 210. Cho hình vuông ABCD. F là trung điểm cạnh CD. E là một điểm nằm trên đường thẳng
AB. Đường thẳng qua F song song với DE cắt BC tại H. Chứng minh rằng EH tiếp xúc với đường
tròn nội tiếp hình vuông ABCD.

Bài 211. Cho hình thoi ABCD ngoại tiếp đường tròn (I). Gọi M, N, P, Q lần lượt là các điểm nằm
trên AD, AB, BC, CD sao cho M N, P Q cùng tiếp xúc với (I). Chứng minh rằng M P k N Q.

Bài 212. Cho tam giác ABC ngoại tiếp đường tròn (I). Gọi (Oa ) là đường tròn qua I, cắt đoạn AI
và tiếp xúc với AB, AC lần lượt tại X, Y . Tương tự có (Ob ), (Oc ).(Ob ) tiếp xúc với BA, BC tại N, M ,
(Oc ) tiếp xúc với CA, CB tại Z, T . Chứng minh rằng XT, Y M, ZN đồng quy.

Bài 213. (St. Petersburg MO). Cho tam giác ABC ngoại tiếp đường tròn (I). Một đường tròn tâm I
cắt BC tại A1 , A2 ; CA tại B1 , B2 ; AB tại C1 , C2 sao cho A1 , A2 , B1 , B2 , C1 , C2 theo thứ tự xoay vòng.
Gọi A3 , B3 , C3 lần lượt là trung điểm cung A1 A2 , B1 B2 , C1 C2 . C4 là giao của A2 A3 và B1 B3 , A4 là
giao của B2 B3 và C1 C3 , B4 là giao của C2 C3 và A1 A3 . Chứng minh rằng A3 A4 , B3 B4 , C3 C4 đồng quy.

Bài 214. Cho lục giác lưỡng tâm ABCDEF . Phân giác các góc B, D, F cắt AC, CE, EA lần lượt tại
M, N, P . Chứng minh rằng AN, CP, EM đồng quy.

12 Đường tròn mixtilinear


12.1 Giới thiệu
Đường tròn mixtilinear nội tiếp (bàng tiếp) là đường tròn tiếp xúc với hai cạnh tam giác và tiếp
xúc trong (ngoài) với đường tròn ngoại tiếp tam giác đó. Đường tròn mixtilinear là một vấn đề khá
kinh điển trong hình học phẳng, nó bắt đầu được nghiên cứu bởi người Nhật Bản ngay từ thế kỷ
XVII, trong các bài toán được khắc trên những ngôi đền cổ (xem [1]). Cách định nghĩa rất đặc biệt
của đường tròn này tạo ra nhiều điều thú vị ẩn chứa bên trong. Trong bài viết này, chúng ta sẽ tìm

103
hiểu một số tính chất của đường tròn mixtilinear và đường tròn Thebault (đường tròn mixtilinear mở
rộng).

12.2 Tính chất


Trong mục này chủ yếu chúng ta sẽ tìm hiểu các tính chất liên quan đến đường tròn mixtilinear
nội tiếp. Thực tế hầu hết những tính chất đúng với nội tiếp thì cũng đúng với bàng tiếp.
Xét tam giác ABC nội tiếp đường tròn (O) và ngoại tiếp đường tròn (I). Kí hiệu ωa , ωb , ωc (với tâm
tương ứng là Oa , Ob , Oc ) lần lượt là các đường tròn mixtilinear nội tiếp ứng với góc A, B, C; X, Y, Z
lần lượt là tiếp điểm của ωa , ωb , ωc với (O); Ab , Ac lần lượt là tiếp điểm của ωa với AB, AC. Tương tự
ta xác định Ba , Bc , Ca , Cb ; D, E, F lần lượt là điểm chính giữa các cung BC, CA, AB.

Tính chất 40. (Bổ đề Sawayama-Thebault). I là trung điểm của Ab Ac .

A
E

F
Ac
I
O
Ab

Oa

B C

Chứng minh. Hiển nhiên E, F lần lượt là giao điểm của XAc , XAb với (O). Suy ra BE giao CF tại I.
Áp dụng định lý Pascal cho lục giác AF BXCE ta có (AB ∩ XF ), (AC ∩ XE), (BE ∩ CF ) thẳng
hàng hay Ab , I, Ac thẳng hàng.
Mặt khác tam giác AAb Ac cân tại A có AI là phân giác ∠Ab AAc nên I là trung điểm Ab Ac .

Nhận xét. Xem cách giải khác tại [2]. Ngoài ra tính chất 2.1 còn là hệ quả của tính chất 3.1.

Tính chất 41. XI đi qua điểm chính giữa cung BAC.

104
Ma
A

F
Ac
O
Ab I

Oa

B C

1
Chứng minh. Do XAb giao CI tại F nằm trên (O) nên ∠BXAb = ∠F CB = ∠ACB = ∠BIA−90◦ =
2
∠Ab IB.
Suy ra tứ giác Ab IXB nội tiếp. Tương tự tứ giác Ac IXC nội tiếp.
Ta thu được ∠BXI = ∠AAb I = ∠AAc I = ∠IXC, suy ra XI là phân giác ∠BXC hay XI đi qua
điểm chính giữa Ma của cung BAC.

Tính chất 42. Ab Ac , BC, XD, Ob Oc , Y Z đồng quy tại A1 .

Y
A

Ob
Z
Oc Ac
I
O
Ab
Oa

A1 B Cb Bc C

X
D

Chứng minh. Gọi A1 là giao của Ab Ac với BC.


A1 B Ac A Ab B Ab B
Áp dụng định lý Menelaus ta có = . = .
A1 C Ac C Ab A Ac C
BX BX AX BAb AAc BAb
Do XAb , XAc lần lượt là phân giác ∠BXA và ∠CXA nên = . = . = .
CX AX CX AAb CAc CAc
A1 B BAb BX
Từ đó = = , hay XA1 là phân giác ngoài ∠BXC, tức là XA1 đi qua D.
A1 C CAc CX
Mặt khác, dễ dàng nhận thấy ZCb giao Y Bc tại D và DCb .DZ = DB 2 = DBc .DY nên tứ giác
ZY Bc Cb nội tiếp.
Do A1 I ⊥ AI, Cb I ⊥ CI nên ∠A1 ICb = 180◦ − ∠AIC = 90◦ − ∠IBC = ∠IBc B.
Lại có A1 nằm trên trục đẳng phương của đường tròn (I, 0) và (O) (xem [3]) nên ta thu được
A1 B.A1 C = A1 I 2 = A1 Cb .A1 Bc . Suy ra A1 thuộc trục đẳng phương của (ZY Bc Cb ) và (O). Tức là
A1 ∈ Y Z.

105
Áp dụng định lý Monge-D’Alembert cho 3 đường tròn ωb , ωc , (O) suy ra A1 là tâm vị tự ngoài của
ωb và ωc , tức là A1 , Ob , Oc thẳng hàng.

Tính chất 43. Gọi A2 là tiếp điểm của (I) với BC. XA2 giao (O) tại L. Khi đó AL k BC.

Ma
A L

Ac
I
Ab O

Oa

A1 B A2 C

X
D

Chứng minh. Theo tính chất 2.2 ta có XI đi qua điểm chính giữa Ma của cung BAC nên ∠IXD = 90◦ .
Lại theo tính chất 2.3, DX, Ab Ac , BC đồng quy tại A1 nên tứ giác A1 IA2 X nội tiếp đường tròn
đường kính IA1 . Suy ra ∠LA2 C = ∠A1 IX = ∠A1 DX = ∠ALA2 . Từ đó AL k BC.

Tính chất 44. (AF Ab ) giao (AEAc ) tại K. Khi đó AEKF là hình bình hành.

F
K
I Ac
Ab O

Oa

B C

Chứng minh. Ta có (KA, KAb ) ≡ (F A, F Ab ) ≡ (F A, F X) ≡ (EA, EX) ≡ (EA, EAc ) ≡ (KA, KAc )
(mod π).
Suy ra K ∈ Ab Ac .
Từ đó ∠F AK = ∠XAb Ac = ∠AAc E = ∠AKE, suy ra AF k KE. Tương tự, AE k KF hay
AEKF là hình bình hành.

Tính chất 45. Đường tròn mixtilinear nội tiếp ứng với góc A của các tam giác ABX và ACX tiếp
xúc nhau.

Chứng minh. Cách 1 (Jean Louis Ayme). (xem [7])

106
A
E

F K
J
Ac
L
N
Ab I2 M

I1

B P C

Gọi I1 , I2 lần lượt là tâm đường tròn nội tiếp các tam giác ABX, ACX; M, N lần lượt là giao của
BI1 , CI2 với (O). Các đường thẳng qua I1 và vuông góc với AI1 , qua I2 và vuông góc với AI2 giao
nhau tại P . M F giao N E tại L, M F giao AI1 tại J, N E giao AI2 tại K.
Ta có F A = F I1 , M A = M I1 nên F M là trung trực đoạn thẳng AI1 , suy ra F M ⊥ AI1 hay
F M k I1 P , tương tự, EN k I2 P.
Mặt khác, gọi T là giao của AX với ωa .
Ta có XAb T Ac là tứ giác điều hòa và XF AE là ảnh của XAb T Ac qua phép vị tự tâm X nên
XF AE là tứ giác điều hòa.
F I1 FA EA EI2
Suy ra = = = . Từ đó EF k I1 I2 .
FX FX EX EX
Hai tam giác F LE và I1 P I2 có cạnh tương ứng song song nên F I1 , EI2 , LP giao nhau tại tâm vị
tự X của hai tam giác, hay X, P, L thẳng hàng.
Lại có JK là đường trung bình của tam giác AI1 I2 nên JK k I1 I2 . Hai tam giác JLK và I1 P I2 có
cạnh tương ứng song song nên I1 J, I2 K, LP giao nhau tại tâm vị tự A của hai tam giác, hay A, L, P
thẳng hàng.
Vậy A, P, X thẳng hàng, suy ra đpcm.
Cách 2.
Xét phép nghịch đảo cực A phương tích k bất kì IA k : B 7→ B 0 , C 7→ C 0 , (O) 7→ B 0 C 0 , BC 7→

(AB 0 C 0 ), ωa 7→ ωa0 .
Do ωa tiếp xúc trong với (O) và tiếp xúc với các cạnh AB, AC nên ωa0 là đường tròn bàng tiếp góc
A của tam giác AB 0 C 0 .
Bài toán được đưa về dạng: Cho tam giác ABC, N là tiếp điểm của đường tròn bàng tiếp góc A
với BC. Chứng minh rằng đường tròn bàng tiếp góc A của các tam giác AN B và AN C tiếp xúc nhau.

107
A

N C
B

Ta biết rằng AN là đường thẳng chia tam giác ABC thành hai tam giác có chu vi bằng nhau.
Gọi J, J 0 lần lượt là tiếp điểm của đường tròn bàng tiếp góc A của các tam giác AN B, AN C với
AN.
Kí hiệu p(XY Z) là nửa chu vi tam giác XY Z, ta có AJ = p(AN B) = p(AN C) = AJ 0 . Do đó
J ≡ J 0 , hay hai đường tròn bàng tiếp góc A của các tam giác AN B và AN C tiếp xúc nhau tại J. Ta
có đpcm.

Tính chất 46. IX giao BC tại M , AX giao Ab Ac tại N , khi đó M N k AI và (XN I) tiếp xúc với
(O).

Ma
A

Ac
N I
Ab

A1 B M A3 C

X
D

Chứng minh. Gọi A3 là trung điểm BC. Ta có tứ giác A1 IA3 D nội tiếp đường tròn đường kính A1 D.
Suy ra ∠AXI = ∠IDA3 = ∠IA1 A3 . Từ đó tứ giác A1 N M X nội tiếp.
Suy ra ∠A1 N M = ∠A1 XM = 90◦ . Vậy M N k AI.
Mặt khác, do AMa và N I cùng vuông góc với AI nên hai tam giác XN I và XAMa có tâm vị tự
X. Suy ra (XN I) tiếp xúc với (XAMa ).

Tính chất 47. AX, BY, CZ đồng quy tại tâm vị tự ngoài của (O) và (I).

Lời giải cho tính chất này bạn đọc xem tại [4].

Tính chất 48. (AIX) trực giao với (O).

108
Chứng minh. Ta có ∠OAD = ∠ODA = ∠AXI, do đó OA là tiếp tuyến của (AIX). Suy ra đpcm.

Tính chất 49. Các đường tròn (AIX), (BIY ), (CIZ) đồng trục.

Chứng minh. Theo tính chất 2.9, phương tích từ O đến 3 đường tròn này đều bằng R2 , lại có 3 đường
tròn cùng đi qua I nên OI là trục đẳng phương của 3 đường tròn.
Ta cũng có thể chứng minh dựa vào tính chất 2.8, dễ dàng suy ra tâm vị tự ngoài của (O) và (I)
cũng có cùng phương tích với 3 đường tròn.

Tính chất 50. Gọi A0 , B 0 , C 0 lần lượt là giao của AI, BI, CI với BC, CA, AB. Khi đó các đường tròn
(AXA0 ), (BXB 0 ), (CXC 0 ) đồng trục.

A''

X'

Ib
Ma
A
Ic

O'a I
O'
B A' C
X
J

Ia

Chứng minh. Gọi Ia , Ib , Ic là 3 tâm bàng tiếp của tam giác ABC. Khi đó I là trực tâm tam giác Ia Ib Ic .
Xét phép nghịch đảo cực I phương tích k = IA.IIa :
A 7→ Ia , X 7→ X 0 , A0 7→ A00 . Tương tự xác định Y 0 , Z 0 , B 00 , C 00 .
Gọi J là trung điểm IIa . Ta đã chứng minh được XX 0 đi qua điểm chính giữa Ma của cung BAC
và phép vị tự tâm I tỉ số 2 biến đường tròn (O) thành (Ia Ib Ic ).
Suy ra IX.IX 0 = IA.IIa = 2IA.IJ = IX.2IMa .
Suy ra Ma là trung điểm IX 0 , tức là X 0 ∈ (Ia Ib Ic ). Do X 0 đối xứng với trực tâm của tam giác
Ia Ib Ic qua trung điểm Ib Ic nên X 0 đối xứng với Ia qua tâm ngoại tiếp O0 của tam giác Ia Ib Ic .
Qua phép nghịch đảo IIk : B 7→ Ib , C 7→ Ic nên BC 7→ (IIb Ic ). Suy ra A00 ∈ (IIb Ic ), từ đó A00 là
điểm đối xứng với Ia qua Ib Ic .
Như vậy tâm của (Ia X 0 A00 ) là giao của đường thẳng qua O0 và vuông góc với Ia O0 với Ib Ic .
Ta chú ý tới kết quả sau: Cho tam giác ABC và một điểm X bất kì. Đường thẳng qua X và vuông
góc với AX giao BC tại A0 . Tương tự xác định B 0 , C 0 . Khi đó A0 , B 0 , C 0 thẳng hàng. (lời giải xem tại
[3]).
Áp dụng vào bài toán này cho tam giác Ia Ib Ic và điểm O0 suy ra tâm của các đường tròn
(Ia X 0 A00 ), (Ib Y 0 B 00 ), (Ic Z 0 C 00 ) thẳng hàng.
Mặt khác hiển nhiên phương tích từ O0 đến 3 đường tròn này bằng nhau. Do đó (Ia X 0 A00 ), (Ib Y 0 B 00 ),
(Ic Z 0 C 00 ) đồng trục, suy ra (AXA0 ), (BXB 0 ), (CXC 0 ) đồng trục.

Tính chất 51. Gọi l là tiếp tuyến của ωa sao cho l nằm trên nửa mặt phẳng bờ BC chứa A đồng
thời l k BC. Gọi P là tiếp điểm của l với ωa . Khi đó AX, AP là hai đường đẳng giác trong góc BAC.

109
Ma
A

P
Q

I O

B C

Chứng minh. Gọi Q là giao điểm thứ hai của AX với ωa .


Do XI giao (O) tại điểm chính giữa Ma của cung BAC và tiếp tuyến của (O) tại Ma song song
với BC nên P là ảnh của Ma qua phép vị tự tâm X. Từ đó X, I, P thẳng hàng.
Ta có IA ⊥ AMa , P Q là ảnh của AMa qua phép vị tự tâm X nên P Q k AMa , suy ra P Q ⊥ AI.
Như vậy P và Q đối xứng nhau qua AI, hay AX và AP là hai đường đẳng giác trong ∠BAC.

Tính chất 52. XOa , XOb là hai đường đẳng giác trong góc BXC.

Ob
Oc T
I

A1

B M C

Chứng minh. Gọi M là giao của IX với BC, T là giao của IX với Ob Oc .
Ta có XA1 và XM lần lượt là phân giác ngoài và trong góc BXC nên (BCM A1 ) = −1.
Suy ra I(BCM A1 ) = −1, hay I(Ob Oc T A1 ) = −1, nghĩa là (Ob Oc T A1 ) = −1.
Vậy X(Ob Oc T A1 ) = −1. Ta lại có ∠IXA1 = 90◦ nên XI là phân giác ∠Ob XOc .
Theo tính chất 2.2, XI là phân giác ∠BXC nên ta có đpcm.

Nhận xét. Tính chất 2.13 có thể tổng quát: Cho AB là một dây cung bất kì của đường tròn (O).
Hai đường tròn (X), (Y ) nằm cùng phía với (O) sao cho chúng cùng tiếp xúc trong với (O) và tiếp xúc
với AB lần lượt tại C, D. H là giao điểm của XY và AB, M là điểm chính giữa cung AB không chứa
(X), (Y ). HM cắt (O) lần thứ hai tại I, khi đó IX, IY là hai đường đẳng giác trong góc AIB.
Lời giải cho bài toán tổng quát, xem [6].

Tính chất 53. Ba Ab Cb Bc Ac Ca là lục giác ngoại tiếp đường tròn (I).

110
A

Ca
Ba

Ac

Ab

B Cb Bc C

Chứng minh. Kẻ tiếp tuyến Ab J tới (I). Ta có ∠JAb Ac = ∠AAb Ac = ∠AAc Ab nên Ab J k AC.
Như vậy tiếp tuyến kẻ từ Ab tới (I) song song với AC, tương tự suy ra Ab Cb là tiếp tuyến song
song với AC của (I).
Chứng minh tương tự ta có đpcm.

Tính chất 54. Đường tròn (A, AAb ) giao (O) tại hai điểm R, S. Khi đó RS là tiếp tuyến của ωa .

Chứng minh. Cách 1.

A
S

Ac
R L
O
Ab

B
C

Gọi Q là giao của AX với ωa , K, L lần lượt là giao của AX với (A, AAb ).
Do hai đường tròn (A, AAb ) và ωa trực giao nên (KLQX) = −1. Mà A là trung điểm KL nên theo
định lý Maclaurin, QK.QL = QA.QX, hay Q thuộc trục đẳng phương của (A, AAb ) và (O). Nghĩa là
Q ∈ RS.
Ta có X là tâm vị tự của hai đường tròn ωa và (O) nên tiếp tuyến tại A của (O) và tiếp tuyến tại
Q của ωa song song với nhau. Mà tiếp tuyến tại A của (O) và RS cùng vuông góc với OA nên RS là
tiếp tuyến của ωa .
Cách 2.
Xét phép nghịch đảo cực A phương tích bất kì, ta chuyển được bài toán về dạng mới như sau:
Cho tam giác ABC, đường tròn ω bàng tiếp góc A tiếp xúc với AB, AC lần lượt tại F, E. Đường
tròn (A, AE) giao BC tại hai điểm M, N . Khi đó (AM N ) tiếp xúc với ω.

111
A

M B D C N

Chứng minh.
Gọi D là tiếp điểm của ω với BC. Áp dụng định lý Casey cho 4 đường tròn (A, 0), (M, 0), (N, 0), ω
ta có (AM N ) tiếp xúc với ω khi và chỉ khi AM.N D + AN.M D = AF.M N (xem [2]).
Nhưng AM = AN = AF nên điều này tương đương N D + M D = M N , hiển nhiên đúng. Ta có
đpcm.

Tính chất 55. Gọi M, N là giao điểm của (Ia ) và ωa . Đường tròn (A, AM ) giao (O) tại U, V . Khi đó
U V là tiếp tuyến chung của (I) và (Ia ).

Ab
I Oa C' V
B C
B' U M N

Ia

Chứng minh. Gọi H là tiếp điểm của (Ia ) với AB. Xét phép nghịch đảo cực A phương tích k =
AAb .AH :
B 7→ B 0 , C 7→ C 0 , (Oa ) 7→ (Ia ), (Ia ) 7→ (Oa ), (O) 7→ B 0 C 0 , BC 7→ (AB 0 C 0 ).
Gọi L là giao điểm thứ hai của AM với (Ia ).
Ta có ∠Ab M A = ∠HKA = ∠AHM , suy ra AN 2 = AM 2 = AAb .AH.
Như vậy đường tròn (A, AM ) không thay đổi qua phép nghịch đảo IA k.

112
Do U, V là giao điểm của (A, AM ) với (O) nên U, V là hai điểm không thay đổi qua phép nghịch
đảo IAk , tức là U, V ∈ B 0 C 0 .

Ta có (Oa ) tiếp xúc với (O) nên (Ia ) tiếp xúc với B 0 C 0 , tức (Ia ) tiếp xúc với U V.
AI rp 2SABC
Mặt khác, AAb .AH = .p = = = bc = AB.AB 0 = AC.AC 0 .
cos A/2 sin A/2. cos A/2 sin A
Suy ra AB 0 = AC, AC 0 = AB. Mà BC là tiếp tuyến chung của (I) và (Ia ) nên qua phép đối xứng
trục, B 0 C 0 là tiếp tuyến chung thứ hai của hai đường tròn này.

Nhận xét.
1. Từ tính chất trên suy ra ảnh của (I) qua phép nghịch đảo IA k chính là đường tròn mixtilinear

bàng tiếp.
2. Một cách tương tự ta cũng có kết quả sau:
Gọi M 0 , N 0 là giao điểm của (I) và ωa . Đường tròn (A, AM 0 ) giao (O) tại U 0 , V 0 . Khi đó U 0 V 0 là
tiếp tuyến của (I). Ngoài ra có thể chứng minh I nằm trên (A, AM 0 ).

Tính chất 56. Gọi A2 là tiếp điểm của (I) với BC, A4 là trung điểm IA2 . Khi đó A4 D là trục đẳng
phương của ωb và ωc .

Ca
Ba B2

C2
I

A4

B Cb A2 Bc C

Chứng minh. Theo lời giải tính chất 2.3 chúng ta đã chứng minh được D nằm trên trục đẳng phương
của ωb và ωc .
Gọi B2 , C2 lần lượt là tiếp điểm của (I) với AC, AB. Xét hai đường tròn (I) và ωb có C2 Ba và A2 Bc
là hai tiếp tuyến chung ngoài. Do đó đường thẳng nối trung điểm C2 Ba và A2 Bc hay đường trung bình
l1 của hình thang C2 Ba Bc A2 chính là trục đẳng phương của (I) và ωb . Hiển nhiên l1 đi qua A4 .
Tương tự, đường trung bình l2 của hình thang Ca B2 A2 Cb là trục đẳng phương của (I) và ωc . Ta
chứng minh được l1 và l2 giao nhau tại A4 .
Như vậy A4 chính là tâm đẳng phương của 3 đường tròn (I), ωb , ωc . Từ đó A4 D là trục đẳng phương
của ωb và ωc .

Tính chất 57. Tâm đẳng phương J của ωa , ωb , ωc nằm trên OI và chia OI theo tỉ số OJ : JI = 2R :
−r.

Chứng minh. Gọi B4 , C4 lần lượt là trung điểm IB2 , IC2 .


Dễ dàng chứng minh hai tam giác DEF và A4 B4 C4 có cạnh tương ứng song song nên DA4 , EB4 , F C4
đồng quy tại tâm vị tự J của hai tam giác, tâm vị tự này cũng chính là tâm vị tự của đường tròn ngoại
tiếp hai tam giác DEF và A4 B4 C4 . Chú ý rằng bán kính đường tròn ngoại tiếp tam giác A4 B4 C4 bằng
r
, vậy OJ : JI = 2R : −r.
2

113
Tính chất 58. Tâm đẳng phương của 3 đường tròn mixtilinear ngoại tiếp và tâm đẳng phương của
3 đường tròn mixtilinear nội tiếp đối xứng nhau qua O.

A6

A5
Ib
Ma
A
Ic B2
A7
C2 I
B7 Mc
C
A2 7 C
B Mb C5
A8
B5 C6

Ia

B6

Chứng minh. Gọi Ma , Mb , Mc lần lượt là điểm chính giữa các cung BAC, ABC, ACB. Một cách tương
tự ta cũng chứng minh được Ma , Mb , Mc lần lượt nằm trên trục đẳng phương của từng cặp các đường
tròn mixtilinear ngoại tiếp (kí hiệu là ωa0 , ωb0 , ωc0 ).
Gọi Ia , Ib , Ic là tâm 3 đường tròn bàng tiếp của tam giác ABC. A0b , A0c là tiếp điểm của ωa0 với
AB, AC. Tương tự ta xác định Ba0 , Bc0 , Ca0 , Cb0 .
Trục đẳng phương của (I) và ωc0 là đường trung bình của hình thang A2 B2 Ca Cb , trục đẳng phương
của (I) và ωb0 là đường trung bình của hình thang A2 C2 Ba Bc . Hai đường trung bình này giao nhau tại
tâm đẳng phương A5 của 3 đường tròn (I), ωb0 , ωc0 .
Tương tự ta xác định được B5 , C5 .
Gọi A6 B6 C6 là tam giác tạo bởi giao điểm của các đường thẳng A0b A0c , Ba0 Bc0 , Ca0 Cb0 ; A7 B7 C7 là tam
giác trung tuyến của tam giác A2 B2 C2 .
Do I là trực tâm của tam giác Ia Ib Ic nên dễ dàng thu được I là tâm đường tròn ngoại tiếp tam
giác A6 B6 C6 .
Ta có (O) là đường tròn Euler của tam giác Ia Ib Ic , (Ia Ib Ic ) là đường tròn Euler của tam giác
A6 B6 C6 nên R(A6 B6 C6 ) = 4R.
Hai tam giác A6 B6 C6 và A2 B2 C2 có chung tâm ngoại tiếp I và có cạnh tương ứng song song nên
I là tâm vị tự trong của hai tam giác đó. Lại có A7 , Ia lần lượt là trung điểm B2 C2 và A6 B6 nên ta
IA7 R(A2 B2 C2 ) r
thu được = = .
IIa R(A6 B6 C6 ) 4R
Gọi A8 là trung điểm Ia A7, tương tự có B8 , C8 .
IA8 A7 A8 − A7 I A7 I a A7 I 4R + r r 4R − r
Suy ra = = − = − = .
IIa IIa 2IIa IIa 8R 4R 8R
Do hai tam giác A8 B8 C8 và Ia Ib Ic có tâm vị tự I nên hai tam giác A5 B5 C5 và A6 B6 C6 cũng có
IA8 4R − r r
tâm vị tự I. Từ đó R(A5 B5 C5 ) = R(A6 B6 C6 ). = 4R. = 2R − .
IIa 8R 2
Dễ dàng chứng minh được hai tam giác Ma Mb Mc và A5 B5 C5 có cạnh tương ứng song song nên
Ma A5 , Mb B5 , Mc C5 đồng quy tại tâm vị tự J 0 của hai tam giác, J 0 nằm trên đường nối hai tâm ngoại

114
r
tiếp I, O đồng thời chia IO theo tỉ số J 0 I : J 0 O = (2R − ) : R. Ta cũng có J 0 chính là tâm đăng
2
phương của ωa0 , ωb0 , ωc0 .
Kết hợp với tính chất 2.18 suy ra J và J 0 đối xứng nhau qua O.

Tính chất 59. Tỉ số giữa bán kính của ωa và ωa0 là α, tương tự có β, γ. Khi đó α + β + γ = 1.

Chứng minh. Theo tính chất 2.1 thì I là trung điểm Ab Ac , Ia là trung điểm A0b , A0c . Từ đó α chính là
tỉ số giữa bán kính đường tròn nội tiếp và đường tròn bàng tiếp góc A. Bài toán đưa về việc chứng
AI BI CI SI II + SIa IIc + SIa IIb
minh + + = 1, hay b c = 1, hiển nhiên đúng.
AIa BIb CIc SIa Ib Ic

Tính chất 60. (Cosmin Pohoata). P là điểm bất kì chuyển động trên (O). Gọi R, S là giao điểm của
hai tiếp tuyến kẻ từ P tới (I) với BC. Khi đó (P RS) luôn đi qua X.

N
A
K

I
M

L
R S
B
C

X
P

Chứng minh. Gọi X 0 là giao của (P RS) với (O). Dựa theo tính chất 2.2 ta chỉ cần chứng minh X 0 I là
phân giác ∠BX 0 C thì sẽ có X 0 ≡ X.
Gọi M, N là giao của P R, P S với (O), L, K lần lượt là giao của P I với (P RS), (O).
Xét hai tam giác M BX 0 và CSX 0 có ∠BM X 0 = ∠SCX 0 , ∠M BX 0 = 180◦ − ∠X 0 P R = 180◦ −
∠RSX 0 = ∠X 0 SC.
Suy ra ∠BX 0 M = ∠SX 0 C.
Lại có ∠M X 0 K = ∠M P K = ∠LP S = ∠LX 0 S.
X 0K KI
Do đó X 0 I là phân giác ∠BX 0 C khi và chỉ khi X 0 I là phân giác ∠KX 0 L, tương đương 0 = .
XL LI
(1)
Do I là tâm bàng tiếp của tam giác P RS và L là điểm chính giữa cung RS nên LI = LR.
Theo định lý Poncelet (xem [5]), M N tiếp xúc với (I), suy ra K là tâm ngoại tiếp tam giác M IN
hay KI = KM.
KI KM
Như vậy = .
LI LR
Xét hai tam giác KM X 0 và LRX 0 có ∠M X 0 K = ∠RP L = ∠RX 0 L, ∠M KX 0 = ∠M P X 0 =
∠RLX 0 . Do đó 4KM X 0 ∼ 4LRX 0 .
KM KX 0
Suy ra = , nghĩa là (1) đúng. Ta có đpcm.
LR LX 0

115
Nhận xét. Bài toán trên có thể được mở rộng như sau: (Vladimir Zajic). Cho tam giác ABC nội
tiếp đường tròn (O), ngoại tiếp đường tròn (I). P là một điểm bất kì nằm ngoài (I). Các tiếp tuyến kẻ
từ P tới (I) giao BC tại E, F . P I giao (O) tại P 0 . Khi đó (P 0 EF ) luôn đi qua tiếp điểm của đường
tròn A-mixtilinear nội tiếp với (O).

Tính chất 61. (Iran 1997). P là điểm bất kì chuyển động trên cung BC không chứa A. Gọi I1 , I2 lần
lượt là tâm đường tròn nội tiếp các tam giác AP B, AP C. Khi đó (I1 I2 P ) luôn đi qua X.

A
E

I1
I2
B C

X
P

Chứng minh. Hiển nhiên P I1 , P I2 lần lượt cắt (O) tại F, E và F, E lần lượt là tâm đường tròn ngoại
tiếp các tam giác ABI1 , ACI2 . Gọi X 0 là giao của (I1 I2 P ) với (O).
Ta có ∠X 0 F I1 = ∠X 0 EI2 , ∠F I1 X 0 = ∠EI2 X 0 , do đó 4F I1 X 0 ∼ 4EI2 X 0 .
AF F I1 X 0F
Suy ra = = 0 . Từ đó tứ giác AF X 0 E điều hòa, theo phép chứng minh tính chất 2.6
AE EI2 XE
ta có tứ giác AF XE điều hòa, suy ra X 0 ≡ X.

12.3 Đường tròn Thebault


Đường tròn Thebault được định nghĩa như sau: Cho tam giác ABC nội tiếp đường tròn (O). Một
đường thẳng l qua A cắt cạnh BC tại D. Đường tròn tiếp xúc với các tia DA, DC và tiếp xúc trong
với (O) gọi là đường tròn Thebault nội tiếp của tam giác ABC ứng với đường thẳng l và đỉnh C. Một
cách tương tự ta cũng định nghĩa được các đường tròn Thebault nội tiếp còn lại và các đường tròn
Thebault bàng tiếp.

D C
B

116
Đường tròn Thebault thực chất là dạng mở rộng của đường tròn mixtilinear. Rất nhiều tính chất
trong mục 2 có thể mở rộng cho đường tròn Thebault, và bởi tính đa dạng và phong phú của chúng
nên thật khó để tìm được điểm dừng. Trong mục này chúng ta chỉ tìm hiểu một số tính chất tiêu biểu
của đường tròn Thebault, một số mở rộng khác xin được nhường lại cho bạn đọc tự khám phá.

Tính chất 62. (Bổ đề Sawayama). Cho tam giác ABC nội tiếp đường tròn (O), ngoại tiếp đường
tròn (I). D là một điểm bất kì trên cạnh BC. Gọi (O0 ) là đường tròn Thebault của tam giác ABC
ứng với AD và đỉnh C. (O0 ) tiếp xúc với các đoạn thẳng CD, AD lần lượt tại E, F . Khi đó I, E, F
thẳng hàng.

O'

E J

N O

C
B
D F

Chứng minh. Gọi K là tiếp điểm của (O0 ) với (O). KE, KF lần lượt cắt (O) lần thứ hai tại N, M .
AM cắt EF tại J. Khi đó M là điểm chính giữa cung BC.
Do K là tâm vị tự ngoài của (O) và (O0 ) nên hiển nhiên EF k M N.
Do đó ∠AKE = ∠AM N = ∠AJE, suy ra tứ giác AKJE nội tiếp.
Mà (O0 ) tiếp xúc với AD nên ∠EF K = ∠AEK = ∠AJK, nghĩa là đường tròn ngoại tiếp tam giác
JKF tiếp xúc với AM .
Suy ra M J 2 = M F.M K. Mặt khác, ∠M KC = ∠BCM nên M C 2 = M F.M K, ta thu được
M J 2 = M C 2 hay M B = M C = M J.
Lại có M là tâm đường tròn ngoại tiếp tam giác BIC, suy ra J ≡ I hay I ∈ EF.

Nhận xét. Khi D ≡ B ta thu được tính chất 2.1.

Tính chất 63. (Định lý Sawayama-Thebault). Cho tam giác ABC nội tiếp đường tròn (O), ngoại
tiếp đường tròn (I). D là một điểm bất kì trên cạnh BC. Gọi (O1 ), (O2 ) lần lượt là các đường tròn
Thebault của tam giác ABC ứng với đường thẳng AD và các đỉnh B, C. Khi đó I, O1 , O2 thẳng hàng.

117
A

O2
F
I
O1
H

L
B G D E C

Chứng minh. Gọi L là giao điểm của O1 O2 và BC; G, H là tiếp điểm của (O1 ) với DB, DA; E, F là
tiếp điểm của (O2 ) với DC, DA.
Theo tính chất 3.1, I là giao điểm của EF và GH.
Gọi I 0 là giao của GH và O1 O2 . Do GH và DO2 cùng vuông góc với DO1 nên GH k DO2 .
LI 0 LG
Suy ra = . (1)
LO2 LD
LO1 LG
Mặt khác, O1 G k O2 E nên = . (2)
LO2 LE
LI 0 LE
Chia theo vế của (1) cho (2) ta thu được = hay O1 D k EI 0 .
LO1 LD
Mà EI và O1 D cùng vuông góc với O2 D nên EI k O1 D. Từ đó I 0 ≡ I hay O1 , I, O2 thẳng hàng.
Bài toán được chứng minh.

Nhận xét. Từ tính chất trên suy ra (O1 ), (I), (O2 ) có một tiếp tuyến chung khác BC (IMO
Shortlist 1969).
Tính chất 64. Nếu D là chân phân giác góc A thì (O1 ) tiếp xúc với (O2 ).
A

O2
I
O1

D
B
C

Chứng minh. Gọi G là điểm chính giữa cung BC không chứa A.


Chúng ta biết rằng phương tích từ G đến họ các đường tròn ω tiếp xúc trong với (O) và tiếp xúc
với BC sao cho ω và G nằm khác phía so với BC đều bằng GB 2 = GI 2 . Do (O1 ) tiếp xúc với GI nên
I chính là tiếp điểm của (O1 ) với AG. Tương tự suy ra (O1 ) và (O2 ) tiếp xúc nhau tại I.

118
Tính chất 65. (Jean-Pierre Ehrmann và Cosmin Pohoata). Cho tam giác ABC, P là một điểm nằm
trong tam giác. Gọi TAb , TAc là hai đường tròn Thebault của tam giác ABC ứng với đường thẳng AP .
Tương tự có TBc , TBa , TCa , TCb . Khi đó 6 đường tròn TAb , TAc , TBc , TBa , TCa , TCb bằng nhau khi và chỉ khi
P là điểm Nagel của tam giác ABC.

O1 O2
I

D E C
B G L

Chứng minh. Gọi D là giao của AP với BC. Trước tiên ta giả sử rằng R(TAb ) = R(TAc ) (tương ứng với
hai đường tròn (O1 ) và (O2 )). Hiển nhiên khi đó bán kính của 2 đường tròn bằng r.
Gọi G, H là tiếp điểm của (O1 ) với DB, DA; E, F là tiếp điểm của (O2 ) với DC, DA. L là tiếp
điểm của (I) với BC.
Do R(O1 ) = R(O2 ) nên BO1 O2 C là hình thang cân, suy ra BG = CE.
O1 ILG là hình chữ nhật, O1 IED là hình bình hành nên GL = O1 I = DE.
Vậy BL = BG + GL = CE + DE = CD. Suy ra D là tiếp điểm của đường tròn bàng tiếp góc A
với BC.
Ngược lại ta luôn dựng được đường tròn (O1 ) tiếp xúc trong với (O), BC và có bán kính bằng r.
Khi đó kẻ tiếp tuyến từ A đến (O1 ) cắt BC tại D. Dựng đường tròn Thebault (O2 ) của tam giác ABC
ứng với AD và đỉnh C. Suy ra bán kính của (O2 ) cũng bằng r, tức là R(O1 ) = R(O2 ) và D là tiếp
điểm của đường tròn bàng tiếp góc A với BC.
Như vậy R(O1 ) = R(O2 ) = r khi và chỉ khi D là tiếp điểm của đường tròn bàng tiếp góc A với
BC. Xét tương tự với các đường tròn còn lại ta có đpcm.

Tính chất 66. AD giao (O) lần thứ hai tại E. Khi đó bán kính của hai đường tròn Thebault của
tam giác ABC ứng với AD bằng nhau khi và chỉ khi bán kính đường tròn nội tiếp hai tam giác EDB
và EDC bằng nhau.

119
A

D
B C

Chứng minh. Theo tính chất 3.4 thì bán kính của hai đường tròn Thebault của tam giác ABC ứng
với AD bằng nhau khi và chỉ khi D là tiếp điểm của đường tròn bàng tiếp góc A với BC.
SEDB SEDC p(EDB) DB
Ta có rEDB = rEDC khi và chỉ khi = hay = . (1)
p(EDB) p(EDC) p(EDC) DC
p(CDE) CD p(BDE) BD
Do hai tam giác ADB và CDE đồng dạng nên = , tương tự = .
p(ADB) AD p(ADC) AD
p(EDB) p(ADC) BD
Như vậy = . . Suy ra (1) tương đương p(ADC) = p(ADB) hay D là tiếp
p(EDC) p(ADB) CD
điểm của đường tròn bàng tiếp góc A với BC. Ta có đpcm.

Nhận xét. Có thể chứng minh tính chất 3.5 dựa vào bài toán sau:
(Vladimir Zajic). Cho tam giác ABC nội tiếp đường tròn (O). P là điểm bất kì nằm trên cạnh
BC, AP cắt (O) lần thứ hai tại D. Gọi (I1 ), (I2 ) lần lượt là đường tròn nội tiếp tam giác AP B, AP C;
(J1 ), (J2 ) lần lượt là đường tròn tiếp xúc với các cặp tia (P B, P D), (P C, P D) và cùng tiếp xúc trong
với (O). Khi đó I1 I2 , J1 J2 , BC đồng quy.
Lời giải cho bài toán này bạn đọc xem tại [4].

Tính chất 67. Gọi G, E lần lượt là tiếp điểm của (O1 ) và (O2 ) với BC, AD giao (O) lần thứ hai tại
L, X là tiếp điểm của đường tròn A-mixtilinear nội tiếp với (O). Khi đó G, E, L, X cùng thuộc một
đường tròn.

120
A S

O2
K
O1 I O
R

Q G E
P B A1 D C

A'
L

Chứng minh. Gọi A0 là giao của AI với (O), A0 X giao BC tại P, A1 là tiếp điểm của (I) với BC. Ta
đã chứng minh được P I ⊥ AI, P A0 ⊥ IX do đó ∠P A1 X = ∠P IX = ∠P A0 I = ∠XLA. Suy ra tứ giác
XA1 DL nội tiếp.
Gọi R, S lần lượt là tiếp điểm của (O1 ), (O2 ) với (O). RS giao BC tại Q. Theo định lý Monge-
D’Alembert ta có Q là tâm vị tự ngoài của (O1 ) và (O2 ), cũng chính là cực của phép nghịch đảo của
hai đường tròn. Suy ra tứ giác RSEG nội tiếp.
Gọi K là hình chiếu của D trên O1 O2 .
Ta có ∠O1 IG = ∠KO2 D = ∠KED, suy ra tứ giác GIKE nội tiếp. Đồng thời ta cũng có tứ giác
IKDA1 nội tiếp đường tròn đường kính ID. Do đó QR.QS = QA1 .QE = QI.QK = QA1 .QD.
Vậy tứ giác RSDA1 nội tiếp. Áp dụng định lý về tâm đẳng phương cho 3 đường tròn (O), (RSEA1 ),
(XA1 DL) suy ra LX, BC, RS đồng quy tại Q.
Suy ra QG.QE = QR.QS = QX.QL, tức là G, E, L, X cùng thuộc một đường tròn.
Tính chất 68. Gọi A0 là điểm chính giữa cung BC không chứa A, J là trung điểm ID. Khi đó A0 J
là trục đẳng phương của (O1 ) và (O2 ).
A

O2
O1
I

B D C

A'

Chứng minh. Việc chứng minh A0 thuộc trục đẳng phương của hai đường tròn này khá đơn giản. Do
đó chúng ta chỉ quan tâm đến điểm J.

121
Do I nằm trên đường thẳng nối tiếp điểm của (O1 ) với AD, BD và J là trung điểm ID nên J
thuộc trục đẳng phương của (O1 ) và (D, 0) (xem [3]). Tương tự, J thuộc trục đẳng phương của (O2 )
và (D, 0). Theo định lý về tâm đẳng phương suy ra J thuộc trục đẳng phương của (O1 ) và (O2 ). Ta
có đpcm.

Nhận xét. Từ lời giải trên suy ra đường tròn đường kính AD trực giao với (O1 ) và (O2 ).

12.4 Ứng dụng của bổ đề Sawayama-Thebault


Bài 215. (IMO 2012 Turkey Preparation). Cho tam giác ABC nội tiếp đường tròn (O), các đường cao
BB 0 , CC 0 . Tiếp tuyến của (O) tại B, C giao nhau tại X. B 0 C 0 giao XB, XC lần lượt tại Y, Z. Chứng
minh rằng (XY Z) tiếp xúc với (O).

A
Z

B'

C'
Y

B I C

Chứng minh. Gọi I là trung điểm BC. Do I là tâm của đường tròn ngoại tiếp tứ giác BC 0 B 0 C nên
IB 0 = IC.
Lại có ∠B 0 CZ = ∠ABC = ∠ZB 0 C nên ZB 0 = ZC. Suy ra ZI là phân giác ∠Y ZX.
Hiển nhiên XI là phân giác ∠Y XZ, do đó I là tâm nội tiếp tam giác XY Z, mà X là trung điểm
BC nên theo bổ đề Sawayama, (ABC) là đường tròn mixtilinear nội tiếp của tam giác XY Z, tức là
(O) tiếp xúc với (XY Z).

Bài 216. (All-Russian MO 2013). Cho tam giác ABC nội tiếp đường tròn (O), ngoại tiếp đường tròn
(I). Gọi X, Y là giao điểm của (BIC) và (I), Z là tâm vị tự ngoài của (I) và (BIC). Chứng minh rằng
(XY Z) tiếp xúc với (O).

122
M

Z
O
K H
Y
I
L
N X
T
B C

Chứng minh. Gọi F là giao của AI với (O), T là tâm vị tự trong của (O) và (I), K là tâm của (XY Z).
ZI TI r YI XI
Ta có = = = = nên X, Y, Z, T cùng nằm trên đường tròn Apollonius của
ZF TF R YF XF
r
đoạn thẳng EF ứng với tỉ số , nói cách khác là đường tròn Apollonius của tam giác IY F.
R
Tâm K của (XY ZT ) là giao của tiếp tuyến tại Y của (IY F ) với IF . Suy ra ∠KY I = ∠IF Y.
Ta thu được ∠KY F + ∠IY F = ∠IF Y + 2∠IY F = 180◦ , suy ra Y F là phân giác ngoài ∠KY I.
FI IY
Nghĩa là = hay F là tâm vị tự ngoài của (I) và (XY Z).
FK KI
Kẻ tiếp tuyến F N, F H tới (XY Z), F N, F H lần lượt cắt (O) tại L, M.
Do (ZT IF ) = −1 nên theo hệ thức Newton ta có KN 2 = KT 2 = KI.KF , suy ra I là hình chiếu
của N trên KF , nghĩa là I là trung điểm N H.
Theo định lý Poncelet, (I) là đường tròn nội tiếp tam giác LF M nên áp dụng bổ đề Sawayama suy
ra (XY Z) là đường tròn mixtilinear nội tiếp của tam giác F LM , hay (XY Z) tiếp xúc với (O).

Bài 217. (Sharygin Geometry Olympiad 2009). Cho tứ giác nội tiếp ABCD. Biết rằng 4 đường tròn
tiếp xúc với các đường chéo và tiếp xúc trong với đường tròn ngoại tiếp tứ giác ABCD có bán kính
bằng nhau. Chứng minh rằng ABCD là hình vuông.

A B
Oa
Ic Id

Od Ob

Ib Ia

Oc
D C

Chứng minh. Kí hiệu Ia , Ib , Ic , Id lần lượt là tâm nội tiếp các tam giác BCD, ACD, ABD, ABC, hai
đường chéo giao nhau tại I, (Oa ), (Ob ), (Oc ), (Od ) lần lượt là 4 đường tròn tiếp xúc với các cặp tia

123
IA, IB; IB, IC; IC, ID; ID, IA và cùng tiếp xúc trong với (ABCD). Áp dụng định lý Sawayama-
Thebault suy ra Ia ∈ Ob Oc . Do R(Oa ) = R(Ob ) = R(Oc ) = R(Od ) nên R(Ia ) = R(Ib ) = R(Ic ) = R(Id ).
Từ đó hai tứ giác Ia Ib Ic Id và ABCD có cạnh tương ứng song song.
Ta đã biết Ia Ib Ic Id là hình chữ nhật, do đó ABCD là hình chữ nhật.
Mặt khác, áp dụng tính chất 3. cho tam giác ABC với hai đường tròn Thebault (Oa ) và (Ob ) suy
ra I là tiếp điểm của đường tròn bàng tiếp góc B với AC. Mà I là trung điểm AC nên tam giác ABC
cân tại B. Suy ra ABCD là hình vuông.

Bài 218. (CGMO 2013). Cho hai đường tròn (O1 ) và (O2 ) tiếp xúc ngoài nhau tại T . Tứ giác ABCD
nội tiếp (O1 ) sao cho AD, BC tiếp xúc với (O2 ) lần lượt tại E, F . Phân giác ∠ABF giao EF tại N, F T
giao (O1 ) lần thứ hai tại M. Chứng minh rằng M là tâm ngoại tiếp tam giác BCN.

B O1 O2
B'
T

A
M
D
N
C'
E

Chứng minh. Gọi B 0 , C 0 lần lượt là giao của BT, CT với (O2 ). Do T là tâm vị tự của (O1 ) và (O2 ) suy
ra BC k B 0 C 0 .
Từ đó F là điểm chính giữa cung B 0 C 0 . Phép vị tự tâm T biến B 0 7→ B, C 0 7→ C, F 7→ M nên M
là điểm chính giữa cung BC.
Do (O2 ) là đường tròn Thebault bàng tiếp của tam giác ABC ứng với đường thẳng AD và N là
giao của phân giác ∠ABC với EF nên theo bổ đề Sawayama-Thebault, N là tâm bàng tiếp góc B của
tam giác ABC.
Ta đã biết điểm chính giữa cung BAC tâm đường tròn ngoại tiếp tam giác ABN , suy ra đpcm.

Bài 219. Cho tam giác nhọn ABC nội tiếp đường tròn (O), trực tâm H. Đường tròn ω có tâm E tiếp
xúc với các đoạn thẳng HB, HC và tiếp xúc với (O). Chứng minh rằng trung điểm HE là tâm đường
tròn nội tiếp tam giác BHC.

H
J F
G
I

B
E C

124
Chứng minh. Gọi D là giao của CH với (O). Suy ra D đối xứng với H qua AB.
Gọi F, G lần lượt là tiếp điểm của ω với HB, HC; I, J lần lượt là tâm đường tròn nội tiếp các tam
giác BHC, BDC.
Theo bổ đề Sawayama-Thebault suy ra J, F, G thẳng hàng.
1
Ta có ∠BIJ = 180◦ − ∠BIC = 90◦ − ∠BHC = ∠HF G = ∠JF B.
2
Suy ra tứ giác BJF I nội tiếp.
1 1
Ta thu được ∠HF I = 180◦ − ∠BF I = 180◦ − ∠BJC = 90◦ − ∠BDC = 90◦ − ∠BHD = ∠F HI.
2 2
Suy ra I là trung điểm HE. Ta có đpcm.

Bài 220. Cho đường tròn (O) ngoại tiếp tam giác ABC. Một đường tròn ω tiếp xúc với các cạnh
AB, AC lần lượt tại L, K và tiếp xúc ngoài với (BOC). Chứng minh rằng LK chia đôi AI với I là
tâm đường tròn nội tiếp tam giác ABC.

A N

K
L
M
O
E
I

B C

Chứng minh. Gọi E là giao điểm thứ hai của AC với (BOC). Ta có ∠BEA = 180◦ − ∠BEC =
180◦ − ∠BOC = 180◦ − 2∠BAC.
Do đó tam giác AEB cân tại E. Suy ra OE là trung trực của đoạn thẳng AB và OE giao CI tại
điểm chính giữa M của cung AB.
Do ω là đường tròn Thebault của tam giác BEC ứng với đường thẳng BA và M là tâm đường
tròn bàng tiếp góc C của tam giác BEC nên M, L, K thẳng hàng.
Tương tự gọi N là điểm chính giữa cung AC thì N, L, K thẳng hàng. Mà M N là trung trực của
đoạn thẳng AI nên LK chia đôi AI.

13 Đường tròn Hagge


13.1 Giới thiệu
Năm 1907, Karl Hagge viết một bài báo về lớp các đường tròn luôn đi qua trực tâm của tam
giác. Bài viết của ông đã mở rộng được đường thẳng Wallace-Simson khi điểm ứng với nó không nằm
trên đường tròn ngoại tiếp. Kết quả này được gọi là đường tròn Hagge, phát biểu như sau.
Cho tam giác ABC nội tiếp đường tròn (O) với trực tâm H. Gọi P là điểm bất kì trong mặt
phẳng. AP, BP, CP giao (O) lần thứ hai tại A1 , B1 , C1 . Gọi A2 , B2 , C2 lần lượt đối xứng với A1 , B1 , C1
qua BC, CA, AB. Khi đó H, A2 , B2 , C2 cùng thuộc một đường tròn. Đường tròn đi qua bốn điểm
H, A2 , B2 , C2 được gọi là đường tròn P -Hagge.

125
A

B1
C1
A2
H
O
P
C2

B2
B C

A1

13.2 Chứng minh


(Nsato).

L
A

Y
Z

B3
C3 A2
Q
O
H P

B C
A3
X

A3 A1

Gọi Q là điểm liên hợp đẳng giác của P ứng với 4ABC; X, Y, Z lần lượt là điểm đối xứng với
A, B, C qua Q; A3 , B3 , C3 là giao của AQ, BQ, CQ với (O), A4 B4 C4 là tam giác trung tuyến của
4ABC.
Phép đối xứng tâm Q lần lượt biến A, B, C thành X, Y, Z nên biến 4ABC thành 4XY Z. Mặt
1
khác A4 B4 C4 là tam giác trung tuyến của 4ABC nên tồn tại tâm vị tự K tỉ số biến 4XY Z thành
2
4A4 B4 C4 .
Gọi L là giao điểm thứ hai của A1 A2 với (O). Ta có AL k HA2 .
Mà LA1 vuông góc với BC và A3 A1 song song với BC nên A3 A1 ⊥ LA1 , suy ra AA3 ⊥ AL hay
AA3 ⊥ HA2 .
Dễ thấy A4 là trung điểm của A2 A3 nên XA2 KA3 là hình bình hành. Suy ra KA2 k AA3 .
Vậy KA2 ⊥ HA2 hay A2 nằm trên đường tròn đường kính KH. Chứng minh tương tự ta có đpcm.

126
13.3 Tính chất
Trong phần này để thuận tiện cho việc cộng góc ta sẽ sử dụng góc định hướng.
Trong lời giải bài toán rõ ràng A2 , B2 , C2 lần lượt là điểm đối xứng với A3 , B3 , C3 qua trung điểm
các cạnh của tam giác ABC. Vì vậy ta có tính chất sau.

Tính chất 69. (China TST 2006). Cho tam giác ABC nội tiếp đường tròn (O) với trực tâm H. Gọi
P là điểm bất kì trong mặt phẳng. AP, BP, CP giao (O) lần thứ hai tại A1 , B1 , C1 . Gọi A2 , B2 , C2 lần
lượt đối xứng với A1 , B1 , C1 qua trung điểm BC, CA, AB. Khi đó H, A2 , B2 , C2 cùng thuộc một đường
tròn.

Sau đây ta sẽ chứng minh tính chất này theo một cách khác.

Chứng minh.
Bổ đề 6. (B2 C2 , B2 A2 ) ≡ (P C, P A) (mod π)
Chứng minh.

C2
M
Y B1
Z
B2
A2
C1 O
A' N
B'
P
C'

B X C

A1

Gọi M, N lần lượt là điểm đối xứng với A2 , C2 qua trung điểm AC thì 4M B1 N đối xứng với
4A2 B2 C2 qua trung điểm AC.
Do AM k CA2 , AM = CA2 và BA1 k CA2 , BA1 = CA2 nên AM A1 B là hình bình hành. Tương
tự BC1 N C là hình bình hành.
Gọi A0 , B 0 , C 0 lần lượt là trung điểm của AA1 , BB1 , CC1 thì A0 , C 0 là trung điểm của BM, BN .
1
Xét phép vị tự HB2 : M 7→ A0 , B1 7→ B 0 , N 7→ C 0 , suy ra 4M B1 N 7→ 4A0 B 0 C 0 . Điều này nghĩa là
(B1 M, B1 N ) ≡ (B 0 A0 , B 0 C 0 ) (mod π).
Mặt khác, OA0 , OB 0 , OC 0 lần lượt vuông góc với AA1 , BB1 , CC1 nên O, A0 , B 0 , C 0 , P đồng viên,
suy ra (B2 A2 , B2 C2 ) ≡ (B1 M, B1 N ) ≡ (B 0 A0 , B 0 C 0 ) ≡ (P A0 , P C 0 ) ≡ (P A, P C) (mod π).
Trở lại bài toán.

127
A3

A
B1

C4 C2 B4

Y
C1 Z
O
P H A2

B2

B X C

B3
A1 C3
A4

Dựng các đường thẳng lần lượt qua A1 , B1 , C1 vuông góc với AA1 , BB1 , CC1 , chúng cắt nhau tạo
thành tam giác A3 B3 C3 và giao (O) lần thứ hai tại A4 , B4 , C4 .
Chú ý rằng AA4 , BB4 , CC4 là đường kính của (O) nên A4 , C4 lần lượt đối xứng với H qua trung điểm
BC, AB. Ta thu được HA2 A4 A1 , HC2 C4 C1 là các hình bình hành, suy ra HA2 k A1 A4 , HC2 k C1 C4 .
Mà P C1 B3 A1 là tứ giác nội tiếp nên áp dụng bổ đề trên suy ra (HC2 , HA2 ) ≡ (B3 C1 , B3 A1 ) ≡
(P C1 , P A1 ) ≡ (P C, P A) ≡ (B2 C2 , B2 A2 ) hay H, A2 , B2 , C2 đồng viên.

Tính chất 70. Với kí hiệu như bài toán ban đầu. Chúng ta có 4A2 B2 C2 ∼ 4A1 B1 C1 .
A

B1

A2
H
C1 O
P
C2

B2
B C

A1

Chứng minh. Ta có H, A2 , B2 , C2 đồng viên.


Do A2 đối xứng với A1 qua BC ta thu được HA2 là đường thẳng Steiner của A1 . Tương tự HB2
là đường thẳng Steiner của B1 . Từ đó (C2 A2 , C2 B2 ) ≡ (HA2 , HB2 ) ≡ (C1 A1 , C1 B1 ) (mod π). Chứng
minh tương tự ta thu được sự đồng dạng của hai tam giác A1 B1 C1 và A2 B2 C2 .

128
Tính chất 71. Gọi A3 , B3 , C3 lần lượt là giao điểm thứ hai của AH, BH, CH với (A2 B2 C2 ). Khi đó
A2 A3 , B2 B3 , C2 C3 đồng quy tại P .

B3 B1
C1 A2
H O

C2 C3
P

B B2 C
A3

A1

Chứng minh. (A2 C2 , A2 A3 ) ≡ (HC2 , HA3 ) ≡ (HC2 , BC) + (BC, HA) ≡ π/2 + (HC2 , BC) (mod π).
Gọi L, Z lần lượt là hình chiếu của C1 trên BC, AB thì LZ là đường thẳng Simson của C1 . Ta thu
được HC2 k LZ
Từ đó (HC2 , BC) ≡ (LZ, LC) ≡ (C1 Z, C1 B) (mod π)
Suy ra (A2 C2 , A2 A3 ) ≡ π/2 + (C1 Z, C1 B) ≡ (BA, BC1 ) ≡ (CA, CC1 ) (mod π)
Vậy H, A3 , B3 , C3 đồng viên, ta thu được (C3 B3 , C3 A3 ) ≡ (HB3 , HA3 ) ≡ (CA, CB) (mod π) .
Chứng minh tương tự ta có 4A3 B3 C3 ∼ 4ABC. Theo tính chất 2, 4A2 B2 C2 ∼ 4A1 B1 C1 .
Vậy tồn tại một phép đồng dạng biến lục giác A3 C2 B3 A2 C3 B2 thành AC1 BA1 CB1 , suy ra
AP A3 P 0
A2 A3 , B2 B3 , C2 C3 đồng quy tại P 0 . Hơn nữa, = nên P P 0 k AH. Tương tự, P P 0 k BH, CH.
A1 P A2 P 0
Nghĩa là P ≡ P 0 . Ta có đpcm.

Tính chất 72. Gọi I là tâm đường tròn ngoại tiếp tam giác A2 B2 C2 . K đối xứng với H qua I.
AK, BK, CK giao (I) lần lượt tại A4 , B4 , C4 . Khi đó A3 A4 , B3 B4 , C3 C4 đồng quy.

A4
H
O
B3 C3

P I

B C
B4
A3 K
C4

129
Chứng minh. Do KA3 ⊥ AH nên (C3 C4 , C3 A3 ) ≡ (KC4 , KA3 ) ≡ (CK, CB) (mod π).
Tương tự, (C3 C4 , C3 B3 ) ≡ (CK, CA) (mod π). Chứng minh tương tự và áp dụng định lý Ceva-sin
cho tam giác A3 B3 C3 ta có đpcm.

Tính chất 73. Cho tam giác ABC nội tiếp đường tròn (O). Gọi P và Q là hai điểm liên hợp đẳng giác
trong tam giác ABC, O1 , O2 lần lượt là tâm của đường tròn P-Hagge và Q-Hagge. Khi đó P Q k O1 O2
và P Q = O1 O2 .

Chứng minh.
Bổ đề 7. Cho tam giác ABC với trực tâm H, tâm ngoại tiếp O và điểm P bất kì trong mặt phẳng.
AP, BP, CP giao (O) lần lượt tại A1 , B1 , C1 . Gọi A2 , B2 , C2 lần lượt đối xứng với A1 , B1 , C1 qua trung
điểm BC, CA, AB, O2 là tâm ngoại tiếp của (A2 B2 C2 ). Khi đó OO2 HP là hình bình hành.
Chứng minh.

B1
A O'2

C1 C'

H'
Y U
A2 Z A'
H

O
X

C2
B O2 C
B2

A1

Gọi A0 , C 0 lần lượt đối xứng với A2 , C2 qua trung điểm AC. Khi đó 4A0 B1 C 0 đối xứng với 4A2 B2 C2
qua trung điểm AC.
Do AA0 k A2 C, AA0 = A2 C, A2 C k BA1 , A2 C = BA1 ta có AA0 A1 B là hình bình hành. (1)
Tương tự, BCC 0 C1 là hình bình hành. (2)
Gọi X, Y, Z lần lượt là trung điểm AA1 , BB1 , CC1 suy ra X, Y, Z nằm trên (OP ).
Từ (1) và (2) ta có X, Z lần lượt là trung điểm của BA0 , BC 0 .
1
Xét phép vị tự HB2 : A0 7→ X, B1 7→ Y, C 0 7→ Z suy ra 4A0 B1 C 0 7→ 4XY Z.
Xét phép đối xứng tâm là trung điểm BC:
S : B 7→ B 0 , H 7→ H 0 là trực tâm tam giác AB 0 C, O2 7→ O20 . Suy ra O2 H 7→ O20 H 0 .
1
Dễ thấy B, O, H 0 thẳng hàng. Do đó phép vị tự HB2 : H 0 7→ O, O20 7→ U là tâm ngoại tiếp tam giác
XY Z.
Ta thu được OP k= O20 H k= O2 H. Vậy HP OO2 là hình bình hành.
Trở lại bài toán.
Từ bổ đề trên, OO1 k= QH, OO2 k= P H suy ra P Q k= O1 O2 . Ta có đpcm.

Nhận xét. Bán kính của đường tròn P-Hagge bằng khoảng cách từ Q đến O.

130
Tính chất 74. Cho tam giác ABC, đường tròn Euler (E) và điểm P bất kì nằm trên (E). Gọi Q là
điểm liên hợp đẳng giác với P trong 4ABC. Khi đó tâm I của đường tròn Q-Hagge nằm trên (E), cụ
thể là điểm đối xứng với P qua tâm Euler E.

Chứng minh. Bài toán là hệ quả trực tiếp của tính chất 5. Chú ý rằng tâm của đường tròn Q-Hagge
luôn đối xứng với P qua trung điểm OH.

Tính chất 75. Cho tam giác ABC và tâm ngoại tiếp O. P và Q là hai điểm liên hợp đẳng giác trong
tam giác ABC sao cho O, P, Q thẳng hàng. Khi đó đường tròn P-Hagge và Q-Hagge tiếp xúc nhau.

Chứng minh. Tính chất 7 cũng là hệ quả trực tiếp của tính chất 5. Xin để lại lời giải cho bạn đọc.

Tính chất 76. Cho tam giác ABC nội tiếp đường tròn (O, R). Một đường tròn tâm O bán kính r < R
giao BC, CA, AB lần lượt tại A1 , A2 , B1 , B2 , C1 , C2 . Gọi E, F là hai điểm Miquel của tam giác ABC
ứng với bộ điểm (A1 , B1 , C1 ) và (A2 , B2 , C2 ). Khi đó hai đường tròn E-Hagge và F-Hagge có bán kính
bằng nhau.

Chứng minh.
Bổ đề 8. Cho tam giác ABC nội tiếp đường tròn (O, R). Một đường tròn tâm O bán kính r < R giao
BC, CA, AB lần lượt tại A1 , A2 , B1 , B2 , C1 , C2 . Gọi P, Q là hai điểm Miquel của tam giác ABC ứng
với bộ điểm (A1 , B1 , C1 ) và (A2 , B2 , C2 ). Khi đó P và Q là hai điểm liên hợp đẳng giác và OP = OQ.
Chứng minh.

B2

X
C1

Q
C2
P O B1
Y

B A1 A2 C

Gọi X, Y, Z lần lượt là giao điểm thứ hai của A1 P, B1 P, C1 P với (O).
Ta có (ZY, ZX) ≡ (A1 Y, A1 X) ≡ (A1 Y, P Y ) + (P Y, A1 X) ≡ (A2 C, A2 B1 ) + (CA2 , CB1 ) ≡
(B1 B2 , B1 A2 ) (mod π)
Suy ra XY = A2 B2 . Chứng minh tương tự, Y Z = B2 C2 , XZ = A2 C2 . Vậy 4XY Z = 4A2 B2 C2 .
Mặt khác, (P X, P Y ) ≡ (QA2 , QB2 ) (mod π), (P Y, P Z) ≡ (QB2 , QC2 ) (mod π) nên tồn tại phép
quay tâm O biến 4XY Z thành 4A2 B2 C2 và P thành Q. Ta thu được OP = OQ.
Hơn nữa, (AP, AB) ≡ (B1 P, B1 C1 ) ≡ (ZY, ZP ) ≡ (C2 B2 , C2 Q) ≡ (AB2 , AQ) (mod π).
Tương tự suy ra P và Q liên hợp đẳng giác trong 4ABC.
Trở lại bài toán.
Áp dụng bổ đề trên, P và Q liên hợp đẳng giác trong 4ABC và OP = OQ. Áp dụng nhận xét
trong tính chất 5 ta thu được đpcm.

131
Tính chất 77. Cho tam giác ABC với trực tâm H. Gọi P, Q là hai điểm liên hợp đẳng giác trong
4ABC. Dựng đường kính HK của đường tròn P-Hagge. Gọi G là trọng tâm tam giác ABC. Khi đó
1
Q, G, K thẳng hàng và QG = GK.
2

B1
C1
Q
C2 A2

H G O

K
B Ma C
X B2

A1

Chứng minh. Gọi X là điểm đối xứng với A qua Q, Ma là trung điểm BC nên theo lời giải bài toán
về đường tròn Hagge ta có Ma là trung điểm XK.
Áp dụng định lý Menelaus cho tam giác AXMa với đường thẳng (Q, G, K) ta có Q, G, K thẳng hàng.
1
Lại áp dụng định lý Menelaus cho tam giác QXK với đường thẳng (A, G, Ma ) thì QG = GK.
2
Tính chất 78. Trọng tâm G nằm trên đường tròn L-Hagge với L là điểm Lemoine.
B2

A
B1

C1
C2 C3 A3
G
O

B3

C
B

A2 A1

Chứng minh. Gọi A1 , B1 , C1 lần lượt là giao của AG, BG, CG với (O), A2 , B2 , C2 lần lượt là giao của
AL, BL, CL với (O); A3 , B3 , C3 lần lượt đối xứng với A2 , B2 , C2 qua BC, CA, AB.
Dễ thấy A3 , B3 , C3 lần lượt nằm trên AG, BG, CG.
Theo tính chất 2, 4A3 B3 C3 ∼ 4A2 B2 C2 nên (A2 B2 , A2 C2 ) ≡ (A3 C3 , A3 B3 ) (mod π)

132
1 _ _ 1 _ _
Mặt khác, (GC3 , GB3 ) ≡ (CB1 + C1 B) ≡ (B2 A + AC2 ) ≡ (A2 B2 , A2 C2 ) ≡ (A3 C3 , A3 B3 )
2 2
(mod π).
Điều này nghĩa là G, A3 , B3 , C3 đồng viên.
Một số kết quả khác.
1. Khi P ≡ I là tâm đường tròn nội tiếp tam giác ABC, đường tròn P-Hagge được gọi là đường tròn
Fuhrmann của 4ABC. Đoạn thẳng nối trực tâm H và điểm Nagel N là đường kính của đường tròn
Fuhrmann (xem lời giải tại [8]). Gọi O là tâm đường tròn ngoại tiếp tam giác ABC thì HN = 2OI.
2. Khi P nằm trên đường tròn ngoại tiếp tam giác ABC, đường tròn P-Hagge suy biến thành
đường thẳng Steiner của P .
3. Khi điểm liên hợp đẳng giác Q của P nằm trên đường tròn ngoại tiếp tam giác ABC, ta thu
được bài toán mới:
Cho tam giác ABC và điểm P bất kì nằm trên đường tròn (O). Gọi A0 , B 0 , C 0 lần lượt là các điểm
đối xứng với P qua trung điểm BC, CA, AB. Khi đó H, A0 , B 0 , C 0 cùng thuộc một đường tròn.
4. Khi P là điểm vô cùng, ta phát biểu lại bài toán đường tròn Hagge dưới dạng:
Cho tam giác ABC nội tiếp đường tròn (O) với trực tâm H. Gọi A1 , B1 , C1 lần lượt là các điểm
nằm trên (O) sao cho AA1 k BB1 k CC1 , A2 , B2 , C2 lần lượt đối xứng với A1 , B1 , C1 qua BC, CA, AB.
Khi đó H, A2 , B2 , C2 cùng thuộc một đường tròn.
5. Khi P ≡ H là trực tâm 4ABC, đường tròn P-Hagge suy biến thành điểm H.
6. Khi P ≡ O là tâm ngoại tiếp 4ABC, tâm của đường tròn P-Hagge trùng với O.

13.4 Từ các kết quả về ba đường tròn bằng nhau


Trên tạp chí hình học Forum Geometricorum, tác giả Bùi Quang Tuấn viết một bài viết với tựa
đề "two triads of congruent circles from reflections" (xem [2]). Trong bài viết này, tác giả đã xây dựng
được hai bộ ba đường tròn bằng nhau sao cho mỗi đường tròn đều đi qua một đỉnh của tam giác. Bộ
thứ nhất liên quan đến phép đối xứng qua qua đường cao, bộ thứ hai liên quan đến phép đối xứng qua
phân giác. Năm 2011, tác giả Trần Quang Hùng cũng dựng được một bộ ba tương tự liên quan đến
phép đối xứng qua đường thẳng nối đỉnh và tâm đường tròn Euler, được xuất bản trong bộ sách "Tìm
tòi và Sáng tạo" của trường THPT chuyên KHTN, ĐHKHTN-ĐHQGHN. Trong phần này chúng ta
sẽ tiếp cận các bộ ba này dưới một góc nhìn mới.
Bài 221. Cho tam giác ABC với trực tâm H, tâm ngoại tiếp O. Gọi Ba , Ca lần lượt là các điểm đối
xứng với B, C qua AH. Tương tự xác định Cb , Ab , Ac , Bc . Khi đó bán kính đường tròn ngoại tiếp các
tam giác ACb Bc , BAc Ca , CAb Ba bằng nhau và bằng bán kính đường tròn O-Hagge.

Ca

Ba

H Ac
O

Bc
B
Cb
C

Ab

133
Nhận xét. Trong [2], Bùi Quang Tuấn đã chứng minh rằng các đường tròn (ACb Bc ), (BAc Ca ), (CAb Ba )
có bán kính bằng bán kính đường tròn (H, HO). Theo nhận xét của tính chất 5, HO là bán kính của
đường tròn O-Hagge.

Bài 222. Gọi I là tâm đường tròn nội tiếp tam giác ABC. Gọi Ba , Ca lần lượt là điểm đối xứng với
B, C qua AI, tương tự xác định Ab , Cb , Ac , Bc . Khi đó bán kính đường tròn ngoại tiếp các tam giác
ACb Bc , BAc Ca , CAb Ba bằng nhau và bằng bán kính đường tròn Fuhrmann.

Cb

A Bc B1

C1
C2 A2

I Ba
O

Ac B2
B Ab C

Ca A1

Nhận xét. Tác giả Bùi Quang Tuấn cũng chứng minh bán kính của các đường tròn (ACb Bc ),
(BAc Ca ), (CAb Ba ) cùng bằng bán kính đường tròn (I, IO). Lại theo nhận xét tính chất 5, IO là bán
kính đường tròn Fuhrmann.

Bài 223. Cho tam giác ABC với tâm đường tròn Euler E. Gọi Ba , Ca lần lượt là điểm đối xứng với
B, C qua AE. Tương tự xác định Cb , Ab của C, A qua đường thẳng BE và Ac , Bc của A, B qua đường
thẳng CE. Khi đó bán kính đường tròn ngoại tiếp các tam giác ACb Bc , BAc Ca , CAb Ba bằng nhau và
bằng bán kính đường tròn E-Hagge.

134
Cb

A B1

Bc
Oc

O
C1 C2
E A2

Q
H
B C

B2

A1

Chứng minh. Gọi A1 , B1 , C1 lần lượt là giao của AE, BE, CE với (O), A2 , B2 , C2 lần lượt là đối xứng
của A1 , B1 , C1 qua BC, CA, AB. Gọi H là trực tâm tam giác ABC.CH cắt đường tròn (O) ngoại tiếp
tam giác ABC lần thứ hai tại Q, (Oc ) là đường tròn ngoại tiếp tam giác AHB.
Do (Oc ) và (O) đối xứng nhau qua AB, C2 đối xứng với C1 qua AB nên C2 ∈ (Oc ). Dễ thấy Oc
nằm trên CE. Mà B và Bc đối xứng nhau qua CE nên Bc ∈ (Oc ).
Ta có (BA, BBc ) ≡ (CQ, CC1 ) (mod π) do CQ ⊥ AB, BBc ⊥ CC1 . Nghĩa là góc (BA, BBc ) bằng
một nửa số đo cung QC1 hay HC2 . Điều này nghĩa là ABc = HC2 hay AHC2 Bc là hình thang cân.
Chứng minh tương tự, AHB2 Cb là hình thang cân. Ta thu được (ABc Cb ) là ảnh của (HC2 A2 ) qua
phép đối xứng trục là đường trung trực của đoạn AH. Nghĩa là bán kính của đường tròn (ABc Cb ) và
đường tròn E-Hagge bằng nhau. Chứng minh tương tự ta có đpcm.

Nhận xét. Lời giải sơ cấp khác cho sự bằng nhau của các đường tròn (ACb Bc ), (BAc Ca ), (CAb Ba )
có thể xem tại [6] hoặc [7]. Ta có một số tính chất sau.
1. Bán kính của các đường tròn (ACb Bc ), (BAc Ca ), (CAb Ba ) bằng khoảng cách từ O tới điểm
Kosnita K (điểm liên hợp đẳng giác của tâm Euler E).
2. Trực tâm tam giác Ia Ib Ic là điểm đối xứng với H qua tâm của đường tròn N-Hagge (Ia , Ib , Ic
lần lượt là tâm đường tròn ngoại tiếp các tam giác ACb Bc , BAc Ca , CAb Ba ).
3. N là tâm của đường tròn K-Hagge.

14 Định lý Fontené
14.1 Giới thiệu
Fontené là một định lý khá đẹp trong hình học sơ cấp, tuy nhiên còn có ít tài liệu đề cập đến.
Chính vì vậy, trong mục này tác giả xin giới thiệu định lý trên và một số hệ quả thú vị của nó. Hi
vọng bạn đọc sẽ cảm nhận được vẻ đẹp của hình học thông qua từng bài toán.
Định lý Fontené 1. Cho tam giác ABC. P là điểm bất kì trên mặt phẳng. Gọi A1 , B1 , C1 thứ tự
là trung điểm BC, CA, AB; A2 B2 C2 là tam giác pedal của P đối với tam giác ABC.X, Y, Z thứ tự là
giao điểm của B1 C1 với B2 C2 , A1 C1 với A2 C2 , A1 B1 với A2 B2 . Khi đó A2 X, B2 Y, C2 Z đồng quy tại
giao điểm của (A1 B1 C1 ) với (A2 B2 C2 ).

135
L A

Q B2

C1 B1
X

C2 P E
F O
O'

B A2 A1 C

Chứng minh. Gọi E là tâm (A1 B1 C1 ), O0 là tâm (A2 B2 C2 ), F là giao điểm của OP với đường tròn
đường kính OA, L là điểm đối xứng của A2 qua B1 C1 thì AL k BC. Do đó ∠ALP = 90◦ .
Ta có ∠AF P = ∠AB2 P = ∠AC2 P = ∠ALP = 90◦ nên L, F, B2 , C2 cùng thuộc (AP ).
∠F C1 X = ∠F AB1 = ∠B2 C2 F suy ra tứ giác F XC1 C2 nội tiếp.
Gọi L0 là giao của F X với (AP ) thì tứ giác AL0 C2 F nội tiếp. Mà F XC1 C2 nội tiếp nên AL0 k B1 C1
hay L0 ≡ L. Vậy L, X, F thẳng hàng.
Gọi Q là giao của A2 X với (E).F 0 là điểm đối xứng với Q qua B1 C1 .
Xét phép đối xứng trục B1 C1 : (AO) 7→ (E), mà Q ∈ (E) suy ra F 0 ∈ (AO).
Mặt khác cũng qua phép đối xứng trục B1 C1 : A2 7→ L. Vì A2 , X, Q thẳng hàng nên L, X, F 0 thẳng
hàng. Suy ra F 0 ≡ F .
Ta thu được tứ giác A2 LQF là hình thang cân.
Suy ra XQ.XA2 = XL.XF = XB2 .XC2 .
Vậy Q nằm trên (O0 ). Tương tự B2 Y, C2 Z cũng đi qua Q. Ta có đpcm.

Định lý Fontené 2. Nếu điểm P di chuyển trên một đường thẳng d cố định đi qua tâm đường
tròn ngoại tiếp O của tam giác ABC thì đường tròn ngoại tiếp tam giác pedal của P đối với tam giác
ABC cắt đường tròn 9 điểm của tam giác ABC tại một điểm cố định.

Chứng minh. Theo chứng minh định lý Fontené 1, giao điểm Q của (E) và (O0 ) là điểm đối xứng của
điểm F nằm trên OP qua đường thẳng B1 C1 . Dễ thấy O là trực tâm của tam giác A1 B1 C1 nên Q là
điểm Anti-Steiner của đường thẳng d. Vậy Q cố định. Ta có đpcm.

Định lý Fontené 3. Gọi P 0 là điểm liên hợp đẳng giác của P trong tam giác ABC. Khi đó đường
tròn ngoại tiếp tam giác pedal của P tiếp xúc với đường tròn Euler của tam giác ABC khi và chỉ khi
O, P, P 0 thẳng hàng.

Chứng minh. Theo định lý Fontené 2 ta có thể tương tự chứng minh rằng giao điểm thứ hai Q0 của
(O0 ) và (E) là điểm Anti-Steiner của OP 0 . Từ đó Q0 ≡ Q tương đương OP ≡ OP 0 hay O, P, P 0 thẳng
hàng. Ta có đpcm.

Nhận xét. Điểm Feuerbach là hệ quả của định lý Fonenté 3, khi P trùng với tâm đường tròn nội
tiếp hoặc tâm 3 đường tròn bàng tiếp.

14.2 Hệ quả
Hệ quả 1. O0 là trực tâm tam giác XY Z.

136
Chứng minh. Theo định lý Fontené 1, A2 X, B2 Y, C2 Z đồng quy tại Q nằm trên đường tròn (O0 ). Suy
ra XZ là đường đối cực của Y trong (O0 ), XY là đường đối cực của Z trong (O0 ). Vậy O0 là trực tâm
tam giác XY Z.

Hệ quả 2. Gọi A3 B3 C3 là tam giác pedal của điểm liên hợp đẳng giác P 0 của P trong tam giác ABC.
Khi đó đường thẳng Simson của Q ứng với tam giác A3 B3 C3 song song với đường thẳng Simson của
Q ứng với tam giác A1 B1 C1 .

L A

Q B2

C3 S
X T
C1
B1
R
B3
C2 F O
P
P'
A1
B A2 A3 C

Chứng minh. Gọi R, S lần lượt là hình chiếu của Q trên B3 C3 , A3 B3 . Kéo dài QR cắt (O0 ) tại W, AF ∩
QW = {T }. Ta có ∠QRS = ∠QB3 S = ∠QW A3 ⇒ RS k A3 W .
Mặt khác ∠P 0 C3 B3 = ∠P 0 AB3 = ∠P AB nên AP ⊥ B3 C3 ⇒ AP k QW .
⇒ ∠T W A3 + ∠F T W = ∠QA2 A3 + ∠P AF = ∠QA2 A3 + ∠P LF = ∠QA2 A3 + ∠P A2 Q = 90◦ .
⇒ SR k A3 W k OP.(1)
Lại theo chứng minh định lý Fontené 2, Q là điểm Anti-Steiner của đường thẳng OP ứng với tam
giác A1 B1 C1 nên đường thẳng Simson của Q ứng với tam giác A1 B1 C1 song song với OP.(2)
Từ (1) và (2) ta có đpcm.

Hệ quả 3. Cho tam giác ABC nội tiếp (O) với trực tâm H. Gọi N là điểm Nagel của tam giác
ABC.ON cắt (O) tại Q.Khi đó đường thẳng Simson của Q ứng với tam giác ABC song song với N H.

Chứng minh. Trước hết ta phát biểu và chứng minh một bổ đề.
Bổ đề 9. Gọi I, G, N lần lượt là tâm đường tròn nội tiếp, trọng tâm và điểm Nagel của tam giác
ABC. Khi đó I, G, N thẳng hàng và IN = 3IG.
Chứng minh.

137
A

T
Y
B'

I Y'
G
N
R
C
B X A' X'
K
P

Ia

Gọi X, Y là tiếp điểm của (I) với BC, AC.XI cắt (I) tại T . Ta sẽ chứng minh A, T, N thẳng hàng.
Thật vậy, gọi Ia là tâm đường tròn bàng tiếp góc A, (Ia ) tiếp xúc với BC, AC tại X 0 , K. Ta có
IT IY IA
0
= = .
Ia X Ia K Ia A
Suy ra A, T, X 0 thẳng hàng hay A, T, N thẳng hàng.
Trên AI lấy P sao cho I là trung điểm AP . Gọi B 0 là trung điểm AC.IY cắt (I) tại R, BR
cắt AC tại Y 0 . Do IB 0 đồng thời là đường trung bình của tam giác Y RY 0 và tam giác AP C nên
BN k IB 0 k P C 0 . Tương tự ta cũng có CN k BP . Do đó BN CP là hình bình hành. Suy ra A0 là trung
điểm N P .
GA GI
Gọi G0 là giao điểm của AA0 và IN . Do IA0 là đường trung bình tam giác AP N nên 0
= = 2.
GA GN
Trở lại bài toán.
C' A B'

O
I G
H N

B C
P

A'

Qua A, B, C kẻ đường song song với cạnh đối diện, 3 đường thẳng này cắt nhau tạo thành tam
giác A0 B 0 C 0 . Ta có (ABC) là đường tròn Euler của tam giác A0 B 0 C 0 .
−2
Xét phép vị tự HG : (ABC) 7→ (A0 B 0 C 0 ), O 7→ H, I 7→ N . Do O là tâm của (ABC) nên H là tâm
0 0 0
của (A B C ). I là tâm đường tròn nội tiếp tam giác ABC nên N là tâm đường tròn nội tiếp tam giác
A0 B 0 C 0 . Suy ra P là điểm Feuerbach của tam giác A0 B 0 C 0 . Theo chứng minh định lý Fontené 2 thì P
là điểm Anti-Steiner của đường thẳng N H ứng với tam giác ABC. Do đó đường thẳng Simson của P
ứng với tam giác ABC song song với N H.

138
Hệ quả 4. Cho tam giác ABC.P là điểm bất kì nằm trong tam giác. A1 B1 C1 là tam giác Pedal của
P ứng với 4ABC. A2 , B2 , C2 lần lượt là trung điểm BC, CA, AB.A3 , B3 , C3 lần lượt là điểm đối xứng
với A1 , B1 , C1 qua A2 , B2 , C2 . Khi đó ba đường tròn (A1 B1 C1 ), (A2 B2 C2 ), (A3 B3 C3 ) đồng quy.

C3 Y B1

C2 B2
Q
O
C1 P E
B3

A2
B A1 A3 C

Chứng minh. Do A1 B1 C1 là tam giác pedal của P ứng với tam giác ABC nên theo định lý Carnot ta
có:
BA21 − CA21 + CB12 − AB12 + AC12 − BC12 = 0.
Suy ra CA23 − BA23 + BC32 − AC32 + AB32 − CB32 = 0. Từ đó A3 B3 C3 là tam giác pedal của Q ứng
với tam giác ABC.
Áp dụng định lý Thales, dễ thấy rằng trung trực của đoạn BC đi qua trung điểm P Q. Tương tự
ta suy ra tâm ngoại tiếp O của tam giác ABC là trung điểm P Q.
Gọi Y là điểm Anti-Steiner của đường thẳng OP ứng với tam giác A2 B2 C2 thì theo chứng minh
định lý Fontené 2, Y ∈ (A1 B1 C1 ). Mà O, P, Q thẳng hàng nên Y là điểm Anti-Steiner của đường thẳng
OQ ứng với tam giác A2 B2 C2 . Lại có A3 B3 C3 là tam giác pedal của Q ứng với tam giác ABC nên áp
dụng chứng minh định lý Fontené 2 lần thứ hai, Y ∈ (A3 B3 C3 ).

Hệ quả 5. Cho tam giác ABC. Gọi A1 là hình chiếu của A trên BC, A2 , B2 , C2 lần lượt là trung
điểm BC, CA, AB. P là điểm bất kì trên mặt phẳng, A0 B 0 C 0 là tam giác pedal của P ứng với
4ABC.(A0 B 0 C 0 ) ∩ (A2 B2 C2 ) = {F, F 0 }. đường thẳng qua A0 song song với AP cắt AA1 tại A00 .
Khi đó đường tròn đường kính A0 A00 đi qua một trong hai điểm F, F 0 .

A''

I'
V M B2
C2 L N
C' I
B'
P
F
A2
B A1 A' C

139
Chứng minh. Gọi V là giao điểm của B 0 C 0 và B2 C2 . Áp dụng định lý Fontené 1 ta thu được A0 , V, F
thẳng hàng.
Mặt khác, gọi I 0 , L lần lượt là trung điểm của AP, AA0 .
Do AA00 A0 P là hình bình hành nên I, L, I 0 thẳng hàng và II 0 ⊥ BC. Mà L ∈ B2 C2 suy ra I 0 đối
xứng với I qua B2 C2 . Vậy (I 0 , I 0 A) đối xứng với (I, IA00 ) qua B2 C2 . Suy ra giao điểm M, N của hai
đường tròn nằm trên đường thẳng B2 C2 .
Lại có B 0 , C 0 ∈ (I 0 , I 0 A) nên V F .V A0 = V B 0 .V C 0 = V M .V N , suy ra F, A0 , M, N cùng thuộc một
đường tròn hay (I, IA00 ) đi qua F .

Hệ quả 6. (Nguyễn Văn Linh). Cho tam giác ABC nội tiếp (O), trực tâm H. P là một điểm bất
kì trên mặt phẳng, A1 , B1 , C1 là giao điểm thứ hai của AP, BP, CP với (O), A2 B2 C2 là tam giác
pedal của P đối với 4ABC, A3 , B3 , C3 thứ tự là điểm đối xứng của A1 , B1 , C1 qua A2 , B2 , C2 . Khi đó
H, A3 , B3 , C3 cùng thuộc một đường tròn.

Chứng minh. Trước tiên ta phát biểu và chứng minh một bổ đề:
Bổ đề 10. Với các kí hiệu như hệ quả 6, ta có 4A1 B1 C1 ∼ 4A2 B2 C2 ∼ 4A3 B3 C3 .
Chứng minh.

B3
C1 C2 C3 B2
O B1
A3
P

B A2 C

A1 C4

B4

Kết quả 4A1 B1 C1 ∼ 4A2 B2 C2 là quen thuộc. Vì vậy chúng ta chỉ chứng minh 4A3 B3 C3 ∼
4A1 B1 C1 .
Gọi B4 , C4 lần lượt là điểm đối xứng của B3 , C3 qua A2 suy ra 4A1 B4 C4 ∼ 4A3 B3 C3 .(1)
Do C2 A2 là đường trung bình của tam giác C1 C3 C4 nên C1 C4 k C2 A2 . Tương tự, B1 B4 k B2 A2 .
Gọi R = B1 B4 ∩ C1 C4 thì ∠B1 RC1 = ∠B2 A2 C2 = ∠B1 A1 C1 . Suy ra R ∈ (O).
⇒ ∠A1 C1 C4 = ∠A1 B1 B4 .
A1 C1 A2 C 2 C1 C4
Mặt khác, = = . Do đó 4A1 C1 C4 ∼ 4A1 B1 B4 .
A1 B1 A2 B 2 B1 B4
Trở lại bài toán.

140
A

M
Z Y

R
C2 B2
O
H
A3 P
S
L
A2
B N
X C

A1

Gọi L là giao điểm của (A2 B2 C2 ) với đường tròn 9 điểm của tam giác ABC, X, Y, Z thứ tự là trung
điểm BC, CA, AB. LA2 ∩ (XY Z) = {L, N }.Gọi M là trung điểm AH, K là hình chiếu của A trên
BC, R là trung điểm AA1 .
Vì OX k= AM nên AO k= XM .
Gọi T là giao của đường thẳng qua A2 song song với AA1 và AK. Áp dụng hệ quả 5 ta thu được
(A2 T ) đi qua L.
Suy ra ∠T A2 L = ∠T KL = ∠M N L kéo theo M N k A2 T k AP . Vậy ∠XM N = ∠OAR. Nhưng
∠XN M = ∠ORA = 90◦ nên 4XM N = 4OAR.
1
⇒ OR k= XN suy ra ORN X là hình bình hành, do đó RN k= OX k= AH.
2
Vì vậy N là trung điểm HA1 hay N A2 là đường trung bình của tam giác HA3 A1 .
⇒ HA3 k A2 L. Tương tự ta cũng có HB3 k B2 L.
⇒ ∠A3 HB3 = ∠A2 LB2 = ∠A2 C2 B2 .
Áp dụng bổ đề 2 ta thu được ∠A3 HB3 = ∠A3 C3 B3 hay A3 , B3 , C3 , H cùng thuộc một đường
tròn.

Hệ quả 7. (Trần Quang Hùng). Cho tam giác ABC. P là một điểm bất kì trên mặt phẳng, A1 B1 C1
là tam giác pedal của P ứng với 4ABC. Gọi R là bán kính (A1 B1 C1 ).P 0 là điểm liên hợp đẳng giác
của P trong 4ABC. Tia A1 P, B1 P, C1 P lần lượt giao (P 0 , 2R) tại X, Y, Z. Khi đó AX, BY, CZ đồng
quy tại một điểm nằm trên (P 0 , 2R).

141
A3

X
B2
T A
B1

C2
C1
P O P' P''
L

B A1 C

C3
B3
H A2 A4

Chứng minh. Gọi A2 , B2 , C2 thứ tự là điểm đối xứng của P qua BC, CA, AB. Do tâm O của (A1 B1 C1 )
là trung điểm P P 0 suy ra OA1 là đường trung bình của tam giác P P 0 A2 . Vì vậy P 0 A2 = 2R hay
A2 ∈ (P 0 ). Tương tự B2 , C2 ∈ (P 0 ).
Phép vị tự HP2 : A1 7→ A2 , B1 7→ B2 , C1 7→ C2 , A 7→ A3 , B 7→ B2 , C 7→ C2 nên (O, R) 7→ (P 0 , 2R)
và (P 0 ) là đường tròn pedal của P trong 4A3 B3 C3 .
Gọi P 00 là điểm đối xứng của P qua P 0 .A4 là hình chiếu của P 00 trên B3 C3 thì A4 ∈ (P 0 ). Gọi H
là hình chiếu của A4 trên B3 C3 , T là điểm nằm trên A3 H sao cho A3 T = P 00 A3 . Áp dụng hệ quả 5,
đường tròn đường kính A4 T đi qua giao điểm L của đường tròn Euler của tam giác A3 B3 C3 và (P 0 ).
Hay T L ⊥ A4 L.
Mặt khác, tia A1 P giao (P 0 ) tại X nên XA4 là đường kính của (P 0 ) , ta thu được XL ⊥ A4 L.
Do đó L, T, X thẳng hàng.
Lại có P X k= P 00 A4 k= A3 T suy ra A3 XP T là hình bình hành. Mà A là trung điểm của P A3 nên
T, A, X thẳng hàng.
Vậy L ∈ AX . Tương tự BY, CZ cũng đi qua L. Ta có đpcm.

15 Định lý Monge-D’Alembert
15.1 Giới thiệu
Định lý Monge-D’Alembert đặt theo tên hai nhà toán học Pháp Gaspard Monge và Jean-le-Rond
D’Alembert, được phát biểu như sau.
Định lý 1. Cho ba đường tròn C1 (O1 , R1 ), C2 (O2 , R2 ), C3 (O3 , R3 ) phân biệt trên mặt phẳng. Khi
đó tâm vị tự ngoài của các cặp đường tròn (C1 , C2 ), (C2 , C3 ), (C3 , C1 ) cùng thuộc một đường thẳng.
Hai tâm vị tự trong của hai trong ba cặp đường tròn trên và tâm vị tự ngoài của cặp đường tròn còn
lại cùng thuộc một đường thẳng. [1][2]

142
O1

O2

O3

A1 A2 A3

Chứng minh. Ta chứng minh định lý trong trường hợp ba tâm vị tự ngoài, trường hợp còn lại chứng
minh tương tự.
Gọi tâm vị tự của các cặp đường tròn (C1 , C2 ), (C2 , C3 ), (C3 , C1 ) lần lượt là A3 , A1 , A2 . Khi đó:
V RA13 : O2 → O1 ; V RA32 : O1 → O3 ; V RA21 : O3 → O2 .
R2 R1 R3

A1 O2 A2 O3 A3 O1 R2 R3 R1
Suy ra: . . = . . = 1.
A1 O3 A2 O1 A3 O2 R3 R1 R2
Theo định lý Menelaus thu được A1 , A2 , A3 thẳng hàng.

Sau đây ta xét một trường hợp đặc biệt. Cho hai đường tròn phân biệt C1 (O1 ) và C2 (O2 ) có hai
tâm vị tự A1 và A2 . C2 cắt O1 O2 tại một điểm cố định còn O2 có thể chuyển động, đường tròn C1 cố
định. Thực hiện việc kéo dài O1 O2 đến vô cùng, khi đó A1 và A2 tiền dần đến giao điểm của O1 O2
và C1 . Như vậy khi O2 ở vô cùng, C2 trở thành đường thẳng l, A1 , A2 là giao điểm của đường thẳng
qua O1 và vuông góc với l và C1 . Việc xác định A1 hay A2 là tâm vị tự trong hay ngoài phụ thuộc
vào việc ta coi O1 nằm bên nửa mặt phẳng nào của l. Như vậy định lý Monge-D’Alembert có thể phát
biểu cho trường hợp hai đường tròn và một đường thẳng như sau:
Định lý 2. Cho hai đường tròn C1 (O1 ), C2 (O2 ) và đường thẳng l. Gọi A1 , A2 lần lượt là các điểm
−−−→ −−−→
nằm trên C1 và C2 sao cho O1 A1 ⊥ l, O2 A2 ⊥ l. Khi đó nếu O1 A1 ↑↑ O2 A2 thì tâm vị tự ngoài của C1
−−−→ −−−→
và C2 , A1 , A2 thẳng hàng. Nếu O1 A1 ↑↓ O2 A2 thì tâm vị tự trong của C1 và C2 , A1 , A2 thẳng hàng.
Định lý 2 có vẻ hiển nhiên và chứng minh khá đơn giản dựa theo định lý Thales nên ta không trình
bày ở đây.
Xét một trường hợp đặc biệt khác là hai trong ba đường tròn C1 , C2 , C3 có cùng bán kính (ví dụ
C1 và C2 ). Khi đó tiếp tuyến chung ngoài của C1 và C2 song song. Định lý Monge-D’Alembert phát
biểu rằng đường thẳng nối hai tâm vị tự ngoài (hoặc trong) của hai cặp đường thẳng còn lại song song
với hai tiếp tuyến chung ngoài của C1 và C2 . Có nghĩa là ta đã coi tâm vị tự ngoài của C1 và C2 nằm
ở vô cùng.

15.2 Ứng dụng


Bài 224. Cho tam giác ABC nội tiếp đường tròn (O). Kí hiệu ωa là đường tròn tiếp xúc với các tia
AB, AC và tiếp xúc trong với (O). Tương tự ta xác định ωb , ωc . Gọi A1 , B1 , C1 lần lượt là tiếp điểm
của ωa , ωb , ωc với (O). Chứng minh rằng AA1 , BB1 , CC1 đồng quy.
Ba đường tròn ωa , ωb , ωc còn gọi là đường tròn "Mixtilinear incircle".

143
B1

C1
P I
O

C
B

A1

Chứng minh. Gọi (I) là đường tròn nội tiếp tam giác ABC; P là tâm vị tự ngoài của (I) và (O).
Xét ba đường tròn ωa , (I), (O) có các tâm vị tự ngoài lần lượt là A, A1 , P nên theo định lý Monge-
D’Alembert suy ra AA1 đi qua P .
Tương tự ta thu được AA1 , BB1 , CC1 đồng quy tại P .

Nhận xét 1. Bằng phương pháp tương tự, ta cũng chứng minh được bài toán tổng quát như sau: Cho
tam giác ABC. Gọi ωa , ωb , ωc lần lượt là các đường tròn tiếp xúc với các cặp tia (AB, AC), (BA, BC),
(CA, CB); (O) là đường tròn cùng tiếp xúc trong (hoặc ngoài) với ωa , ωb , ωc lần lượt tại A1 , B1 , C1 .
Chứng minh rằng AA1 , BB1 , CC1 đồng quy.
Bài 225. Cho tam giác ABC. Kí hiệu ωa , ωb , ωc lần lượt là các đường tròn tiếp xúc với các cặp tia
(AB, AC), (BC, BA), (CA, CB) sao cho ωa , ωb , ωc tiếp xúc ngoài nhau. Gọi A1 , B1 , C1 lần lượt là tiếp
điểm của các cặp đường tròn (ωb , ωc ), (ωa , ωc ), (ωa , ωb ). Chứng minh rằng AA1 , BB1 , CC1 đồng quy.
Ba đường tròn ωa , ωb , ωc còn gọi là đường tròn Malfatti.

B1
C1

A1

B C

Chứng minh. Gọi A2 , B2 , C2 lần lượt là tâm vị tự ngoài của các cặp đường tròn (ωb , ωc ), (ωa , ωc ), (ωa , ωb ).
Theo định lý Monge-D’Alembert suy ra A2 , B2 , C2 thẳng hàng.
Do B1 , C1 lần lượt là tâm vị tự trong của các cặp đường tròn (ωa , ωc ), (ωa , ωb ), A2 là tâm vị tự
ngoài của (ωb , ωc ) nên theo định lý Monge-D’Alembert, A2 , B1 , C1 thẳng hàng. Tương tự, A1 , B2 , C1
thẳng hàng, A1 , B1 , C2 thẳng hàng.
Mặt khác, BC, CA, AB lần lượt là tiếp tuyến chung ngoài của ba đường tròn trên nên A2 , B2 , C2
lần lượt nằm trên BC, CA, AB.

144
Theo định lý Desargues, hai tam giác ABC và A1 B1 C1 thấu xạ, hay AA1 , BB1 , CC1 đồng quy.

Bài 226. Cho ba đường tròn C1 , C2 , C3 ngoài nhau cùng tiếp xúc trong với đường tròn (O). Kẻ 3 tiếp
tuyến chung ngoài của 3 đường tròn C1 , C2 , C3 sao cho cả 3 đường tròn đều không nằm trên một nửa
mặt phẳng có bờ là tiếp tuyến trên. Gọi A, B, C lần lượt là tiếp điểm của C1 , C2 , C3 với (O); A1 B1 C1
là tam giác tạo bởi giao điểm của các tiếp tuyến (các đỉnh theo thứ tự tương ứng với A, B, C). Chứng
minh rằng AA1 , BB1 , CC1 đồng quy.

A1 C1
B1 C A2
B

B2

C2

Chứng minh. Gọi A2 , B2 , C2 lần lượt là tâm vị tự ngoài của các cặp đường tròn (C2 , C3 ), (C1 , C3 ), (C1 , C2 ).
Áp dụng định lý Monge-D’Alembert cho 3 đường tròn (O), C1 , C2 ta suy ra A, B, C2 thẳng hàng.
Tương tự, A, B2 , C thẳng hàng, A2 , B, C thẳng hàng.
Mặt khác, hiển nhiên các bộ ba sau thẳng hàng: (A1 , B1 , C2 ), (A2 , B1 , C1 ), (A1 , B2 , C1 ).
Như vậy hai tam giác ABC và A1 B1 C1 có giao điểm các cặp cạnh tương ứng thẳng hàng. Theo
định lý Desargues, hai tam giác ABC và A1 B1 C1 thấu xạ hay AA1 , BB1 , CC1 đồng quy.

Bài 227. (IMO 2008). Cho tứ giác lồi ABCD (BA 6= BC). Gọi đường tròn nội tiếp các tam giác
ABC và ADC lần lượt là k1 , k2 . Giả sử tồn tại một đường tròn k tiếp xúc với tia đối của các tia AB
và CB, đồng thời tiếp xúc với AD, CD. Chứng minh rằng giao điểm của hai tiếp tuyến chung ngoài
của k1 và k2 nằm trên k.

C
O1
N
M O2 P
Q
HD I
A

145
Chứng minh. Gọi E, F, P, Q lần lượt là tiếp điểm của k(J) với AB, CD, AD, BC; M, N lần lượt là tiếp
điểm của k1 (O1 , R1 ) và k2 (O2 , R2 ) với AC. G, H lần lượt là điểm đối xứng với M, N qua O1 , O2 . I là
−−→ −→
điểm nằm trên k sao cho O1 G ↑↑ JI.
Theo định lý 2 suy ra B, G, I thẳng hàng.
Mặt khác, do BE = BQ nên BA + AE = BC + CQ.
Tương đương BA+AP = BC +CF ⇔ BA+AD+DP = BC +CD+DF ⇔ BA+AD = BC +CD.
AB + AC − BC CD + AC − AD
Từ đó AM = = = CN.
2 2
Suy ra N là tiếp điểm của đường tròn k3 bàng tiếp góc A của tam giác ABC với AC.
Áp dụng định lý 2 cho k1 , k3 , AC suy ra B, G, N thẳng hàng.
Chứng minh tương tự, I, D, H, M thẳng hàng.
IO1 R1
Mà GM k N H, O1 , O2 lần lượt là trung điểm GM, N H nên O1 , O2 , I thẳng hàng và = .
IO2 R2
Vậy I là tâm vị tự của k1 và k2 hay giao điểm của hai tiếp tuyến chung ngoài của k1 và k2 nằm
trên k.

Bài 228. Cho ba cung γ1 , γ2 , và γ3 cùng dựng trên dây AC sao cho chúng cùng nằm trên một nửa
mặt phẳng bờ AC, cung γ2 nằm giữa hai cung γ1 và γ3 . B là một điểm nằm trên AC, h1 , h2 , h3 là ba
tia bắt đầu từ B sao cho chúng cùng nằm trên nửa mặt phẳng chứa ba cung tròn, đồng thời h2 nằm
giữa h1 và h3 . Gọi Vij là giao điểm của hi và γj (i = 1, 3). Gọi V\ ij Vkj Vkl Vil là tứ giác cong có cạnh là
\
các đoạn thẳng Vij Vil , Vkj Vkl và cung Vij Vkj , Vil Vkl . Ta gọi tứ giác cong đó là tứ giác ngoại tiếp nếu
tồn tại một đường tròn tiếp xúc với hai đoạn thẳng và hai cung. Chứng minh rằng nếu các tứ giác
cong V\11 V21 V22 V12 , V12 V22 V23 V13 , V21 V31 V32 V22 ngoại tiếp thì tứ giác V22 V32 V33 V23 cũng ngoại tiếp.
\ \ \ \ \ \ \

Chứng minh.
Bổ đề 11. Cho hai dây cung γ1 và γ2 cùng dựng trên dây AC sao cho chúng cùng nằm trên một nửa
mặt phẳng bờ AC. Gọi C1 , C2 là hai đường tròn tiếp xúc với γ1 và γ2 . Khi đó tâm vị tự ngoài của C1
và C2 nằm trên AC.

I2

Z
I1
Y

X O2

I
A C

O1

Chứng minh. Gọi (O1 , R1 ) và (O2 , R2 ) lần lượt là đường tròn chứa các dây cung γ1 , γ2 ; I là tâm vị
tự ngoài của C1 và C2 . X, Y là tiếp điểm của C1 , C2 với γ1 , Z, T là tiếp điểm của C1 , C2 với γ2 .
Áp dụng định Monge-D’Alembert cho bộ ba đường tròn ((O1 ), γ1 , γ2 ) và ((O2 ), γ1 , γ2 ) suy ra I, X, Y
thẳng hàng và I, Z, T thẳng hàng.
Gọi IA1 A2 là tiếp tuyến chung ngoài của γ1 , γ2 .
Phép nghịch đảo IIIA1 .IA2 : γ1 7→ γ2 , đồng thời các cặp đường thẳng I1 Z và I2 T , I1 X và I2 Y không
song song nên IZ.IT = IA1 .IA2 = IX.IY . Suy ra X, Y, Z, T đồng viên.

146
Xét ba đường tròn (O1 ), (O2 ), (XY T Z) có trục đẳng phương lần lượt là AC, XY, ZT nên AC, XY, ZT
đồng quy. Từ đó I ∈ AC.
Trở lại bài toán.

h2

h1 h3
V23

V33
V13

V22 V32

V12

V11 V31
V21

A B C

Kí hiệu ω1 , ω2 , ω3 lần lượt là đường tròn nội tiếp các tứ giác V\


11 V21 V22 V12 , V12 V22 V23 V13 , V21 V31 V32 V22 ;
\ \ \ \ \
ω4 là đường tròn tiếp xúc với γ3 , γ2 và h2 . Như vậy ta cần chứng minh ω4 tiếp xúc với h3 .
Gọi I13 , I23 , I24 lần lượt là tâm vị tự ngoài của ω1 và ω3 , ω2 và ω3 , ω2 và ω4 .
Áp dụng bổ đề 1 suy ra I13 , I24 ∈ AC. Do B ∈ AC là tâm vị tự ngoài của ω1 và ω2 nên theo định
lý Monge-D’Alembert ta thu được I23 ∈ AC.
Lại áp dụng định lý Monge-D’Alembert cho ba đường tròn ω2 , ω3 , ω4 suy ra tâm vị tự ngoài của
ω3 và ω4 nằm trên AC. Do tiếp tuyến chung ngoài của ω3 và ω4 giao AC tại B nên B chính là tâm vị
tự ngoài của ω3 và ω4 . Suy ra ω4 tiếp xúc với h3 .

Nhận xét 2. Bằng phương pháp tương tự ta có thể chứng minh bài toán sau đây.
Bài 5.1. Cho tứ giác lồi ABCD. AB giao CD tại E, AD giao BC tại F . Gọi l1 , l2 là hai đường
thẳng qua E, l3 , l4 là hai đường thẳng qua F . Giả sử các tứ giác tạo bởi giao điểm của các bộ đường
thẳng (l1 , l3 , AD, CD), (l2 , l3 , AD, AB), (l2 , l4 , AB, BC) ngoại tiếp. Chứng minh rằng tứ giác tạo bởi
giao điểm của bộ đường thẳng (l1 , l4 , BC, CD) ngoại tiếp.

Bài 229. (Bài toán Apollonius). Dựng đường tròn tiếp xúc với ba đường tròn phân biệt trên mặt
phẳng.

147
D
l
A
O2 C2
J2
C1 J1
O1 M
B C

I1
L I2
N
F
C3 O3 J3

Chứng minh. (Gergonne). Để ngắn gọn chúng ta tìm lời giải cho bài toán trong trường hợp ba đường
tròn C1 (O1 ), C2 (O2 ), C3 (O3 ) đôi một ngoài nhau. Các trường hợp còn lại có thể chứng minh tương tự.
Giả sử ta đã dựng được các đường tròn trên. Gọi (I1 ) là đường tròn tiếp xúc ngoài với C2 và
tiếp xúc trong với C1 , C3 , (I2 ) là đường tròn tiếp xúc trong với C2 và tiếp xúc ngoài với C1 , C3 .
(A, B), (C, D), (E, F ) lần lượt là tiếp điểm của C1 , C2 , C3 với (I1 ), (I2 ).
Gọi M, N lần lượt là tâm vị tự trong của các cặp đường tròn C1 và C2 , C2 và C3 , P là tâm vị tự
ngoài của C1 và C3 .
Theo định lý Monge-D’Alembert, M, N, P thẳng hàng.
Mặt khác, áp dụng định lý Monge-D’Alembert cho ba đường tròn C1 , C2 , (I1 ) suy ra A, M, C thẳng
hàng. Tương tự, B, M, D thẳng hàng.
Gọi KM H là tiếp tuyến chung của C1 và C2 . Đặt M K.M H = k. Phép nghịch đảo IM k : C 7→ C
1 2
và cặp đường thẳng AO1 và CO2 không song song nên M A.M C = k. Tương tự suy ra A, B, C, D cùng
thuộc một đường tròn và M thuộc trục đẳng phương của (I1 ) và (I2 ). Như vậy đường thẳng l đi qua
M, N, P là trục đẳng phương của (I1 ) và (I2 ).
Gọi L là giao của AB và CD thì LA.LB = LC.LD.
Nghĩa là L thuộc trục đẳng phương của C1 và C2 . Tương tự ta thu được L là tâm đẳng phương
của C1 , C2 , C3 .
Mặt khác, dễ thấy cực của các đường thẳng AB, CD, EF ứng với C1 , C2 , C3 lần lượt nằm trên
trục đẳng phương của (I1 ) và (I2 ) nên AB, CD, EF lần lượt đi qua cực của đường thẳng l ứng với
C1 , C2 , C3 .
Như vậy ta có cách dựng như sau:
Dựng đường thẳng l đi qua ba tâm vị tự M, N, P của C1 , C2 , C3 ; J1 , J2 , J3 lần lượt là cực của l
ứng với C1 , C2 , C3 .
Dựng tâm đẳng phương L của C1 , C2 , C3 .
LJ1 , LJ2 , LJ3 lần lượt cắt C1 , C2 , C3 tại A, B, C, D, E, F . Hai đường tròn ngoại tiếp tam giác ACE
và BDF là đường tròn cần dựng.
Rõ ràng ứng với mỗi đường thẳng đi qua ba tâm vị tự của C1 , C2 , C3 ta dựng được hai đường tròn,
mà có tất cả 4 đường thẳng đi qua ba tâm vị tự của C1 , C2 , C3 nên bài toán có 8 nghiệm hình.

Bài 230. (Yetti). Cho tam giác ABC nội tiếp đường tròn (O). P là điểm bất kì nằm trên cạnh BC, AP
cắt (O) lần thứ hai tại D. Gọi (I1 ), (I2 ) lần lượt là đường tròn nội tiếp tam giác AP B, AP C; (J1 ), (J2 )
lần lượt là đường tròn tiếp xúc với các cặp tia (P B, P D), (P C, P D) và cùng tiếp xúc trong với (O).
Chứng minh rằng I1 I2 , J1 J2 , BC đồng quy.

Chứng minh. Trước tiên ta phát biểu một bổ đề.

148
Bổ đề 12. Cho tứ giác ABCD nội tiếp đường tròn (O). AC giao BD tại E. Gọi (I) là đường tròn nội
tiếp tam giác AEB, (J) là đường tròn tiếp xúc trong với (O) tại N và tiếp xúc với các tia EC, ED.
Gọi M là điểm chính giữa cung AB không chứa N . Khi đó M, I, N thẳng hàng.

B
M

A K
L I

E
O
H
G I2
I1

D J

N C

Chứng minh.
Gọi G, H lần lượt là tiếp điểm của (J) với BD, AC. L, K lần lượt là điểm chính giữa cung BD
chứa A, cung AC chứa B. BI giao AL tại I1 , AI giao BK tại I2 .
Áp dụng định lý 2 cho (J), BD, (O) suy ra N, G, L thẳng hàng. Tương tự, N, H, K thẳng hàng.
Ta có AI1 là phân giác ngoài của ∠N AB nên I2 là tâm đường tròn bàng tiếp góc B của 4ABD.
Tương tự, I2 là tâm đường tròn bàng tiếp góc A của tam giác ABC.
Theo định lý Sawayama-Thébault, I1 , I2 , G, H thẳng hàng.
Ta có ∠N GH = ∠N HC = ∠N BK suy ra tứ giác N GBI2 nội tiếp.
1 1
∠BII2 = (∠BAE + ∠ABE) = 90o − ∠AEB = ∠BGI2 .
2 2
Suy ra tứ giác GIBI2 nội tiếp. Từ đó 5 điểm N, G, I, B, I2 cùng thuộc một đường tròn.
Suy ra ∠IN B = ∠II2 B. (1)
Tương tự, ∠IN A = ∠II1 A. (2)
Mặt khác, AI, BI lần lượt đi qua điểm chính giữa các cung BC, AD nên ∠I1 AI2 = ∠I1 BI2 .
Từ đó tứ giác I1 ABI2 nội tiếp, suy ra ∠II1 A = ∠II2 B. (3)
Từ (1), (2), (3) suy ra N I là phân giác góc AN B hay M, I, N thẳng hàng.
Trở lại bài toán.
A
M
N

I1
I2
B C
L
D

J1
K
J2

E G

Gọi K, G lần lượt là tiếp điểm của (J1 ), (J2 ) với (O). M, N lần lượt là điểm chính giữa các cung
AC, AB.

149
Áp dụng định lý Monge-D’Alembert cho ba đường tròn (O), (J1 ), (J2 ) suy ra BC, KG, J1 J2 đồng
quy tại L.
Áp dụng định lý Pascal cho 6 điểm N, M, B, C, K, G suy ra giao điểm của các cặp đường thẳng
(N C, M B), (N G, M K), (BG, CK) thẳng hàng.
Từ đó theo định lý Desargues, I1 I2 , BC, J1 J2 đồng quy. Suy ra đpcm.

Nhận xét 3. Dựa vào lời giải trên, ta có thể chứng minh một bài toán của tác giả, một điều khá thú
vị và trùng hợp là bài toán về sau nằm trong cuộc thi Olympic toàn nước Nga 2010.

Bài 231. Cho tứ giác ABCD nội tiếp đường tròn (O). AC giao BD tại K. Gọi (I1 ), (I2 ), (I3 ), (I4 ) lần
lượt là đường tròn nội tiếp tam giác ABC, BCD, CDA, DAB. M1 , M2 , M3 , M4 lần lượt là điểm chính
giữa các cung AB, BC, CD, DA. Chứng minh rằng I1 M1 , I2 M2 , I3 M3 , I4 M4 , OK đồng quy.

B
M1
L N1 N2

A I1 M2

K I2
N4
J4
M4
O

J3
C

N3 M3

Chứng minh. Gọi (J1 ), (J2 ), (J3 ), (J4 ) lần lượt là các đường tròn tiếp xúc với các cặp tia (KA, KB),
(KB, KC), (KC, KD), (KD, KA) và tiếp xúc trong với (O) lần lượt tại N1 , N2 , N3 , N4 .
Theo bổ đề 2 ta có M1 , I1 , N3 thẳng hàng và M2 , I2 , N4 thẳng hàng. Do đó ta chỉ cần chứng minh
I1 N3 , I2 N4 , OK đồng quy.
Theo kết quả của bài toán trên, N3 N4 , J3 J4 , I1 I2 đồng quy tại một điểm trên AC.
Áp dụng định lý Desargues cho ba đường thẳng N3 N4 , J3 J4 , I1 I2 suy ra giao điểm của các cặp
đường thẳng (I1 N3 , I2 N4 ), (N4 J4 , N3 J3 ), (I1 J3 , I2 J4 ) thẳng hàng, tức là giao điểm của I1 N3 và I2 N4
nằm trên OK.
Chứng minh tương tự ta thu được I1 M1 , I2 M2 , I3 M3 , I4 M4 , OK đồng quy.

Bài 232. (IMO Shortlist 2007). Cho tứ giác lồi ABCD. P là một điểm trên cạnh AB. Gọi (I), (I1 ), (I2 )
lần lượt là là đường tròn nội tiếp tam giác DP C, AP D, BP C. Giả sử (I) tiếp xúc với (I1 ), (I2 ) lần
lượt tại K, L. AC giao BD tại E, AK giao BL tại F . Chứng minh rằng E, I, F thẳng hàng.

150
A
P
M
B
I1
K
E I2 Q
L
I
F
J

D
N
C

Chứng minh. (Cosmin Pohoata). Gọi (J) là đường tròn tiếp xúc với AB và các tia AD, BC.
Áp dụng định lý Monge-D’Alembert cho ba đường tròn (J), (I), (I1 ) suy ra AK đi qua tâm vị tự
trong của (I) và (J). Tương tự ta thu được F là tâm vị tự trong của (I) và (J).
Như vậy ta chỉ cần chứng minh E nằm trên IJ.
Gọi M, Q lần lượt là tiếp điểm của (I2 ) với AB, BC; N là tiếp điểm của (I) với CD.
Ta có P M = P L = P K, CQ = CL = CN . Do đó P B + DC = P M + M B + DN + N C =
P K + BQ + DK + CQ = DP + BC.
Suy ra tứ giác DP BC ngoại tiếp.
Tương tự, tứ giác AP CD ngoại tiếp đường tròn ω1 .
Áp dụng định lý Monge-D’Alembert cho ba đường tròn (J), (I), ω1 suy ra AC đi qua tâm vị tự
ngoài của (I) và (J).
Tương tự suy ra giao điểm của AC và BD là tâm vị tự ngoài của (I) và (J). Nghĩa là E ∈ IJ. Ta
có đpcm.

15.3 Bài tập tự luyện


Bài 233. Cho tam giác ABC nội tiếp đường tròn (O), ngoại tiếp đường tròn (I). Gọi A1 , B1 , C1 lần
lượt là tiếp điểm của (I) với BC, CA, AB; A2 , B2 , C2 lần lượt là điểm chính giữa các cung BC, CA, AB
chứa A, B, C. Chứng minh rằng A1 A2 , B1 B2 , C1 C2 đồng quy.
Bài 234. (ELMO Shortlist 2011). Cho tam giác ABC. Kí hiệu ωa là đường tròn tiếp xúc với các tia
AB, AC; ωb , ωc được định nghĩa tương tự. Gọi Pa là tâm vị tự trong của ωb và ωc . Tương tự với Pb , Pc .
Chứng minh rằng APa , BPb , CPc đồng quy.
Bài 235. (ELMO Shortlist 2011). Cho ba đường tròn ω, ω1 (O1 ), ω2 (O2 ) sao cho ω1 và ω2 tiếp xúc
ngoài nhau tại P và cùng tiếp xúc trong với ω tại A và B. Gọi X là hình chiếu vuông góc của P trên
AB. Chứng minh rằng ∠O1 XP = ∠O2 XP .
Bài 236. Cho hai đường tròn trực giao (O1 ) và (O2 ). Gọi X, Y lần lượt là hai điểm nằm trên (O1 ) và
(O2 ) sao cho giao điểm của O1 Y và O2 X nằm trên trục đẳng phương của (O1 ) và (O2 ). Chứng minh
rằng XY đi qua tâm vị tự ngoài của (O1 ) và (O2 ).
Bài 237. Cho hình sao ABCDE và một đường tròn (O) chứa ABCDE. Gọi (Oa ), (Ob ), (Oc ), (Od ), (Oe )
lần lượt là đường tròn tiếp xúc trong với (O) tại A1 , B1 , C1 , D1 , E1 và lần lượt tiếp xúc với tia đối của các
cặp tia (AC, AD), (BD, BE), (CA, CE), (DB, DA), (EC, EB). Chứng minh rằng AA1 , BB1 , CC1 , DD1 ,
EE1 đồng quy.

151
Bài 238. (Nguyễn Văn Linh). Cho tứ giác ABCD nội tiếp đường tròn (O), ngoại tiếp đường tròn
(I). Gọi X, Y, Z, T lần lượt là tiếp điểm của (I) với AB, BC, CD, DA. (O1 ) là đường tròn tiếp xúc với
(O) và tiếp xúc với cạnh AB tại X sao cho (O1 ) và C, D nằm khác phía đối với đường thẳng AB.
(O2 ), (O3 ), (O4 ) được định nghĩa tương tự. Chứng minh rằng các đường thẳng kẻ từ A, B, C, D và lần
lượt vuông góc với O4 O1 , O1 O2 , O2 O3 , O3 O4 đồng quy tại một điểm nằm trên OI.

Bài 239. (Petrisor Neagoe). Cho tam giác ABC. ω là một đường tròn bất kì qua hai điểm B, C.
AB, AC cắt ω lần thứ hai tại E, F . Gọi X, Y là điểm chính giữa cung BE, CF lần lượt không chứa
C, B. AX, AY cắt ω lần thứ hai tại P, Q. Gọi γ là đường tròn tiếp xúc với các cạnh AB, AC tại M, N
và tiếp xúc trong với ω. Gọi U, V là giao điểm của (P XM ) và (QY N ). Chứng minh rằng tứ giác
XU Y V là hình bình hành.

Bài 240. (Nguyễn Văn Linh). Cho tam giác ABC nội tiếp đường tròn (O). Kẻ đường tròn (O0 ) tiếp
xúc trong với (O) tại A và tiếp xúc với cạnh BC. Từ B, C kẻ tiếp tuyến khác BC tới (O0 ) và cắt nhau
tại D. DA giao (O) tại E. Hai tiếp tuyến của (O0 ) qua E giao BC tại G, H. Chứng minh rằng đường
tròn (EGH) tiếp xúc với (O).

Bài 241. (Romanian Masters in mathematics 2010). Cho tứ giác lồi A1 A2 A3 A4 không có cặp cạnh
nào song song. Kí hiệu ωi là đường tròn tiếp xúc với cạnh Ai Ai+1 tại Ti và tiếp xúc với các tia
Ai−1 Ai , Ai+2 Ai+1 (i = 1, 4, A0 ≡ A4 , A1 ≡ A5 , A2 ≡ A6 ). Chứng minh rằng A1 A2 , A3 A4 , T2 T4 đồng
quy khi và chỉ khi A2 A3 , A4 A1 , T1 T3 đồng quy.

Bài 242. (Nguyễn Văn Linh). Cho tam giác nhọn ABC. P là điểm nằm trong tam giác sao cho
AB + BP = AC + CP . BP, CP giao AC, AB lần lượt tại Y, Z. Chứng minh rằng tâm vị tự ngoài của
đường tròn nội tiếp các tam giác AP Y và AP Z nằm trên BC.

Bài 243. (Nguyễn Văn Linh). Cho tam giác ABC nội tiếp đường tròn (O), ngoại tiếp đường tròn
(I). Gọi D, E, F lần lượt là tiếp điểm của (I) với BC, CA, AB. D0 , E 0 , F 0 đối xứng với D, E, F qua I.
Dựng đường tròn tiếp xúc trong với (I) tại D0 và tiếp xúc với (O) tại A1 . A2 là tiếp điểm của đường
tròn "Mixtilinear incircle" ứng với đỉnh A với (O); A3 là giao của đường tròn đường kính AI với (O).
Tương tự ta xác định B1 , B2 , B3 , C1 , C2 , C3 . Chứng minh rằng AA1 , BB1 , CC1 , A2 A3 , B2 B3 , C2 C3 , OI
đồng quy.

16 Định lý Casey
16.1 Giới thiệu
Định lý Casey đặt theo tên nhà toán học John Casey, hay còn gọi là định lý Ptolemy mở rộng [1],
được phát biểu như sau:
Cho bốn đường tròn Ci (i = 1, 4). Kí hiệu tij là độ dài tiếp tuyến chung của hai đường tròn Ci và
Cj . Khi đó bốn đường tròn Ci cùng tiếp xúc với một đường tròn (hoặc đường thẳng) C khi và chỉ khi

t12 t34 ± t13 t42 ± t14 t23 = 0

152
t12

C1

C2
t24
t23
C
t14

t13 C3
C4

t34

Chú ý rằng tiếp tuyến được chọn của hai đường tròn Ci , Cj là tiếp tuyến chung ngoài khi và chỉ
khi cả hai đường tròn Ci , Cj cùng tiếp xúc trong (hoặc ngoài) với C, là tiếp tuyến chung trong khi và
chỉ khi trong hai đường tròn Ci , Cj có một đường tròn tiếp xúc trong, một đường tròn tiếp xúc ngoài
với C. Dấu của tij tkl là dấu "+" khi và chỉ khi các đoạn thẳng nối hai tiếp điểm của Ci và Cj , Ck và
Cl không cắt nhau, là dấu "−" khi và chỉ khi ngược lại.
Dễ dàng nhận thấy khi bốn đường tròn trên cùng suy biến thành đường tròn điểm, định lý Casey
trở thành định lý Ptolemy [2].
Khi ba đường tròn suy biến thành đường tròn điểm, định lý Casey trở thành định lý Purser [3].

16.2 Chứng minh định lý


Lời giải sau dựa theo [4].
Ta phát biểu và chứng minh một bổ đề.

Bổ đề 13. Cho hai đường tròn (O1 , R1 ) và (O2 , R2 ) không chứa nhau. I là một điểm nằm ngoài hoặc
nằm trong cả hai đường tròn. Phép nghịch đảo cực I phương tích R2 lần lượt biến (O1 , R1 ), (O2 , R2 )
thành (O10 , R10 ), (O20 , R20 ). Gọi T12 , T12
0 là độ dài tiếp tuyến chung ngoài (hoặc trong nếu có) của hai
2
T12 T 02
đường tròn (O1 , R1 ) và (O2 , R2 ); (O10 , R10 ) và (O20 , R20 ). Khi đó = 0 12 0 .
R1 R2 R1 R2

J
N

D'
M
A'

O P

D
A
O1 B C O2

B' C'

Chứng minh. Ta chỉ chứng minh trường hợp tiếp tuyến chung ngoài, trường hợp chung trong chứng
minh tương tự.

153
Gọi (O) là đường tròn trực giao với (O1 ) và (O2 ). (O) giao (O1 ) tại A0 , B 0 , giao (O2 ) tại C 0 , D0 .
Lấy J trên (O) sao cho J nằm trên trục đẳng phương của (O1 ) và (O2 ). Gọi k là phương tích từ J đến
hai đường tròn (O1 ) và (O2 ). Xét phép nghịch đảo IJk :
A0 7→ A, B 0 7→ B, C 0 7→ C, D0 7→ D.
Do A0 , B 0 , C 0 , D0 ∈ (O) nên A, B, C, D thẳng hàng. Phép nghịch đảo bảo toàn độ lớn góc giữa hai
đường cong tại giao điểm nên A, O1 , B thẳng hàng và C, O2 , D thẳng hàng. Từ đó A, B, C, D nằm trên
đường nối tâm O1 O2 .
Không mất tổng quát giả sử A, B, C, D nằm trên O1 O2 theo thứ tự, R1 < R2 . Gọi M N là tiếp
tuyến chung ngoài của (O1 ) và (O2 )(M ∈ (O1 ), N ∈ (O2 )).
Gọi P là hình chiếu vuông góc của O1 trên O2 N .
Ta có M N 2 = O1 P 2 = O1 O22 − P O22 = O1 O22 − (R1 − R2 )2 = (O1 O2 − R1 + R2 )(O1 O2 + R1 − R2 ) =
BD.CA. √
B 0 D0 JD0 JB 0 JD0 .JB 0 k
Xét = = = √ = .
BD JB JD JD.JB JD.JB
B 0 D0 .JD.JB C 0 A0 .JC.JA
2
T12 BD.CA . B 0 D0 .C 0 A0
Tương tự ta thu được = = 0 0 k 0 0
k = 0 0 0 0.
4R1 R2 AB.CD A B .JA.JB C D .JC.JD A B .C D
.
2
k k
IIR : (O) 7→ (O0 ), A0 7→ A00 , B 0 7→ B 00 , C 0 7→ C 00 , D0 7→ D00 .
Khi đó (O0 ) là đường tròn trực giao với (O10 ) và (O20 ).
02
T12 B 00 D00 .C 00 A00 B 0 D0 .C 0 A0
Một cách tương tự ta cũng chứng minh được = = .
4R10 .R20 A00 B 00 .C 00 D00 A0 B 0 .C 0 D0
2
T12 T 02
Vậy = 0 12 0 .
R1 R2 R1 .R2

Trở lại bài toán.


Trước tiên ta chứng minh chiều thuận của định lý Casey cho trường hợp cả bốn đường tròn Ci đều
tiếp xúc trong với C, các trường hợp còn lại chứng minh tương tự.
Kí hiệu ri , Oi là bán kính và tâm của đường tròn Ci ; r, O là bán kính và tâm của đường tròn C.
Không mất tổng quát giả sử r4 = M in{ri }. Ta nhận thấy các đường tròn (O1 , r1 − r4 ), (O2 , r2 −
r4 ), (O3 , r3 − r4 ), (O4 , 0) cùng tiếp xúc với đường tròn (O, r − r4 ) và độ dài tiếp tuyến chung của hai
đường tròn (Oi , ri − k), (Oj , rj − k)(k ∈ (0, M in{ri , rj })) bằng tij . Do đó ta chỉ cần chứng minh chiều
thuận của định lý Casey cho bốn đường tròn (O1 , r1 − r4 ), (O2 , r2 − r4 ), (O3 , r3 − r4 ), (O4 , 0) (Kí hiệu
là Q1 , Q2 , Q3 , O4 với bán kính lần lượt là R1 , R2 , R3 , 0).
Xét phép nghịch đảo IO R2 : Q 7→ Q0 , Q 7→ Q0 , Q 7→ Q0 , (O, r − r ) 7→ l và Q0 , Q0 , Q0 cùng tiếp
4 1 1 2 2 3 3 4 1 2 3
xúc một phía với l. Gọi t0ij là độ dài tiếp tuyến chung ngoài của Q0i và Q0j . Không mất tổng quát giả sử
tiếp điểm của Q02 và l nằm trênsđoạn thẳng nối hai tiếp điểm của Q01 và Q03 với l. Ta có t012 +t023 −t013 = 0.
Ri .Rj
Áp dụng bổ đề 1,tij = t0ij . (i, j = 1, 3).
Ri0 .Rj0
s
Ri0 t04i t04i .t4i R2 Ri
= = 2 = 2 do đó t4i = R. .
Ri t4i t4i t4i Ri0
s
R1 .R2 .R3
Từ đó t12 .t34 + t23 .t14 − t13 .t24 = (t012 + t023 − t013 ).R = 0.
R10 .R20 .R30
Chiều thuận được chứng minh.
Bây giờ giả sử t12 .t34 + t23 .t14 − t13 .t24 = 0. Ta sẽ chứng minh Q1 , Q2 , Q3 , (O4 , 0) cùng tiếp xúc với
một đường tròn.
Sử dụng phép nghịch đảo IO R2 ta cũng suy ra t0 + t0 − t0 = 0.
4 12 23 13
Không mất tổng quát giả sử R30 = M in{R10 , R20 , R30 }. Cùng giảm bán kính ba đường tròn Q01 , Q02 , Q03
một đoạn R30 ta được ba đường tròn (I1 , q1 ), (I2 , q2 ), (I3 , 0).

154
Ta tìm quỹ tích các điểm I3 sao cho t013 − t023 = t012 .
Tập hợp 0
p các điểm sao cho tiếp tuyến kẻ từ điểm đó đến (I1 , q1 ) bằng t13 là đường tròn tâm I1 ,
bán kính q12 + t02 0
13 , tập
p hợp các điểm sao cho tiếp tuyến kẻ từ điếm đó đến (I2 , q2 ) bằng t23 là đường
2 02
tròn tâm I2 , bán kính q2 + t23 . Hai đường tròn này cắt nhau tại hai điểm nằm trên tiếp tuyến chung
ngoài của (I1 , q1 ) và (I2 , q2 ).
Vậy I3 nằm trên tiếp tuyến chung ngoài của (I1 , q1 ) và (I2 , q2 ). Suy ra Q01 , Q02 , Q03 cùng tiếp xúc với
một đường thẳng. Nghĩa là tồn tại một đường tròn đi qua O4 và tiếp xúc với Q1 , Q2 , Q3 .
Vậy tồn tại một đường tròn tiếp xúc với bốn đường tròn Ci .
Định lý Casey được chứng minh.

Chiều thuận của bài toán có thể chứng minh theo hướng sau đây không sử dụng phép nghịch đảo,
được coi là hệ quả của định lý Ptolemy.

Bổ đề 14. Cho hai đường tròn C1 (O, r1 ) và C2 (O2 , r2 ) cùng tiếp xúc với C(O, R) lần lượt tại hai điểm
A, B. Khi đó độ dài tiếp tuyến chung (trong hoặc ngoài) của C1 và C2 được tính bởi công thức:

AB p
t12 = (R ± r1 )(R ± r2 )
R

N
M
O O2

B
O1

Chứng minh. Ta chứng minh bổ đề trong trường hợp C1 và C2 cùng tiếp xúc trong với C.
Dễ thấy t212 = O1 O22 − (r1 − r2 )2 .
Áp dụng định lý hàm số cos cho tam giác OO1 O2 và AOB ta có:
O1 O22 = OO12 + OO22 − 2OO1 .OO2 . cos ∠O1 OO2 .
AB 2 = 2R2 (1 − cos ∠O1 OO2 )
AB 2
Từ đó t212 = (R − r1 )2 + (R − r2 )2 − 2(R − r1 )(R − r2 )(1 − ) − (r1 − r2 )2
2R2
AB 2 AB 2
= (r1 − r2 )2 − (r1 − r2 )2 + 2(R − r1 )(R − r2 ) = (R − r1 )(R − r2 )
2R2 R2
AB p
Vậy t12 = (R − r1 )(R − r2 ).
R
AB p
Tương tự nếu C1 và C2 cùng tiếp xúc ngoài với C, t12 = (R + r1 )(R + r2 ), nếu C1 và C2 tiếp
R
xúc khác phía với C, chẳng hạn C1 tiếp xúc trong còn C2 tiếp xúc ngoài, t12 là độ dài của tiếp tuyến
AB p
chung trong được tính bởi công thức t12 = (R − r1 )(R + r2 ).
R
Trở lại bài toán.
Gọi A, B, C, D lần lượt là tiếp điểm của C1 , C2 , C3 , C4 với C. Áp dụng bổ đề 2 ta có:
AB.CD + AD.BC − AC.BD p
t12 .t34 + t14 .t23 − t13 .t24 = . (R − r1 )(R − r2 )(R − r3 )(R − r4 ).
R2

155
Do tứ giác ABCD nội tiếp nên theo định lý Ptolemy, AB.CD + AD.BC − AC.BD = 0.
Vậy t12 .t34 + t14 .t23 − t13 .t24 = 0. Chiều thuận của định lý Casey được chứng minh.

16.3 Ứng dụng


Bài 244. Cho tam giác ABC nội tiếp đường tròn (O). GọiCa , Cb , Cc lần lượt là các đường tròn tiếp xúc
trong với (O) tại điểm chính giữa cung nhỏ BC, CA, AB và lần lượt tiếp xúc với các cạnh BC, CA, AB.
Gọi tbc , tca , tab là độ dài các tiếp tuyến chung ngoài của các cặp đường tròn (Cb , Cc ), (Cc , Ca ), (Ca , Cb ).
a+b+c
Khi đó tbc = tca = tab = .
4

Cb

Cc

B C
Ca

Chứng minh. Gọi ta , tb , tc là độ dài các tiếp tuyến kẻ từ A, B, C tới Ca , Cb , Cc . Do Ca , Cb , Cc tiếp xúc
trong với (O) tại điểm chính giữa các cung nhỏ BC, CA, AB nên ba đường tròn lần lượt tiếp xúc với
BC, CA, AB tại trung điểm mỗi cạnh.
Áp dụng định lý Casey cho bốn đường tròn Ca , (A, 0), (B, 0), (C, 0) ta có
a a
ta .a = .b + .c
2 2
b+c a+c a+b
Suy ra ta = . Tương tự, tb = , tc = .
2 2 2
Lại áp dụng định lý Casey cho bốn đường tròn Ca , Cc , (A, 0), (C, 0) ta có
c a
ta .tc = . + tac .b
2 2
ac (b + c)(a + b) ac
ta .tc − −
Từ đó tac = 4 = 4 4 = a + b + c.
b b 4
Tương tự suy ra đpcm.

Bài 245. (Định lý Feuerbach) Chứng minh rằng đường tròn Euler của tam giác tiếp xúc với các đường
tròn nội tiếp và bàng tiếp tam giác đó.

156
A

B2

C1 B1

I
C2
E

B A2 A1 C

Chứng minh. Ta chứng minh bài toán cho trường hợp đường tròn nội tiếp. Các trường hợp khác chứng
minh tương tự.
Giả sử ta có tam giác ABC với A1 , B1 , C1 lần lượt là trung điểm các cạnh BC, CA, AB; A2 , B2 , C2
lần lượt là tiếp điểm của đường tròn nội tiếp (I) với BC, CA, AB. Đặt độ dài các cạnh tam giác ABC
lần lượt là a, b, c.
Áp dụng định lý Casey cho bốn đường tròn (I), (A1 , 0), (B1 , 0), (C1 , 0) với độ dài các tiếp tuyến
a b c |b − c| |c − a| |a − b|
chung lần lượt là B1 C1 = , C1 A1 = , A1 B1 = , A1 A2 = , B1 B2 = , C1 C2 = .
2 2 2 2 2 2
Ta có thể chọn các dấu "+","−" sao cho a|b − c| ± b|c − a| ± c|a − b| = 0 nên tồn tại đường tròn
tiếp xúc với (I), (A1 , 0), (B1 , 0), (C1 , 0) hay đường tròn Euler của tam giác ABC tiếp xúc với (I).

Bài 246. Cho tam giác ABC ngoại tiếp (I) có A1 , B1 , C1 lần lượt là trung điểm BC, CA, AB. F là
tiếp điểm của (I) và đường tròn Euler (E). Khi đó ta có thể chọn các dấu "+","−" sao cho

F A1 ± F B1 ± F C1 = 0.

Chứng minh. Gọi R, r lần lượt là bán kính đường tròn Euler và (I).
Áp dụng bổ đề 2 cho đường tròn (I) và (B1 , 0) ta có

R R R
B1 F = p .B1 B2 , C1 F = p .C1 C2 , A1 F = p .A1 A2 .
(R − r)R (R − r)R (R − r)R

R
Do đó A1 F ± B1 F ± C1 F = p .(A1 A2 ± B1 B2 ± C1 C2 )
(R − r)R
R
= p .(|b − c| ± |c − a| ± |a − b|) = 0.
2 (R − r)R

Bài 247. (Đường tròn Hart). Cho ba đường tròn C1 , C2 , C3 cắt nhau lần lượt tại các cặp điểm (A, A0 ),
(B, B 0 ), (C, C 0 ). Gọi (I), (IA ), (IB ), (IC ) lần lượt là đường tròn nội tiếp tam giác cong ABC, A0 BC,
AB 0 C, ABC 0 . Khi đó tồn tại một đường tròn tiếp xúc ngoài với (IA ), (IB ), (IC ) và tiếp xúc trong với
(I), gọi là đường tròn Hart. Chú ý rằng có tất cả 8 đường tròn Hart ứng với ba đường tròn cho trước.
[4][5]

157
A'

IA
C2
B
C1
C
I
IC
IB

A C'

B'

C3

Chứng minh. Kí hiệu tij là độ dài tiếp tuyến chung ngoài, t0ij là độ dài tiếp tuyến chung trong của hai
đường tròn (Ii ) và (Ij ).
Do (IA ), (IB ), (IC ), (I) cùng tiếp xúc với C1 nên tAI .t0BC + tAB .t0CI − t0AC .tBI = 0.
Tương tự, bốn đường tròn cùng tiếp xúc với C2 nên tAC .t0BI + t0BC .tAI − t0AB .tCI = 0.
Bốn đường tròn cùng tiếp xúc với C3 nên tBC .t0AI + t0AB .tCI − t0AC .tBI = 0.
Từ ba đẳng thức trên suy ra tAC .t0BI − tBC .t0AI − tAB .t0CI = 0.
Lưu ý dấu "+" hay "−" trong các đẳng thức trên không quan trọng, có thể đảo dấu nhưng phải
thoả mãn trong một đẳng thức tồn tại cả "+" và "−".
Theo định lý Casey dạng đảo, tồn tại một đường tròn tiếp xúc ngoài với ba đường tròn (IA ), (IB ), (IC )
và tiếp xúc trong với (I).

Bài 248. (IMO 2011). Cho tam giác ABC nội tiếp đường tròn (O). l là tiếp tuyến bất kì của (O). Gọi
la , lb , lc lần lượt là đường thẳng đối xứng với l qua ba cạnh BC, CA, AB. la , lb , lc cắt nhau tạo thành
tam giác A0 B 0 C 0 . Khi đó đường tròn ngoại tiếp tam giác A0 B 0 C 0 tiếp xúc với (O).

A'
B1

C'
A
I

P O'

B
A1
M C

B'
C1

Chứng minh. Kí hiệu dX /l là khoảng cách từ X đến đường thẳng l. l cắt BC, CA, AB lần lượt tại
A1 , B1 , C1 . P là tiếp điểm của l và (O).
Do lc và l đối xứng nhau qua AB nên dB /l = dB /lc , la và l đối xứng nhau qua BC nên dB /l = dB /lA .
Từ đó dB /lA = dB /lc hay B 0 B là phân giác góc B 0 .

158
Tương tự suy ra AA0 , BB 0 , CC 0 đồng quy tại tâm nội tiếp I của tam giác A0 B 0 C 0 .
Mặt khác, (B 0 A0 , B 0 C 0 ) ≡ (B 0 A0 , BA) + (BA, BC) + (BC, B 0 C 0 ) ≡ (BA, l) + (BA, BC) + (l, BC) ≡
2(BA, BC) (mod π).
∠C 0 B 0 A0
Do đó ∠C 0 IA0 = 90o + = 180o − ∠ABC. Suy ra I ∈ (O).
2
Gọi pa , pb , pc lần lượt là phương tích từ A0 , B 0 , C 0 tới (O). Áp dụng định lý Casey cho bốn đường
tròn (A0 , 0), (B 0 , 0), (C 0 , 0) và (O), ta có (A0 B 0 C 0 ) tiếp xúc với (O) khi và chỉ khi:
√ √ √
B 0 C 0 . pa ± A0 C 0 . pb ± A0 B 0 . pc = 0
Đặt bán kính đường tròn ngoại tiếp tam giác ABC và A0 B 0 C 0 lần lượt là R và R0 , bán kính đường
tròn nội tiếp tam giác A0 B 0 C 0 là r.
B0C 0
Theo định lý hàm số sin, = 2R0 nên B 0 C 0 = 2R0 . sin(180o −2∠BAC) = 2R0 . sin 2∠BAC.
sin ∠B 0 A0 C 0
r r
A0 I = 0 0
=
sin ∠B A I cos ∠BAC
A0 A AB1 AB1 . sin ∠P B1 A dA /l
0
= 0 0
nên A0 A = = .
sin ∠AB1 A sin ∠C A I cos ∠BAC cos BAC
PA
dA /l = P A sin ∠B1 P A = P A sin ∠P CA = P A. .
2R
PA 2
Do đó A0 A = .
2R cos ∠BAC √ √
√ r r
Suy ra B 0 C 0 . AA0 .A0 I = 2R0 . sin 2∠BAC.P A. √ = P A. sin ∠BAC.4R0 . √
√ 2R cos ∠BAC 2R
0 r
= P A.BC.2R . √
2R.R
Tương tự và áp dụng định lý Ptolemy suy ra

0 0 √ 0 0 √ 0 0 √ 0 r
B C . pa ± A C . pb ± A B . pc = 2R . √ (P A.BC ± P B.AC ± P C.AB) = 0
2R.R
Vậy (A0 B 0 C 0 ) tiếp xúc với (O).

Bài 249. (Bổ đề Thébault). Cho tam giác ABC ngoại tiếp (I), nội tiếp (O). Một đường tròn (E) tiếp
xúc trong với (O) và tiếp xúc với các cạnh AB, AC lần lượt tại P, Q. Khi đó I là trung điểm P Q.
Chứng minh. Trước tiên ta giới thiệu một bổ đề.
Bổ đề 15. (Định lý Cristea). Gọi D, E, F lần lượt là ba điểm nằm trên các cạnh BC, CA, AB of
4ABC và M ∈ AD. Khi đó EF đi qua M khi và chỉ khi
FB EC MD
DC · + BD · = BC ·
FA EA MA
Chứng minh.
Giả sử M ∈ EF. Gọi {P } ≡ EF ∩ BC. Áp dụng định lý Menelaus cho 4DAB ứng với đường
thẳng P M F và P EM ta có:
FB MD PB EC PC MD
DC · = DC · · , BD · = BD · ·
FA MA PD EA PD MA
FB EC M D (DC · P B + BD · P C)
⇒ DC · + BD · = ·
FA EA MA PD
M D [(P D − P C) · P B + (P B − P D) · P C]
= ·
MA PD
FB EC M D P D · (P B − P C) MD
⇒ DC · + BD · = · = BC ·
FA EA MA PD MA
Chiều đảo của bổ đề chứng minh tương tự dựa theo chiều đảo của định lý Menelaus.
Trở lại bài toán.

159
A

I
P

B D C

Áp dụng định lý Casey cho bốn đường tròn (A, 0), (B, 0), (C, 0), (E) ta có:
c.(b − AP ) + b.(c − AP ) 2bc
AP.BC = AB.CQ + AC.BP hay AP = , suy ra AP = .
a a+b+c
DI a ab ac
Gọi {D} ≡ AI ∩ BC,ta có = , CD = , BD = .
AI b+c b+c b+c
Áp dụng bổ đề 3, ta cần chứng minh
BP CQ DI
CD · + BD · = BC ·
PA AQ AI
.
2bc 2bc
ab c− ac b−
Hay · a + b + c + · a + b + c = a · a , hoàn toàn đúng sau một số biến đổi.
b+c 2bc b+c 2bc b+c
a+b+c a+b+c
Suy ra I ∈ P Q. Mà tam giác AP Q cân tại A có phân giác AI nên I là trung điểm P Q.

Bài 250. (Bổ đề Sawayama). Cho tam giác ABC nội tiếp đường tròn (O), ngoại tiếp đường tròn (I).
D là điểm bất kì trên BC. Một đường tròn ω tiếp xúc với các tia DC, DA lần lượt tại E, F và tiếp
xúc trong với (O). Khi đó I nằm trên đường thẳng EF .

K
I

B J D E C

Chứng minh. Gọi J là giao của AI và BC. Theo định lý Menelaus, I, E, F thẳng hàng khi và chỉ khi
EJ F D IA
. . =1
ED F A IJ

160
EJ IA
Tương đương . = 1.
F A IJ
IA b+c
Mà = nên ta cần chứng minh a.F A = (b + c).EJ (1)
IJ a
Áp dụng định lý Casey cho bốn đường tròn (A, 0), (B, 0), (C, 0), ω ta có
AF.BC + AB.CE = AC.BE
Hay a.AF + c.(BC − BE) = b.BE ⇔ a.AF + ac = BE.(b + c).
ac
Do BJ = nên a.AF + (b + c).BJ = BE.(b + c)
b+c
Từ đó a.AF = (b + c).(BE − BJ) = (b + c).EJ. Như vậy (1) đúng, tức là I nằm trên đường thẳng
EF .

Bài 251. Cho tam giác ABC ngoại tiếp đường tròn (I). Gọi ωa là đường tròn qua hai điểm B, C và
tiếp xúc với (I), ωa0 là đường tròn tiếp xúc với các tia AB, AC và tiếp xúc ngoài với ωa . Tương tự ta xác
định ωb0 , ωc0 . Gọi r, ra , rb , rc lần lượt là bán kính các đường tròn (I), ωa0 , ωb0 , ωc0 . Khi đó r = ra + rb + rc .

C2 B2
Ia

B' C'
A2 T B1

C1
I

B A1 C

Chứng minh. Kẻ tiếp tuyến qua T của đường tròn (I) và song song với BC, cắt AB, AC lần lượt tại
B 0 , C 0 . Ta chứng minh đường tròn nội tiếp tam giác AB 0 C 0 là ωa0 .
Gọi A1 , B1 , C1 là tiếp điểm của (I) với BC, CA, AB; A2 , B2 , C2 là tiếp điểm của đường tròn nội
tiếp (Ia ) của tam giác AB 0 C 0 với B 0 C 0 , C 0 A, AB 0 . Kí hiệu p, p0 là nửa chu vi tam giác ABC, AB 0 C 0 .
p−a
Ta có p0 = AB1 = p − a nên hai tam giác AB 0 C 0 và ABC đồng dạng theo tỉ số .
p
(p − a)a
Suy ra B 0 C 0 = .
p
(p − a)a (p − a)2
BC2 = c − AC2 = p0 − B 0 C 0 = c − [(p − a) − ]=c− .
p p
(p − a)2
Tương tự, CB2 = b − .
p
(p − a)a (p − a)b (p − a)
A2 T = B 0 C 0 − 2B 0 A2 = − 2(p − a − ) = (b − c) (với giả thiết b > c).
p p p
Áp dụng định lý Casey cho bốn đường tròn (B, 0), (C, 0), (I), (Ia ) ta có:

(p − a)2 (p − a) (p − a)2
BA1 .CB2 + BC.A2 T − BC2 .CA1 = (p − b).(b − ) + a.(b − c) − (c − ).(p − c) = 0
p p p

161
nên (Ia ) tiếp xúc ngoài với ωa hay (Ia ) ≡ ωa0 .
ra p−a rb p − b rc p−c
Từ đó = . Tương tự = , = .
r p r p r p
ra + rb + rc 3p − a − b − c
Suy ra = = 1 hay r = ra + rb + rc .
r p

Bài 252. (Juan Carlos). Gọi AB và CD là hai dây cung song song của đường tròn (O). Hai đường
tròn (O1 ) và (O2 ) cùng tiếp xúc ngoài với (O) và có AB là tiếp tuyến chung sao cho (O1 ), (O2 )
và CD nằm khác phía với AB. Kí hiệu tP Oi là tiếp tuyến kẻ từ P tới đường tròn (Oi ). Khi đó
tCO1 + tCO2 = tDO1 + tDO2 .

C D

H A B I

K J

P O2
O1

Chứng minh. Gọi I, J, H, K lần lượt là tiếp điểm của (O2 ), (O1 ) với AB, (O).
Gọi P là giao điểm của HK và (O). Do K là tâm vị tự của (O) và (O1 ) nên OP k O1 A hay
OP ⊥ AB.
Suy ra P là điểm chính giữa cung AB. Tương tự suy ra HK, IJ, (O) đồng quy tại P .
1 1
Mặt khác, ∠AHK = ∠HO1 K = ∠KOP = ∠KJP nên tứ giác HKJI nội tiếp.
2 2
Từ đó P nằm trên trục đẳng phương của hai đường tròn (O1 ) và (O2 ), suy ra tP O1 = tP O2 .(1)
Áp dụng định lý Casey cho bốn đường tròn (C, 0), (P, 0), (O1 ), (O2 ) ta có

CP.HI = tCO1 .tP O2 + tCO2 .tP O1

Lại áp dụng định lý Casey cho bốn đường tròn (D, 0), (P, 0), (O1 ), (O2 ) ta có

DP.HI = tDO1 .tP O2 + tDO2 .tP O1

Do CD k AB và C là trung điểm cung AB nên CP = DP , từ đó

tCO1 .tP O2 + tCO2 .tP O1 = tDO1 .tP O2 + tDO2 .tP O1 .

Kết hợp với (1) suy ra tCO1 + tCO2 = tDO1 + tDO2 .

Bài 253. (Iran TST 2012). Cho hình bình hành ABCD. Gọi w1 , w2 lần lượt là hai đường tròn tiếp
xúc với các cặp đoạn thẳng AB và AD, BC và CD. Giả sử tồn tại một đường tròn tiếp xúc với đường
thẳng AD và DC và tiếp xúc ngoài với w1 và w2 . Khi đó tồn tại một đường tròn tiếp xúc với đường
thẳng AB và BC và tiếp xúc ngoài với w1 và w2 .

162
A Q T D
P
w3
M

w1 N
S' U
Q'
w2
S
B R C

Chứng minh. Gọi w3 là đường tròn tiếp xúc với w1 , w2 , AD, DC; R1 , R2 , R3 lần lượt là bán kính của
w1 , w2 , w3 ; h1 , h2 là hai đường cao của hình bình hành ABCD ứng với các cạnh AB, AD.
Gọi M, N, P, Q, R, S, T, U lần lượt là giao điểm của w1 ∩ w3 , w2 ∩ w3 , AB ∩ w1 , AD ∩ w1 , BC ∩
w2 , CD ∩ w2 , AD ∩ w3 , CD ∩ w3 ; Q0 , S 0 là điểm đối xứng với Q, S qua tâm các đường tròn w1 , w2 .
Dựa vào phép vị tự dễ dàng chứng minh các bộ ba điểm (P, M, U ), (Q0 , M, T ), (T, N, R), (U, N, S 0 )
thẳng hàng.
Gọi l là độ dài tiếp tuyến chung ngoài của w1 , w2 ; l1 , l2 là độ dài tiếp tuyến kẻ từ U, T tới các đường
tròn w1 , w2 . √ √ √ √
Ta có T Q = T M · T Q0 , U S = U N · U S 0 và l1 = U M · U P , l2 = T N · T R.
Theo định lý Casey, tồn tại w3 đi qua U, T và tiếp xúc với w1 , w2 nên

l · T U + T Q · U S = l1 · l2
√ √
Do đó l · T U + T M · T Q0 · U N · U S 0 = U M · U P · T N · T R
Áp dụng định lý hàm số sin,
∠ADC √
l · 2R3 · sin(90 − ) + 2R3 U S 0 · sin ∠N U S · T Q0 · sin ∠M T Q
√ 2
= 2R3 · U P · sin ∠M U S · T R · sin ∠N T Q
Tương đương
∠ADC p p
l · cos( ) + 2R1 · 2R2 = h1 · h2
2
.
Một cách tương tự, tồn tại đường tròn w4 tiếp xúc với w1 , w2 , AB, BC khi và chỉ khi
∠ABC p p
l · cos( ) + 2R1 · 2R2 = h1 · h2
2
Vậy tồn tại w3 khi và chỉ khi tồn tại w4 .

16.4 Bài tập tự luyện


Bài 254. Cho hình vuông ABCD nội tiếp đường tròn (O). E là một điểm nằm trên cung AC chứa B.
Đường tròn (O0 ) tiếp xúc với AC và tiếp xúc với (O) tại E. Kẻ tiếp tuyến DT tới (O0 ). Chứng minh
rằng DT = DA.

Bài 255. Cho ba đường tròn C1 , C2 , C3 bằng nhau và đôi một tiếp xúc nhau đồng thời cùng tiếp xúc
trong với đường tròn (O) lần lượt tại X, Y, Z. Gọi A là điểm bất kì nằm trên cung XY không chứa Z,
d1 , d2 , d3 lần lượt là độ dài tiếp tuyến kẻ từ A đến C1 , C2 , C3 . Chứng minh rằng d1 + d2 = d3 .

163
Bài 256. (Hongkong 2009). Cho tam giác ABC vuông tại C, đường cao CD. Đường tròn ω tiếp xúc
với các cạnh AC, AB lần lượt tại N, M và tiếp xúc ngoài với đường tròn đường kính BC. Chứng minh
rằng:
i. BD.CN + BC.DM = CD.BM .
ii. BM = BC.
Bài 257. (Kostas Vittas). Cho đường tròn (O) đường kính AB. P, Q là hai điểm bất kì trên (O) và
khác phía với AB. Kẻ QT ⊥ AB. P C, P D lần lượt là tiếp tuyến kẻ từ P đến đường tròn đường kính
AT, BT . Chứng minh rằng P C + P D = P Q.
Bài 258. Cho tam giác ABC ngoại tiếp đường tròn (I), nội tiếp đường tròn ω. ωa là đường tròn tiếp
xúc trong với ω và tiếp xúc với các cạnh AB, AC. AI giao ω lần thứ hai tại S. Kẻ tiếp tuyến ST tới
ST |b − c|
ωa . Chứng minh rằng = .
SA b+c
Bài 259. (Iran 2009). Hai đường tròn C1 và C2 có bán kính bằng nhau và cắt nhau tại hai điểm. Một
đường thẳng l lần lượt cắt C1 , C2 tại các điểm theo thứ tự A, B, C, D (A, C ∈ C2 , B, D ∈ C1 ). Dựng
hai đường tròn ω1 và ω2 sao cho hai đường tròn cùng tiếp xúc ngoài với C1 , tiếp xúc trong với C2 và
cùng tiếp xúc khác phía với l. Giả sử ω1 và ω2 tiếp xúc với nhau. Chứng minh rằng AB = CD.
Bài 260. Cho hai đường tròn ω1 và ω2 tiếp xúc ngoài với nhau tại I và cùng tiếp xúc trong với ω.
Tiếp tuyến chung ngoài của ω1 và ω2 giao ω tại B, C. Tiếp tuyến chung tại I cắt ω tại A sao cho A và
I nằm cùng một phía đối với BC. Chứng minh rằng I là tâm nội tiếp tam giác ABC.
Bài 261. AB là dây cung bất kì của đường tròn (O). Hai đường tròn C1 và C2 cùng tiếp xúc trong
với (O) và tiếp xúc khác phía với AB. Chứng minh rằng AB = 2t12 ( t12 là độ dài tiếp tuyến chung
ngoài của C1 và C2 ) khi và chỉ khi C1 hoặc C2 tiếp xúc với AB tại trung điểm AB.
Bài 262. Cho đường tròn (O). (O1 ) và (O2 ) cùng tiếp xúc trong với (O) và tiếp xúc ngoài nhau tại
X. Tiếp tuyến chung tại X của hai đường tròn cắt (O) tại A và B. Kí hiệu t12 là độ dài tiếp tuyến
1 1 2
chung ngoài của (O1 ) và (O2 ). Chứng minh rằng + = .
XA XB t12
Bài 263. (Thébault). Cho tam giác ABC ngoại tiếp đường tròn (I, r) và nội tiếp đường tròn (O). D
là một điểm nằm trên cạnh BC. Đường tròn ω1 bán kính r1 tiếp xúc với các tia DA, DB và tiếp xúc
trong với (O), đường tròn ω2 bán kính r2 tiếp xúc với các tia DA, DC và tiếp xúc trong với (O). Đặt
α α
∠ADB = α. Chứng minh rằng r1 . cos2 + r2 . sin2 = r.
2 2
Bài 264. (Luis González). Cho tam giác đều ABC cạnh a. Gọi (I) và (O) lần lượt là đường tròn nội
tiếp và ngoại tiếp tam giác ABC. P là điểm bất kì trên (I), XY Z là tam giác pedal của P ứng với
tam giác ABC. Các đường tròn C1 , C2 , C3 lần lượt tiếp xúc với cạnh BC, CA, AB tại X, Y, Z và tiếp
xúc với các cung BC, CA, AB không chứa đỉnh đối diện. Gọi T12 , T23 , T13 lần lượt là độ dài các tiếp
35
tuyến chung ngoài của C1 , C2 , C3 . Chứng minh rằng T12 + T23 + T31 = a.
16
Bài 265. (Lev Emelyanov). Cho ba điểm A1 , B1 , C1 lần lượt thuộc ba cạnh BC, CA, AB của tam giác
ABC. Dựng các đường tròn ωa , ωb , ωc lần lượt tiếp xúc với ba cạnh BC, CA, AB tại A1 , B1 , C1 và tiếp
xúc với đường tròn ngoại tiếp (O) của tam giác ABC tại các điểm nằm trên cung không chứa A, B, C.
Chứng minh rằng tồn tại một đường tròn tiếp xúc ngoài với ω1 , ω2 , ω3 và tiếp xúc với đường tròn nội
tiếp tam giác ABC khi và chỉ khi AA1 , BB1 , CC1 đồng quy.

17 Tam giác paralogic


17.1 Lời mở đầu
Cho tam giác ABC. Một đường thẳng d bất kì cắt ba cạnh BC, CA, AB lần lượt tại A1 , B1 , C1 .
Các đường thẳng lần lượt qua A1 , B1 , C1 và vuông góc với BC, CA, AB cắt nhau tạo thành tam giác

164
A2 B2 C2 . Khi đó A2 B2 C2 được gọi là tam giác paralogic của tam giác ABC. Dễ thấy rằng ABC cũng
là tam giác paralogic của tam giác A2 B2 C2 .
Trong bài viết này, chúng ta sẽ tìm hiểu một số tính chất liên quan đến tam giác paralogic. Để đơn
giản, trong bài viết sẽ không sử dụng góc định hướng, các trường hợp hình vẽ khác bạn đọc chứng
minh tương tự.

17.2 Một số tính chất


Tính chất 79. Hai tam giác ABC và A2 B2 C2 đồng dạng.

Tính chất này khá đơn giản nên xin được nhường lại cho bạn đọc.

Tính chất 80. Các đường tròn ngoại tiếp tam giác ABC và A2 B2 C2 giao nhau tại hai điểm, một
điểm là điểm Miquel của tứ giác toàn phần ABCA1 B1 C1 , một điểm là giao của AA2 , BB2 , CC2 .

B1
K
B2
O

C1 A2
O2 B
A1
C

J
C 2
Chứng minh.
Gọi K là điểm Miquel của tứ giác toàn phần ABCA1 B1 C1 .
Ta có K ∈ (A1 BC1 ), mà BB2 là đường kính của (A1 BC1 ) nên K ∈ (A1 B2 C1 ).
Tương tự, K ∈ (A2 B1 C1 ), tức là K là điểm Miquel của tứ giác toàn phần A1 B1 C1 A2 B2 C2 . Từ đó
suy ra K ∈ (A2 B2 C2 ).
Gọi J là giao điểm của AA2 và CC2 .
Ta có ∠BAJ = ∠C1 AA2 = ∠C1 B1 A2 = ∠A1 B1 C2 = ∠A1 CC2 = ∠BCJ.
Suy ra J ∈ (ABC). Chứng minh tương tự ta có AA2 , BB2 , CC2 đồng quy tại J. Mặt khác, theo
tính chất 1 ta suy ra ∠BJA = ∠BCA = ∠B2 C2 A2 nên J ∈ (A2 B2 C2 ).

Tính chất 81. Hai đường tròn (ABC) và (A2 B2 C2 ) trực giao.

Chứng minh. Gọi O và O2 lần lượt là tâm ngoại tiếp các tam giác ABC và A2 B2 C2 .
Ta có ∠OJO2 = ∠OJA + ∠A2 JO2 = 90o − ∠ACJ + 90o − ∠A2 C2 J
= 180o − (∠ACJ + ∠A2 C2 J) = 180o − 90o = 90o .
Vậy (ABC) và (A2 B2 C2 ) trực giao.

Tính chất 82. Đường thẳng d chia đôi đoạn thẳng nối trực tâm H và H2 của tam giác ABC và
A2 B2 C2 (định lý Sondat).

Chứng minh. Cách 1 (Jean Louis Ayme).

165
A

B1
H

M A1
C
B
P
N

C2
J
C1
H2
A2
B2

Gọi J là giao điểm của AA2 và CC2 , M, N lần lượt là hình chiếu của J trên BC, AB.
Ta có ∠BM N = ∠BJN = ∠BB2 C1 = ∠BA1 C1 nên M N k d hay đường thẳng Simson và đường
thẳng Steiner của J ứng với 4ABC song song với d.
Tương tự, đường thẳng Steiner của J ứng với 4A2 B2 C2 song song với d.
Gọi P, Q là điểm đối xứng với J qua AB, A2 B2 . Dễ thấy C1 là trung điểm của P Q. Chú ý rằng d
là đường trung bình của hình thang P HH2 Q nên d chia đôi HH2 .
Cách 2.
Bổ đề 16. Cho tứ giác toàn phần ABCDEF. Khi đó điểm Miquel và tâm của các đường tròn ngoại
tiếp tam giác AEF, CDE, ABC, BDE cùng nằm trên 1 đường tròn (đường tròn Miquel của tứ giác
toàn phần).
Chứng minh.

A
M

O1

F P3 O2

E P2 P1
O3

O4
B C D

Gọi O1 , O2 , O3 và O4 lần lượt là tâm đường tròn ngoại tiếp các tam giác AEF, CDE, ABC và
BDE.
Gọi P1 , P2 , P3 lần lượt là chân các đường vuông góc kẻ từ M tới O2 O4 , O2 O3 và O3 O4 .
Do P1 , P2 , P3 là trung điểm của M D, M C, M B nên chúng thẳng hàng.
Theo định lí đảo về đường thẳng Simson ta có M, O2 , O3 , O4 cùng nằm trên một đường tròn.
Tương tự suy ra đpcm.
Trở lại bài toán.

166
A

B1
H O
X

A1
B L C
M
C2 Z
N

Y
C1 H2 K
A2
B2

Gọi X, Y, Z, L lần lượt là trung điểm của AA2 , BB2 , CC2 , HH2 . Gọi M, N lần lượt là trung điểm
của HA2 , HB2 .
Ta có 4A2 H2 B2 ∼ 4AHB.
1
Do 4M LN là ảnh của 4A2 H2 B2 qua phép vị tự tâm H tỉ số nên 4M LN ∼ ∆A2 H2 B2 .
2
XM AH BH YN
Ta có ∠XM L = (AH, A2 H2 ) = 90o = (BH, B2 H2 ) = ∠LN Y và = = =
LM A2 H2 B 2 H2 LN
nên 4XM L ∼ 4Y N L từ đó 4A2 H2 B2 ∼ 4M LN ∼ 4XLY.
Chứng minh tương tự, 4Y LZ ∼ 4B2 H2 C2 , 4ZLX ∼ 4C2 H2 A2 .
Suy ra L là trực tâm tam giác XY Z.
Gọi K là điểm Miquel của tứ giác toàn phần ABCA1 B1 C1 . Áp dụng bổ đề 1 ta có X, Y, Z, K đồng
viên.
Chú ý rằng A1 , B1 , C1 là điểm đối xứng với K qua Y Z, ZX, XY nên d là đường thẳng Steiner của
K ứng với 4XY Z. Điều này nghĩa là d đi qua trực tâm của 4XY Z hay d đi qua trung điểm của
HH2 .

Tính chất 83. Nếu đường thẳng d đi qua trực tâm H của tam giác ABC thì (ABC) và (A2 B2 C2 )
cắt nhau tại điểm Anti-Steiner của đường thẳng d ứng với tam giác ABC.

Chứng minh. Gọi J là giao điểm của AA2 , BB2 , CC2 .


Theo lời giải 1 của tính chất 4 thì đường thẳng Steiner của J ứng với tam giác ABC song song với
d. Do đó nếu d đi qua H thì hiển nhiên J là điểm Anti-Steiner của đường thẳng d ứng với tam giác
ABC.

Tính chất 84. Cho tam giác ABC nội tiếp (O) với trực tâm H. Một đường thẳng d qua H cắt (O)
tại hai điểm U, W . Gọi F là điểm Anti-Steiner của tam giác ABC ứng với đường thẳng d. d1 , d2 lần
lượt là trung trực của F U, F W . Khi đó các đường tròn ngoại tiếp tam giác paralogic của tam giác
ABC ứng với đường thẳng d, d1 , d2 tiếp xúc nhau tại F .

167
W
A

Y1 F
C2 Y
H
O
Z
X
B X1 C

I U

O2

Z1
A2

2 B
Chứng minh.
Gọi A2 B2 C2 , X1 Y1 Z1 lần lượt là tam giác paralogic của tam giác ABC ứng với đường thẳng d, d1 .
Theo tính chất 5 ta có (A2 B2 C2 ) đi qua F.
Giả sử d1 cắt BC, CA, AB lần lượt tại X, Y, Z.
Ta có các điểm đối xứng với F qua Y Z, AB, AC đều nằm trên d nên theo định lý đảo về đường
thẳng Simson ta suy ra F ∈ (AY Z). Điều này nghĩa là F là điểm Miquel của tứ giác toàn phần
ABCXY Z. Theo tính chất 2 ta suy ra F ∈ (X1 Y1 Z1 ). Lại áp dụng tính chất 3 ta thu được OF là tiếp
tuyến chung của hai đường tròn (A2 B2 C2 ) và (X1 Y1 Z1 ). Chứng minh tương tự suy ra đpcm.

Tính chất 85. Cho tam giác ABC nội tiếp đường tròn (O) với trực tâm H. Hai đường thẳng d1 và
d2 bất kì vuông góc với nhau và đi qua H. d1 cắt BC, CA, AB lần lượt tại X1 , Y1 , Z1 . Gọi A1 B1 C1 ,
A2 B2 C2 lần lượt là tam giác paralogic của tam giác ABC ứng với d1 và d2 . Khi đó đường tròn ngoại
tiếp các tam giác A1 B1 C1 và A2 B2 C2 tiếp xúc với nhau tại một điểm nằm trên (O).

Chứng minh.
Bổ đề 17. Cho tam giác ABC nội tiếp (O) với trực tâm H. Hai đường thẳng d1 và d2 bất kì vuông
góc với nhau và đi qua H. d1 cắt BC, CA, AB lần lượt tại X1 , Y1 , Z1 . Tương tự ta xác định X2 , Y2 , Z2 .
Khi đó hai tứ giác toàn phần ABCX1 Y1 Z1 và ABCX2 Y2 Z2 có chung điểm Miquel.
Chứng minh.

168
Y2

Hb
A

Z2
Y1

M
H

Z1

X1
B X2 C

Ha
K

Gọi Ha , Hb lần lượt là điểm đối xứng với H qua BC, CA. Suy ra X1 Ha , Y1 Hb cắt nhau tại K -
điểm Anti-Steiner của tam giác ABC ứng với d1 . Hơn nữa, theo tính chất đối xứng ta hiển nhiên có
Ha ∈ (X1 HX2 ), Hb ∈ (Y1 HY2 ).
Do Ha , H, Hb lần lượt nằm trên các cạnh của tam giác X1 KY1 nên (HX1 Ha ), (Ha KHb ), (Hb Y1 H)
đồng quy tại điểm Miquel M của tam giác X1 KY1 ứng với bộ ba điểm (Ha , H, Hb ).
Do (X1 HX2 ) và (Y1 HY2 ) cắt nhau tại M nên M là điểm Miquel của tứ giác toàn phần X1 Y1 CX2 HY2 .
Từ đó M ∈ (X1 CY1 ).
Suy ra (X1 CY1 ) và (ABC) giao nhau tại M hay M là điểm Miquel của tứ giác toàn phần
ABCX1 Y1 Z1 . Tương tự ta có đpcm.
Trở lại bài toán.

169
C2

O2

A
V
H2
A2

T Y1
B1 B2
H
O
H1 Z1 A1
X1 C
U B

O1

C1

Gọi T là điểm Miquel của tứ giác toàn phần ABCX1 Y1 Z1 .


Theo tính chất 2 ta suy ra T ∈ (A1 B1 C1 ).
Áp dụng bổ đề 2 suy ra T cũng là điểm Miquel của tứ giác toàn phần ABCX2 Y2 Z2 . Tương tự ta
có T là giao điểm của (A1 B1 C1 ) và (A2 B2 C2 ).
Theo tính chất 3 ta thu được T O là tiếp tuyến chung của hai đường tròn (A1 B1 C1 ) và (A2 B2 C2 ).
Tức là (A1 B1 C1 ) và (A2 B2 C2 ) tiếp xúc nhau tại T .

Nhận xét. Bổ đề 2 chính là bước quan trọng để chứng minh định lý về đường thẳng Droz-Farny:
Trung điểm các đoạn thẳng X1 X2 , Y1 Y2 , Z1 Z2 thẳng hàng.
Trong bài toán trên, nếu gọi H1 , H2 lần lượt là trực tâm của các tam giác A1 B1 C1 và A2 B2 C2 thì
dựa vào tính chất 4 ta nhận thấy H1 ∈ d1 , H2 ∈ d2 . Đồng thời có thể chứng minh được trung điểm của
HH1 và HH2 cùng nằm trên đường thẳng Droz-Farny của tam giác ABC ứng với hai đường thẳng
d1 , d2 . Những tính chất thú vị này xin nhường lại cho bạn đọc.

18 Một số vấn đề về đa giác lưỡng tâm


18.1 Giới thiệu
Một đa giác lồi được gọi là lưỡng tâm khi đa giác đó vừa nội tiếp vừa ngoại tiếp đường tròn. Những
đa giác lưỡng tâm chúng ta thường gặp là tam giác và đa giác đều.
Bài viết này sẽ giới thiệu một số định lý liên quan đến đa giác lưỡng tâm và khai thác một số tính
chất liên quan.

170
18.2 Tính chất
Để tiện theo dõi chúng ta quy ước đường tròn nội tiếp là (I, r), đường tròn ngoại tiếp là (O, R),
khoảng cách OI = d.

18.2.1 Tam giác


Đây là trường hợp quen thuộc nên chúng ta chỉ quan tâm một số công thức liên quan đến đường
tròn nội tiếp và đường tròn ngoại tiếp.

Tính chất 86. Giữa đường tròn nội tiếp và ngoại tiếp tam giác ABC có các hệ thức sau:

R2 − 2Rr = d2 (2.1.1)

1 1 1
+ = (2.1.2)
R+d R−d r
√ √ √
R − d − r + R + d − r = 2R (2.1.3)
  
R+d R−d
−1 − 1 = 1 (2.1.4)
r r

Chứng minh. Qua một số phép biến đổi đơn giản ta nhận thấy bốn hệ thức trên tương đương nhau.
Vì vậy ta chỉ chứng minh hệ thức (2.1.1), hay còn gọi là hệ thức Euler.
Bổ đề 18. Cho tam giác ABC ngoại tiếp đường tròn (I), nội tiếp đường tròn (O). AI cắt (O) lần
thứ hai tại E. Khi đó E là tâm đường tròn ngoại tiếp tam giác BIC.
Chứng minh.
Ta có ∠IBE = ∠IBC + ∠CBE = ∠IBA + ∠EAC = ∠IBA + ∠BAI = ∠BIE.
Suy ra EI = EB. Tương tự EI = EC. Vậy E là tâm đường tròn ngoại tiếp tam giác BIC.

Trở lại bài toán.

171
A

F
I

B C

Gọi E là giao của AI và (O). F là hình chiếu của I trên AB.


BE
Theo định lý hàm số sin, = 2R.
sin ∠BAE
BE
Lại có r = IF = AI. sin ∠BAE nên 2Rr = .AI. sin ∠BAE = BE.AI = IA.IE (theo bổ
sin ∠BAE
đề 1),
Mặt khác, IA.IE = R2 − OI 2 nên R2 − 2Rr = d2 .

18.2.2 Tứ giác
Tính chất 87. Giữa đường tròn nội tiếp và ngoại tiếp tứ giác lưỡng tâm ABCD có các hệ thức sau:
1 1 1
2
+ 2
= 2 (2.2.1)
(R + d) (R − d) r

(R2 − d2 )2 = 2r2 (R2 + d2 ) (2.2.2)

(R + r + d)(R + r − d)(R − r + d)(R − r − d) = r4 (2.2.3)

2 !  !
R−d 2
 
R+d
−1 − 1 = 1 (2.2.4)
r r
p
d2 = R2 + r2 − r r2 + 4R2 (2.2.5)

B
N

A
I
O

D
C
M

172
Chứng minh. Sau một số biến đổi có thể thấy 5 hệ thức trên tương đương nhau, vì vậy ta chỉ chứng
minh hệ thức (2.2.1), hay còn gọi là định lý Fuss.
Kéo dài BI, DI cắt (O) lần lượt tại M, N .
1
Ta có ∠M N C + ∠N M C = ∠IBC + ∠IDC = (∠ADC + ∠ABC) = 90o .
2
Suy ra O là trung điểm M N .
Áp dụng công thức tính đường trung tuyến trong tam giác IM N ta có:
IM 2 IN 2 M N 2 IM 2 IN 2
OI 2 = + − = + − R2 .
2 2 4 2 2
1 1 2(R2 + d2 ) IM 2 + IN 2 IM 2 IN 2
Do đó + = = = +
(R + d)2 (R − d)2 (R2 − d2 )2 (PI /(O))2 IM 2 .IB 2 IN 2 .ID2
∠B ∠D
1 1 sin2 sin2
= + = 2 + 2 = 1.
IB 2 ID 2 r 2 r 2 r2
1 1 1 2(R2 + d2 ) 2 2
Nhận xét 4. Từ định lý Fuss, 2 = 2
+ 2
= 2 2 2
≥ 2 2
≥ 2 , ta suy ra
√ r (R + d) (R − d) (R − d ) R −d R
R ≥ 2r.

Vì mục đích của bài viết không đi sâu vào tính toán và chứng minh các công thức nên tác giả chỉ
đề cập và không chứng minh các tính chất sau. Bạn đọc có thể coi như bài tập tự luyện để thử sức
mình.

Tính chất 88. Trong mọi tứ giác lưỡng tâm ABCD, các đẳng thức sau đây thoả mãn.

AB CD AD BC
= , = (2.3.1)
IA.IB IC.ID IA.ID IB.IC
AC BD 2R
= = 2 (2.3.2)
IA.IC IB.ID R − d2
1 1 1 1 1
2
+ 2
= 2
+ 2
= 2 (2.3.3)
IA IC IB ID r

AC 2 BD2
= (2.3.4)
IA2 + IC 2 IB 2 + ID2
 
1 1
8Rr + = AB + BC + CD + DA (2.3.5)
AC BD

8R2 r2
AC.BD = (2.3.6)
R2 − d2

IA.IB.IC.ID = 2r2 (R2 − d2 ) (2.3.7)

AB + BC + CD + DA R2 − d2
= (2.3.8)
AC + BD Rr

pq 4R2
− = 1 (2.3.9)
4r2 pq

xz = yt (2.3.10)

p x+z
= (2.3.11)
q y+t

173
Với p, q là độ dài hai đường chéo AC và BD của tứ giác ABCD, x, y, z, t lần lượt là độ dài của
bốn đoạn thẳng nối đỉnh tứ giác với tiếp điểm đường tròn nội tiếp
Tính chất 89. Gọi a, b, c, d lần lượt là độ dài các cạnh tứ giác lưỡng tâm ABCD. Diện tích của tứ
giác ABCD được cho bởi một trong các công thức:

S= abcd (2.4.1)


4
S= xyzt(x + y + z + t) (2.4.2)

S = AI.CI + BI.DI (2.4.3)


 
2 1 1
S = 2r + (2.4.4)
sin A sin B

Tính chất 90. Bán kinh đường tròn nội tiếp và ngoại tiếp của tứ giác lưỡng tâm được cho bởi các
công thức:
√ √
abcd abcd
r= = (2.5.1)
a+c b+d
√ √
r= xz = yt (2.5.2)
r
1 (ab + cd)(ac + bd)(ad + bc)
R= (2.5.3)
4 abcd
Một số công thức khác, xem [2].
Tính chất 91. Cho tứ giác ABCD ngoại tiếp đường tròn (I). Gọi M, N, P, Q lần lượt là tiếp điểm
của AB, BC, CD, DA với (I). Khi đó tứ giác ABCD lưỡng tiếp khi và chỉ khi M P ⊥ N Q.

Q M

D
E

P
B

N
C

Chứng minh. Gọi E là giao điểm của M P và N Q.


1
Do ∠DP M = ∠AM P nên ∠DP E = 180o − (∠CDA + ∠DAB).
2
o 1
Tương tự, ∠DQE = 180 − (∠CDA + ∠DCB).
2
1
Do đó ∠P DQ + ∠P EQ = 360o − (∠DQE + ∠DP E) = ∠CDA + (∠DAB + ∠DCB)
2
Suy ra tứ giác ABCD nội tiếp khi và chỉ khi ∠DAB + ∠DCB = 90o ⇔ ∠P EQ = 90o hay
M P ⊥ N Q.

174
Nhận xét 5. Tính chất 2.2.6 chính là một phương pháp đơn giản để dựng tứ giác lưỡng tâm:
Dựng đường tròn (I), trên đó lấy bốn điểm X, Y, Z, T sao cho XZ ⊥ Y T . Các tiếp tuyến của (I)
tại X, Y, Z, T cắt nhau tạo thành tứ giác ABCD. Khi đó tứ giác ABCD lưỡng tâm.

Tính chất 92. Cho tứ giác ABCD ngoại tiếp đường tròn (I). Gọi E, F lần lượt là giao của các đường
thẳng AB và CD, AD và BC. Khi đó tứ giác ABCD lưỡng tiếp khi và chỉ khi ∠EIF = 90o .

Chứng minh. Gọi M, N, P, Q lần lượt là tiếp điểm của (I) với AB, BC, CD, DA. Do IE ⊥ M P, IF ⊥
N Q nên tính chất 2.2.7 là hệ quả trực tiếp của tính chất 2.2.6

Tính chất 93. Cho tứ giác ABCD ngoại tiếp đường tròn (I) sao cho không có hai đỉnh đối diện nào
đối xứng nhau qua đường chéo. Khi đó tứ giác ABCD lưỡng tiếp khi và chỉ khi đường thẳng Newton
của tứ giác ABCD vuông góc với đường thẳng Newton của tứ giác tiếp điểm M N P Q.

Q M

K
J D T
G I

P L
B

N
C

Chứng minh. Gọi K, L lần lượt là giao điểm của IE và M P , IF và N Q; T là giao điểm của M P và
N Q; J là trung điểm EF .
Đường thẳng Newton của tứ giác ngoại tiếp đi qua tâm nội tiếp của tứ giác đó nên IJ là đường
thẳng Newton của tứ giác ABCD. Mặt khác, K, L lần lượt là trung điểm M P , N Q nên KL là đường
thẳng Newton của tứ giác M N P Q.
Xét đường tròn (I) có M P là đường đối cực của E, N Q là đường đối cực của F suy ra EF là
đường đối cực của T . Từ đó IT ⊥ EF .
Do IK.IE = IL.IF = r2 nên tứ giác EKLF nội tiếp.
Suy ra IJ ⊥ KL ⇔ IT đi qua trung điểm G của KL. Trung điểm KL nằm trên đường kính T I của
đường tròn ngoại tiếp tứ giác T LIK khi và chỉ khi hoặc G là trung điểm T I hoặc KL ⊥ T I. Trường
hợp KL ⊥ T I không xảy ra do IT ≡ IJ nên tứ giác ABCD có hai đỉnh đối diện đối xứng nhau qua
đường chéo. Vì vậy G là trung điểm IT . Điều này tương đương ∠EIF = 90o nên theo tính chất 2.2.7
ta có đpcm.
r2 d
Tính chất 94. Trọng tâm G của tứ giác tiếp điểm XY ZT nằm trên OI và IG = .
R2 − d2

175
B

X
A
A' B'

T Y
O' O
I
D'
C'

D Z

Chứng minh. Gọi A0 , B 0 , C 0 , D0 lần lượt là trung điểm T X, XY, Y Z, ZT .


2
Ta có IA.IA0 = IB.IB 0 = IC.IC 0 = ID.ID0 = r2 nên phép nghịch đảo NIr : A 7→ A0 , B 7→
B 0 , C 7→ C 0 , D 7→ D0 và đường tròn ngoại tiếp tứ giác ABCD thành đường tròn ngoại tiếp tâm O0 của
tứ giác A0 B 0 C 0 D0 . Do A0 B 0 C 0 D0 là hình bình hành và nội tiếp nên O0 chính là trung điểm của A0 C 0 .
Từ đó O0 ≡ G.
−→ r2 −→ r2 −→
Theo phép nghịch đảo, IG = .IO = 2 2
.IO.
PI /(O) d −R
r2 d
Vậy G nằm trên OI và IG = 2 .
R − d2
2r2 d
Tính chất 95. Giao điểm P của hai đường chéo AC và BD nằm trên OI, đồng thời IP = .
R2 − d2
Chứng minh. Cách 1. Gọi U, V lần lượt là trung điểm XZ, Y T . Dễ dàng có IU P V là hình chữ nhật
nên trọng tâm tứ giác XY ZT là trung điểm IP . Tính chất 2.2.10 thực chất là hệ quả của tính chất
2.2.9.
Sau đây chúng ta sẽ chứng minh tính chất 2.2.10 theo một hướng khác để tìm ra tính chất mới.
Cách 2.
B
C1
D1
A

P
I
O
L

B1
A1
C

Gọi A1 , B1 , C1 , D1 lần lượt là giao điểm của AI, BI, CI, DI với (O). Theo phép chứng minh tính
chất 2.2.1, A1 C1 và B1 D1 là các đường kính của (O).
Gọi L là giao điểm của A1 B và D1 C.
Áp dụng định lý Pascal cho 6 điểm A1 , B1 , C1 , D1 , B, C suy ra I, O, L thẳng hàng.

176
Lại áp dụng định lý Pascal cho 6 điểm A, B, C, D, A1 , D1 suy ra I, P, L thẳng hàng. Như vậy P
nằm trên OI.
Mặt khác, gọi X, Y, Z, T lần lượt là tiếp điểm của AB, BC, CD, DA với (I). M, N lần lượt là giao
điểm của XY và ZT , XT và Y Z; E, F là giao điểm của AB và CD, AD và BC.
Theo một kết quả quen thuộc của tứ giác ngoại tiếp, AC, BD, XZ, Y T đồng quy tại P .
Áp dụng định lý Pascal cho 6 điểm X, Y, Z, T, X, Z suy ra E, M, N thẳng hàng. Tương tự suy ra
E, F, M, N thẳng hàng.

P'
B
A X
E
T
P
I Y
D

O
Z

Gọi P 0 là giao của OI với M N. Ta thấy P 0 lần lượt nằm trên đường đối cực của P ứng với (I), (O)
nên IP.IP 0 = r2 và OP.OP 0 = R2 .
r2 R2 R2
Từ đó IP 0 = , IP 0 + OI = OP 0 = = .
IP OP IP + OI
R 2 r 2
Đặt IP = x suy ra d = − .
x+d x
2r2 d R2 r2
Bây giờ ta thay x = 2 và tính K = 2 − .
R − d2 2r d 2r2 d
+d
R2 − d2 R2 − d2
2 2
(R − d ) 2
Từ công thức (2.2.2) suy ra 2r2 =
R2 + d2
Suy ra

R2 R2 − d2 R2 (R2 + d2 ) R2 − d2 R2 + d2 − R2 + d2
K= − = − = = d.
(R2 2
− d )d 2d d(R2 − d2 + R2 + d2 ) 2d 2d
+d
R2 + d2
2r2 d
Vậy IP = .
R2 − d2
Nhận xét 6. Theo phép chứng minh trên, điểm P được gọi là điểm "giới hạn" của hai đường tròn (I)
và (O), nghĩa là điểm nghịch đảo của P ứng với hai đường tròn trùng nhau. Với mỗi cặp đường tròn
(O), (I) đều tồn tại hai điểm "giới hạn" là P và P 0 . Ta có thể dựng điểm "giới hạn" của hai đường
tròn (O) và (I) chứa nhau như sau: dựng trục đẳng phương d của (O) và (I). d giao OI tại E. Dựng
đường tròn tâm E bán kính bằng độ dài tiếp tuyến từ E tới (O), (I), đường tròn này giao OI tại hai
điểm "giới hạn" của (O) và (I).

Tính chất 96. Phép nghịch đảo cực P hoặc P 0 phương tích bất kì biến (O) và (I) thành hai đường
tròn đồng tâm.

177
A

D
O' P
P' I
I' O

C
B

Chứng minh. Xét phép nghịch đảo cực P phương tích k. NPk : (O) 7→ (O0 ), (I) 7→ (I 0 ).
P I0 k k k
Ta có = = 0
suy ra P I 0 =
PI P P /(I) P I.P P PP0
PO 0 k k k
= = 0
suy ra P O0 =
PO PP /(O) P O.P P PP0
Từ đó P I = P O hay I ≡ O . Chứng minh tương tự với phép nghịch đảo cực P 0 ta có đpcm.
0 0 0 0

Tính chất 97. IP là đường đối trung của các tam giác IAC và IBD.

P I O

1
Chứng minh. Ta có ∠IAO = ∠IAB −∠OAB = ∠DAB −(90o −∠ACB) = ∠ACB −∠ICB = ∠ACI.
2
Do đó OA là tiếp tuyến của (AIC). Chứng minh tương tự suy ra O là giao điểm của hai tiếp tuyến
tại A, C của (AIC).
Theo tính chất 2.2.10, O, I, P thẳng hàng do đó IP là đường đối trung của tam giác IAC. Tương
tự với tam giác IBD.

Tính chất 98. Một đường thẳng qua I và song song với một cạnh của tứ giác ABCD cắt hai cạnh
đối diện còn lại tại M, N . Khi đó độ dài M N không phụ thuộc vào việc chọn cạnh của tứ giác để kẻ
song song với nó.

178
A

N
D

K
O
I L

Chứng minh. Gọi M N và LK là hai đường thẳng qua I và lần lượt song song với AD, BC (M, K ∈
CD; N, L ∈ AB).
Ta có ∠IN L = ∠DAB = 180o − ∠DCB = ∠M KI nên tứ giác M KN L nội tiếp, từ đó hai tam
giác KIM và N IL đồng dạng.
Do hai tam giác này có đường cao hạ từ I đều bằng r nên IK = IN, IM = IL, KM = N L.
Dễ thấy tam giác AN I cân tại N nên N A = N I. Tương tự, LI = LB, IK = KC, IM = M D.
Suy ra 2M N = M N + KL = AN + LB + AM + KC = AB − N L + CD + KM = AB + CD =
1
(AB + BC + CD + DA).
2
1
⇒ M N = (AB +BC +CD +DA) và không phụ thuộc vào việc chọn cạnh của tứ giác ABCD.
4
Tính chất 99. (Juan Carlos). Gọi X, Y, Z, T là tiếp điểm của (I) với AB, BC, CD, DA. H, K là hình
chiếu của I trên BD, AC. Khi đó SXKT = SXHY .

T
X
D
K I
O
H

B
Y
C

Chứng minh. Ta có tứ giác T KXA và XBY H lần lượt nội tiếp đường tròn đường kính IA, IB suy ra
∠KXT = ∠CAD = ∠CBD = ∠HXY .
1
SXKT XT.XK. sin ∠T XK XT.XK IA2 . sin ∠DAB. sin ∠CAB IA2 .BD.BC
Từ đó = 2 = = =
SXHY 1 XY.XH IB 2 . sin ∠ABC. sin ∠DBA IB 2 .AC.AD
XY.XH. sin ∠HXY
2

179
IA2 AC.AD
Như vậy ta cần chứng minh 2
= .
IB BD.BC
Để được như vậy trước tiên ta chứng minh các công thức (2.3.1) và (2.3.2).
Gọi A0 , B 0 , C 0 , D0 lần lượt là giao điểm thứ hai của AI, BI, CI, DI với (O). Dễ thấy A0 B 0 C 0 D0 là
hình chữ nhật.
A0 B 0 IA0 k AB A0 B 0 CD C 0 D0
Đặt PI /(O) = k. Ta có = = ⇔ = . Tương tự = . Do
AB IB IA.IB IA.IB k IC.ID k
A0 B 0 C 0 D0 là hình chữ nhật nên A0 B 0 = C 0 D0 . Công thức (2.3.1) được chứng minh. Công thức (2.3.2)
chứng minh tương tự.
AC.AD IA.IC IA.ID IA2
Như vậy = . = . Suy ra đpcm.
BD.BC IB.ID IB.IC IB 2
Tính chất 100. Gọi H1 , H2 , H3 , H4 lần lượt là trực tâm các tam giác AIB, BIC, CID, DIA; K1 , K2 ,
K3 , K4 lần lượt là trực tâm các tam giác XIY, Y IZ, ZIT, T IX. Khi đó các bộ (H1 , H2 , H3 , H4 ) và
(K1 , K2 , K3 , K4 ) lần lượt thẳng hàng và hai đường thẳng này vuông góc với nhau tại P .

T X

M
H4
D K3
Q O
P I

N B

Y
C

Chứng minh. Gọi M, Q lần lượt là hình chiếu của Z trên IT , A trên ID. N là trung điểm Y Z.
Theo tính chất 2.2.6, XZ ⊥ Y T . Do đó ∠N P Y = ∠T Y Z = ∠T ID = ∠T ZM = ∠T P M . Suy ra
M, P, N thẳng hàng.
Ta thu được ∠M P Q = ∠ZP N = ∠P ZN = 90o − ∠ZY T = ∠T DI = ∠T H4 A, suy ra tứ giác
P M H4 Q nội tiếp.
Mặt khác, M, Q thuộc đường tròn đường kính K3 H4 .
Như vậy 5 điểm K3 , P, Q, H4 , M cùng thuộc một đường tròn. Suy ra ∠K3 P H4 = 90o . Chứng minh
tương tự suy ra H1 , H2 , H3 , H4 cùng nằm trên đường thẳng qua P và vuông góc với P K3 . Lại theo
phép tương tự suy ra K1 , K2 , K3 , K4 cùng nằm trên đường thẳng qua P và vuông góc với đường thẳng
đi qua H1 , H2 , H3 , H4 .

Nhận xét 7. Tính chất 2.2.14 là sự tổng hợp của hai bài toán sau:
Bài toán 1. Cho tứ giác ABCD ngoại tiếp đường tròn (I), AC giao BD tại P . Khi đó trực tâm
của các tam giác AIB, BIC, CID, DIA cùng thuộc một đường thẳng đi qua P .
Bài toán 2. Cho tứ giác ABCD nội tiếp đường tròn (O) sao cho AC vuông góc với BD tại P . Khi
đó trực tâm của các tam giác AOB, BOC, COD, DOA cùng thuộc một đường thẳng đi qua P .
Cả hai bài toán đều có thể chứng minh bằng cách áp dụng định lý Pascal.

Tính chất 101. Tâm đường tròn nội tiếp của 8 tam giác ABC, BCD, CDA, DAB, AP B, BP C,
CP D, DP A cùng thuộc một đường tròn có tâm nằm trên đường thẳng OI.

180
A

D P

Chứng minh. Phép chứng minh cho bài toán tổng quát sẽ được giới thiệu ở mục sau.

Nhận xét 8. Tính chất 2.2.15 là sự tổng hợp của hai bài toán sau:
Bài toán 3. Cho tứ giác ABCD ngoại tiếp đường tròn (I), AC giao BD tại P . Khi đó tâm đường
tròn nội tiếp của 4 tam giác AP B, BP C, CP D, DP A cùng thuộc một đường tròn.
Bài toán 4. Cho tứ giác ABCD nội tiếp đường tròn (O). Khi đó tâm đường tròn nội tiếp của 4
tam giác ABC, BCD, CDA, DAB tạo thành 4 đỉnh của một hình chữ nhật.

Cuối cùng chúng ta sẽ mở rộng bổ đề 1 cho tứ giác lưỡng tâm.

Tính chất 102. Kéo dài AI, BI, CI, DI cắt (O) lần lượt tại A0 , B 0 , C 0 , D0 . Khi đó tâm đường tròn
ngoại tiếp các tam giác AIB, BIC, CID, DIA lần lượt nằm trên các cạnh của tứ giác A0 B 0 C 0 D0 , tâm
đường tròn ngoại tiếp các tam giác A0 IB 0 , B 0 IC 0 , C 0 ID0 , D0 IA0 cùng nằm trên đường tròn tâm I bán
R2 − d2
kính .
2r
A C'

O'2

B'

D I O

D'

O2
C
A'

Chứng minh. Gọi O1 , O2 , O3 , O4 lần lượt là tâm đường tròn ngoại tiếp các tam giác AIB, BIC, CID,
DIA; O10 , O20 , O30 , O40 lần lượt là tâm đường tròn ngoại tiếp các tam giác A0 IB 0 , B 0 IC 0 , C 0 ID0 , D0 IA0 .
Ta có ∠IO2 C = 2∠IBC = ∠ABC = ∠IA0 C, suy ra tứ giác IO2 A0 C nội tiếp. Suy ra ∠IO2 A0 =
∠ICA0 = 90o .
Tương tự suy ra O2 là hình chiếu của I trên A0 D0 .
Như vậy O1 , O2 , O3 , O4 là hình chiếu của I trên các cạnh của tứ giác A0 B 0 C 0 D0 .

181
Mặt khác, đặt k = PI /(O).
B0C 0 B0C 0
Ta có IO20 = = .
2 sin ∠B 0 IC 0 2 sin ∠BIC
k.BC k.BC k R2 − d2
Dễ dàng chứng minh được B 0 C 0 = , suy ra IO20 = = = .
IB.IC 2IB.IC. sin ∠BIC 2r 2r
2
R −d 2
Chứng minh tương tự suy ra O10 , O20 , O30 , O40 nằm trên đường tròn tâm I bán kính .
2r

18.2.3 Đa giác nhiều hơn 4 đỉnh


Trong mục này chúng ta sẽ tìm hiểu một số tính chất thú vị của các đa giác lưỡng tâm có số đỉnh
nhiều hơn 4.

Tính chất 103. Cho ngũ giác lưỡng tâm ABCDE. Gọi A1 , B1 , C1 , D1 , E1 lần lượt là giao điểm
của các cặp đường thẳng (BD, CE), (CE, DA), (DA, EB), (EB, AC), (AC, BD). Khi đó ngũ giác
A1 B1 C1 D1 E1 ngoại tiếp.

Chứng minh. Cách 1.

E B4
B3

B2 C3

D C2
A4
A3
F I
O
L K

C A'4
A
E2

E3
D2

B D3
B'4

Gọi A2 , B2 , C2 , D2 , E2 lần lượt là tiếp điểm của CD, DE, EA, AB, BC với (I). A3 , B3 , C3 , D3 , E3
lần lượt là điểm chính giữa các cung CD, DE, EA, AB, BC không chứa đỉnh nào của ngũ giác;
A4 , B4 , C4 , D4 , E4 là giao của AI, BI, CI, DI, EI với (O); A04 , B40 , C40 , D40 , E40 là điểm đối xứng với
A4 , B4 , C4 , D4 , E4 qua O.
Áp dụng định lý Pascal cho 6 điểm A, B, B3 , B4 , A3 , A4 ta thu được giao điểm L của AA3 và BB3 ,
I, giao điểm F của B3 A4 và A3 B4 thẳng hàng.
Lại áp dụng định lý Pascal cho 6 điểm A4 , A04 , B4 , B40 , B3 , A3 suy ra F, O, giao điểm K của B3 B40
và A3 A04 thẳng hàng.
Việc còn lại là chứng minh K ∈ OI.
Dễ thấy B3 B40 ⊥ A3 C3 . Do đó ta chỉ cần chứng minh đường vuông góc kẻ từ B3 tới A3 C3 , A3 tới
E3 B3 giao nhau tại OI.
Lại do hai ngũ giác A3 B3 C3 D3 E3 và A2 B2 C2 D2 E2 có cạnh tương ứng song song nên tâm vị tự của
chúng nằm trên đường nối hai tâm ngoại tiếp OI. Vì vậy ta chỉ cần chứng minh đường vuông góc kẻ
từ A2 tới E2 B2 và B2 tới A2 C2 giao nhau tại một điểm trên OI.

182
A

C2
A5
D2
E N
E5 M

B2 O
Q
J I

D B

A2
E2

Gọi A5 , B5 , C5 , D5 , E5 lần lượt là trung điểm C2 D2 , D2 E2 , E2 A2 , A2 B2 , B2 C2 .


2
Phép nghịch đảo NIr : A 7→ A5 , B 7→ B5 , C 7→ C5 , D 7→ D5 , E 7→ E5 nên ngũ giác A5 B5 C5 D5 E5
nội tiếp đường tròn tâm J và J ∈ OI.
Gọi M, N lần lượt là trung điểm B2 D2 , A5 E5 , Q là điểm đối xứng với I qua J.
Ta có C2 , M, N thẳng hàng nên N J là đường trung bình của hình thang C2 QIM , từ đó C2 Q ⊥
B2 D2 . Tương tự suy ra các đường vuông góc kẻ từ A2 tới E2 B2 và B2 tới A2 C2 giao nhau tại Q.
Từ đó K ∈ OI. Suy ra F ∈ OI, kéo theo L ∈ OI. Chứng minh tương tự suy ra phân giác các góc
CAD, DBE, ECA, ADB, BEC đồng quy tại L. Suy ra đpcm.

Để chứng minh kết quả ngũ giác A1 B1 C1 D1 E1 ngoại tiếp ta có thể tiếp cận theo phương pháp
khác như sau.
Cách 2.
E

C2

B2
A
E3 C1
B1
D D3
A3
A1 L I O
A2
C3 D1
B3
E1
C D2

E2

Gọi A3 , B3 , C3 , D3 , E3 lần lượt là giao điểm của B2 D2 và C2 E2 , D2 A2 và C2 E2 , E2 B2 và D2 A2 ,


A2 C2 và E2 B2 , B2 D2 và A2 C2 .
Áp dụng định lý Pascal cho 6 điểm A2 , B2 , C2 , D2 , C2 , D2 suy ra A nằm trên đường thẳng nối giao
điểm của B2 D2 và A2 C2 , B2 C2 và A2 D2 . Tương tự với D. Từ đó E3 ∈ AD. Tương tự với A3 , B3 , C3 , D3 .
Ta có ∠C1 E3 A3 = ∠EB2 D2 − ∠EDA = ∠AD2 B2 − ∠EBA = ∠C1 A3 E3 .
Từ đó C1 A3 = C1 E3 . Tương tự, D1 A3 = D1 B3 , suy ra AE3 + AB3 = PAC1 D1 .
AD2 . sin ∠E3 D2 A AC2 . sin ∠B3 C2 A
Mặt khác, AE3 = , AB3 = .
sin ∠AE3 D2 sin ∠C2 B3 A

183
AD2 . sin ∠E3 D2 A AC2 . sin ∠B3 C2 A
Ta sẽ chứng minh AE3 = AB3 , khi và chỉ khi =
sin ∠AE3 D2 sin ∠C2 B3 A
sin ∠E3 D2 A sin ∠B3 C2 A sin ∠E3 D2 A sin ∠AE3 D2 sin ∠D2 A3 B
⇔ = ⇔ = = .
sin ∠AE3 D2 sin ∠C2 B3 A sin ∠B3 C2 A sin ∠C2 B3 A sin ∠E2 A3 B
BE2 BA3 BD2 BA3
Ta có = , = .
sin ∠E2 A3 B sin ∠A3 E2 B sin ∠BA3 D2 sin ∠A3 D2 B
sin ∠D2 A3 B sin ∠A3 D2 B sin ∠E3 D2 A
Suy ra = = .
sin ∠BA3 E2 sin ∠A3 E2 B sin ∠B3 C2 A
Vậy AE3 = AB3 , hay A3 , B3 , E3 là tiếp điểm của đường tròn bàng tiếp góc A của tam giác AC1 D1
với ba cạnh. Chứng minh tương tự suy ra các tam giác AC1 D1 , BD1 E1 ,...,EB1 C1 có chung đường tròn
bàng tiếp, hay ngũ giác A1 B1 C1 D1 E1 ngoại tiếp.

Tính chất 104. (Trần Quang Hùng). Gọi (Oa ), (Ob ), (Oc ), (Od ), (Oe ) là 5 đường tròn tiếp xúc trong
với (O) tại A2 , B2 , C2 , D2 , E2 và lần lượt tiếp xúc với các cặp tia (A1 C, A1 D), (B1 D, B1 E), (C1 E, C1 A),
(D1 A, D1 B), (E1 B, E1 C). Khi đó A1 A2 , B1 B2 , C1 C2 , D1 D2 , E1 E2 đồng quy tại một điểm trên OI.

C2
E

B2 Oc
A
Ob
B1
C1
D
A1 L I
A2 Oa O
D1
E1 Od

C Oe

E2 D2

Chứng minh. Tính chất 2.3.2 thực chất là hệ quả của 2.3.1.
Gọi (L) là đường tròn nội tiếp ngũ giác A1 B1 C1 D1 E1 .
Áp dụng định lý Monge-D’Alembert (xem [4]) cho 3 đường tròn (O), (Oc ), (L) ta thu được C1 C2
đi qua tâm vị tự trong của (O) và (L). Chứng minh tương tự ta có đpcm.

Tính chất 105. Cho lục giác lưỡng tâm A1 A2 A3 A4 A5 A6 . Gọi tiếp điểm của A1 A2 , A2 A3 , ..., A6 A1 với
(I) lần lượt là B12 , B23 , ..., B61 . Khi đó A1 A4 , A2 A5 , A3 A6 , B12 B45 , B23 B56 , B34 B61 đồng quy tại điểm
"giới hạn" của hai đường tròn (O) và (I).

184
X

A1
B12
B61
A6
A2
B56
P I O

B23
A5
B45
A3
A4 B34
Y

Chứng minh. Do lục giác A1 A2 A3 A4 A5 A6 ngoại tiếp nên theo định lý Brianchon, A1 A4 , A2 A5 , A3 A6
đồng quy.
Áp dụng định lý Desargues suy ra giao điểm X ≡ A1 A2 ∩A4 A5 , Y ≡ A1 A6 ∩A3 A4 , Z ≡ A2 A3 ∩A5 A6
thẳng hàng.
Do B12 B45 , B34 B61 , B23 B56 lần lượt là đường đối cực của X, Y, Z ứng với (I) nên chúng đồng quy
tại cực P của đường thẳng qua X, Y, Z ứng với (O).
Áp dụng định lý Pascal cho 6 điểm B45 , B34 , B45 , B34 , B12 , B61 suy ra A4 nằm trên đường nối giao
điểm B61 B45 và B12 B34 với P . Tương tự suy ra A1 A4 đi qua P . Tương tự ta thu được A1 A4 , A2 A5 ,
A3 A6 , B12 B45 , B23 B56 , B34 B61 đồng quy tại P . Do đường thẳng (X, Y, Z) cũng là đường đối cực của
P ứng với (I) nên P là điểm "giới hạn" của (O) và (I).

Tính chất 106. Gọi I12 , I23 , ..., I61 lần lượt là tâm đường tròn nội tiếp các tam giác P A1 A2 , P A2 AP3 ,...,P A6 A1 .
Khi đó lục giác I12 I23 ...I61 nội tiếp đường tròn có tâm nằm trên OI.

A1
B61
A6

I61
B56 B12
I56 I12

A5 P I O
I45
B45
A2
I23
I34
A4
B23
B34

A3

Chứng minh. Ta có ∠A2 P B12 = 180o − (∠B12 P A2 + ∠P A2 B12 ) = 180o − (∠P B45 A4 + ∠P A4 B45 ) =
∠A4 P B45 = ∠A1 P B12 .
Do đó P B12 là phân giác góc A1 P A2 hay P B12 đi qua I12 . Tương tự P B23 đi qua I23 .
I12 P A2 P A2 P I23 P
Ta có = = = .
I12 B12 A2 B12 A2 B23 I23 B23
Từ đó I12 I23 k B12 B23 . Tương tự suy ra hai lục giác I12 I23 ...I61 và B12 B23 ...B61 có tâm vị tự là P .
Mà B12 B23 ...B61 nội tiếp đường tròn (I) nên I12 I23 ...I61 nội tiếp đường tròn có tâm nằm trên P I hay
OI.

185
Tính chất 107. (Trần Quang Hùng). Gọi (J12 ), (J23 ),...,(J61 ) là các đường tròn tiếp xúc trong với (O)
lân lượt tại L12 , L23 , ..., L61 và lần lượt tiếp xúc với các cặp tia (P A1 , P A2 ),(P A2 , P A3 ),...,(P A6 , P A1 ).
Khi đó L12 , L45 , I12 , I45 cùng thuộc một đường tròn ω1 , tương tự với ω2 , ω3 và ba đường tròn này giao
nhau tại 2 điểm trên OI (bộ đường tròn coaxal).

Chứng minh.
Bổ đề 19. Cho tứ giác ABCD nội tiếp đường tròn (O), AC giao BD tại P . Gọi (I1 ), (I2 ) lần lượt là
đường tròn nội tiếp tam giác AP B, CP D, (J1 ), (J2 ) là đường tròn tiếp xúc trong với (O) lần lượt tại
H, L và tiếp xúc với các cặp tia (P A, P B); (P C, P D). Khi đó H, L, I1 , I2 cùng thuộc một đường tròn
trực giao với (O).
Chứng minh.

B N
K
H

A
I1

I2

D Q
L
M C

Gọi M, N lần lượt là điểm chính giữa cung DC chứa và không chứa L; K, Q lần lượt là điểm chính
giữa cung AB chứa và không chứa H.
Theo bổ đề 2 tại [4] ta suy ra các bộ ba sau thẳng hàng: (H, I2 , M ), (H, I1 , Q), (L, I2 , N ), (L, I1 , K).
Suy ra ∠I1 HI2 = ∠QHM = ∠KLN = ∠I1 LI2 . Từ đó tứ giác I1 HLI2 nội tiếp.
Mặt khác, ∠OHI2 = ∠OHM = ∠HM O = ∠HM N = ∠HLN = ∠HLI2 . Suy ra OH là tiếp tuyến
của (HI1 I2 ). Tương tự suy ra đường tròn ngoại tiếp tứ giác I1 HLI2 trực giao với (O).
Trở lại bài toán.

A1 L12

L61

A2
A6 I12
I61

L56 I56
I
O I23
P

I45
A5
L23
I34
L45

A4 A3
L34

186
Từ bổ đề 2 suy ra L12 , L45 , I12 , I45 cùng thuộc đường tròn ω1 , tương tự với ω2 , ω3 .
Theo tính chất 2.3.4 ta thu được P I12 .P I45 = P I23 .P I56 = P I34 .P I61 nên P có cùng phương tích
với ω1 , ω2 , ω3 .
Mặt khác, cũng theo bổ đề 2 thì ω1 , ω2 ω3 cùng trực giao với (O). Do đó PO /(ω1 ) = PO /(ω2 ) =
PO /(ω3 ) = R2 .
Vậy OP là trục đẳng phương của 3 đường tròn ω1 , ω2 ω3 hay chúng cắt nhau tại 2 điểm nằm trên
OI.

Nhận xét 9. Từ tính chất 2.3.5, nếu gọi Q là giao của L12 L45 và L34 L61 thì QL12 .QL45 = QL34 .QL61 ,
suy ra Q thuộc trục đẳng phương của ω1 và ω3 hay Q ∈ OI.
Tương tự suy ra L12 L45 , L34 L61 , L23 L56 đồng quy tại Q.

Tính chất 108. 6 đường thẳng J12 L45 , J23 L56 , J34 L61 , J45 L12 , J56 L23 , J61 L34 đồng quy tại một điểm
trên OI.

T
L12 A2
A1
J12 L23

J23
L61 J61

A6
P O
Q V I
L56 J56 A3

A5 J45 J34
L45
L34
A4

Chứng minh. Qọi T là giao của J34 J61 và L34 L61 . Áp dụng định lý Monge-D’Alembert cho 3 đường
tròn (O), (J34 ), (J61 ) ta thu được T là tâm vị tự ngoài của (J34 ) và (J61 ). Gọi V là giao của J61 L34 và
J34 L61 .
Ta có (J34 J61 P T ) = −1 và OP giao L34 L61 tại Q nên (QP V O) = −1.
Tương tự ta cũng suy ra giao điểm của các cặp đường thẳng (J12 L45 , J45 L12 ) và (J23 L56 , J56 L23 )
là V . Suy ra đpcm.

Tính chất 109. Tâm vị tự ngoài của các cặp 2 đường tròn bất kì trong 6 đường tròn (J12 ), (J23 ), ..., (J61 )
cùng nằm trên một đường thẳng.

L12
A1 A2

J12
L61
J61
L23
A6 J23

P O
L56 Q V I
M J56

A5 J45
A3
L45 J34

A4 L34

187
Chứng minh. Ta sẽ chứng minh các tâm vị tự trên cùng nằm trên đường đối cực của Q ứng với (O).
Áp dụng định lý Desargues cho 3 đường thẳng (L12 L61 , J12 J61 , L34 L45 ) ta có O ≡ L12 J12 ∩
L61 J61 , Q ≡ L61 L34 ∩ L12 L45 , V ≡ J61 L34 ∩ J12 L45 thẳng hàng. Suy ra L12 L61 , J12 J61 , L34 L45 đồng quy
tại M .
Áp dụng định lý Monge-D’Alembert cho 3 đường tròn (J12 ), (J61 ), (O) ta có M là tâm vị tự ngoài
của (J12 ) và (J61 ) và hiển nhiên M nằm trên đường đối cực d của Q ứng với (O).
Chứng minh tương tự cho (J12 ) và (J23 ), lại theo định lý Monge-D’Alembert suy ra tâm vị tự ngoài
của (J61 ) và (J23 ) nằm trên d. Tương tự cho các cặp đường tròn ở vị trí như vậy.
Cuối cùng, một lần nữa áp dụng định lý Monge-D’Alembert cho 3 đường tròn (J61 ), (J12 ), (J34 ) ta
suy ra tâm vị tự ngoài của (J61 ) và (J34 ) nằm trên d. Tương tự thu được đpcm.

Chúng ta tạm thời dừng các tính chất của ngũ giác và lục giác lưỡng tâm tại đây. Xung quanh hai
lớp đa giác này còn khá nhiều tính chất thú vị, tuy nhiên trong khuôn khổ có hạn, bài viết chỉ đề cập
một số tính chất cơ bản và đẹp. Bạn đọc có thể dựa vào chúng để tìm ra những bài toán mới của riêng
mình.
Quay lại tính chất 2.2.15. Ban đầu ý tưởng để tìm ra tính chất này khá tự nhiên. Bắt nguồn từ
hai bài toán 3 và 4 rất quen thuộc rồi kết hợp chúng lại tạo ra cách phát biểu bài toán mới vô cùng lạ
mắt và độc đáo. Phép chứng minh cho trường hợp tứ giác lưỡng tâm có thể xem tại [9]. Một ý tưởng
mới nảy sinh là liệu với các đa giác có nhiều cạnh hơn, tính chất trên có còn đúng. Tác giả đã thử mở
rộng và chứng minh thành công cho trường hợp ngũ giác, sau đó nhận ra lời giải của ngũ giác có thể
áp dụng cho trường hợp đa giác bất kì.

Tính chất 110. Cho n−giác lưỡng tâm A1 A2 A3 ...An (≥ 3). Kí hiệu Ii là tâm đường tròn nội tiếp
của tam giác Ai−1 Ai Ai+1 ; Ai(i+1) là giao điểm của Ai Ai+2 và Ai−1 Ai+1 ; Ii(i+1) là tâm đường tròn nội
tiếp của tam giác Ai Ai(i+1) Ai+1 (i = 1, n). Khi đó 2n điểm I1 , I2 , ..., In , I12 , I23 , ..., In1 cùng thuộc một
đường tròn.

A1 O1

I1 I12
On A2
K
A12 I2 O2

A23
An
O A3
I3
I

A4

Chứng minh. Gọi Oi là điểm chính giữa của cung Ai Ai+1 . Do I1 , I2 lần lượt là tâm đường tròn nội
tiếp của tam giác An A1 A2 , A1 A2 A3 nên O1 A1 = O1 A2 = O1 I1 = O1 I2 hay A1 , A2 , I1 , I2 cùng thuộc
(O1 , O1 A1 ). Tương tự với các đường tròn (O2 ), (O3 ), ..., (On ).

188
Lại có A1 I1 , A2 I2 , ..., An In đồng quy tại tâm đường tròn nội tiếp I của n-giác A1 A2 A3 ...An nên
phép nghịch đảo cực I, phương tích IA1 .II1 biến đường tròn ngoại tiếp n-giác A1 A2 ...An thành đường
tròn đi qua I1 , I2 , ...., In hay I1 , I2 , ..., In cùng thuộc một đường tròn.
Mặt khác, gọi K là giao của I1 I2 và A1 A3 .
Ta có ∠KI1 I12 = ∠I2 A1 A2 = ∠KA1 I12 nên tứ giác I1 A1 I12 K nội tiếp.
Từ đó ∠I1 I12 A1 = ∠I1 KA1 = ∠I2 I1 I − ∠I1 A1 A3 .
Mà ∠II1 I2 = ∠A1 A2 I2 = ∠I2 A2 A3 = ∠I2 I3 I, ∠I1 A1 A3 = ∠I1 I3 I, ta thu được ∠I1 I12 A1 =
∠I2 I3 I − ∠I1 I3 I = ∠I1 I3 I2 . Vậy I12 nằm trên (I1 I2 I3 ). Chứng minh tương tự suy ra đpcm.

Tính chất 111. (Định lý Poncelet về đa giác lưỡng tâm - Poncelet’s porism). Cho hai đường tròn
chứa nhau thoả mãn: tồn tại một n−giác nội tiếp đường tròn lớn và ngoại tiếp đường tròn nhỏ. Khi
đó tồn tại vô số n−giác vừa nội tiếp đường tròn lớn vừa ngoại tiếp đường tròn nhỏ, đồng thời bất kì
một điểm nào thuộc đường tròn lớn đều có thể lấy làm đỉnh cho những n−giác như thế.

Chứng minh. Phép chứng minh sau dựa theo [5].


Ta phát biểu hai bổ đề.
Bổ đề 20. Một đường thẳng d nào đó cắt hai đường tròn (O) và (I) theo thứ tự tại các cặp điểm
A, A0 và B, B 0 . Khi đó các giao điểm của các tiếp tuyến với đường tròn thứ nhất tại A và A0 và các
tiếp tuyến với đường tròn thứ hai tại B và B 0 cùng nằm trên một đường tròn có tâm thẳng hàng với
các tâm của hai đường tròn đã cho.
Chứng minh.
Q
P A'
B'
N

A I
O

Giả sử các tiếp tuyến cắt nhau tạo thành tứ giác M N P Q như hình vẽ.
Do ∠N M Q = ∠N AB − ∠P BA0 = ∠P A0 B − ∠A0 B 0 Q = ∠P QN nên tứ giác M N P Q nội tiếp. Việc
còn lại là chứng minh tâm của (M N P Q) nằm trên OI.
MA sin ∠M BA p
Ta có = = ⇒ q 2 M A2 − p2 M B 2 = 0.
MB sin ∠M AB q
Từ đó q 2 (M O2 − R2 ) − p2 (M I 2 − r2 ) = 0 ⇒ q 2 M O2 − p2 M I 2 = q 2 R2 − p2 r2 . Chứng minh tương
tự với N, P, Q.
Đến đây ta chú ý đến một quỹ tích quen thuộc: Cho hai điểm O và I. Quỹ tích các điểm M thoả
mãn xM O2 − yM I 2 = k cho trước nếu là một đường tròn thì tâm của đường tròn này là một điểm J
−→ −
→ → −
được xác định bởi hệ thức xJO − y JI = 0 , tức là J thẳng hàng với O và I. Áp dụng vào bài toán ta
có đpcm.
−→ −→ → −
Chú ý. Có thể lấy điểm J trên OI sao cho q 2 JO − p2 JI = 0 và tính độ dài OM . Ngoài ra ta cũng
có thể chứng minh (O), (I), (J) là một bộ đường tròn coaxal.

189
Bổ đề 21. Cho hai đường tròn đựng nhau (O) và (I). Trên (O) cho hai đường gấp khúc đơn A1 A2 ...An
và B1 B2 ...Bn nội tiếp và có cùng hướng sao cho các đốt A1 A2 , A2 A3 , ..., An−1 An và B1 B2 , B2 B3 , ..., Bn−1 Bn
của chúng đều tiếp xúc với (I). Khi đó các đường thẳng A1 B1 , A2 B2 , ..., An Bn cùng tiếp xúc với một
đường tròn nào đó có tâm thẳng hàng với các tâm của (O) và (I).
Chứng minh.
L

A2

B1
C2
B12
A12
C1 K
A23 B2
J O
A1 I

B23

B3 C3 A3

Gọi A12 , A23 , B12 , B23 lần lượt là tiếp điểm của A1 A2 , A2 A3 , B1 B2 , B2 B3 với (I); A1 B1 ∩ A2 B2 =
{L}.
A12 A23 giao A1 B1 , A2 B2 lần lượt tại C1 , C2 .
Ta có ∠LC1 C2 = ∠LA1 A2 + ∠C1 A12 A1 = ∠LB2 B1 + ∠C2 B12 B2 = ∠LC2 C1 . Suy ra LC1 = LC2 .
Từ đó tồn tại đường tròn (J) tiếp xúc với A1 B1 và A2 B2 tại C1 , C2 .
Theo bổ đề 3 suy ra O, I, J thẳng hàng.
Tiếp theo ta chứng minh (J) tiếp xúc với A3 B3 .
Thật vậy, gọi K là giao của A12 A23 và B12 B23 thì A2 , C2 , B2 nằm trên đường đối cực của K ứng
với (I). Suy ra A23 , B23 , C2 thẳng hàng. Gọi C3 là giao của A23 B23 với B3 A3 . Chứng minh tương tự
ta thu được tồn tại một đường tròn tiếp xúc với A2 B2 và A3 B3 lần lượt tại C2 , C3 . Như vậy (J) tiếp
xúc với A3 B3 tại C3 . Chứng minh tương tự suy ra (J) tiếp xúc với A1 B1 , A2 B2 , ..., An Bn .

Trở lại bài toán.


Giả sử n−giác A1 A2 ...An nội tiếp đường tròn (O) và ngoại tiếp đường tròn (I). Lấy một điểm B1
tuỳ ý trên (O) và lần lượt kẻ các tiếp tuyến B1 B2 , B2 B3 , ..., Bn−1 Bn , Bn Bn+1 tới (I). Ta cần chứng
minh Bn+1 ≡ B1 .
Theo bổ đề 2, tồn tại một đường tròn (J) tiếp xúc với A1 B1 , A2 B2 , ..., An Bn , An+1 Bn+1 . Do An+1 ≡
A1 nên (J) tiếp xúc với A1 Bn+1 . Mà (J) tiếp xúc với A1 B1 nên Bn+1 ≡ B1 . Bài toán được chứng
minh.

Để kết thúc bài viết, xin giới thiệu tới bạn đọc một định lý có dạng gần giống định lý Poncelet.

Tính chất 112. (Định lý Steiner- Steiner’s porism). Cho hai đường tròn đựng nhau (O) và (I). Giả
sử tồn tại chuỗi đường tròn ω1 , ω2 , ..., ωn sao cho n đường tròn cùng tiếp xúc với (O) và (I), đồng thời
ω2 tiếp xúc ngoài với ω1 , ω3 tiếp xúc ngoài với ω2 , ..., ωn tiếp xúc ngoài với ω1 . Khi đó tồn tại vô số
chuỗi ω1 , ω2 , ..., ωn thoả mãn điều kiện trên, đồng thời có thể chọn đường tròn ω1 bất kì cùng tiếp xúc
với (O) và (I).

190
O' P I O

Chứng minh. Gọi P là điểm "giới hạn" của (O) và (I). Theo tính chất 2.2.10, phép nghịch đảo cực
P phương tích k bất kì biến (O) và (I) thành hai đường tròn đồng tâm O0 ; chuỗi ω1 , ω2 , ..., ωn thành
chuỗi W = (ω10 , ω20 , ..., ωn0 ) cùng tiếp xúc với hai đường tròn đồng tâm O0 và lần lượt tiếp xúc ngoài
nhau. Do chuỗi W đóng kín nên có thể cho ω10 chuyển động mà không làm thay đổi tính chất của W .
Từ đó theo phép nghịch đảo, tồn tại vô số chuỗi ω1 , ω2 , ..., ωn thoả mãn điều kiện trên và có thể chọn
vị trí bất kì cho ω1 .

19 Hình sao năm cánh


19.1 Giới thiệu
Chắc hẳn bạn đọc đều quen thuộc với một hình sao năm cánh. Nó xuất hiện ở nhiều nơi, đặc biệt
có trên Quốc kì của Việt Nam. Xung quanh hình sao có một số bài toán khá kinh điển và đẹp mắt,
mời các bạn cùng chiêm ngưỡng.

19.2 Đường tròn Miquel trong hình sao


Chúng ta sẽ tiếp cận đường tròn Miquel theo một chuỗi bài toán khá thú vị.

Bài 266. (Hong Kong TST 2008) Cho ngũ giác lồi ABCDE. Gọi A1 là giao của CE và BD, B1 là
giao của AD và CE, tương tự định nghĩa C1 , D1 , E1 . Gọi A2 là giao của (AC1 E) và (AD1 B). Tương
tự định nghĩa B2 , C2 , D2 , E2 . Chứng minh rằng AA2 , BB2 , CC2 , DD2 , EE2 đồng quy.

191
S

A2 Q

B
C1 D1
E
E2 B2
X
E1
B1

C2
A1
D2
D
C

Chứng minh. Gọi S là giao điểm thứ hai của (AB2 C) và (ED2 C).
Ta có ∠ED2 C = 180◦ − ∠D2 EB1 − ∠D2 CA1 = 180◦ − ∠B1 DD2 − ∠A1 DD2 = 180◦ − ∠B1 DA1 .
Tương tự ∠DB2 C = 180◦ − ∠D1 BE1 . Do đó ∠ESA = ∠ESC + ∠ASC = 180◦ − ∠ED2 C + 180◦ −
∠AB2 C = ∠C1 DB + ∠C1 BD = 180◦ − ∠EC1 A, nghĩa là S nằm trên (AC1 E).
Gọi Q là giao của (CD2 E) và EB. QC cắt AD tại R. Ta có S là điểm Miquel của tam giác C1 RQ
ứng với bộ 3 điểm A, E, C. Suy ra R nằm trên (AB2 C).
Ta có ∠B1 DD2 = ∠D2 EC = ∠D2 QR nên R, D, D2 , Q cùng thuộc một đường tròn, tương tự
R, B2 , B, Q cùng thuộc một đường tròn.
Mặt khác, ∠BQR = 180◦ − ∠ED2 C = ∠ADB nên R, D, B, Q cùng thuộc một đường tròn.
Suy ra D, D2 , B2 , B cùng thuộc một đường tròn ω1 .
Tương tự, E, E2 , B2 , B cùng thuộc đường tròn ω2 .
Áp dụng định lý về tâm đẳng phương cho 3 đường tròn ω1 , ω2 , (EB1 D) suy ra EE2 , DD2 , BB2 đồng
quy. Chứng minh tương tự, AA2 , BB2 , DD2 đồng quy, BB2 , CC2 , EE2 đồng quy. Ta có đpcm.

Bài 267. Chứng minh rằng nếu ABCDE là ngũ giác nội tiếp thì A2 B2 C2 D2 E2 cũng là ngũ giác nội
tiếp.

Chứng minh. Gọi X là giao điểm của AA2 , BB2 , CC2 , DD2 , EE2 . Đặt k = XA.XA2 = ... = XE.XE2 .
k : A 7→ A , ..., E 7→ E . Do đó (ABCDE) 7→ (A B C D E ).
Phép nghịch đảo IX 2 2 2 2 2 2 2

Bài 268. Gọi A3 là giao của (B1 DA1 ) và (E1 CA1 ). Tương tự xác định B3 , C3 , D3 , E3 . Chứng minh
rằng A3 , B3 , C3 , D3 , E3 cùng thuộc một đường tròn. (Đường tròn Miquel trong hình sao).

192
A

C3 D3

D1 B
E C1

B1 E1
B3

E3
A1

A3
D
C

Chứng minh. Áp dụng định lý điểm Miquel cho tứ giác toàn phần AB1 CEC1 D1 ta có C3 là giao của
(EB1 C1 ) và (AD1 C1 ). Suy ra C3 ∈ (AB1 C). Tương tự, A3 ∈ (AB1 C).
Ta có ∠D3 E3 A3 = ∠D3 E3 E1 + ∠A3 E3 E1 = ∠AD1 D3 + ∠DA1 A3 = ∠AC3 D3 + ∠AB1 A3
= 180◦ − ∠A3 B1 A + ∠AC3 D3 = 180◦ − ∠A3 C3 A + ∠AC3 D3 = 180◦ − ∠A3 C3 D3 .
Do đó tứ giác A3 C3 D3 E3 nội tiếp. Chứng minh tương tự ta có đpcm.

Nhận xét 1. Có thể chứng minh bài toán dựa theo bổ đề sau:
Bổ đề. Cho 4 điểm A, B, C, D theo thứ tự nằm trên một đường thẳng. Gọi (O1 ), (O2 ), (O3 ), (O4 )
lần lượt là đường tròn qua các cặp điểm (A, B);(B, C);(C, D);(D, A). X, Y, Z, T lần lượt là giao điểm
thứ hai của các cặp đường tròn (O1 ) và (O2 ), (O2 ) và (O3 ), (O3 ) và (O4 ), (O4 ) và (O1 ). Khi đó
X, Y, Z, T cùng thuộc một đường tròn.
Bổ đề trên có thể chứng minh đơn giản bằng phép nghịch đảo cực A phương tích ki bất kì biến mô
hình bài toán thành bài toán điểm Miquel của tam giác.
Nhận xét 2. Có thể mở rộng bài toán hình sao Miquel cho n đường thẳng (xem chương chuỗi
đường tròn).
Bài 269. Chứng minh rằng nếu ngũ giác ABCDE nội tiếp thì AA3 , BB3 , CC3 , DD3 , EE3 đồng quy.

D3

B
C3 C1 D1
E
B1
E1

B3 A1 E3

A3
D C

193
Chứng minh. Theo phép chứng minh bài toán 3 ta có các ngũ giác AC3 B1 A3 C, BE3 A1 B3 E nội tiếp.
Ta có ∠A3 B3 B = ∠A3 B3 A1 + ∠A1 B3 B = ∠A3 B1 A1 + ∠CEB = ∠A3 AC + ∠CAB = ∠CAB.
Do đó tứ giác CB3 A3 B nội tiếp đường tròn ω1 .
Chứng minh tương tự tứ giác AEA3 E3 nội tiếp đường tròn ω2 .
Áp dụng định lý tâm đẳng phương cho 3 đường tròn ω1 , ω2 , (BE3 A1 B3 E) suy ra AA3 , EE3 , BB3
đồng quy.
Chứng minh tương tự ta có đpcm.

19.3 Hình sao Morley


Năm 1907, trên tạp chí Transactions of the American Mathematical Society , Frank Morley
đưa ra bài toán sau: Cho năm điểm A1 , A2 , A3 , A4 , A5 trên đường tròn (O). Gọi O1 , O2 , O3 , O4 , O5
lần lượt là điểm chính giữa cung A3 A4 , A4 A5 , A5 A1 , A1 A2 , A2 A3 ; (O1 ), (O2 ), (O3 ), (O4 ), (O5 ) lần lượt
là (O1 , O1 A4 ), (O2 , O2 A5 ), (O3 , O3 A1 ), (O4 , O4 A2 ), (O5 , O5 A3 ) ;B1 , B2 , B3 , B4 , B5 lần lượt là giao điểm
thứ hai của (O3 ) và (O4 ), (O4 ) và (O5 ), (O5 ) và (O1 ), (O1 ) và (O2 ), (O2 ) và (O3 ). Các cặp đường thẳng
(B2 B3 , B4 B5 ), (B3 B4 , B1 B5 ), (B4 B5 , B1 B2 ), (B1 B5 , B2 B3 ), (B1 B2 , B3 B4 ) cắt nhau tạo thành ngũ giác
X1 X2 X3 X4 X5 . Khi đó các đỉnh của ngũ giác trên đều lần lượt nằm trên (O1 ), (O2 ), (O3 ), (O4 ), (O5 ).
Sau đây chúng ta sẽ chứng minh lại bài toán trên và khai thác một số tính chất trong trường hợp
ngũ giác Morley là ngũ giác đặc biệt.

19.3.1 Chứng minh


X4

O4
A1

X3 A2

O3 B1 B2
X5
O5

B5
A5
B3
A3

B4

O2
A4 O1

X2
X1

Chứng minh. Ta có O1 O5 đồng thời là phân giác của ∠A3 O1 B3 và ∠A3 O1 A2 nên O1 , B3 , A2 thẳng
hàng.
Chứng minh tương tự, A3 , B5 , O4 thẳng hàng. Suy ra ∠B2 B3 A2 = ∠B2 A3 A2 = ∠O4 O1 A2 hay
B2 B3 k O1 O4 .
Tương tự B1 B5 k O2 O4 . Từ đó ∠B1 X4 B2 = ∠O1 O4 O2 . Mà O1 , O2 lần lượt là điểm chính giữa
1
cung A3 A4 , A4 A5 nên ∠O1 O4 O2 = ∠A3 O4 A5 , nghĩa là X4 nằm trên (O4 ). Chứng minh tương tự ta
2
có đpcm.

194
Chúng ta giả sử rằng X1 X2 X3 X4 X5 là một ngũ giác nội tiếp. Khai thác bài toán ta thu được một
số tính chất sau:

19.3.2 Khai thác


.

Tính chất 113. X1 A1 , X2 A2 , X3 A3 , X4 A4 , X5 A5 đồng quy tại một điểm (kí hiệu là P ).

X4

A1
O4
X3

A2
O3 B1 B2
X5
O5
P
B5
A5 B3

A3
B4

O2 O1
A4

X1
X2

1
Chứng minh. Ta có B1 B5 k O2 O4 nên ∠B1 O4 O2 = ∠X4 B1 O4 = 90 ◦ − ∠B1 O4 X4 .
2
1 ◦
Mà ∠B1 O4 O2 = ∠A5 O4 A4 nên ∠A5 O4 A4 + ∠B1 O4 X4 = 180 hay X4 , O4 , A4 thẳng hàng. Tương
2
tự các bộ ba (X5 , O5 , A5 ), (X3 , O3 , A3 ) thẳng hàng. (1)
Mặt khác, ∠X4 X3 X5 = ∠X4 X2 X5 = ∠O4 O2 O5 = ∠O4 O3 O5 , mà X3 X5 k O3 O5 nên O3 O4 k X3 X4 .
Tương tự O4 O5 k X4 X5 nên hai tam giác O3 O4 O5 và X3 X4 X5 có cạnh tương ứng song song. Suy ra
O3 X3 , O4 X4 , O5 X5 đồng quy tại P . Kết hợp với (1) ta có đpcm. Điểm P chính là tâm vị tự của hai
ngũ giác X1 X2 X3 X4 X5 và O1 O2 O3 O4 O5 .

Tính chất 114. Các đường vuông góc kẻ từ X1 tới B3 B4 , X2 tới B4 B5 , X3 tới B1 B5 , X4 tới B1 B2 , X5
tới B2 B3 đồng quy tại một điểm (kí hiệu là Q)

195
X4

A1
O4
X3

A2
O3
B1 B2

H5 X5
O5
H3
B5
A5 Q P B3

H1 A3
B4
O1
O2
A4

X1
X2

Chứng minh. Gọi H1 , H2 , H3 , H4 , H5 lần lượt là hình chiếu của X1 trên B3 B4 , X2 trên B4 B5 , X3 trên
B1 B5 , X4 trên B1 B2 , X5 trên B2 B3 . Ta có X1 X5 k O1 O5 , A3 B3 ⊥ O1 O5 nên A3 B3 ⊥ X1 X5 . Suy
ra ∠A3 X1 X5 = ∠B3 A3 X1 − 90◦ . Mặt khác ∠B4 X1 H1 = 90◦ − ∠X1 B4 H1 = ∠B3 A3 X1 − 90◦ . Suy ra
∠A3 X1 X5 = ∠B4 X1 H1 .
Tương tự ∠A3 X5 X1 = ∠H5 X5 X3 . Do đó giao điểm Q của X1 H1 và X5 H5 là điểm liên hợp đẳng
giác của A3 trong tam giác X1 X3 X5 . Lại có X3 , O3 , A3 thẳng hàng nên X3 Q và X3 O3 là hai đường
đẳng giác trong góc B1 X3 B5 . Suy ra X3 , H3 , Q thẳng hàng. Vậy X1 H1 , X3 H3 , X5 H5 đồng quy. Chứng
minh tương tự ta có đpcm.

Tính chất 115. B1 B2 B3 B4 B5 là ngũ giác nội tiếp.

X4

A1
O4
X3

A2
O3
B1 B2

H5 X5
O5
P
B5
A5 Q B3

A3
H1
B4
O1
O2
A4

X1
X2

Chứng minh. Theo tính chất 2 ta có B3 là trực tâm tam giác QX1 X5 , suy ra QB3 ⊥ X1 X5 . Mà A3 B3 ⊥
X1 X5 nên Q, B3 , A3 thẳng hàng. Chứng minh tương tự ta thu được A1 B1 , A2 B2 , A3 B3 , A4 B4 , A5 B5

196
đồng quy tại Q. Suy ra Q là tâm đẳng phương của năm đường tròn (O1 ), (O2 ), (O3 ), (O4 ), (O5 ). Gọi
k là phương tích của Q đối với năm đường tròn trên. Xét phép nghịch đảo tâm Q, phương tích
k : A1 7→ B1 , A2 7→ B2 , A3 7→ B3 , A4 7→ B4 , A5 7→ B5 . Mà A1 A2 A3 A4 A5 nội tiếp nên B1 B2 B3 B4 B5 nội
tiếp.

Tính chất 116. A1 A2 A3 A4 A5 là một ngũ giác lưỡng tâm.

Chứng minh. (Hình vẽ xem tính chất 3).


Ta có ∠A4 A5 B5 = ∠A4 B4 X1 = ∠A4 B3 X1 = ∠B2 B3 O5 .
= ∠B3 B2 O5 = ∠A1 B2 X4 = ∠A1 B1 X4 = ∠B5 A5 A1 .
Suy ra đường phân giác các góc của ngũ giác A1 A2 A3 A4 A5 đồng quy tại Q, hay ngũ giác A1 A2 A3 A4 A5
ngoại tiếp.

Tính chất 117. Các cặp tam giác sau đồng dạng: P B1 B5 và X3 A5 A1 , P B1 B2 và X4 A2 A1 , P B2 B3 và
X5 A3 A2 , P B3 B4 và X1 A4 A3 , P B4 B5 và X2 A5 A4 .

X4

A1
X3 O4

A2
O3
B1 B2

O5 X5
H3
Q P
A5 B3
B5
A3
B4 H1
O1
O2
A4

X1
X2

Chứng minh. Ta chỉ chứng minh cặp tam giác P B1 B5 và X3 A5 A1 đồng dạng.
Do các cặp X1 P, X1 Q và X4 P, X4 Q là hai đường đẳng giác trong ∠B5 X1 X4 v∠B5 X4 X1 nên P, Q
là hai điểm liên hợp đẳng giác trong tam giác B5 X1 X4 . Suy ra ∠P B5 B1 = ∠QB5 X1 = ∠A5 B5 X3 =
∠X3 A1 A5 .
Tương tự ∠X3 A5 A1 = ∠P B1 B5 . Từ đó suy ra đpcm.

Tính chất 118. Tập hợp 10 hình chiếu của P, Q trên năm cạnh B1 B2 , B2 B3 , B3 B4 , B4 B5 , B5 B1 cùng
thuộc một đường tròn tâm là trung điểm P Q.

197
X4

A1
X3 O4

A2
O3 H4 K
B1 4
B2
K3
O5 X5
H3 H5
Q P K5
A5 O B3
B5
H2 A3
K2 K1
B4 H1
O1
O2
A4

X1
X2

Chứng minh. Gọi K1 , K2 , K3 , K4 , K5 lần lượt là hình chiếu của P trên các cạnh B3 B4 , B4 B5 , B5 B1 ,
B1 B2 , B2 B3 .
Ta có ∠B5 K2 K3 = ∠B5 P K3 = 90◦ − ∠P B5 K3 = 90◦ − ∠QB5 H2 = ∠B5 QH2 = ∠K3 H3 H2 .
Suy ra H2 , K3 , H3 , K2 cùng thuộc một đường tròn. Dễ thấy đường trung trực d3 của K3 H3 là đường
trung bình của hình thang K3 P QH3 nên d3 đi qua trung điểm O của P Q. Tương tự đường trung
trực d2 của K2 H2 đi qua trung điểm O của P Q. Do đó O là tâm của (H2 , K3 , H3 , K2 ), tương tự O là
tâm của (K3 , K4 , H3 , H4 ), (K4 , K5 , H4 , H5 ), (K5 , K1 , H5 , H1 ) nên 10 điểm K1 , K2 , ..., K5 , H1 , H2 , ..., H5
cùng nằm trên đường tròn tâm O.

Tính chất 119. Gọi I1 , I2 , I3 , I4 , I5 lần lượt là tâm đường tròn ngoại tiếp của các tam giác X3 B1 X4 ,
X4 B2 X5 , X5 B3 X1 , X1 B4 X2 , X2 B5 X4 . Khi đó hai ngũ giác I1 I2 I3 I4 I5 và B1 B2 B3 B4 B5 có tâm vị tự
P.

X4
I1

A1
X3 O4
I'1
A2
O3 B1
B2

S T1 X5
O5
B5 P
A5 Q B3
T5

I5 A3
B4

O2 O1
A4

X1
X2

198
Chứng minh. Gọi T1 là giao của O2 O4 và O3 O5 ; S là giao của (I5 ) với X3 Q, I10 là tâm (O3 T1 O4 ). Ta
có B5 là trực tâm của tam giác X3 QX2 nên ∠X3 QX2 = 180◦ − ∠X3 B5 X2 = ∠X2 SQ.
Suy ra S là điểm đối xứng với Q qua X2 X4 . Tương tự ta chứng minh được S nằm trên (I1 ). Mà
X3 Q ⊥ X2 X4 suy ra I1 I5 k B1 B5 . Tương tự I1 I2 k B1 B2 , ..., I4 I5 k B4 B5 .(1)
Mặt khác ta có P là tâm vị tự của hai ngũ giác X1 X2 X3 X4 X5 và O1 O2 O3 O4 O5 nên P, T1 , B1 thẳng
hàng.
Lại có I1 O4 ⊥ X2 X4 và X2 X4 k O2 O4 suy ra ∠I1 O4 T1 = 90◦ . Tương tự ∠I1 O3 T1 = 90◦ , ta thu
được I10 là trung điểm I1 T1 .
Ta có ∠I10 T1 O4 = 90◦ − ∠T1 O3 O4 = 90◦ − ∠X4 X3 X5 = 90◦ − ∠B5 X2 B4 = ∠O2 B5 B4
= ∠X3 B5 A1 = ∠X3 B1 A1 = ∠QB1 B2 = ∠P B1 B5 = ∠B1 T1 O4
Do đó T1 , B1 , I10 thẳng hàng. Ta thu được P, B1 , I1 thẳng hàng. Tương tự các bộ ba (P, B5 , I5 ),
(P, B2 , I2 ), (P, B3 , I3 ), (P, B4 , I4 ) thẳng hàng. (2)
P I1
Từ (1) và (2) suy ra I1 I2 I3 I4 I5 là ảnh của B1 B2 B3 B4 B5 qua phép vị tự tâm P tỉ số .
P B1

20 Chuỗi đường tròn


20.1 Chuỗi đường tròn Clifford
20.1.1 Giới thiệu
Để ngắn gọn trong bài viết này tập hợp đường thẳng đã cho được ngầm hiểu là không có ba đường
thẳng nào đồng quy và không có hai đường thẳng nào song song.
Chúng ta bắt đầu từ một định lý quen thuộc về điểm Miquel: Trên mặt phẳng cho 4 đường thẳng cắt
nhau tạo thành 4 tam giác. Khi đó đường tròn ngoại tiếp 4 tam giác đồng quy tại một điểm gọi là điểm
Miquel của 4 đường thẳng. Miquel cũng chứng minh trong trường hợp 5 đường thẳng rằng 5 điểm Miquel
của mỗi bộ 4 trong 5 đường thẳng cùng nằm trên một đường tròn, gọi là đường tròn Miquel của 5 đường
thẳng. Cũng xin nêu một trường hợp rất đẹp là đường tròn Miquel của hình sao năm cánh, được phát
biểu như sau: Cho ngũ giác lồi B1 B2 B3 B4 B5 . Gọi A1 , A2 , A3 , A4 , A5 lần lượt là giao điểm của các cặp
đường thẳng (B2 B3 , B4 B5 ), (B3 B4 , B1 B5 ), (B4 B5 , B1 B2 ), (B2 B3 , B5 B1 ), (B1 B2 , B3 B4 ). Gọi C1 là giao
điểm của (A4 B1 B2 ) và (A3 B1 B5 ). Tương tự ta xác định C2 , C3 , C4 , C5 . Khi đó 5 điểm C1 , C2 , C3 , C4 , C5
cùng thuộc một đường tròn [2]. Một câu hỏi đặt ra là liệu có thể tổng quát định lý nêu trên không?

P14 A4
C2

P
C1
C3 P13
l4 C2
C1
P34 l3
l1 A3 B1
A5
B2
P12 l2 P24 P23
C5
B5 B3 C3

B4
C4

A2 A1
C4

Năm 1870, W.K.Clifford, một nhà toán học Anh, đã tổng quát bài toán cho n đường thẳng. Cụ
thể, với n = 6, 6 đường tròn Miquel của mỗi bộ 5 trong 6 đường thẳng đồng quy tại một điểm, gọi là

199
điểm Clifford của 6 đường thẳng. Với n = 7, 7 điểm Clifford của mỗi bộ 6 trong 7 đường thẳng cùng
thuộc một đường tròn, gọi là đường tròn Clifford của 7 đường thẳng. Với n = 8, 8 đường tròn Clifford
của mỗi bộ 7 trong 8 đường thẳng đồng quy tại một điểm, gọi là điểm Clifford của 8 đường thẳng. Bài
toán cũng đúng với trường hợp n bất kì lớn hơn 3.
Có hai trường hợp xảy ra. Nếu n chẵn, n đường tròn Clifford của mỗi bộ n − 1 trong n đường thẳng
đồng quy tại điểm Clifford của n đường thẳng. Nếu n lẻ, n điểm Clifford của mỗi bộ n − 1 trong n
đường thẳng cùng thuộc đường tròn Clifford của n đường thẳng.
Quay lại định lý về điểm Miquel, sử dụng phép nghịch đảo phương tích bất kì có tâm là điểm
bất kì nằm ngoài 4 đường thẳng và không nằm trên 4 đường tròn ngoại tiếp 4 tam giác. Định lý
Miquel trở thành bài toán: Cho một điểm P bất kì trên mặt phẳng và 4 đường tròn Ci (i = 1, 4) đi
qua P . Gọi Pij là giao điểm thứ hai của Ci và Cj ; Cijk là đường tròn qua 3 điểm Pij , Pjk , Pik . Khi
đó 4 đường tròn C234 , C134 , C124 , C123 đồng quy tại điểm P1234 gọi là điểm Clifford của 4 đường tròn
Ci (i = 1, 4). Bằng một số suy luận đơn giản ta cũng nhận thấy P là điểm Clifford của 4 đường tròn
C234 , C134 , C124 , C123 . Sử dụng phép nghịch đảo tương tự trong trường hợp 5 đường thẳng, gọi C5 là
đường tròn thứ 5 đi qua P . Khi đó 5 điểm P2345 , P1345 , P1245 , P1235 , P1234 cùng thuộc đường tròn C12345
gọi là đường tròn Clifford của 5 đường tròn. Tổng quát, cho n đường tròn Ci (i = 1, n) đi qua P . Trường
hợp n chẵn, n đường tròn C23...n , C13..n , ..., C12...n−1 đồng quy tại điểm P12...n . Trường hợp n lẻ, n điểm
P23...n , P13...n , ..., P12...n−1 cùng thuộc đường tròn C12...n . Đó là dạng phát biểu thứ hai của định lý về
chuỗi đường tròn Clifford.

C4

P14 C134
C124

C1
P24
P34

P
P1234

C3
P12 P13
C234

P23

C2
C123

Sau đây chúng ta sẽ chứng minh dạng phát biểu thứ hai của định lý chuỗi đường tròn Clifford.
Bạn đọc cũng có thể tìm thấy lời giải khác cho dạng phát biểu thứ nhất trong [7].

Chứng minh. Trường hợp n = 4 là một định lý quen thuộc nên xin nhường lại cho bạn đọc.
Với n = 5. Gọi C là đường tròn qua 3 điểm P2345 , P1345 , P1245 .
Có 4 đường tròn qua điểm P2345 là C, C345 , C245 , C235 .
3 đường tròn C345 , C245 , C235 có các giao điểm thứ hai lần lượt là P45 , P35 , P25 . 3 điểm này cùng
thuộc đường tròn C5 .
3 đường tròn C, C345 , C245 có các giao điểm thứ hai lần lượt là P1345 , P1245 , P45 . 3 điểm này cùng
thuộc đường tròn C145 .
Lại có C5 giao C145 tại 2 điểm P15 và P45 . Như vậy P15 là điểm Clifford của 4 đường tròn
C, C345 , C245 , C235 .

200
Mặt khác, 3 đường tròn C, C345 , C235 giao nhau tại 3 điểm P1345 , X, P35 (2.1). Ta suy ra X thuộc
đường tròn qua 3 điểm P1345 , P35 , P15 (đường tròn C135 ). Lại có X thuộc đường tròn C235 nên X là
điểm P1235 hoặc P35 . Do X và P35 phân biệt theo (2.1) nên X là P1235 , tức là P2345 , P1345 , P1245 , P1235
cùng thuộc một đường tròn. Tương tự suy ra trường hợp n = 5 đúng.
Với n = 6, ta chứng minh C23456 , C13456 , C12456 , C12356 đồng quy.
Giao điểm của C13456 , C12456 , C12356 lần lượt là P1456 , P1356 , P1256 . 3 điểm này cùng thuộc đường
tròn C156 .
Giao điểm của C23456 , C12456 , C12356 lần lượt là P2456 , P2356 , P1256 . 3 điểm này cùng thuộc đường
tròn C256 .
Giao điểm của C23456 , C13456 , C12356 lần lượt là P3456 , P2356 , P1356 . 3 điểm này cùng thuộc đường
tròn C356 .
Giao điểm của C23456 , C13456 , C12456 lần lượt là P3456 , P2456 , P1456 . 3 điểm này cùng thuộc đường
tròn C456 .
4 đường tròn C156 , C256 , C356 , C456 cùng đi qua P56 nên áp dụng định lý điểm Clifford của 4
đường tròn ta thu được C23456 , C13456 , C12456 , C12356 đồng quy tại điểm Clifford của 4 đường tròn
C156 , C256 , C356 , C456 . Chứng minh tương tự suy ra trường hợp n=6 đúng.
Trường hợp tổng quát hoàn toàn chứng minh tương tự hai trường hợp n = 5; 6. Bằng phép nghịch
đảo suy ra dạng phát biểu thứ nhất của đường tròn Clifford đúng. Bài toán được chứng minh.
Một điều thú vị là ta đã xây dựng một tập hợp gồm Cn1 +Cn3 +... = 2n−1 đường tròn và Cn2 +Cn4 +... =
2 n−1 điểm. Trong đó mỗi đường tròn đều đi qua n điểm và mỗi điểm đều nằm trên n đường tròn.
Thật vậy, xét đường tròn bất kì Ck1 k2 ...kj (kq ∈ {1, 2, ..., n}, q = 1, j). Ck1 k2 ...kj đi qua Pk10 k20 ...kj−1
0 (kq0 ∈
{k1 , k2 , ..., kj }, q = 1, j − 1) và Pk1 k2 ...kj ki (i ∈ {1, 2, ..., n}, i > j). Như vậy Ck1 k2 ...kj đi qua j+(n−j) = n
điểm. Tương tự ta cũng chứng minh được mỗi điểm đều thuộc n đường tròn.

20.1.2 Khai thác một dạng phát biểu khác của chuỗi đường tròn Clifford
Chúng ta bắt đầu từ một bài toán trong mục đề ra kì này của tạp chí Toán học và tuổi trẻ.
Bài 270. Trong mặt phẳng cho 4 điểm A1 , A2 , A3 , A4 và một điểm P bất kì. Khi đó đường tròn pedal
của P ứng với các tam giác A2 A3 A4 , A1 A3 A4 , A1 A2 A4 , A1 A2 A3 đồng quy tại một điểm [8].

A2 C123
P12
C2
A1

P1234
P23
C134 P13

A3
C1
P

P14 C3
P34

C4
A4

Chứng minh. Gọi C1 , C2 , C3 , C4 lần lượt là đường tròn đường kính P A1 , P A2 , P A3 , P A4 . Kí hiệu Pij là
giao điểm thứ hai của Ci và Cj (i; j = 1, 4). Cijk là đường tròn ngoại tiếp tam giác Pij Pjk Pki (k = 1, 4).
Theo định lý đường tròn Clifford ta thu được C234 , C134 , C124 , C123 đồng quy tại P1234 .

201
Mặt khác ta nhận thấy Pij Pjk Pki là tam giác pedal của tam giác Ai Aj Ak (k = 1, 4). Từ đó suy ra
đpcm.

Nhận xét 10. Bài toán 1 có thể coi là một dạng phát biểu khác của đường tròn Clifford. Một cách
tương tự ta có thể tổng quát bài toán 1 cho n điểm Ai (i = 1, n). Với n = 5, cho điểm P và 5 điểm
Ai (i = 1, 5). Gọi Pijkm là giao điểm của các đường tròn Cjkm , Cikm , Cijm , Cijk (i, j, k, m = 1, 5). Khi
đó 5 điểm P2345 , P1345 , P1245 , P1235 , P1234 cùng thuộc đường tròn C12345 . Các điểm và đường tròn được
định nghĩa tương tự với n bất kì (n > 3), chia ra hai trường hợp n chẵn hoặc lẻ.

Bài 271. Cho tứ giác ABCD nội tiếp đường tròn (O) và một điểm P bất kì không nằm trên (O). Khi
đó tâm các đường tròn pedal của P ứng với các tam giác ABC, BCD, CDA, DAB cùng thuộc một
đường tròn.

Mab
Od
Oc A

Mbd
Oa
Mad
O
Ob
P

D
C

Chứng minh. Gọi (Oa ), (Ob ), (Oc ), (Od ) lần lượt là đường tròn pedal của P ứng với các tam giác
BCD, ACD, ABD, ABC; Mab , Mad , Mbd lần lượt là trung điểm AB, AD, BD.
Theo bài toán 1, (Oa ), (Ob ), (Oc ), (Od ) đồng quy tại K.
Do Oc Od ⊥ Mab K, Oc Ob ⊥ Mad K nên ta có:
(Oc Ob , Oc Od ) ≡ (KMad , KMab ) ≡ (Mbd Mad , Mbd Mab ) ≡ (AMab , AMad ) ≡ (AB, AD) (mod π)
Tương tự, (Oa Ob , Oa Od ) ≡ (CB, CD) (mod π)
Mà tứ giác ABCD nội tiếp nên (Oc Ob , Oc Od ) ≡ (AB, AD) ≡ (CB, CD) ≡ (Oa Ob , Oa Od ) (mod π).
Từ đó suy ra đpcm.

Nhận xét 11. Khi P ≡ O suy ra (Oa ), (Ob ), (Oc ), (Od ) lần lượt là đường tròn Euler của các tam giác
R
BCD, ACD, ABD, ABC và bán kính của bốn đường tròn đều bằng hay một nửa bán kính (O). Lại
2
R
có (Oa ), (Ob ), (Oc ), (Od ) đồng quy tại K nên Oa , Ob , Oc , Od ∈ (K, ), được gọi là đường tròn Euler
2
của tứ giác nội tiếp ABCD. Ta cũng có thể tổng quát đường tròn Euler cho đa giác n đỉnh.

Bài 272. Ta định nghĩa đường tròn Euler của dây A1 A2 của (O, R) là đường tròn đi qua trung điểm
R
A1 A2 có bán kính . Đường tròn Euler của tam giác A1 A2 A3 nội tiếp (O) là đường tròn có tâm là
2
R
giao điểm của ba đường tròn Euler của ba dây A1 A2 , A2 A3 , A1 A3 , bán kính . Tổng quát, đường tròn
2
Euler của n−giác A1 A2 ...An nội tiếp (O) là đường tròn có tâm là giao điểm của n đường tròn Euler

202
R
của (n − 1)−giác có đỉnh là một trong n điểm Ai (i = 1, n), bán kính. Đồng thời đường tròn Euler
2
của n−giác nội tiếp đi qua tâm của n đường tròn Euler của (n − 1)−giác có đỉnh là (n − 1) trong n
đỉnh trên.

C23

O23

C2
O2 C3

O3
O123

O12
O13
C12

O1 C13

C1

Chứng minh. Giả sử bài toán đúng với (n − 1)−giác. Ta chứng minh bài toán đúng với n−giác.
Trước tiên cho n−giác A1 A2 ...An nội tiếp (O). Ta kí hiệu Ci1 i2 ...ik là đường tròn Euler của (n −
k)−giác có đỉnh thuộc tập hợp {A1 , A2 , ..., An /Ai1 , Ai2 , ..., Aik }(i1 , i2 , ..., ik ∈ {1, 2..., n}); Oi1 i2 ...ik là
tâm của Ci1 i2 ...ik .
Như vậy ta chỉ cần chứng minh n đường tròn Ci (i = 1, n) đồng quy.
Từ giả thiết quy nạp suy ra ba đường tròn C23 , C13 , C12 đồng quy tại O123 và O1 , O2 , O3 lần lượt
là giao điểm của C12 và C13 , C12 và C23 , C13 và C23 .
R R
Do C12 , C13 , C23 có bán kính đều bằng nên (O123 , ) là đường tròn ngoại tiếp của tam giác
2 2
O23 O13 O12 .
C23 và C12 giao nhau tại O123 và O2 nên O2 là điểm đối xứng với O123 qua O12 O23 . Tương tự O1 , O3
lần lượt là điểm đối xứng với O123 qua O13 O12 , O13 O23 . Theo một kết quả quen thuộc C1 , C2 , C3 đồng
quy tại trực tâm của tam giác O23 O13 O12 .
Chứng minh tương tự suy ra n đường tròn Ci (i = 1, n) đồng quy. Từ đó suy ra đpcm.

Để kết thúc mục 1.2, xin giới thiệu và không chứng minh một hệ quả tổng quát. Bài toán có thể
được chứng minh dựa theo bài toán 1, điểm Euler-Poncelet của 4 điểm và phương pháp quy nạp.

Bài 273. Trong mặt phẳng cho 4 điểm A1 , A2 , A3 , A4 sao cho bất kì 3 điểm nào trong số đó đều không
thẳng hàng và không có điểm nào là trực tâm của tam giác tạo bởi ba điểm còn lại. Ta định nghĩa
điểm An là trực tâm của tam giác Ai Aj Ak (1 ≤ i < j < k ≤ n − 1, n > 4). Từ đó tạo thành tập hợp
điểm {A1 , A2 , ..., An }. Khi đó tất cả đường tròn pedal của bất kì điểm Ax ứng với tam giác tạo bởi
3 trong số n − 1 điểm còn lại và tất cả đường tròn Euler của các tam giác tao bởi 3 trong số n điểm
đồng quy. Như vậy chúng ta có Cn−1 3 .n + C 3 đường tròn đồng quy tại một điểm.
n

20.2 Chuỗi đường tròn Morley và điểm de Longchamps


Trong phần này chúng ta đề cập đến một chuỗi đường tròn và điểm tương tự như chuỗi Clifford.

203
Năm 1877, G. de Longchamps đưa ra bài toán như sau: Trong mặt phẳng cho n đường thẳng. Với
n = 4, 4 đường thẳng cắt nhau tạo thành 4 tam giác. Khi đó tâm của các đường tròn ngoại tiếp 4 tam
giác này cùng thuộc một đường tròn, gọi là đường tròn Morley của 4 đường thẳng. Đường tròn Morley
cũng đồng thời đi qua điểm Miquel của 4 đường thẳng đó. Với n = 5, 5 đường tròn Morley của mỗi
bộ 4 trong 5 đường thẳng đó đồng quy tại một điểm gọi là điểm de Longchamps của 5 đường thẳng,
đồng thời tâm của 5 đường tròn Morley cùng thuộc một đường tròn, gọi là đường tròn Morley của 5
đường thẳng. Tổng quát với n bất kì (n ≥ 4), n đường tròn Morley của mỗi bộ n − 1 trong n đường
thẳng đó đồng quy tại một điểm gọi là điểm de Longchamps của n đường thẳng, đồng thời tâm của n
đường tròn Morley cùng thuộc một đường tròn, gọi là đường tròn Morley của n đường thẳng [3]. Lời
giải sau đây dựa theo [5] hoặc [6].

C1

P14 C2
P

C3 O2
P13 O1
P34
O3
l4 l3
l1
O4

l2
P12 P24 P23

C4

Chứng minh. Trước tiên cho n đường thẳng li (i = 1, n). Kí hiệu Ci là đường tròn Morley của tập hợp
n − 1 đường thẳng ngoại trừ li ; Oi là tâm của Ci ; Cij là đường tròn Morley của tập hợp n − 2 đường
thẳng ngoại trừ li , lj ; Oij là tâm của Cij ; Pij là giao điểm của li và lj .
Trường hợp n = 4. Ta có C1 , C2 , C3 , C4 đồng quy tại M là điểm Miquel của 4 đường thẳng.
Do O2 O3 ⊥ P14 M, O1 O3 ⊥ P24 M nên (O3 O2 , O3 O1 ) ≡ (M P14 , M P24 ) ≡ (P12 P14 , P12 P24 ) ≡ (l1 , l2 )
(mod π).
Tương tự (O4 O2 , O4 O1 ) ≡ (l1 , l2 ) (mod π). Từ đó suy ra O1 , O2 , O3 , O4 cùng thuộc một đường
tròn C. Kết quả C đi qua M thu được từ một số biến đổi góc đơn giản, xin không trình bày ở đây.
Như trên ta đã chứng minh (O4 O2 , O4 O1 ) ≡ (l1 , l2 ) (mod π) hay tổng quát (Ok Oi , Ok Oj ) ≡ (li , lj )
(mod π)(4.1) đúng với n = 4.
Giả sử (4.1) đúng với n − 1 đường thẳng (n ≥ 5) , ta chứng minh nó cũng đúng với n đường thẳng.
Thật vậy, xét trường hợp có n đường thẳng. Hai đường tròn C1 và C2 giao nhau tại O12 và một
điểm L.
Ta có (LO23 , LO13 ) ≡ (LO23 , LO12 ) + (LO12 , LO13 ) (mod π).
Do L, O12 , O23 ∈ C2 và theo điều giả sử (4.1) đúng với n−1 đường thẳng nên (LO23 , LO12 ) ≡ (l3 , l1 )
(mod π). Tương tự (LO12 , LO13 ) ≡ (l2 , l3 ) (mod π).
Suy ra (LO23 , LO13 ) ≡ (l2 , l3 ) + (l3 , l1 ) ≡ (l2 , l1 ) (mod π).
Từ đó L ∈ C3 , tương tự suy ra n đường tròn Ci (i = 1, n) đồng quy tại L.
Mặt khác do C1 và C2 giao nhau tại O12 và L nên O1 O2 ⊥ LO12 . Tương tự, O1 O3 ⊥ LO13 .
Từ đó (O1 O2 , O1 O3 ) ≡ (LO12 , LO13 ) ≡ (l2 , l3 ) (mod π)
Tương tự (Oi O2 , Oi O3 ) ≡ (l2 , l3 ) (mod π). Ta thu được n điểm Oi (i = 1, n) cùng thuộc một đường
tròn.
Đồng thời cũng suy ra (4.1) đúng với n đường thẳng.

204
Theo nguyên lý quy nạp bài toán được chứng minh.

20.3 Chuỗi đường tròn Steiner và Pappus


20.3.1 Chuỗi Steiner
Chuỗi Steiner mang tên nhà toán học Jakob Steiner, là một tập hợp gồm n đường tròn C1 , C2 , ..., Cn
cùng tiếp xúc với hai đường tròn không giao nhau (O, R) và (I, r), sao cho Ci tiếp xúc với Ci−1 và
Ci+1 (i = 1, n, C0 ≡ Cn , C1 ≡ Cn+1 ). Do tính chất này nên tập hợp (C1 , C2 , ..., Cn ) còn được gọi là
một chuỗi đóng. Chuỗi Steiner có một số tính chất thú vị sau.

Tính chất 120. (Định lý Steiner-Steiner’s Porism). Cho hai đường tròn đựng nhau (O) và (I). Giả
sử tồn tại một chuỗi Steiner ω1 , ω2 , ..., ωn ứng với (O) và (I). Khi đó tồn tại vô số chuỗi Steiner ứng
với hai đường tròn trên, đồng thời có thể chọn vị trí bất kì cho đường tròn ω1 .

O' P I O

Chứng minh. Định lý Steiner là một ứng dụng đặc sắc của phép nghịch đảo.
Gọi P là điểm "giới hạn" của (O) và (I). Theo tính chất 2.2.10 của chương [], phép nghịch đảo cực
P phương tích k bất kì biến (O) và (I) thành hai đường tròn đồng tâm O0 ; chuỗi ω1 , ω2 , ..., ωn thành
chuỗi W = (ω10 , ω20 , ..., ωn0 ) cùng tiếp xúc với hai đường tròn đồng tâm O0 và lần lượt tiếp xúc ngoài
nhau. Do chuỗi W đóng kín nên có thể cho ω10 chuyển động mà không làm thay đổi tính chất của W .
Từ đó theo phép nghịch đảo, tồn tại vô số chuỗi ω1 , ω2 , ..., ωn thoả mãn điều kiện trên và có thể chọn
vị trí bất kì cho ω1 .

Tính chất 121. Gọi tâm của C1 , C2 , ..., Cn lần lượt là O1 , O2 , ..., On . Khi đó O1 , O2 , ..., On cùng nằm
trên một ellipse hoặc hyperbol.

Chứng minh. Dễ thấy Oi O + Oi I = R + r hoặc |Oi O − Oi I| = |R − r| tuỳ thuộc vào trường hợp (O)
và (O0 ) đựng nhau hoặc ngoài nhau. Do đó O1 , O2 , ..., On cùng nằm trên ellipse hoặc hyperbol có hai
tiêu điểm O và I.

Tính chất 122. Gọi tiếp điểm của Ci và Ci+1 là Ai(i+1) . Khi đó A12 , A23 , ..., A(n−1)n , An1 cùng thuộc
một đường tròn.

205
O' P
I O

Chứng minh. Theo lời giải tính chất 3.1.1, phép nghịch đảo tâm là điểm "giới hạn" P của (O) và (I)
biến C1 , C2 , ..., Cn lần lượt thành C10 , C20 , ..., Cn0 cùng tiếp xúc với hai đường tròn đồng tâm O0 . Do đó
A12 , A23 , ..., A(n−1)n , An1 thành các tiếp điểm A012 , A023 , ..., A0(n−1)n , A0n1 cùng nằm trên một đường tròn
tâm O0 . Theo phép nghịch đảo suy ra A12 , A23 , ..., A(n−1)n , An1 cùng thuộc một đường tròn. Hơn nữa
tâm của đường tròn này nằm trên đường thẳng OI.

20.3.2 Chuỗi Pappus


Cho hai đường tròn (O, R), (O0 , R0 ) tiếp xúc trong với nhau tại A (R > R0 ). Chuỗi Pappus là một
tập hợp vô hạn các đường tròn ..., C−3 , C−2 , C−1 , C0 , C1 , C2 , C3 , ... cùng tiếp xúc với (O) và (O0 ) sao
cho Ci tiếp xúc với Ci−1 và Ci+1 , đồng thời tâm của C0 nằm trên OO0 . Có thể thấy rằng chuỗi Pappus
có dạng gần giống chuỗi Steiner, đồng thời hai tính chất 3.1.2 và 3.1.3 vẫn đúng cho chuỗi Pappus.

Tính chất 123. Kí hiệu Ri là bán kính của đường tròn Ci , hi là khoảng cách từ tâm của Ci tới OO0 .
Khi đó hi = 2|i|.Ri .

206
A

Chứng minh. Gọi A là tiếp điểm của (O, R) và (O0 , R0 ). Phép nghịch đảo cực A phương tích bất kì
biến (O) và (O0 ) thành hai đường thẳng song song d và d0 cùng vuông góc với OO0 , Ci thành Ci0 tiếp
xúc với d và d0 . Gọi h0i , Ri0 lần lượt là khoảng cách từ tâm của Ci0 tới OO0 và bán kính của Ci0 .
Rõ ràng h0i = 2|i|Ri0 .
hi h0
Do A là tâm vị tự của Ci và Ci0 nên = i0 . Suy ra hi = 2|i|Ri .
Ri Ri

Nhận xét 12. 1. Miền diện tích được tạo bởi 3 nửa đường tròn như hình vẽ còn được gọi là một
"arbelos". Các tính chất của arbelos bạn đọc có thể xem tại [12].

2. Dựa vào phép nghịch đảo, chúng ta cũng có thể tìm được bán kính của đường tròn Ci . Đặt
R0 (1 − r)r
r= thì ri = . Các tính chất khác của chuỗi Pappus có thể xem tại [10][11].
R 2[i (1 − r)2 + r]
2

20.4 Chuỗi đường tròn Apollonius


Chuỗi Apollonius, hay còn gọi là bài toán 9 đường tròn, được phát biểu như sau.
Cho 3 đường tròn (O1 ), (O2 ), (O3 ) trên mặt phẳng. Ta xây dựng một chuỗi đường tròn xoay vòng
như sau. Gọi C12 là đường tròn tiếp xúc với (O1 ) và (O2 ), C23 là đường tròn tiếp xúc với C12 , (O2 ),
(O3 ), C34 là đường tròn tiếp xúc với C23 , (O3 ), (O1 ), C45 là đường tròn tiếp xúc với C34 , (O1 ), (O2 ),
C56 là đường tròn tiếp xúc với C45 , (O2 ), (O3 ), C61 là đường tròn tiếp xúc với C5 , (O3 ), (O1 ). Khi đó
C12 , C23 , ..., C61 là một chuỗi đóng, hay C61 tiếp xúc với C12 .

Chứng minh. Ta chứng minh bài toán trong trường hợp 3 đường tròn (O1 ), (O2 ), (O3 ) đôi một ngoài
nhau. Các trường hợp khác chứng minh tương tự.

207
Gọi P1 , P2 lần lượt là tiếp điểm của C12 với (O1 ) và (O2 ); P3 là tiếp điểm của C23 với (O3 ); P4 là
tiếp điểm của C34 với (O1 ), tương tự với P5 , P6 , P7 . Như vậy ta cần chứng minh P7 ≡ P1 .
Trước tiên ta phát biểu và chứng minh một bổ đề.
Bổ đề 22. Cho hai đường tròn (O1 ) và (O2 ) không chứa nhau. Gọi C1 , C2 là hai đường tròn tiếp xúc
ngoài với (O1 ), (O2 ) lần lượt tại A, B và C, D. Khi đó A, B, C, D cùng thuộc một đường tròn.
Chứng minh.

c1

B
A
O2

O1
T

C D

c2

Áp dụng định lý Monge-D’Alembert cho 3 đường tròn (O1 ), (O2 ), C1 suy ra AB đi qua tâm vị tự
ngoài T của (O1 ) và (O2 ). Tương tự ta cũng có CD đi qua T .p p
Như ta đã biết phép nghịch đảo cực T phương tích k = PT /(O1 ). PT /(O2 ) biến (O1 ) thành
(O2 ). Do O1 A và O2 B không song song, O1 C và O2 D không song song nên T A.T B = T C.T D = k.
Suy ra A, B, C, D cùng thuộc một đường tròn.
Trở lại bài toán.

O3
C56
C61

C34 P6
C12
P1 P3
O1
P4
C23
C45

P5 P2

O2

Áp dụng bổ đề 1 cho hai đường tròn C12 và C34 cùng hai đường tròn tiếp xúc C23 và (O1 ) suy ra
P1 , P2 , P3 , P4 cùng thuộc một đường tròn.
Áp dụng bổ đề 1 cho hai đường tròn (O2 ) và (O3 ) với hai đường tròn tiếp xúc C23 và C56 suy ra
P2 , P3 , P5 , P6 cùng thuộc một đường tròn.

208
Tiếp tục áp dụng bổ đề 1 cho hai đường tròn (O1 ) và (O2 ) với hai đường tròn tiếp xúc C45 và C12
suy ra P1 , P2 , P4 , P5 cùng thuộc một đường tròn.
Từ các điều trên ta thu được P1 , P2 , P3 , P4 , P5 , P6 cùng thuộc một đường tròn ω.
Áp dụng bổ đề 1 cho hai đường tròn (O1 ) và (O3 ) với hai đường tròn tiếp xúc C61 và C34 suy ra
P3 , P4 , P6 , P7 cùng thuộc một đường tròn. Từ đó P7 là giao của ω với (O1 ) hay P7 ≡ P1 . Như vậy C61
và C12 tiếp xúc nhau. Ta có đpcm.

Chuỗi Apollonius có một hệ quả thú vị, được gọi là bài toán 6 đường tròn.
Cho tam giác ABC và một đường tròn C1 tiếp xúc với hai cạnh AB, AC. Dựng đường tròn C2 tiếp
xúc với C1 và tiếp xúc với CA, CB. Tiếp tục dựng C3 tiếp xúc với C2 và tiếp xúc với BC, BA, C4 tiếp
xúc với C3 và tiếp xúc với AB, AC, C5 tiếp xúc với C4 và tiếp xúc với CA, CB, C6 tiếp xúc với C5 và
tiếp xúc với BC, BA. Khi đó C6 tiếp xúc với C1 .
A

c1

c2

c4 c6
c5

c3

B C

Chứng minh. Thực chất bài toán này thu được từ phép nghịch đảo chuỗi Apollonius trong trường hợp
3 đường tròn (O1 ), (O2 ), (O3 ) đồng quy tại A, với A là cực nghịch đảo, phương tích bất kì.

21 Các tam giác có dạng AB + AC = kBC


21.1 Mở đầu
Các bài toán về tam giác có điều kiện đặc biệt AB + AC = kBC (k = 2, 3) đã xuất hiện trong một
số cuộc thi và tạp chí trên thế giới. Tuy nhiên các tính chất liên quan đến nó còn khá ít và đơn giản.
Trong bài viết này, tác giả xin khai thác và tìm hiểu tính chất của hai dạng tam giác AB + AC = 2BC,
AB + AC = 3BC, đồng thời xem xét một số mở rộng trong trường hợp tổng quát AB + AC = kBC.
Nếu không có chú thích gì thêm, chúng ta kí hiệu (O), (I) lần lượt là đường tròn ngoại tiếp, đường
tròn nội tiếp tam giác ABC.

21.2 Các tam giác có dạng AB + AC = 2BC


Tính chất 124. AI ⊥ OI.

Chứng minh. Cách 1.

209
A

F
O
I

B Ma C

Gọi J là điểm chính giữa cung BC không chứa A, Ma là trung điểm BC, F là tiếp điểm của (I)
với AB.
AB + AC − BC 2BC − BC 1
Ta có AF = = = BC = BMa .
2 2 2
Đồng thời ∠IAF = ∠JBMa nên 4AF I = 4BMa J. Do đó AI = BJ = IJ hay I là trung điểm
AJ. Suy ra OI ⊥ AJ.
Cách 2.
Sử dụng hình vẽ của cách 1. Áp dụng định lý Ptolemy cho tứ giác ABJC ta có
AB · JC + AC · JB = AJ · BC.
Do JB = JC = JI nên JI(AB + AC) = AJ · BC. Theo giả thiết AB + AC = 2BC nên AJ = 2JI
hay I là trung điểm AJ. Suy ra OI ⊥ AJ.

Tính chất 125. Gọi G là trọng tâm tam giác ABC. Khi đó IG k BC.

F
G O
I

B D K Ma C

Chứng minh. Gọi K là giao của AI với BC. D là tiếp điểm của (I) với BC.
Theo lời giải 1 của tính chất 1 ta thu được Ma J = IF = ID nên theo định lý Thales, KI = KJ.
AI AG
Do đó =2= . Suy ra IG k BC.
IK GMa
Tính chất 126. Đường tròn (I) tiếp xúc với BC, CA, AB lần lượt tại D, E, F . Gọi Mb , Mc lần lượt
là trung điểm AC, AB. Mb Mc cắt EF tại L. Khi đó đường tròn (L, LI) tiếp xúc với (I) và (AO).

210
A

E
Mc L
Mb

F O
I

B D Ma C

Chứng minh. Gọi Ma là trung điểm BC. AI cắt (O) tại K.


Do ∠AIO = 90◦ nên I ∈ (AMb Mc ), từ đó EF là đường thẳng Simson của I ứng với tam giác
AMb Mc . Suy ra IL ⊥ Mb Mc hay I, L, D thẳng hàng, suy ra A, L, Ma thẳng hàng hay L là trung điểm
Mb Mc .
Từ đó dễ dàng có (L, LI) tiếp xúc với (AO).
1
Do IA = IK, M K = r, nên IL = r. Từ đó (L, LI) tiếp xúc với (I).
2
Tính chất 127. (Hidetosi Fukagawa, tạp chí CRUX năm 1988) Xét trường hợp tam giác ABC
nhọn. Kẻ AK ⊥ BC. Ma là trung điểm BC. Dựng đường tròn (O1 ) tiếp xúc với tia KA, KB và tiếp
xúc trong với (O), đường tròn (O2 ) tiếp xúc với tia KA, KC và tiếp xúc trong với (O). Khi đó tiếp
tuyến chung trong thứ hai của (O1 ) và (O2 ) đi qua Ma .

T E O2

Mc Mb

F I O

R J
O1
Y
Z
B X K D Ma C

Chứng minh. Gọi D, E, F là tiếp điểm của đường tròn (I) với BC, CA, AB, Mb , Mc là trung điểm
AC, AB, J là tâm đường tròn Euler của tam giác ABC. Kéo dài AK cắt (O) tại L.
BC
Ta có AE = AF = nên AE + AF = BC = AMb + AMc , suy ra EMb = F Mc . Từ đó
2
4IEMb = 4IF Mc , ta thu được IMb = IMc . Mà JMb = JMc nên IJ là trung trực của Mb Mc hay
trung trực của KM.

211
Gọi X, Y là tiếp điểm của (O1 ) với BC, AH, Z, T là tiếp điểm của (O2 ) với BC, AH.
Theo định lý Sawayama-Thebault, XY cắt ZT tại I và I ∈ O1 O2 . Do ∠IXZ = ∠IZX = 45◦ nên
IX = IZ. Mà IK = IM nên XK = ZMa . Đồng thời ID là đường trung bình của hình thang O1 XZO2
nên IO1 = IO2 . Kết hợp với ∠O1 KO2 = 90◦ ta suy ra IO2 = IO2 = IK = IMa hay ∠O1 Ma O2 = 90◦ .
Từ Ma kẻ tiếp tuyến Ma R khác BC tới (O1 ). Suy ra Ma O1 là phân giác ∠XMa R, từ đó Ma O2 là
phân giác ∠RMa Z hay Ma R là tiếp tuyến của (O2 ). Vậy Ma R là tiếp tuyến chung của (O1 ) và (O2 ).

Tính chất 128. Gọi H là trực tâm. Khi đó H là tâm vị tự trong của (O1 ) và (O2 ).

Chứng minh. Trước tiên ta phát biểu một bổ đề.


Bổ đề 23. Cho hai đường tròn (O1 ), (O2 ) ngoài nhau và cùng tiếp xúc trong với (O). Gọi AE, BF
là hai tiếp tuyến chung trong của (O1 ) và (O2 ) sao cho A, B khác phía với E, F bờ O1 O2 . CD là tiếp
tuyến chung ngoài của (O1 ) và (O2 ) sao cho CD cùng phía với EF bờ O1 O2 . Khi đó CD k EF.

O2

I1
T
I2 O

O1

C D

E L F

Chứng minh.
Gọi L là điểm chính giữa cung CD không chứa A, B. AL, BL cắt O1 O2 lần lượt tại I1 , I2 . AE cắt
BF tại T.
Áp dụng định lý Thebault suy ra I1 , I2 lần lượt là tâm nội tiếp các tam giác ACD, BCD. Suy ra
LI1 = LD = LI2 . Từ đó ∠AI1 T = ∠BI2 T.
Mà ∠BT I2 = ∠AT I1 nên ∠I2 BT = ∠I1 AT hay ∠LBF = ∠LAE. Suy ra L là điểm chính giữa
cung EF . Vậy CD k EF.

Trở lại bài toán.

212
A

O2

Mc Mb
O
I

J
H
O1

B K D Ma C

L G

Gọi G là đối xứng của A qua O suy ra Ma là trung điểm HG. Từ đó LG k BC. Áp dụng bổ đề 1
suy ra HG là tiếp tuyến chung trong của (O1 ) và (O2 ). Vậy H là tâm vị tự ngoài của (O1 ) và (O2 ).
Tiếp theo chúng ta đến với một tính chất được tìm ra bởi thành viên mr.danh trên diễn đàn
AoPS.
Tính chất 129. Gọi T, U, V là giao của tiếp tuyến chung ngoài khác BC của hai đường tròn (O1 ) và
(O2 ) với AH, AB, AC. Khi đó T U = T V .

T
S O2
U

H O
O1

B Ma C

L G

Chứng minh. Gọi S là giao điểm thứ hai của HG với (O). Áp dụng bổ đề 1 suy ra AS k U V, GL k BC.
Suy ra G(BCMa L) = −1, chiếu lên (O) ta thu được (BCSL) = −1, từ đó A(BCSL) = −1 hay
A(U V ST ) = −1. Vậy T là trung điểm U V .
Nhận xét. Bằng cách sử dụng tỉ số kép, ta có thể mở rộng tính chất 6 như sau.
Cho tam giác ABC nội tiếp đường tròn (O), M là điểm bất kì trên BC. Gọi (O1 ), (O2 ) là đường
tròn Thebault nội tiếp của tam giác ABC ứng với cát tuyến AM . Gọi d là tiếp tuyến chung trong
khác AM của (O1 ) và (O2 ). d cắt (O) tại D sao cho A và D cùng thuộc một nửa mặt phẳng bờ
BC. DB, DC, d cắt tiếp tuyến chung ngoài thứ hai khác BC của (O1 ) và (O2 ) tại U, V, T . Khi đó
TU MC
= .
TV MB

213
Tính chất 130. Đường tròn đường kính AO và đường tròn đường kính HMa tiếp xúc nhau.

Chứng minh. Cách 1. (Xem hình vẽ tính chất 5)


Theo tính chất 5 ta suy ra HI là phân giác ∠AHMa hay HI là phân giác ngoài ∠KHMa . Mà I
nằm trên trung trực KM nên HIMa K nội tiếp.
Do I nằm trên trung trực KMa nên I nằm trên đường trung bình của hình thang AKMa O hay
đường thẳng đi qua trung điểm AO và HMa . Vậy tâm của (AO), (HMa ) và I thẳng hàng. Suy ra
(AO) tiếp xúc với (HMa ) tại I.
Cách 2 (Dựa theo Telv Cohl).

O
I G
H N

B Ma C

Gọi G là trọng tâm tam giác ABC, N là giao của IG với OMa . Ta có N Ma = r = Ma J. Áp
AI Ma J GN
dụng định lý Menelaus cho tam giác IN J với cát tuyến (A, G, Ma ) ta có · · = 1. Do đó
AJ Ma N GI
GN GH
=2 . Ta thu được IO k HN.
GI GO
Mà AI ⊥ OI nên AI ⊥ HN . Đồng thời N I ⊥ AH nên I là trực tâm tam giác AHN . Suy ra
HI ⊥ AN.
Lại có IMa là đường trung bình của tam giác AJN nên IMa k AN . Suy ra HI ⊥ IMa hay
I ∈ (HMa ). Do I nằm trên đường trung bình của hình thang AOMa H nên I nằm trên đường nối hai
tâm của (AO) và (HMa ). Ta có đpcm.

Tính chất 131. Tứ giác AOMa H là hình thang ngoại tiếp.

Chứng minh. Tiếp tục sử dụng hình vẽ tính chất 5.


Ta có ∠IMa O = ∠IKH = ∠IMa H nên Ma I là phân giác ∠OMa H. Như vậy I là giao của phân
giác các góc HAO, AHMa , HMa O nên I là tâm nội tiếp hình thang AHMa O.

Nhận xét. Từ tính chất 8, áp dụng định lý Pythot, bạn đọc có thể chứng minh tam giác AGH
có AG + HG = 3AH.

Tính chất 132. Gọi J là giao của AI với (O). Khi đó điểm Nagel là đối xứng của J qua BC.

Chứng minh. Trước tiên ta phát biểu một bổ đề.


Bổ đề 24. Tâm đường tròn nội tiếp, trọng tâm và điểm Nagel thẳng hàng.
Chứng minh.

214
A

Mc Mb

I G
N

C
B D Ma L

Gọi D là tiếp điểm của đường tròn nội tiếp (I) với BC, DI cắt (I) lần thứ hai tại T , AT cắt BC
tại L; gọi Ma , Mb , Mc lần lượt là trung điểm BC, CA, AB; N là điểm Nagel của tam giác ABC.
Ta biết rằng Ma là trung điểm DL nên Ma I k AL hay Ma I k AN . Chứng minh tương tự,
Mb I k BN, Mc I k CN . Do đó I là điểm Nagel của tam giác Ma Mb Mc .
IG 1
Do G là tâm vị tự của hai tam giác ABC và Ma Mb Mc nên I, G, N thẳng hàng và = .
GN 2
Trở lại bài toán.

O
G
I
N

B D Ma P C

Gọi N là điểm Nagel, AN cắt (I) tại L, cắt BC tại P .


Theo tính chất 2, IG k BC, từ đó áp dụng bổ đề 2 suy ra IN k BC.
Do I là trung điểm LD do đó N là trung điểm LP .
Lại có Ma là trung điểm DP nên N Ma k ID hay N Ma ⊥ BC và N Ma = r.
Theo phép chứng minh tính chất 1, JMa = r nên J và N đối xứng với nhau qua BC.

Tính chất 133. Gọi Ia là tâm bàng tiếp góc A. Đường tròn (A, AIa ) cắt BC tại E, F sao cho E
thuộc tia CB, F thuộc tia BC. Đường tròn (EBIa ) cắt AB tại M , (F CIa ) cắt AC tại N . Khi đó tứ
giác BCN M là tứ giác lưỡng tâm.

215
A

O
I
E B K C F

Ia

Chứng minh. Gọi E 0 là giao của đường tròn qua A, B và tiếp xúc với AIa với BC, K là giao của AIa
với BC, J là giao của AI với (O).
Ta có AI = IJ = JIa , AI = 2IK nên AK = KIa . Từ đó KIa2 = KA2 = KB · KE 0 , nghĩa là
0
(E BIa ) tiếp xúc với AIa .
Như vậy ∠AE 0 Ia = ∠AE 0 K + ∠KE 0 Ia = ∠BAI + ∠BIa I = ∠BAI + ∠BCI.
Mà ∠E 0 AIa = ∠ABC nên ∠E 0 AIa = 180◦ −2∠AE 0 Ia hay tam giác AE 0 Ia cân tại A. Từ đó E 0 ≡ E.
Suy ra ∠BIa M = 180◦ −∠M BIa −∠BM Ia = 180◦ −(90◦ −∠IBC)−∠BIa I = 90◦ +∠IBC −∠ICB.
Chứng minh tương tự, ∠CIa N = 90◦ + ∠ICB − ∠IBC. Suy ra ∠BIa M + ∠CIa N = 180◦ hay tứ
giác BCN M ngoại tiếp.
Mặt khác, AIa là tiếp tuyến chung của (BIa M ) và (CIa N ) nên AB · AM = AIa2 = AC · AN hay
tứ giác BCN M nội tiếp. Ta có đpcm.

Để kết thúc phần này chúng ta đến với một bài toán nằm trong đề chọn đội tuyển IMO của Thổ
Nhĩ Kỳ năm 2015.
Bài toán. Cho tam giác ABC nội tiếp đường tròn (O), ngoại tiếp (I). (I) tiếp xúc với BC, CA, AB
lần lượt tại D, E, F . Gọi E1 , F1 lần lượt đối xứng với A qua E, F . Đường tròn ω1 đi qua E1 tiếp xúc với
(I) tại D cắt AC lần thứ hai tại E2 ; đường tròn ω2 đi qua F1 tiếp xúc với (I) tại D cắt AB tại F2 . Gọi
P, Q lần lượt là trung điểm OE, IF . Chứng minh rằng E2 F2 ⊥ P Q khi và chỉ khi AB + AC = 2BC.

216
E3

E2

E
Q P
F O E1
I
B
D C
F1

F2

Chứng minh. AB + AC = 2BC khi và chỉ khi AI ⊥ OI hay OI k EF hay OI k P Q.


Như vậy ta sẽ chứng minh E2 F2 ⊥ OI.
Gọi E3 là giao của DF với AC. E20 là trung điểm E3 E. Do (E3 EAC) = −1 nên sau một số phép
tính toán suy ra CE 2 = CE1 .CE20 hay (E20 E1 D) tiếp xúc với (I) hay E20 ≡ E2 . Theo hệ thức Newton,
E2 E 2 = E2 A.E2 C hay E2 thuộc trục đẳng phương của (O) và (I). Chứng minh tương tự suy ra
E2 F2 ⊥ OI.
Nhận xét. Theo tác giả bài toán này là sự kết hợp cơ học giữa một tính chất của tam giác bất kì
với tính chất 1. Do đó đề thi tuy nhìn đẹp mắt nhưng không phải một đề thi hay.

21.3 Các tam giác có dạng AB + AC = 3BC


Tính chất 134. (IMO Shortlist 2005). Đường tròn (I) tiếp xúc với AC, AB lần lượt tại E, F . Gọi
L, K là điểm đối xứng với E, F qua I. Khi đó B, C, K, L cùng thuộc một đường tròn.
A

O E
F
I
K
L

B C

217
Chứng minh. Gọi J là điểm chính giữa cung BC không chứa A. Áp dụng định lý Ptolemy cho tứ giác
ABJC ta có
AB · JC + AC · JB = AJ · BC hay JI(AB + AC) = AJ · BC.
Do AB + AC = 3BC nên AJ = 3JI. Gọi M là trung điểm AI thì M I = JI. Mà F I = KI nên
JI = M I = F M = JK.
Suy ra K ∈ (BIC). Tương tự suy ra B, C, K, L cùng thuộc một đường tròn.

Tính chất 135. Gọi G là trọng tâm tam giác ABC. Khi đó IG ⊥ BC.
AI AG AI
Chứng minh. Theo lời giải tính chất 11, = 2. Gọi Ma là trung điểm BC thì =2= . Suy
IJ GMa IJ
ra IG k JMa hay IG ⊥ BC.

Tính chất 136. (Iran 2004). Đường tròn (I) nội tiếp tam giác ABC tiếp xúc với AB, AC tại P, Q.
BI, CI giao P Q lần lượt tại L, K. Khi đó (ILK) tiếp xúc với (I).

Chứng minh. Cách 1.

K
Y P
L
Q

X I

B D C

Gọi D là tiếp điểm của (I) với BC. AI giao (ABC) tại J.
r
Ta có (ILK) tiếp xúc với (I) khi và chỉ khi R(ILK) = .
2
Dễ thấy BL ⊥ LC, CK ⊥ BK nên L, K nằm trên (BC).
Bằng cộng góc suy ra I là tâm nội tiếp tam giác DLK. Kẻ IX ⊥ LD, IY ⊥ P Q.
R(ILK) IY IX
Do hai tam giác ILK và IBC đồng dạng nên = = = sin ∠IDX = sin A/2.
R(IBC) ID ID
R(ILK) IP r
Hay = = .
IJ AI AI
r
Do đó R(ILK) = khi và chỉ khi IP = 2IJ hay JA = 3JB = 3JC.
2
Theo lời giải tính chất 11, điều này tương đương AB + AC = 3BC.
Cách 2.

218
A

F T E M
N
K P
L

Q I R

B D C

Gọi R, T là giao của CK với (I). Ta có T P RD là tứ giác điều hoà. Gọi M là giao của IK với DP
thì do T R ⊥ IM nên M T, M R là hai tiếp tuyến của (I). Suy ra IK.IM = r2 .
Tương tự gọi N là giao của IL với DQ thì IL.IN = r2 .
Dễ thấy M, N lần lượt là hình chiếu của A trên IB, IC và M N là đường trung bình ứng với đỉnh
A của tam giác ABC.
2
Xét phép nghịch đảo IIr : (I) 7→ (I), (ILK) 7→ M N do đó (ILK) tiếp xúc với (I) khi và chỉ khi
(I) tiếp xúc với đường trung bình EF của tam giác ABC (E ∈ AC, F ∈ AB) hay (I) là đường tròn
bàng tiếp góc A của tam giác AEF .
1 AB + AC − BC 1
Khi đó AP = AQ = pAEF = pABC , khi và chỉ khi = (BC + CA + AB) hay
2 2 4
AB + AC = 3BC.

Tính chất 137. Đường tròn (I) tiếp xúc với BC, CA, AB lần lượt tại D, E, F . Gọi L, K lần lượt đối
xứng với E, F qua I. T là giao điểm thứ hai của đường tròn đường kính AI với (O). Khi đó (LKT )
tiếp xúc với (O).

T
X

O
E
I K
F
L

P
B D C

Chứng minh. Theo tính chất 11, L, K ∈ (BIC).

219
Kéo dài AT cắt BC tại P suy ra P I ⊥ AI. Suy ra ∠T DP = ∠T IP = ∠T AD = ∠(T J, BC), suy
ra T, D, J thẳng hàng.
2
Xét phép nghịch đảo IJJI : L 7→ L, K 7→ K, T 7→ D. Do đó (T LK) 7→ (I). Mà (O) 7→ BC, BC tiếp
xúc với (I) nên (T LK) tiếp xúc với (O).

Nhận xét. Có một kết quả khá đẹp nằm trong kì thi quốc gia Nga năm 2010 như sau, mời bạn
đọc thử chứng minh.
Cho tam giác ABC nội tiếp đường tròn (O), ngoại tiếp đường tròn (I). Đường tròn ngoại tiếp tam
giác BIC cắt (I) tại hai điểm X, Y . Gọi Z là tâm vị tự ngoài của (I) và (BIC). Chứng minh rằng
(XY Z) tiếp xúc với (O).

Tính chất 138. Đường tròn (I) tiếp xúc với BC tại D. Gọi T là điểm đối xứng với D qua I. BT, CT
cắt AC, AB lần lượt tại M, N . X là trung điểm AI. Khi đó X là tâm đường tròn ngoại tiếp tam giác
T M N.
A

X M
T

O
E
I
F
L K

D
B C

Chứng minh. Ta có ∠BLI = 180◦ − ∠ICB, ∠T LE = ∠T DE = ∠ICB nên B, L, T thẳng hàng. Tương
tự C, K, T thẳng hàng. Suy ra M, N là tiếp điểm của đường tròn bàng tiếp góc B, C với AC, AB.
1
Ta có ∠M T N = ∠BT C = ∠EDF = 90◦ − ∠BAC.
2
Mặt khác, AM = CE = p − c, N F = AB − 2BF = AB − (AB + BC − AC) = AC − BC =
AC + BC − AB
= p − c. Suy ra AM = N F .
2
Từ đó 4AXM = 4F XN (c.g.c). Suy ra ∠F N X = ∠AM X hay AM XN nội tiếp. Mà AX là phân
giác ∠N AM nên XM = XN, đồng thời ∠N XM = 180◦ − ∠BAC = 2∠M T N nên X là tâm ngoại
tiếp tam giác M T N.

Từ lời giải tính chất 15, ta suy ra kết quả sau.

Tính chất 139. Điểm đối xứng với tiếp điểm của (I) với BC qua I là điểm Nagel.

Tính chất 140. Gọi Ia là tâm bàng tiếp góc A. Trên BC lấy hai điểm P, Q sao cho CP = CA, BQ =
BA và theo thứ tự P, B, C, Q. Đường tròn (P BIa ) cắt AB lần thứ hai tại M , đường tròn (QCIa ) cắt
AC lần thứ hai tại N . Khi đó BCN M là tứ giác lưỡng tâm.

220
A

Q
P B D C

Ia

N
M

Chứng minh. Gọi D là điểm chính giữa cung BC. Dễ thấy IA = IQ = IP. Từ giả thiết AB + AC =
3BC suy ra AI = 2ID hay IA = IIa . Vậy AP Ia Q nội tiếp đường tròn tâm I. Bằng một số phép cộng
góc đơn giản suy ra AICQ, AIBP nội tiếp.
Ta có ∠P M B = ∠P Ia B = ∠P Ia A − ∠BIa A = ∠AQB − ∠ICB = ∠Ia IC − ∠ICA = ∠IAC. Suy
1
ra ∠BIa M = 180◦ − ∠BP M = ∠P BM + ∠P M B = ∠B + ∠A.
2
1
Tương tự ∠CIa N = ∠C + ∠A. Ta thu được ∠BIa M + ∠CIa N = 180◦ .
2
Suy ra M N tiếp xúc với (Ia ) hay tứ giác BCN M ngoại tiếp.
Mặt khác, ∠CN M = 2∠CN Ia = 2∠P QIa = 2∠P AI = 2∠IBC = ∠ABC hay tứ giác BCN M nội
tiếp. Ta có đpcm.

Nhận xét. Từ lời giải tính chất 17 ta nhận thấy I là trung điểm AIa . Do đó bán kính đường tròn
nội tiếp bằng một nửa bán kính đường tròn bàng tiếp góc A.
Tiếp theo chúng ta đến với một tính chất được tìm ra độc lập bởi tác giả. Khá thú vị là bài toán
từng nằm trong cuộc thi hình học Sharygin của Nga năm 2014, bài 9.6.

Tính chất 141. Gọi Y, Z lần lượt là điểm chính giữa các cung ABC, ACB. Khi đó Y Z là tiếp tuyến
của đường tròn (BIC).

221
Ib

X
A

Ic

O E
Z
I
F
K
Y L
D
B C

Ia

Chứng minh. Gọi Ia , Ib , Ic lần lượt là tâm bàng tiếp các góc A, B, C của tam giác ABC; D, E, F lần
lượt là tiếp điểm của (I) với BC, AC, AB; T, L, K lần lượt đối xứng với D, E, F qua I; X là điểm
chính giữa cung BAC. Khi đó XY Z là tam giác trung tuyến của tam giác Ia Ib Ic .
1
Do ∠T BI = ∠LBI = 90◦ − ∠LIK = 90◦ − ∠BT C nên BI ⊥ T C. Từ đó I là trực tâm tam giác
2
BT C.
Ta có BI ⊥ Ia Ic nên BI ⊥ XZ. Mà XZ ⊥ ZJ (do JX là đường kính của (O)) nên IB k JZ. Từ
đó JZ ⊥ KC. Vậy ZK = ZC.
1
Mặt khác, dễ thấy EK k AI k F L. Do đó ∠KEC = ∠JAC = ∠JZC = ∠KZC. Như vậy Z là
2
tâm ngoại tiếp của tam giác KEC. Tương tự Y là tâm ngoại tiếp của tam giác F LB.
Suy ra IZ là trung trực của EK, IY là trung trực của F L. Mà EK k F L nên Y, I, Z thẳng hàng
đồng thời Y Z ⊥ JI. Vậy Y Z là tiếp tuyến của (BIC).

Tính chất 142. Gọi X, Y, Z lần lượt là điểm chính giữa cung BAC, ABC, ACB; A0 , B 0 , C 0 lần lượt
là điểm đối xứng với A qua X, B qua Y, C qua Z. Khi đó tiếp tuyến tại B 0 của (Y, Y B) cắt tiếp tuyến
tại C 0 của (Z, ZC) tại A0 .

222
Ib

X A'
A

Ic

C'
R
S

O
B'
I Z

Y
B C

Ia

Chứng minh. Gọi R là giao của đường thẳng qua Y vuông góc với BB 0 và đường thẳng qua Z vuông
góc với CC 0 .
Ta có Y R k JZ, ZR k Y J nên Y RZJ là hình bình hành. Suy ra R đối xứng với J qua trung điểm
Y Z. Gọi S là trung điểm AI suy ra S đối xứng với J qua Y Z. Suy ra SRZY là hình thang cân. Ta
thu được RS ⊥ AI. Từ đó RA = RI.
Gọi A00 là điểm đối xứng với I qua R. Suy ra RA = RI = RA00 hay AA00 ⊥ AI hay AA00 ≡ Ib Ic .
Lại có (O) là đường tròn pedal của I ứng với tam giác Ia Ib Ic , do đó R là điểm liên hợp đẳng
giác của I trong tam giác Ia Ib Ic . Suy ra RX ⊥ Ib Ic hay RX ⊥ AA00 . Như vậy XA = XA00 , nghĩa là
A00 ≡ A0 .
Theo định lý Thales, A0 B 0 k RY k IB, A0 C 0 k RZ k IC. Ta có đpcm.

Nhận xét.
1. Theo tính chất đường tròn pedal, O là trung điểm IR. Như vậy chúng ta thu được kết quả:
đường thẳng OI của tam giác ABC đi qua điểm đối xứng với A qua X.
2. Từ hình vẽ ta phát hiện ra một tính chất thú vị đúng với tam giác bất kì: điểm Nagel là tâm
đẳng phương của các đường tròn (X, XA), (Y, Y B), (Z, ZC). Tác giả Trần Quang Hùng cũng tìm
ra tính chất này tại [8].
DB IY
3. Gọi D là tiếp điểm của (I) với BC, ta có thể chứng minh = , từ đó áp dụng định lý
DC YZ
ERIQ suy ra điểm Nagel của tam giác ABC nằm trên B 0 C 0 .

Tính chất 143. Kẻ AK ⊥ BC. Gọi (O1 ) là đường tròn tiếp xúc với tia KA, KB và tiếp xúc trong với
(O), (O2 ) là đường tròn tiếp xúc với tia KA, KC và tiếp xúc trong với (O). L là tiếp điểm của đường
tròn bàng tiếp góc A với BC. Khi đó tiếp tuyến chung trong khác AK của (O1 ) và (O2 ) đi qua L.

223
A

M G T N

T E
F O2
I

O1 Y
X Z
B K D L C

Chứng minh. Gọi D, E, F lần lượt là tiếp điểm của (I) với BC, CA, AB. DI cắt (I) lần thứ hai tại T .
Qua T kẻ đường song song với BC cắt AB, AC tại M, N.
Ta biết rằng M, N lần lượt là trung điểm của AB, AC.
Mà A, T, L thẳng hàng nên T là trung điểm AL. Kéo dài LI cắt AK tại G suy ra I là trung điểm
GL. Từ đó IK = IG = IL.
Mặt khác, gọi X, Y lần lượt là tiếp điểm của (O1 ) với BC, AK, Z, T lần lượt là tiếp điểm của (O2 )
với BC, AK. Theo định lý Sawayama-Thebault, XY cắt ZT tại I. Mà ∠IXZ = ∠IZX = 45◦ nên tam
giác XIZ vuông cân tại I, từ đó IX = IZ.
Chứng minh hoàn toàn tương tự tính chất 4 suy ra L nằm trên tiếp tuyến chung trong thứ hai của
(O1 ) và (O2 ).

21.4 Trường hợp tổng quát AB + AC = kBC


Quay lại một số tính chất trong phần 1 và 2. Với tính chất 5, tác giả đã thử kiểm tra trong trường
hợp k = 3 thì tâm vị tự trong L của (O1 ) và (O2 ) có tính chất gì đặc biệt. Một điều khá thú vị là khi
đó trực tâm H là trung điểm của KL. Từ đó chúng ta có mở rộng trong trường hợp tổng quát k là số
thực bất kì lớn hơn 1 như sau.

Tính chất 144. Cho tam giác ABC có AB + AC = kBC, trực tâm H. Kẻ AK ⊥ BC. Gọi (O1 ) và
(O2 ) là hai đường tròn Thebault nội tiếp của tam giác ABC ứng với cát tuyến AK; L là tâm vị tự
KH 1
trong của (O1 ) và (O2 ). Khi đó = .
KL k − 1

224
A

O2

N
H I
O
O1
L

B X K M Z C

Q G
J

Chứng minh. Gọi Q là giao của AK với (O). AI cắt (O) lần thứ hai tại J. Tiếp tuyến chung trong
khác AQ của (O1 ) và (O2 ) cắt BC tại M , cắt (O) tại G. Qua M kẻ đường vuông góc với BC cắt AO
tại N . (O1 ), (O2 ) tiếp xúc với BC lần lượt tại X, Z.
Ở phần 1 ta đã chứng minh I là trung điểm O1 O2 . Từ đó IX = IZ, suy ra IK = IM.
Gọi R là giao của trung trực KM với AN , suy ra RA = RN . Dễ thấy 4ARI ∼ 4AOJ nên
RA = RI.
Suy ra ∠AIN = 90◦ .
Mặt khác, ta cũng chứng minh được QG k BC, do đó AG và AQ đẳng giác trong ∠BAC hay AG
là đường kính của (O). Từ đó ∠AJG = 90◦ . Như vậy IN k JG.
KH QK GM GN JI
Vậy = = = = .
KL KL ML NA IA
Áp dụng định lý Ptolemy cho tứ giác ABJC ta có JB(AB + AC) = AJ · BC. Từ đó AJ = kJB
JI 1
hay = . Ta có đpcm.
IA k−1
Nhận xét. Từ lời giải trên dễ thấy I nằm trên (LM ). Từ đó chúng ta có tổng quát cho tính chất
7 như sau.

Tính chất 145. Cho tam giác ABC có AB + AC = kBC nội tiếp (O), trực tâm H. Kẻ AK ⊥ BC,
KH 1
L là điểm nằm trên AK thỏa mãn = . Gọi G là điểm đối xứng với A qua O, LG cắt BC
KL k − 1
tại M . Qua M kẻ đường vuông góc với BC cắt AO tại N . Khi đó hai đường tròn (LM ) và (AN ) tiếp
xúc nhau.

Tiếp theo chúng ta nhận thấy tính chất 9 và tính chất 16 có điểm khá tương đồng nhau, từ đó ta
có bài toán tổng quát.

Tính chất 146. Cho tam giác ABC có AB + AC = kBC. Đường tròn nội tiếp (I) tiếp xúc với BC
NA 2
tại D. Gọi T là điểm đối xứng với D qua I, N là điểm Nagel. Khi đó = .
TA k − 1

225
A

N
I

B D Ma C

Chứng minh. Gọi J là giao của AI với (O), Ma là trung điểm BC, AMa cắt IT tại L.
NA 2Ma I 2Ma L 2JI
Theo bổ đề 2 trong phần 1, AN k IMa và AN = 2IMa . Suy ra = = = =
TA TA LA IA
2
.
k−1
Tính chất 10 và tính chất 17 cũng có những sự tương đồng. Từ đó dẫn đến mở rộng sau với lời giải
tương tự hai tính chất trên.

Tính chất 147. Cho tam giác ABC có Ia là tâm đường tròn bàng tiếp góc A. Đường tròn qua B và
Ia tiếp xúc với AIa cắt AB tại M . Đường tròn qua C và Ia tiếp xúc với AIa cắt AC tại N . Khi đó
BCN M là tứ giác lưỡng tâm.

Cuối cùng dựa vào tính chất J là trung điểm IIa , ta có tính chất sau trong trường hợp tổng quát.

Tính chất 148. Cho tam giác ABC có AB + AC = kBC. Gọi r và ra là bán kính đường tròn nội
r k−1
tiếp và đường tròn bàng tiếp góc A. Khi đó = .
ra k+1

22 Định lý Đào về đường thẳng Simson mở rộng


Năm 2014, tác giả Đào Thanh Oai đề xuất bài toán sau (không kèm lời giải).
Bài toán 1 (Đào Thanh Oai). Cho tam giác ABC nội tiếp đường tròn (O). P là điểm bất kì
nằm trên (O) và l là một đường thẳng bất kì qua O. Gọi A1 , B1 , C1 lần lượt là giao của AP, BP, CP
với l; A2 , B2 , C2 lần lượt là hình chiếu của A1 , B1 , C1 trên BC, CA, AB. Chứng minh rằng A2 , B2 , C2
thẳng hàng và đường thẳng qua A2 , B2 , C2 chia đôi đoạn nối trực tâm tam giác ABC với P.
Chú ý rằng khi l đi qua P ta thu được đường thẳng Simson của P ứng với tam giác ABC.

Chứng minh. Chúng ta phát biểu lại bài toán dưới dạng sau.
Cho tứ giác ABCD nội tiếp đường tròn (O). Một đường thẳng l bất kì qua O cắt AB, BC, CD, DA,
AC, BD lần lượt tại X, Y, Z, T, U, V . Gọi X1 , Y1 , Z1 , T1 , U1 , V1 lần lượt là hình chiếu của X, Y, Z, T, U, V
trên CD, AD, AB, BC, BD, AC. Khi đó X1 , Y1 , Z1 , T1 , U1 , V1 cùng nằm trên một đường thẳng d.
Ngoài ra, nếu ta gọi Ha , Hb , Hc , Hd lần lượt là trực tâm các tam giác BCD, CDA, DAB, ABC
thì AHa , BHb , CHc , DHd đồng quy tại trung điểm K của mỗi đường và d đi qua K.

Trước tiên xin phát biểu hai bổ đề.

226
Bổ đề 25. Quỹ tích các điểm có tỉ số phương tích tới hai đường tròn không đồng tâm cho trước không
đổi là một đường tròn đồng trục với hai đường tròn đã cho.
Bổ đề 1 là một bài toán quen thuộc, lời giải xem tại [3].
Bổ đề 26. Gọi M, N, P, Q lần lượt là trung điểm của AB, BC, CD, DA, dM , dN , dP , dQ lần lượt là
các đường thẳng qua M, N, P, Q và vuông góc với CD, DA, AB, BC. Khi đó AHa , BHb , CHc , DHd ,
dM , dN , dP , dQ đồng quy tại K.

M
A
K

N
Q Ha

O
Hb

D
P
C

Chứng minh. Dễ thấy AHb = 2OP = BHa . Mà AHb k BHa nên AHb Ha B là hình bình hành. Điều
này nghĩa là AHa và BHb có chung trung điểm K. Tương tự AHa , BHb , CHc và DHd đồng quy tại
K. Ngoài ra, M K là đường trung bình của tam giác ABHa nên M K k BHa hay M K ⊥ CD. Tương
tự ta có thể chứng minh N K ⊥ AD, P K ⊥ AB and QK ⊥ BC. Bổ đề 2 được chứng minh.

B X
Z1 A

Y1
Y

T1
O
T

Z D
C
X1

Thật vậy, gọi Z10 , X10 lần lượt là giao của Y1 T1 với AB, CD.
Ta sẽ chứng minh tỉ số phương tích của 4 điểm Z10 , X, X10 , Z đến hai đường tròn (O) và (Y T ) bằng
nhau.

227
PZ10 /(O) PX /(O) Z 0 A · Z10 B XA · XB
Ta có = khi và chỉ khi 01 = .
PZ10 /(Y T ) PX /(Y T ) 0
Z1 Y1 · Z1 T1 XY · XT
Áp dụng định lý hàm số sin trong tam giác suy ra điều này tương đương:
sin ∠Z10 Y1 A sin ∠Z10 T1 B sin ∠XT A sin ∠XY B
0 · 0 = · (1)
sin ∠Z1 AY1 sin ∠Z1 BT1 sin ∠XAT sin ∠XBY
Mà ∠Z10 Y1 A = ∠T Y1 T1 = ∠T Y T1 = ∠BY X, ∠Z10 AY1 + ∠XAT = 180◦ , ∠Z10 T1 B = ∠AT X,
∠Z1 BT1 + ∠Y BX = 180◦ nên (1) hiển nhiên đúng.
0

PZ10 /(O) PX /(O) PX10 /(O) PZ /(O)


Chứng minh tương tự suy ra = = = .
PZ1 /(Y T )
0 PX /(Y T ) PX1 /(Y T )
0 PZ /(Y T )
Suy ra X, Z, X10 , Z10 cùng nằm trên đường tròn ω đồng trục với (O) và (Y T ). Tâm của đường tròn
này nằm trên l nên XZ là đường kính của ω. Suy ra Z10 ≡ Z1 , X10 ≡ X1 .
Vậy X1 , Y1 , Z1 , T1 thẳng hàng. Chứng minh tương tự suy ra 6 điểm X1 , Y1 , Z1 , T1 , U1 , V1 thẳng
hàng.

T1
Y1 K

N
Q

Y
O
T

D
C

Mặt khác, từ bổ đề 2 suy ra QK song song với ON và N K song song với OQ. Suy ra OQKN là
hình bình hành.
KN OQ OT T1 N
Từ đó = = = . Theo định lý Thales, T1 , K, Y1 thẳng hàng. Do đó đường thẳng
Y1 Y Y1 Y TY T1 Y
qua 6 điểm X1 , Y1 , Z1 , T1 , U1 , V1 phải đi qua K. Bài toán được chứng minh.

Nhận xét. Đường thẳng Simson có một tính chất nổi tiếng phát biểu rằng góc giữa hai đường
thẳng Simson của hai điểm P và P 0 bằng một nửa số đo cung P P 0 . Ta thử áp dụng tính chất này vào
bài toán 1 và thu được tính chất tương tự.

Tính chất 149. Gọi P 0 là điểm bất kì khác P trên (O) và định nghĩa A02 , B20 , C20 tương tự A2 , B2 , C2 .
Khi đó góc giữa đường thẳng đi qua A2 , B2 , C2 và đường thẳng đi qua A02 , B20 , C20 bằng một nửa số đo
cung P P 0 .

228
A P

B'2

Y1
L B2
P'
l
Y

Y'1 O
B'1
B1

B C

Chứng minh. Gọi Y là giao điểm của l và AC, Y1 , Y10 lần lượt là hình chiếu của Y trên P B, P 0 B, d và
d0 lần lượt là đường thẳng đi qua A2 , B2 , C2 và A02 , B20 , C20 . d giao d0 tại L.
Từ dạng thứ hai của bài toán 1, Y1 nằm trên d và Y10 nằm trên d0 .
Ta có (d, d0 ) = ∠B20 LB2 = 180◦ − ∠LB2 B20 − ∠LB20 B2 = ∠Y10 B10 B1 − ∠Y1 B2 Y = ∠Y10 B10 B1 −
∠Y1 B1 Y = ∠B10 BB1 = ∠P 0 BP , bằng một nửa số đo cung P P 0 .

Chúng ta lại nhớ đến tính chất của đường thẳng Simson: Đường thẳng qua P vuông góc với BC
cắt đường tròn ngoại tiếp tam giác ABC tại X thì AX song song với đường thẳng Simson của P . Ta
thử tìm tính chất tương tự trong bài toán tổng quát.

Tính chất 150. Gọi P 0 là điểm đối xứng với P qua l. Qua P kẻ đường vuông góc với ba cạnh của
tam giác ABC cắt (O) lần lượt tại X, Y, Z. Khi đó AX, BY, CZ cùng song song với d. Hay nói cách
khác, đường thẳng Simson của P 0 song song với d.

C2
P

B2 T C1 A1
O
B1 E

P'

B C A2

Chứng minh. Gọi E là giao của l với AC. Kẻ ET ⊥ BP thì T ∈ l.


Ta có ∠Y BP = 180◦ − ∠P P 0 Y.
Do P P 0 ⊥ l, P 0 Y ⊥ AC nên ∠P P 0 Y = 180◦ − ∠AEB1 = 180◦ − ∠B2 T B1 .
Suy ra ∠Y BP = ∠B2 T B1 . Vậy BY k d. Chứng minh tương tự suy ra đpcm.

229
Tiếp theo, chú ý rằng trung điểm đoạn nối trực tâm với P nằm trên đường tròn Euler của
tam giác ABC. Theo dạng thứ hai của bài toán 1, K là trung điểm chung của các đoạn thẳng
AHa , BHb , CHc , DHd , do đó K nằm trên đường tròn Euler của các tam giác ABC, BCD, CDA, DAB
hay K là điểm Euler-Poncelet của bộ 4 điểm (A, B, C, D). Ta thu được tính chất sau.
Tính chất 151. Đường thẳng d đi qua điểm Euler-Poncelet của bộ 4 điểm (A, B, C, P ).

C2

P
A1
C1
K B2
O
B1

B C A2

Tính chất 152. Gọi T là giao điểm thứ hai của d với đường tròn Euler của tam giác ABC. l cắt
BC, CA, AB lần lượt tại X, Y, Z. Khi đó các đường tròn đường kính AX, BY, CZ đồng quy tại T.

C2
T
P

M K C1
J
O
Z
L

B C A2

Chứng minh. Gọi M là trung điểm AB, J, L lần lượt là hình chiếu của C, Z trên AB, P C.
Ta có J, M, T, K cùng nằm trên đường tròn Euler của tam giác ABC, L nằm trên d, J, L cùng
nằm trên đường tròn đường kính CZ.
Ta có ∠M JT = ∠M KT . Theo bổ đề 2, M K ⊥ CP do đó M K k ZL. Suy ra ∠M KT = ∠ZLT.
Vậy ∠M JT = ∠T LZ hay T LZJ là tứ giác nội tiếp. Vậy T nằm trên đường tròn đường kính CZ.
Chứng minh tương tự, T cũng nằm trên đường tròn đường kính AX, BY .

Nhận xét.
1. Gọi Ha , Hb , Hc là chân 3 đường cao của tam giác ABC. Dễ thấy HA · HHa = HB · HHb =
HC · HHc nên H nằm trên trục đẳng phương của 3 đường tròn đường kính AX, BY, CZ. Gọi G là
giao điểm thứ hai của 3 đường tròn này ta có HT · HG = HA · HHa = k. Lại có phép nghịch đảo tâm

230
H phương tích k biến đường tròn Euler thành đường tròn (O) nên G nằm trên (O). Vậy ta có tính
chất: Đường tròn đường kính AX, BY, CZ đồng quy tại 2 điểm, một điểm nằm trên đường tròn Euler
và một điểm nằm trên đường tròn ngoại tiếp tam giác ABC.
2. Trong tạp chí THTT tháng 8/2015, bài toán T12 đề ra kì này có nhắc tới khái niệm cực trực
giao. Cực trực giao có một tính chất khá nổi tiếng, phát biểu rằng cực trực giao của một đường thẳng
l đi qua tâm đường tròn ngoại tiếp thì nằm trên đường tròn Euler. Trong trường hợp này, cực trực giao
là điểm Anti-Steiner của tam giác trung tuyến ứng với l và nằm trên đường tròn pedal của mọi điểm
nằm trên l. (xem [4], [5]). Chú ý rằng đường tròn pedal của X, Y, Z ứng với tam giác ABC chính là
đường tròn đường kính AX, BY, CZ. Do đó T chính là cực trực giao của đường thẳng l ứng với tam
giác ABC. Ta thu được tính chất sau.

Tính chất 153. Cực trực giao của các tam giác ABC, BP C, CP A, AP B ứng với đường thẳng l cùng
nằm trên d.

Tính chất 154. Ta gọi dP là đường thẳng Simson mở rộng của P ứng với đường thẳng l. Gọi Q là
điểm đối xứng với P qua O, dQ là đường thẳng Simson mở rộng của Q ứng với l. Đường thẳng qua P
vuông góc với dP giao đường thẳng qua Q vuông góc với dQ tại J. Khi đó J ∈ (O) và đường thẳng
Simson mở rộng của J ứng với l đi qua tâm đường tròn Euler của tam giác ABC.

d'

A S

P
T l
K

O
H
E
d
L
B
Q C
M

Chứng minh. Theo bài toán 1, dP và dQ lần lượt đi qua trung điểm K và L của HP và HQ. Lại theo
tính chất 2, góc giữa dP và dQ bằng một nửa số đo cung P Q, mà P Q là đường kính của (O) nên
dP ⊥ dQ tại cực trực giao T của tam giác ABC ứng với đường thẳng l. Vậy ∠P JQ = 90◦ hay J ∈ (O).
Gọi M là trung điểm của HJ, dJ là đường thẳng Simson mở rộng của J. Suy ra M ∈ dJ . Theo
tính chất 5, dJ đi qua T . Gọi S là giao của dQ và dJ .
Theo tính chất 1, ta có ∠LT M = ∠QP J = ∠OJP = ∠OSQ, do đó dJ k OJ. Suy ra dJ là đường
trung bình của tam giác HOJ hay dJ đi qua tâm đường tròn Euler của tam giác ABC.

Nhận xét. Gọi J 0 là điểm đối xứng với J qua O thì đường thẳng Simson mở rộng dJ 0 của J 0 vuông
góc với dJ tại T . Mà dJ đi qua tâm đường tròn Euler nên dJ 0 là tiếp tuyến của đường tròn Euler. Từ
đó suy ra tính chất sau.

Tính chất 155. Đường thẳng qua P song song với dP giao đường thẳng qua Q song song với dQ tại
J 0 . Khi đó J 0 ∈ (O) và đường thẳng Simson mở rộng của J 0 tiếp xúc với đường tròn Euler của tam
giác ABC.

231
Nhận xét. Chúng ta lại nhớ đến tính chất điểm Feuerbach là điểm Anti-Steiner của tam giác
trung tuyến ứng với đường thẳng OI. Như vậy khi l ≡ OI, K trùng với điểm Feuerbach Fe . Khi đó
đường thẳng Simson mở rộng của J 0 ứng với OI là tiếp tuyến chung của đường tròn Euler và đường
tròn nội tiếp.
Đến đây bài viết xin được dừng lại. Một số nhận xét được tác giả nêu ra và chưa chứng minh đều
là các kết quả rất nổi tiếng trong hình học. Do khuôn khổ có hạn nên chưa có điều kiện giới thiệu lời
giải cho các tính chất này. Mong bạn đọc xem như bài tập và tự mình nghiên cứu.

23 Một số vấn đề khác

232
PHẦN III: MỘT SỐ TÌM TÒI SÁNG TẠO

24 Xung quanh bài 3 VMO 2012


24.1 Giới thiệu
Trong bài viết này tác giả chứng minh lại bài số 3 trong kì thi chọn HSG Quốc gia năm 2012. Đồng
thời chúng ta sẽ khai thác bài toán trong trường hợp tổng quát.

24.2 Chứng minh bài toán


Bài toán.
Trong mặt phẳng, cho tứ giác lồi ABCD nội tiếp đường tròn tâm O và có các cặp cạnh đối không
song song. Gọi M, N tương ứng là giao điểm của các đường thẳng AB và CD, AD và BC. Gọi P, Q, S, T
tương ứng là giao điểm các đường phân giác trong của các cặp M \AN và M\ BN , M\ BN và M \ CN ,
M\ CN và M \DN , M\ DN và M \AN . Giả sử bốn điểm P, Q, S, T đôi một phân biệt.
1) Chứng minh rằng bốn điểm P, Q, S, T cùng nằm trên một đường tròn. Gọi I là tâm của đường
tròn đó.
2) Gọi E là giao điểm của các đường chéo AC và BD. Chứng minh rằng ba điểm E, O, I thẳng
hàng.
Chứng minh.
N
P S

Q
I
T
B

A
E

O
M
D

a. Không mất tổng quát ta có thể giả sử D nằm giữa M và C, B nằm giữa N và C. Các trường
hợp còn lại chứng minh tương tự.
Ta có P[ \ − QCB
QS = QBN \ = 1 (N \ \ = 1 BM
BM − BCD) \ C.
2 2
1\
Tương tự, P[T S = BM C. Do đó P[ T S = P[
QS. Kéo theo 4 điểm P, R, S, T cùng thuộc một đường
2
tròn. Ta có đpcm.
b. Do Q là giao của phân giác góc M CB và M BN nên Q là tâm đường tròn bàng tiếp góc C của
tam giác M BC. Suy ra M Q là phân giác ngoài của góc BM C.
Tương tự, M T là phân giác ngoài của góc BM C. Do đó M, T, Q thẳng hàng.
Do T là tâm đường tròn bàng tiếp góc D của tam giác MDA nên sử dụng cộng góc ta dễ dàng suy
1\
ra M T A = 90o − M
\ DA = QBA.
\
2
Do đó tứ giác T QBA nội tiếp. Ta suy ra M T.M Q = M A.M B hay M nằm trên trục đẳng phương
của hai đường tròn (O) và (I). Tương tự, N nằm trên trục đẳng phương của hai đường tròn (O) và
(I). Suy ra M N ⊥ OI.

233
Để chứng minh O, I, E thẳng hàng, ta sẽ chứng minh OE ⊥ M N . Đây là một kết quả quen thuộc
có tên là định lý Brocard. Có nhiều cách để chứng minh kết quả này. Ở đây chúng tôi giới thiệu cách
giải bằng cực và đối cực.
Do AC giao BD tại E nên E lần lượt nằm trên đường đối cực của M, N đối với đường tròn (O).
Suy ra M N là đường đối cực của E đối với (O), từ đó OE ⊥ M N . Vậy ta có đpcm.
Nhận xét. Bài số 3 là một bài toán không khó nhưng khá thú vị. Câu a tương đối đơn giản, chỉ
cần những biến đổi góc thông thường và không cần điều kiện tứ giác ABCD nội tiếp. Câu b đòi hỏi
sử dụng định lý Brocard nhưng đây là một kết quả quen thuộc. Chứng minh OI ⊥ M N bằng cách sử
dụng phương tích cũng là hướng đi đơn giản và tự nhiên. Tuy nhiên, ý tưởng của bài toán không mới,
nó bắt nguồn từ bài toán 2978 trên tạp chí Crux Mathematicorum năm 2004 volume 30.

24.3 Khai thác bài toán


Chúng ta nhận thấy câu a của bài toán không dùng đến điều kiện "tứ giác ABCD nội tiếp". Điều
đó có nghĩa là nó vẫn đúng trong trường hợp tứ giác ABCD bất kì. Một cách tương tự, bạn đọc có
thể dễ dàng chứng minh các kết quả sau đây:
Cho tứ giác ABCD bất kì thì:
4 phân giác trong góc A, B, C, D cắt nhau tại 4 điểm thuộc đường tròn (I1 ).
4 phân giác ngoài góc A, B, C, D cắt nhau tại 4 điểm thuộc đường tròn (I2 ).
2 phân giác trong góc A, C cắt 2 phân giác ngoài góc B, D tại 4 điểm thuộc đường tròn (I3 ).
2 phân giác trong góc B, D cắt 2 phân giác ngoài góc A, C tại 4 điểm thuộc đường tròn (I4 ).
2 phân giác trong góc A, B cắt 2 phân giác ngoài góc C, D tại 4 điểm thuộc đường tròn (I5 ).
2 phân giác trong góc B, C cắt 2 phân giác ngoài góc D, A tại 4 điểm thuộc đường tròn (I6 ).
2 phân giác trong góc C, D cắt 2 phân giác ngoài góc A, B tại 4 điểm thuộc đường tròn (I7 ).
2 phân giác trong góc D, A cắt 2 phân giác ngoài góc B, C tại 4 điểm thuộc đường tròn (I8 ).
Chúng ta sẽ không chứng minh lại các kết quả trên mà sẽ bàn đến những tính chất xoay quanh 8
đường tròn.
Tính chất 1. Bốn điểm I1 , I2 , I3 , I4 thẳng hàng trên đường thẳng d (ta gọi bốn đường tròn
(I1 ), (I2 ), (I3 ), (I4 ) là bộ đường tròn ω1 ). Bốn điểm I5 , I6 , I7 , I8 thẳng hàng trên đường thẳng l ( ta gọi
bốn đường tròn (I5 ), (I6 ), (I7 ), (I8 ) là bộ đường tròn ω2 ). Đồng thời d ⊥ l.
Chứng minh.
Theo cách định nghĩa trên, ta thấy rằng hai đường tròn bất kì (Ix ) và (Iy ) thuộc hai bộ ω1 và ω2
đều có một điểm chung. Do đó trước tiên ta phát biểu một bổ đề.
Bổ đề 1. Hai đường tròn (Ix ) và (Iy ) trực giao (nghĩa là nếu gọi J là giao điểm của hai đường
tròn thì I\ o
x JIy = 90 .)
Chứng minh bổ đề.

234
I6

H
E
K

A
B
M
L I1
Q
N
P

D C F

Ta chứng minh bổ đề với trường hợp (I1 ) và (I6 ) (Các trường hợp còn lại chứng minh tương tự).
Kí hiệu dX là phân giác trong góc X, lX là phân giác ngoài góc X (X là các đỉnh của tứ giác
ABCD).
Gọi M, N, P, Q, H, K, L lần lượt là giao điểm của các cặp (dA , dB ), (dB , dC ), (dC , dD ), (dD , dA ),
(dC , lD ), (lA , dB ), (lA , lD ); E, F lần lượt là giao điểm của AD và BC, AB và CD.
Ta cần chứng minh I\ o
1 N I6 = 90 .
1\
Chú ý rằng M là tâm đường tròn bàng tiếp góc E của tam giác AEB nên AM \ B = 90o − AEB.
2
o−N o − AM 1\
Từ đó I\ 1 N Q = 90 \ M Q = 90 \ B = AEB. (1)
2
Mặt khác, K là tâm đường tròn bàng tiếp góc A của tam giác AEB nên AKB \ = 1 AEB.\ Từ đó
2
o o 1\
6 N L = 90 − LKN = 90 − AEB. (2)
I\ \
2
Từ (1) và (2) suy ra I\ o
1 N Q + I6 N L = 90 .
\
Vậy I\ o
1 N I6 = 90 .
Trở lại bài toán.
Theo bổ đề trên thì I1 N là tiếp tuyến kẻ từ I1 tới đường tròn (I6 ). Tương tự, I1 M, I1 P, I1 Q lần
lượt là tiếp tuyến kẻ từ I1 tới các đường tròn (I5 ), (I7 ), (I8 ). Vậy I1 có cùng phương tích tới bộ đường
tròn ω2 . Tương tự, I2 có cùng phương tích tới tới bộ đường tròn ω2 . Nghĩa là I1 I2 là trục đẳng phương
của bộ bốn đường tròn ω2 , suy ra bốn điểm I5 , I6 , I7 , I8 cùng nằm trên đường thẳng l vuông góc với
I1 I2 . Tương tự ta cũng thu được I1 , I2 , I3 , I4 thẳng hàng trên đường thẳng d. Vậy d ⊥ l. Ta có đpcm.
Nhận xét. Điều thú vị là bộ đường tròn ω1 có chung trục đẳng phương l còn bộ đường tròn ω2 có
chung trục đẳng phương d.
Trước khi đến với tính chất 2, ta phát biểu lại điểm Miquel của tứ giác toàn phần ABCDEF như
sau:
Cho tứ giác ABCD. Gọi E là giao điểm của AD và BC, F là giao điểm của AB và CD. Khi đó
bốn đường tròn ngoại tiếp các tam giác ABE, CBF, CDE, ADF đồng quy tại một điểm J gọi là điểm
Miquel của tứ giác toàn phần ABCDEF .
Tính chất 2. Sử dụng kí hiệu như lời giải tính chất 1. Ta có giao điểm của d và l là điểm Miquel
của tứ giác toàn phần ABCDEF .
Chứng minh.
Trước tiên ta phát biểu và chứng minh một bổ đề.
Bổ đề 2. I\ o
x JIy = 90 .
Chứng minh bổ đề.

235
I6

E
K
J

U A

M B
L V
T
N I1
Q
P

D C F

Ta sẽ chứng minh bổ đề với Ix ≡ I1 , Iy ≡ I6 . Các trường hợp còn lại được chứng minh tương tự.
Gọi T là giao điểm thứ hai của (I1 ) và (I6 ). dC cắt (EDC) lần thứ hai tại U, F L cắt (ADF ) lần
thứ hai tại V .
Do P là tâm đường tròn nội tiếp của tam giác EDC nên U là tâm đường tròn ngoại tiếp của tam
giác EP D. Mà HEP\ = HDP \ = 90o nên U là trung điểm HQ. Tương tự, V là trung điểm LQ.
o
N = 90 − I\ o
Ta có HT 6 N H, N T P = 90 − I1 N P .
\ \ \
o
Suy ra HT P = HT N + N T P = 180 − I6 N H − I\ o
1 N P = I1 N I6 = 90 .
\ \ \ \ \
Từ đó N\ U T = 2N \ HT = N \ I6 T . Suy ra U nằm trên (N I6 T ) hay đường tròn đường kính I6 I1 .
Tương tự, V nằm trên đường tròn đường kính I6 I1 .
Ta có U[JV = U[ JD − V[ JD = DCU\ − DF \ V =P \ N Q = V\ NU.
Vậy J nằm trên đường tròn đường kính I6 I1 . Suy ra I\ JI o
1 6 = 90 .
Trở lại bài toán.
Theo bổ đề 2, ta có I\ o o
1 JI6 = 90 . Tương tự ta cũng có I1 JI5 = 90 . Vậy ba điểm I5 , I6 , J thẳng
\
hàng hay J ∈ l. Tương tự, J ∈ d. Vậy J là giao điểm của d và l.
Tính chất 3. Kí hiệu Γ1 là phương tích từ J tới bộ đường tròn ω1 , Γ2 là phương tích từ J tới bộ
đường tròn ω2 . Khi đó Γ1 + Γ2 = 0.
Chứng minh.
Từ bổ đề 1 và 2 ta có I\ o
1 N I6 = I1 JI6 = 90 .
\
Suy ra I1 N 2 + I6 N 2 = I1 J 2 + I6 J 2 hay I1 J 2 − I1 N 2 + I6 J 2 − I6 N 2 = 0.
Vậy Γ1 + Γ2 = 0.
Lời bàn. Ta quay lại bài số 3 VMO 2012. Trong trường hợp này, tâm của bộ đường tròn ω1 nằm
trên đường thẳng OE. Gọi J là giao của (N AB) và M N . Dễ dàng chứng minh được tứ giác M JAD
nội tiếp hay J là điểm Miquel của tứ giác toàn phần ABCDEF . Theo tính chất 2, J là giao điểm của
OE và l. Theo định lý Brocard, OE ⊥ M N vậy M N ≡ l. Ta thu được tâm của bộ đường tròn ω2 nằm
trên đường thẳng M N .
Như vậy, từ một bài toán tưởng chừng đơn giản, nếu biết đào sâu suy nghĩ chúng ta có thể tìm ra
được nhiều điều thú vị ẩn chứa bên trong nó.

236
25 Bài toán 6 trong kì thi chọn đội tuyển quốc gia Iran năm 2013
25.1 Giới thiệu
Trong ngày thi thứ 2 của kì thi chọn đội tuyển quốc gia Iran năm 2013 xuất hiện một bài toán khá
thú vị:
Cho 4 điểm A, B, C, D theo thứ tự cùng nằm trên đường thẳng l. Hai cung ω1 , ω2 dựng trên dây
AB và ω3 , ω4 dựng trên dây CD sao cho 4 cung trên cùng nằm trên một nửa mặt phẳng bờ là đường
thẳng l đồng thời ω1 tiếp xúc với ω3 và ω2 tiếp xúc với ω4 . Chứng minh rằng giao điểm của tiếp tuyến
chung ngoài của các cặp cung (ω2 , ω3 ) và (ω1 , ω4 ) nằm trên l.
Ở bài viết này tác giả sẽ đưa ra 4 cách chứng minh cho bài toán trên và lời giải cho bài toán tổng
quát.

25.2 Chứng minh


Cách 1.
Trước tiên ta phát biểu một số bổ đề.

Bổ đề 27. (Định lý Monge-D’Alembert). Cho ba đường tròn C1 (O1 , R1 ), C2 (O2 , R2 ), C3 (O3 , R3 ) phân
biệt trên mặt phẳng. Khi đó tâm vị tự ngoài của các cặp đường tròn (C1 , C2 ), (C2 , C3 ), (C3 , C1 ) cùng
thuộc một đường thẳng. Hai tâm vị tự trong của hai trong ba cặp đường tròn trên và tâm vị tự ngoài
của cặp đường tròn còn lại cùng thuộc một đường thẳng.

Có thể chứng minh bổ đề 1 tương đối đơn giản dựa theo định lý Menelaus, xin phép không trình
bày lại ở đây.

Bổ đề 28. Cho hai dây cung γ1 và γ2 cùng dựng trên dây AC sao cho chúng cùng nằm trên một nửa
mặt phẳng bờ AC. Gọi C1 , C2 là hai đường tròn tiếp xúc với γ1 và γ2 . Khi đó tâm vị tự ngoài của C1
và C2 nằm trên AC.

I2

Z
I1
Y

X O2

I
A C

O1

Chứng minh. Gọi (O1 , R1 ) và (O2 , R2 ) lần lượt là đường tròn chứa các cung γ1 , γ2 ; I là tâm vị tự ngoài
của C1 và C2 . X, Y là tiếp điểm của C1 , C2 với γ1 , Z, T là tiếp điểm của C1 , C2 với γ2 .
Áp dụng định Monge-D’Alembert cho bộ ba đường tròn ((O1 ), γ1 , γ2 ) và ((O2 ), γ1 , γ2 ) suy ra I, X, Y
thẳng hàng và I, Z, T thẳng hàng.
Gọi IA1 A2 là tiếp tuyến chung ngoài của C1 , C2 .
Phép nghịch đảo IIIA1 .IA2 : C1 7→ C2 , đồng thời các cặp đường thẳng I1 Z và I2 T , I1 X và I2 Y
không song song nên IZ.IT = IA1 .IA2 = IX.IY . Suy ra X, Y, Z, T đồng viên.

237
Xét ba đường tròn (O1 ), (O2 ), (XY T Z) có trục đẳng phương lần lượt là AC, XY, ZT nên AC, XY, ZT
đồng quy. Từ đó I ∈ AC.

Bổ đề 29. Cho hai dây cung ω1 và ω2 cùng nằm trên một nửa mặt phẳng bờ là đường thẳng l và
cùng dựng trên dây AB nằm trên l. C, D là hai điểm bất kì trên l và nằm ngoài đoạn thẳng AB. Khi
đó 4 tiếp tuyến kẻ từ C, D tới ω1 và ω2 cắt nhau tạo thành một tứ giác ngoại tiếp.

N
T

Q
Z
M
Y X O2

O1

C A B D

Chứng minh. Gọi X, Z và Y, T là các điểm trên ω1 và ω2 sao cho CX, CY, DZ, DT là các tiếp tuyến
của hai đường tròn (O1 ) và (O2 ) chứa các cung ω1 và ω2 ; M N P Q là tứ giác tạo bởi giao điểm của
CX, CY, DZ, DT như hình vẽ.
Do C, D nằm trên trục đẳng phương của ω1 và ω2 nên CX = CY, DZ = DT . Từ đó ta có
CM + DP = CX + XM + DT + T P = CY + M Z + DZ + P Y = CP + DM.
Theo định lý Pithot suy ra tứ giác M N P Q ngoại tiếp.

Trở lại bài toán.

I F
O3
M

N Q O4
O2
P
O1

Y A B X C D

Gọi (O1 ), (O2 ), (O3 ), (O4 ) lần lượt là các đường tròn chứa các cung ω1 , ω2 , ω3 , ω4 ; P, Q lần lượt là
tiếp điểm của (O1 ) và (O3 ), (O2 ) và (O4 ).
Qua P kẻ tiếp tuyến chung trong của (O1 ) và (O3 ), cắt d tại X.
Ta có XP 2 = XA.XB = XC.XD nên X thuộc trục đẳng phương của (O2 ) và (O4 ), tức là XQ là
tiếp tuyến chung trong của (O2 ) và (O4 ).
Giả sử tiếp tuyến chung ngoài M E của ω2 và ω3 cắt d tại Y , qua Y kẻ tiếp tuyến Y N tới ω1 , Y F
tới ω4 .

238
Áp dụng bổ đề 3 cho hai đường tròn (O1 ) và (O2 ) với hai điểm X, Y suy ra Y M, Y N, XP, XQ cắt
nhau tạo thành một tứ giác ngoại tiếp đường tròn (I).
Lại áp dụng bổ đề 3 cho hai đường tròn (O3 ) và (O4 ) với hai điểm X, Y suy ra Y F tiếp xúc với
đường tròn nội tiếp tam giác tạo bởi các đường thẳng XP, XQ, Y M hay đường tròn (I). Như vậy
Y N, Y F cùng tiếp xúc với (I), suy ra Y, N, F thẳng hàng. Ta có đpcm.
Cách 2.

F
O3

Q O4
N
O2 P

O1

Y A H B X C D G

Với cách kí hiệu tương tự như cách 1, tiếp tuyến M E của ω2 và ω3 cắt l tại Y , lần lượt kẻ các tiếp
tuyến Y N, Y F tới ω2 , ω4 . Ta cần chứng minh Y, N, F thẳng hàng.
Do XP = XQ, Y M = Y N, Y E = Y F nên tồn tại các đường tròn C1 lần lượt tiếp xúc với (O1 )
và (O3 ) tại P , tiếp xúc với (O2 ) và (O4 ) tại Q; đường tròn C2 tiếp xúc với (O2 ) tại M và (O1 ) tại N ,
đường tròn C3 tiếp xúc với (O3 ) tại E và (O4 ) tại F.
Theo bổ đề 2 suy ra M Q cắt N P tại tâm vị tự ngoài G của C1 và C2 và G ∈ l, EP cắt F Q tại
tâm vị tự ngoài H của C1 và C3 và H ∈ l.
Áp dụng định lý Monge-D’Alembert cho 3 đường tròn C1 , C2 , C3 suy ra M E cắt d tại tâm vị tự
ngoài của C2 và C3 , hay Y là tâm vị tự ngoài của C2 và C3 . Suy ra Y, N, F thẳng hàng. Ta có đpcm.
Cách 3.

O3

N O4
Q
O2 P

O1

Y A B X C D

Với ý tưởng tương tự cách 2 ta cần chứng minh Y, N, F thẳng hàng.


Như cách 2 chúng ta đã chứng minh tồn tại các đường tròn C1 , C2 , C3 .

239
Từ đó theo cách chứng minh bổ đề 2 suy ra M, N, P, Q cùng thuộc một đường tròn và E, F, P, Q
cùng thuộc một đường tròn.
Mặt khác, áp dụng định lý Monge-D’Alembert cho 3 đường tròn C1 , (O2 ), (O3 ) suy ra P Q cắt EM
tại tâm vị tự ngoài T của hai đường tròn (O2 ) và (O3 ).
Suy ra T M .T E = T P .T Q, nghĩa là M, E, P, Q cùng thuộc một đường tròn.
Như vậy 6 điểm M, N, E, F, P, Q cùng thuộc một đường tròn. Suy ra tứ giác M N F E nội tiếp.
Giả sử Y N cắt đường tròn (Y, Y E) tại F 0 . Do hai đường tròn (Y, Y M ) và (Y, Y N ) đồng tâm và
hai đường thẳng M E, N F 0 giao nhau tại Y nên M N F 0 E là hình thang cân hay M N F 0 E nội tiếp.
Lại có F và F 0 cùng nằm trên (Y, Y N ) nên F ≡ F 0 , hay Y, N, F thẳng hàng. Ta có đpcm.
Cách 4.
Trước tiên ta phát biểu 2 bổ đề sau.

Bổ đề 30. Cho hai đường tròn (O1 ) và (O2 ) không chứa nhau. Gọi C1 , C2 là hai đường tròn tiếp xúc
ngoài với (O1 ), (O2 ) lần lượt tại A, B và C, D. Khi đó A, B, C, D cùng thuộc một đường tròn.

c1

B
A
O2

O1
T

C D

c2

Chứng minh. Áp dụng định lý Monge-D’Alembert cho 3 đường tròn (O1 ), (O2 ), C1 suy ra AB đi qua
tâm vị tự ngoài T của (O1 ) và (O2 ). Tương tự ta cũng có CD đi qua T .
Từ đó T A.T B = T C.T D = k. Suy ra A, B, C, D cùng thuộc một đường tròn.

Bổ đề 31. (Chuỗi đường tròn Apollonius- bài toán 9 đường tròn). Cho 3 đường tròn (O1 ), (O2 ), (O3 )
trên mặt phẳng. Ta xây dựng một chuỗi đường tròn xoay vòng như sau. Gọi C12 là đường tròn tiếp
xúc với (O1 ) và (O2 ), C23 là đường tròn tiếp xúc với C12 , (O2 ), (O3 ), C34 là đường tròn tiếp xúc với
C23 , (O3 ), (O1 ), C45 là đường tròn tiếp xúc với C34 , (O1 ), (O2 ), C56 là đường tròn tiếp xúc với C45 ,
(O2 ), (O3 ), C61 là đường tròn tiếp xúc với C5 , (O3 ), (O1 ). Khi đó C12 , C23 , ..., C61 là một chuỗi đóng,
hay C61 tiếp xúc với C12 .

240
O3
C56
C61

C34 P6
C12
P1 P3
O1
P4
C23
C45

P5 P2

O2

Chứng minh. Ta chứng minh bài toán trong trường hợp 3 đường tròn (O1 ), (O2 ), (O3 ) đôi một ngoài
nhau. Các trường hợp khác chứng minh tương tự.
Gọi P1 , P2 lần lượt là tiếp điểm của C12 với (O1 ) và (O2 ); P3 là tiếp điểm của C23 với (O3 ); P4 là
tiếp điểm của C34 với (O1 ), tương tự với P5 , P6 , P7 . Như vậy ta cần chứng minh P7 ≡ P1 .
Áp dụng bổ đề 4 cho hai đường tròn C12 và C34 cùng hai đường tròn tiếp xúc C23 và (O1 ) suy ra
P1 , P2 , P3 , P4 cùng thuộc một đường tròn.
Áp dụng bổ đề 4 cho hai đường tròn (O2 ) và (O3 ) với hai đường tròn tiếp xúc C23 và C56 suy ra
P2 , P3 , P5 , P6 cùng thuộc một đường tròn.
Tiếp tục áp dụng bổ đề 4 cho hai đường tròn (O1 ) và (O2 ) với hai đường tròn tiếp xúc C45 và C12
suy ra P1 , P2 , P4 , P5 cùng thuộc một đường tròn.
Từ các điều trên ta thu được P1 , P2 , P3 , P4 , P5 , P6 cùng thuộc một đường tròn ω.
Áp dụng bổ đề 4 cho hai đường tròn (O1 ) và (O3 ) với hai đường tròn tiếp xúc C61 và C34 suy ra
P3 , P4 , P6 , P7 cùng thuộc một đường tròn. Từ đó P7 là giao của ω với (O1 ) hay P7 ≡ P1 . Như vậy C61
và C12 tiếp xúc nhau. Ta có đpcm.

Trở lại bài toán.

F'

O3

N O4
Q
O2
P

O1

Y A B X C D

Chú ý rằng chuỗi Apollonius vẫn đúng trong trường hợp đường tròn suy biến thành đường thẳng.

241
Tiếp tuyến chung M E của (O2 ) và (O3 ) cắt l tại Y . Kẻ tiếp tuyến Y N của ω1 , dựng đường tròn
C3 tiếp xúc với Y M, Y N lần lượt tại E, F 0 . Đường tròn C1 và C2 được định nghĩa giống cách 2,3.
Áp dụng bổ đề 5 cho đường tròn C1 và 2 đường tròn suy biến thành đường thẳng Y M và Y N ta
có:
(O2 ) tiếp xúc với C1 , Y M .
C2 tiếp xúc với (O2 ), Y M, Y N.
(O1 ) tiếp xúc với C2 , Y N, C1 .
(O3 ) tiếp xúc với (O1 ), C1 , Y M.
C3 tiếp xúc với (O3 ), Y M, Y N.
(O40 ) tiếp xúc với C3 , Y N, C1 .
Do chuỗi (O2 ), C2 , (O1 ), (O3 ), C3 , (O40 ) là chuỗi đóng nên (O40 ) tiếp xúc với (O2 ), C1 , Y N.
Điều này nghĩa là (O40 ) tiếp xúc với C1 tại P và tiếp xúc với Y N tại F 0 .
Lại có tiếp tuyến tại Q của (O2 ) cắt tiếp tuyến tại P của (O1 ) tại X và XP = XQ nên X nằm
trên trục đẳng phương của (O40 ) và (O3 ).
Mà Y E = Y F 0 nên Y cũng thuộc trục đẳng phương của (O40 ) và (O3 ). Vậy XY là trục đẳng phương
của (O40 ) và (O3 ) hay (O40 ) đi qua C, D.
Suy ra (O40 ) ≡ (O4 ). Ta có đpcm.

25.3 Mở rộng
Bài toán 1. Cho 2n điểm A1 , B1 , A2 , B2 , ..., An , Bn (n ≥ 2) theo thứ tự nằm trên đường thẳng l.
Hai cung ωk và γk dựng trên dây Ak Bk (k = 1, n) sao cho 2n cung cùng nằm trên một nửa mặt phẳng
bờ l đồng thời ωk tiếp xúc với ωk−1 và ωk+1 , γk tiếp xúc với γk−1 và γk+1 . Khi đó giao điểm của tiếp
tuyến chung ngoài của các cặp cung ω1 và γ2k , γ1 và ω2k nằm trên l; giao điểm của các tiếp tuyến
chung ngoài của các cặp cung ω1 và ω2k+1 , γ1 và γ2k+1 cũng nằm trên l (k = 1, 2, ...).
Chứng minh.
Cách 1.
Để chứng minh bài toán mở rộng trước tiên ta phát biểu một bổ đề sau.

Bổ đề 32. Cho 6 điểm A1 , B1 , A2 , B2 , A3 , B3 cùng nằm trên đường thẳng l sao cho các đoạn thẳng
A1 B1 , A2 B2 , A3 B3 tách rời nhau. Hai cung ωi và γi cùng dựng trên dây Ai Bi (i = 1, 3) sao cho 6 cung
cùng nằm trên một nửa mặt phẳng bờ l. Kí hiệu dij là tiếp tuyến chung ngoài của cặp cung ωi và ωj ,
kij là tiếp tuyến chung ngoài của cặp cung γi và γj . Giả sử giao điểm của d12 và k12 , d23 và k23 cùng
nằm trên l. Khi đó giao điểm của d13 và k13 cũng nằm trên l.

A1 B1 A2 B2 A3 B3 X Z Y

Chứng minh. Gọi X, Y lần lượt là giao của d12 và k12 , d23 và k23 ; Z là giao của d13 với l. Từ Z kẻ tiếp
0 tới γ .
tuyến k13 1

242
Áp dụng bổ đề 3 cho 2 đường tròn ω1 , γ1 và 2 điểm X, Z suy ra tồn tại một đường tròn C1 nội
0 . Tương tự tồn tại đường tròn C nội
tiếp tứ giác tạo bởi giao điểm của 4 đường thẳng d12 , k12 , d13 , k13 2
tiếp tứ giác tạo bởi giao điểm của 4 đường thẳng d12 , k12 , d23 , k23 . Gọi C3 là đường tròn nội tiếp (hoặc
bàng tiếp) tam giác tạo bởi giao điểm của các đường thẳng d13 , d23 , k23 .
Áp dụng định lý Monge-D’Alembert cho 3 đường tròn C1 , C2 , C3 ta có X là tâm vị tự ngoài của
C1 và C2 ; Y là tâm vị tự ngoài của C2 và C3 nên tâm vị tự ngoài của C1 và C3 nằm trên XY .
Do d13 là tiếp tuyến chung của C1 và C3 nên d13 cắt l tại tâm vị tự ngoài Z của C1 và C3 . Mà k13 0

tiếp xúc với C1 nên k13 0 tiếp xúc với C .


3
Từ Z kẻ tiếp tuyến k13 00 tới γ , lại áp dụng bổ đề 3 cho 2 đường tròn ω và γ với hai điểm Y, Z suy
3 3 3
00 . Từ đó k 0 ≡ k 00 ≡ k . Ta có đpcm.
ra C3 tiếp xúc với k13 13 13 13

Trở lại bài toán.

A1 B1 A2 B2 A3 B3

A1 B1 A2 B2 A3 B3 A4 B4

Kí hiệu dij là tiếp tuyến chung ngoài của ωi và γj , kij là tiếp tuyến chung ngoài của γi và ωj ; d0ij
là tiếp tuyến chung ngoài của ωi và ωj , kij 0 là tiếp tuyến chung ngoài của γ và γ (i < j). Gọi X là
i j ij
giao của dij và kij ; Xij0 là giao của d0ij và kij
0 .

Theo bài toán Iran TST 2013 ta có X12 và X23 nằm trên l. Do đó áp dụng bổ đề 6 suy ra X13 0 nằm

trên l. Lại có X34 nằm trên l nên lại áp dụng bổ đề 6 suy ra X14 nằm trên l. Chứng minh tương tự ta
0
thu được X1(2k) và X1(2k+1) nằm trên l.
Cách 2.

243
E

O1

M
O3 F

N O23 O12

P B3 A3 B2 A2 B1 A1

Theo cách 2 cho bài toán Iran TST 2013 thì tồn tại một đường tròn (O12 ) tiếp xúc với ω1 , γ1 , ω2 ,
γ2 ; đường tròn (O23 ) tiếp xúc với ω2 , γ2 , ω3 , γ3 . Gọi P là giao của tiếp tuyến chung ngoài M E của ω1
và ω3 , từ P lần lượt kẻ tiếp tuyến P N, P F tới γ3 , γ1 . Cũng theo cách 2 của bài toán Iran TST 2013
thì tồn tại đường tròn (O1 ) tiếp xúc với ω1 và γ1 lần lượt tại E, F ; đường tròn (O3 ) tiếp xúc với ω3 và
γ3 lần lượt tại M, N.
Theo bổ đề 2, tâm vị tự ngoài của các cặp đường tròn ((O1 ), (O12 )); ((O12 ), (O23 ) nằm trên l do
đó theo định lý Monge-D’Alembert, tâm vị tự ngoài của (O1 ) và (O23 ) nằm trên l.
Cũng theo bổ đề 2, tâm vị tự ngoài của (O23 ) và (O3 ) nằm trên l nên lại áp dụng định lý Monge-
D’Alembert suy ra tâm vị tự ngoài của (O1 ) và (O3 ) nằm trên l. Điều này nghĩa là P, N, F thẳng hàng
hay giao điểm của tiếp tuyến chung ngoài của các cặp (ω1 , ω3 ), (γ1 , γ3 ) nằm trên l. Trường hợp tổng
quát được chứng minh tương tự.
Tuy nhiên chúng ta chưa dừng lại ở đây. Bài toán mở rộng lại đem đến một bài toán thú vị khác.
Bài toán 2. Cho 6 điểm A1 , B1 , A2 , B2 , A3 , B3 cùng nằm trên đường thẳng l sao cho các đoạn
thẳng A1 B1 , A2 B2 , A3 B3 tách rời nhau. Hai cung ωi và γi cùng dựng trên dây Ai Bi (i = 1, 3) sao cho
6 cung cùng nằm trên một nửa mặt phẳng bờ l. Giả sử rằng ω1 tiếp xúc với ω2 , γ1 tiếp xúc với γ2 , ω3
tiếp xúc với ω2 và γ1 , γ3 tiếp xúc với γ2 . Khi đó γ3 tiếp xúc với ω1 .

A1 B1 A2 B2 A3 B3

Chứng minh. Kí hiệu như lời giải bài toán 1. Ta có X13 0 nằm trên l.
0
Gọi γ3 là cung qua A3 , B3 và tiếp xúc với ω1 . Theo bài toán Iran TST 2013 suy ra giao điểm của
tiếp tuyến của các cặp cung ω1 và ω3 , γ1 và γ30 nằm trên l. Tức là tiếp tuyến đi qua X13
0 của γ tiếp
1
0 0
xúc với γ3 , điều này nghĩa là γ3 ≡ γ3 . Ta có đpcm.

Dựa theo bài toán 1 và sử dụng cách chứng minh tương tự trên, ta có thể tổng quát bài toán 2
như sau.
Bài toán 3. Chuỗi đường tròn ω1 , γ1 , ω2 , γ2 , ..., ω2k , γ2k , ω2k+1 , γ2k+1 được xây dựng giống bài toán
1. Khi đó nếu ω2k tiếp xúc với ω1 thì γ2k tiếp xúc với γ1 , nếu ω2k+1 tiếp xúc với γ1 thì γ2k+1 tiếp xúc
với ω1 .

244
Ngoài ra, sử dụng phép nghịch đảo phương tích bất kì với cực là điểm bất kì nằm ngoài các đường
tròn và đường thẳng l trên, ta có thể thu được một mô hình thú vị tương tự bài toán 3 với các điểm
A1 , B1 , A2 , B2 , ..., A2k+1 , B2k+1 cùng nằm trên một đường tròn.
Cuối cùng, trong bài toán Iran TST 2013, ta có thể nhận thấy 4 cung ω1 , ω2 , ω3 , ω4 cùng tiếp xúc
với một đường tròn. Ý tưởng đó giúp chúng ta nghĩ đến một mở rộng khác như sau.
Bài toán 4. Cho 4 điểm A, B, C, D theo thứ tự cùng nằm trên đường thẳng l. Hai cung ω1 , ω2
dựng trên dây AB và ω3 , ω4 dựng trên dây CD sao cho 4 cung trên cùng nằm trên một nửa mặt phẳng
bờ là đường thẳng l đồng thời ω2 nằm giữa ω1 và l, ω4 nằm giữa ω3 và l. Giả sử tồn tại một đường
tròn tiếp xúc với 4 cung trên. Khi đó giao điểm của tiếp tuyến chung ngoài của các cặp cung (ω2 , ω4 )
và (ω1 , ω3 ) nằm trên l.
Cách giải cho bài toán 4 được thực hiện tương tự lời giải 2 của bài toán Iran TST 2013.

M
O4

F
O1
N

O3
Y A B C D
O2

Ta cũng có thể tổng quát cho n đường tròn như sau.


Bài toán 5. Cho 2n điểm A1 , B1 , A2 , B2 , ..., An , Bn (n ≥ 2) theo thứ tự nằm trên đường thẳng l.
Hai cung ωk và γk dựng trên dây Ak Bk (k = 1, n) sao cho 2n cung cùng nằm trên một nửa mặt phẳng
bờ l đồng thời ωk nằm giữa γk và l. Với mỗi giá trị của k (k = 1, n − 1), giả sử tồn tại một đường tròn
tiếp xúc với 4 cung ωk , γk , ωk+1 , γk+1 . Khi đó giao điểm của các tiếp tuyến chung ngoài của ω1 và ωn ,
γ1 và γn nằm trên l.

Đến đây bài viết xin được kết thúc. Xung quanh bài toán Iran TST 2013 còn rất nhiều điều thú
vị, mong bạn đọc tiếp tục khám phá.

26 Bài toán số 5 trong kì thi chọn đội tuyển toán Quốc tế của Việt
Nam năm 2015
26.1 Mở đầu
Trong ngày thi thứ hai của kì thi Việt Nam TST 2015 có một bài toán khá thú vị.
Bài toán. Cho tam giác ABC nhọn có một điểm P nằm trong tam giác sao cho ∠AP B = ∠AP C =
α và α > 180◦ − ∠BAC. Đường tròn ngoại tiếp tam giác AP B cắt AC ở E, đường tròn ngoại tiếp tam
giác AP C cắt AB ở F . Gọi Q là điểm nằm trong tam giác AEF sao cho ∠AQE = ∠AQF = α. Gọi
D là điểm đối xứng với Q qua EF , phân giác góc EDF cắt AP tại T.
a) Chứng minh rằng ∠DET = ∠ABC, ∠DF T = ∠ACB .
b) Đường thẳng P A cắt các đường thẳng DE, DF lần lượt tại M, N . Gọi I, J lần lượt là tâm đường
tròn nội tiếp các tam giác P EM, P F N và K là tâm đường tròn ngoại tiếp tam giác DIJ. Đường thẳng
DT cắt (K) tại H. Chứng minh rằng HK đi qua tâm đường tròn nội tiếp của tam giác DM N.
Trong bài viết này chúng ta sẽ chứng minh bài toán trên và khai thác một số vấn đề xung quanh.

245
26.2 Chứng minh
M

Y
N F
D
K
X J
T

I H
E Q
A

B C

a) Ta có ∠BEC = 180◦ − ∠AP B = 180◦ − ∠AP C = ∠BF C nên tứ giác BEF C nội tiếp. Suy ra
hai tam giác AEF và ABC đồng dạng.
Lại có ∠AP B = ∠AP C = ∠AQE = ∠AQF = α nên AQ và AP đẳng giác trong góc BAC.
Gọi T 0 là điểm liên hợp đẳng giác của Q trong tam giác AEF . Hiển nhiên T 0 ∈ AT.
Gọi X là điểm đối xứng với T 0 qua EF .
Ta có ∠X 0 EF = ∠T 0 EF = ∠AEQ, ∠XF E = ∠T 0 F E = ∠AF Q, suy ra A và X liên hợp đẳng giác
trong tam giác EQF . Mà QA là phân giác ∠EQF nên X, Q, A thẳng hàng.
Do đối xứng của T 0 qua EF nằm trên QA nên T 0 nằm trên đối xứng của QA qua EF . Mà D đối
xứng với Q qua EF, DT là phân giác ∠EDF nên DT đối xứng với QA qua EF . Từ đó T 0 ≡ T.
Suy ra ∠DET = ∠XEQ = ∠F EA = ∠ABC. Tương tự ∠DF T = ∠ACB.

b) Ta có ∠P ED = ∠DEF + ∠F EA + ∠P EA = ∠QEF + ∠ABC + ∠P BA = ∠ABC + ∠P BA +


∠P BC = 2∠BAC. Theo câu a, ∠DET = ∠ABC nên ET là phân giác ∠DEA. Tương tự F T là phân
giác ∠DF A, suy ra tứ giác DEP F ngoại tiếp đường tròn tâm T.
Qua D kẻ tiếp tuyến thứ hai tới (I), cắt P T tại L.
Tứ giác DEP L, DEP F ngoại tiếp nên DF − P F = DE − P E = DL − P L, suy ra tứ giác DLP F
1
ngoại tiếp, hay DL là tiếp tuyến chung của (I) và (J). Từ đó ∠IDJ = ∠EDF.
2
1
Mặt khác, gọi Y là tâm nội tiếp tam giác DM N , ta có ∠IY J = 180 −∠M IN = 90◦ − ∠M DN =

2
1
∠EDF.
2
Suy ra ∠IDJ = ∠IY J, ta thu được D, Y, I, J cùng thuộc một đường tròn. Do DY và DH là phân
giác của hai góc bù nhau nên ∠Y DH = 90◦ , suy ra Y H là đường kính của (DIJ). Vậy Y H đi qua K.

246
26.3 Khai thác
Đầu tiên chúng ta xem xét câu a của bài toán.
Không khó để nhận ra trường hợp tổng quát nếu hai điểm E, F bất kì thỏa mãn EF đối song với
BC, kết luận của bài toán vẫn giữ nguyên.
Tiếp theo chú ý vào dữ kiện ∠AQE = ∠AQF = α, ta liên tưởng tới một điểm đặc biệt có tính
chất như vậy, đó là điểm Fermat. Thực tế đã có một kết quả khá quen thuộc về sự đồng quy liên quan
đến điểm Fermat được phát biểu như sau.

Bài 274. Cho tam giác ABC có F là điểm Fermat. Khi đó các đường thẳng đối xứng của AF, BF, CF
qua BC, CA, AB đồng quy tại điểm liên hợp đẳng giác của F trong tam giác ABC- điểm isodynamic.

Cách chứng minh bài toán 1 hoàn toàn tương tự câu a của bài toán ban đầu. Chú ý rằng điểm
isodynamic là giao của 3 đường tròn Apollonius trong tam giác. Trong trường hợp tổng quát cũng có
tính chất tương tự, ta có bài toán sau.

Bài 275. Cho tam giác ABC và một điểm P bất kì. Gọi Q là điểm đối xứng với P qua BC. Khi đó
AP, AQ đẳng giác trong ∠BAC khi và chỉ khi P nằm trên đường tròn A-Apollonius.

F
B E C

Chứng minh. Gọi E, F là chân phân giác trong góc BAC. Ta có P và Q đối xứng nhau qua BC nên
EP = EQ. AP và AQ đẳng giác trong ∠BAC khi và chỉ khi AE là phân giác ∠P AQ. Mà EP = EQ
và AP 6= AQ nên điều này tương đương A nằm trên (P EQ).
Tâm ngoại tiếp tam giác P EQ nằm trên trung trực P Q tức là nằm trên BC. Gọi F là giao của
(EP Q) với BC suy ra EF là đường kính của (P EQ), mà E là chân phân giác trong nên A nằm trên
(P EQ) tương đương F là chân phân giác ngoài hay (P EQ) là đường tròn Apollonius ứng với đỉnh A
của tam giác ABC. Ta có đpcm.

Như vậy trong câu a, điểm T nằm trên đường tròn A-Apollonius của tam giác AEF.

Tiếp theo ta có thể chứng minh tứ giác DEP F ngoại tiếp theo một cách khác như sau: dễ thấy
phân giác các góc QEP, QF P, EP F giao nhau tại A nên tứ giác EQF P ngoại tiếp đường tròn tâm
A. Từ đó suy ra EQ − F Q = EP − F P , mà Q và D đối xứng qua EF nên EQ − F Q = DE − DF , ta
thu được DE − DF = P E − P F hay tứ giác DEP F ngoại tiếp.
Từ cách chứng minh này, nếu lấy R là điểm đối xứng với P qua EF ta cũng thu được RE − RF =
P E − P F = QE − QF , từ đó tứ giác REQF ngoại tiếp. Ta thu được bài toán tương tự câu a như sau.

Bài 276. Với giả thiết giống bài toán ban đầu. Gọi R là điểm đối xứng với P qua EF , phân giác góc
ERF cắt AQ tại S. Khi đó ∠RES = ∠ABC, ∠RF S = ∠ACB .

247
R

F
S

B
C

Chứng minh. Ta đã chứng minh được tứ giác REQF ngoại tiếp. Do QA là phân giác ∠EQF nên S là
1 1
tâm nội tiếp tứ giác REQF . Suy ra ∠RES = ∠REQ = (∠REF + ∠F EQ = ∠P EF + ∠F EQ) =
2 2
∠F EA = ∠ABC. Chứng minh tương tự, ∠RF S = ∠ACB.

Theo cách chứng minh bài 3, ta có thể tổng quát câu a của bài toán như sau.

Bài 277. Cho tam giác AEF . Một đường tròn ω có tâm A thỏa mãn E và F nằm ngoài ω. Từ E
và F kẻ hai tiếp tuyến tới ω, chúng cắt nhau tại hai điểm P và Q sao cho tứ giác EP F Q có cạnh
không tự cắt nhau. Gọi D là điểm đối xứng với Q qua EF . Phân giác ∠EDF cắt AP tại T . Khi đó
∠DET = ∠AEF, ∠DF T = ∠AF E.

D
F

E T
Q

Tiếp theo chúng ta sẽ khai thác câu b. Có thể thấy với đường thẳng bất kì đi qua P cắt DE tại M
và DF tại N , kết luận của bài toán được giữ nguyên. Mấu chốt của câu b là chứng minh D nằm trên
đường tròn đi qua tâm nội tiếp các tam giác DM N, EM P, P N F . Đây là một kết quả quen thuộc đã
xuất hiện trong bài toán G8, IMO Shortlist 2009, được phát biểu như sau.

Bài 278. Cho tứ giác ngoại tiếp ABCD. Một đường thẳng qua A cắt đoạn thẳng BC tại M và đường
thẳng CD tại N . Gọi I1 , I2 , I3 lần lượt là tâm nội tiếp các tam giác ABM, M N C, N DA. Chứng minh
rằng trực tâm tam giác I1 I2 I3 nằm trên M N .

248
B

A
I1

I
I3 M

I2
D
N
C

1 1
Chứng minh. Ta có ∠I1 I2 I3 = (∠M N C + ∠CM N ) = ∠BCD.
2 2
Gọi K là giao điểm thứ hai của tiếp tuyến kẻ từ C tới (I3 ) với AM . Do tứ giác AKCD ngoại tiếp
nên AK + DC = AD + KC hay AK − KC = AD − DC = AB − CB.
Suy ra AB + KC = BC + AK hay tứ giác ABCK ngoại tiếp.
1 1
Vậy ∠I1 CI3 = ∠I1 CK + ∠I3 CK = (∠BCK + ∠DCK) = ∠BCD.
2 2
Ta thu được ∠I1 I2 I3 = ∠I1 CI3 hay I1 , I2 , I3 , C nằm trên một đường tròn.
Dễ thấy điểm đối xứng với C qua I1 I2 , I3 I2 nằm trên M N nên M N là đường thẳng Steiner của C
ứng với tam giác I1 I2 I3 . Điều đó nghĩa là trực tâm tam giác I1 I2 I3 nằm trên M N .

Ta lại có kết quả tổng quát hơn về sự đồng viên của C, I1 , I2 , I3 như sau.

Bài 279. Cho tứ giác ngoại tiếp ABCD. E, F là hai điểm bất kì nằm trên CD. AF giao BE tại
G. Gọi I1 , I2 , I3 , I4 lần lượt là tâm đường tròn nội tiếp các tam giác ABG, EF G, ADF, BCE. Chứng
minh rằng I1 , I2 , I3 , I4 cùng thuộc một đường tròn.

Chứng minh. Cách 1.

I1

M
R L
K
T
I3 N
G
U
D I4
I2 S
P

E
Q
F
C

Kẻ tiếp tuyến chung M N của (I3 ) và (I4 ). M N giao AF, BE lần lượt tại L, K.
Gọi P, R, T lần lượt là tiếp điểm của (I3 ) với CD, AD, AF ; Q, S, U lần lượt là tiếp điểm của (I4 )
với CD, BC, BE.
Do tứ giác ABCD ngoại tiếp nên theo định lý Pythot, AB + CD = AD + BC.

249
Suy ra AB + DP + P Q + QC = AR + DR + BS + CS, từ đó AB + P Q = AR + BS = AT + BU .
Mặt khác P Q = M N nên AB+M N = AT +BU hay AB+M L+LK +KN = AL+LT +BK +KG.
Suy ra AB + LK = AL + BK, nghĩa là tứ giác ABKL ngoại tiếp.
Vậy (I1 ), (I3 ), (I4 ) có chung một tiếp tuyến LK.
1 1
Ta thu được ∠I3 I2 I4 = ∠EI2 F = 90◦ + ∠EGF, ∠I3 I1 I4 = 90◦ − ∠LGK.
2 2
Suy ra ∠I3 I1 I4 + ∠I3 I2 I4 = 180◦ . Vậy I1 , I2 , I3 , I4 cùng thuộc một đường tròn.
Cách 2. (Vũ Thanh Tùng, HS lớp 11 THPT chuyên Sư Phạm)

K B
A

I1

I
I3
G
D
I4
I2

F
C

1
Ta có ∠I3 AI1 = ∠DAB = ∠IAB nên ∠I3 AI = ∠I1 AB.
2
Trên AB lấy điểm K sao cho 4AI3 I ∼ 4AI1 K.
Suy ra 180◦ − ∠AKI1 = ∠AKI1 = ∠AII3 = 180◦ − ∠BIC.
Từ đó ∠I1 KB = ∠BIC. Tương tự ta cũng có ∠KBI1 = ∠IBI4 nên 4KI1 B ∼ 4II4 B.
Ta thu được 4AI3 I1 ∼ 4AIK, 4BI1 I4 ∼ 4BKI, suy ra ∠AI1 I3 + ∠BI1 I4 = ∠AKI + ∠BKI =
180◦ .
1
Suy ra ∠I3 I1 I4 = 180◦ − ∠AI1 B = 90◦ − ∠AGB = 180◦ − ∠EI2 F = 180◦ − ∠I3 I2 I4 .
2
Vậy I1 , I2 , I3 , I4 cùng thuộc một đường tròn.

Rõ ràng khi E ≡ C ta thu được bài toán 3. Đồng thời không khó nhận ra trong bài toán 4, đường
thẳng Steiner của I2 ứng với tam giác I1 I3 I4 song song với tiếp tuyến chung của (I1 ), (I3 ), (I4 ). Ta
cũng thấy trong cách giải thứ nhất, tam giác GEF thu được bằng cách lấy giao điểm của các đường
thẳng đối xứng với M N qua 3 cạnh của tam giác I1 I3 I4 . Đây là một bổ đề khá quen thuộc và nhiều
ứng dụng.
Bổ đề. Cho tam giác ABC và một đường thẳng d bất kì. Gọi XY Z là tam giác tạo bởi giao điểm
của các đường thẳng đối xứng với d qua BC, CA, AB. Khi đó tâm đường tròn nội tiếp tam giác XY Z
nằm trên đường tròn ngoại tiếp tam giác ABC.
Để kết thúc bài viết, mời bạn đọc chứng minh một số bài toán có cách phát biểu tương tự bài 5.

Bài 280. Cho tứ giác ngoại tiếp ABCD. E là điểm bất kì trên CD. Gọi (I1 ), (I2 ), (I3 ) lần lượt là
đường tròn nội tiếp các tam giác ADE, AEB, BEC. Chứng minh rằng trực tâm tam giác I1 I2 I3 nằm
trên tiếp tuyến chung ngoài khác CD của (I1 ) và (I3 ).

Bài 281. Cho hình bình hành ABCD có góc A nhọn. Trên cạnh BC lấy điểm T sao cho tam giác
AT D nhọn. Gọi O1 , O2 , O3 lần lượt là tâm đường tròn ngoại tiếp của các tam giác ABT, ADT, CDT .
Chứng minh rằng trực tâm của tam giác O1 O2 O3 nằm trên AD.

250
Bài 282. Cho tam giác ABC. Một đường thẳng d cắt BC, CA, AB lần lượt tại X, Y, Z. Gọi O1 , O2 , O3
lần lượt là tâm đường tròn ngoại tiếp các tam giác AY Z, BXZ, CXY. Chứng minh rằng trực tâm tam
giác O1 O2 O3 nằm trên d.

27 Bài toán số 3 trong kì thi chọn HSG Quốc gia của Việt Nam năm
2016
27.1 Giới thiệu
Trong kì thi chọn HSG Quốc gia năm 2016 (VMO 2016) có bài toán số 3 phát biểu như sau.

Bài toán. Cho tam giác ABC có B, C cố định, A thay đổi sao cho tam giác ABC nhọn. Gọi D là
trung điểm BC và E, F tương ứng là hình chiếu vuông góc của D lên AB, AC.
a) Gọi O là tâm của đường tròn ngoại tiếp tam giác ABC. EF cắt AO và BC lần lượt tại M và
N . Chứng minh rằng đường tròn ngoại tiếp tam giác AM N đi qua một điểm cố định.
b) Các tiếp tuyến của đường tròn ngoại tiếp tam giác AEF tại E, F cắt nhau tại T . Chứng minh
T thuộc một đường thẳng cố định.

Trước tiên xin giới thiệu tới bạn đọc lời giải của bài toán.

Chứng minh. a) Xét trường hợp AB < AC. Trường hợp còn lại chứng minh tương tự.
Ta có
∠DAM = ∠DAO = ∠DAC − ∠OAC = ∠DF E − (90◦ − ∠ABC) = ∠ABC − ∠AF E = ∠M N D.
Do đó tứ giác AM DN nội tiếp. Suy ra đường tròn ngoại tiếp tam giác AM N luôn đi qua D cố
định.

A
K

O
M E

F
C
N B D
L

b) Cách 1.
Kéo dài T D cắt (AEF ) lần thứ hai tại K. Do tứ giác KF DE là tứ giác điều hòa nên ta có
A(DKEF ) = −1. Lại có D là trung điểm BC nên AK k BC. Mà ∠AKD = 90◦ nnKD ⊥ BC. Điều
này nghĩa là T nằm trên đường trung trực của BC cố định.
Cách 2.
Kéo dài DE giao AB tại G, DF giao AC tại L.

251
Ta có ∠ET F = 180◦ − 2∠EAF , ∠AGE = ∠ALF = 90◦ − ∠BAC, do đó ∠AGE = ∠ALF =
1
∠ET F . Suy ra T là tâm đường tròn ngoại tiếp tứ giác GF EL.
2
Áp dụng định lý con bướm cho đường tròn (T, T E) với hai dây cung F L và EG cùng đi qua D.
Ta có DB = DC nên T D ⊥ BC. Vậy T nằm trên đường trung trực của BC cố định.

Đây là một bài toán không mới. Ý b) từng xuất hiện trong cuộc thi Kolmogorov của Nga (xem
[1]). Tác giả Trần Quang Hùng cũng đã mở rộng tại [3], [4]. Tác giả khá bất ngờ khi kì thi năm nay
lấy lại đề của nước khác. Vì tính quen thuộc của nó nên đây là một trong những câu dễ kiếm điểm
trong đề. Tuy vậy cấu hình của bài toán khá đẹp mắt và còn nhiều điều để khám phá. Trong bài viết
này tác giả sẽ mở rộng và khai thác những vấn đề xung quanh.

27.2 Khai thác


Trước tiên chúng ta xem xét câu a). Rõ ràng trong lời giải không sử dụng đến dữ kiện D là trung
điểm BC mà chỉ xét tới yếu tố cố định của D. Do đó chúng ta có thể mở rộng câu a) cho điểm D bất
kì trên BC. Đường tròn ngoại tiếp tam giác AM N vẫn luôn đi qua D.
Ở đây chúng ta sẽ tập trung vào câu b). Có thể nhận thấy cấu hình của câu b) rất giống bài toán
1 trong kì thi chọn đội tuyển Quốc tế của Việt Nam năm 2012, phát biểu như sau.

Bài 283. Cho đường tròn (O) và hai điểm B, C cố định trên (O) sao cho BC không phải là đường
kính của (O). A là điểm bất kì trên (O). Gọi D, J, K lần lượt là trung điểm của BC, CA, AB. E, M, N
lần lượt là hình chiếu vuông góc của A trên BC, B trên DJ và C trên DK. Hai tiếp tuyến tại M, N
của đường tròn ngoại tiếp tam giác EM N giao nhau tại T . Chứng minh rằng T là điểm cố định khi
A chuyển động trên (O).

K J
T

M O
H
N

B E D C

Gọi H là trực tâm tam giác ABC. Không khó nhận ra rằng đường tròn (EM N ) chính là đường
tròn đường kính DH. M, N lần lượt là hình chiếu vuông góc của D trên HB và HC. Do đó nếu xét
tam giác HBC thì câu b) thực chất là một phần của bài toán 1. Bài toán 1 khác biệt ở chỗ A chuyển
động trên (O) trong khi ở câu b), A là điểm bất kì trong mặt phẳng.
Như vậy nếu thay giả thiết của câu b) ta thu được bài toán sau.

Bài 284. Cho tam giác ABC nội tiếp đường tròn (O) có B, C cố định, A chuyển động trên (O). Gọi
D là trung điểm BC và E, F tương ứng là hình chiếu vuông góc của D lên AB, AC. Các tiếp tuyến
của đường tròn ngoại tiếp tam giác AEF tại E, F cắt nhau tại T . Chứng minh T là điểm cố định.

252
Ở ý a) chúng ta đã thay trung điểm BC bằng một điểm cố định bất kì trên BC. Ta thử làm như
vậy với bài toán 2 và thu được bài toán tổng quát.

Bài 285. Cho tam giác ABC nội tiếp đường tròn (O) có B, C cố định, A chuyển động trên (O). Gọi
D là một điểm cố định trên BC và E, F tương ứng là hình chiếu vuông góc của D lên AB, AC. Các
tiếp tuyến của đường tròn ngoại tiếp tam giác AEF tại E, F cắt nhau tại T . Chứng minh T chuyển
động trên một đường tròn cố định.

O'
F
D
B C

O1
K L
T
M R
O2
J

Chứng minh. Gọi K là giao của DE với AB, L là giao của DF với AC. Theo cách 2 của câu b) ta thu
được T là trung điểm KL.
Gọi P là giao của hai tiếp tuyến tại B và C của (O). Các điểm O1 , O2 lần lượt nằm trên P B, P C
sao cho DO1 k P C, DO2 k P B. Ta có ∠DO1 B = ∠CP B = 180◦ − 2∠BAC = 2∠BKD suy ra O1 là
tâm đường tròn ngoại tiếp của tam giác DBK. Tương tự, O2 là tâm đường tròn ngoại tiếp của tam
giác DCL.
Qua D kẻ đường thẳng vuông góc với BC cắt P B tại R, P C tại Q. Suy ra BR, CQ lần lượt là
đường kính của (O1 ) và (O2 ). Suy ra giao điểm J của KR và LQ là điểm đối xứng với A qua tâm O0
của (AKL).
Ta có ∠QRP = 90◦ − ∠P BC = 90◦ − ∠P CB = ∠RQP nên P R = P Q. Lại có ∠RP Q =

180 − ∠BP C = 2∠BAC = 2∠RJQ nên J nằm trên (P, P Q). Do D cố định nên Q cố định, suy ra
(P, P Q) cố định.
Ta có D là trực tâm của tam giác AKL nên KDLJ là hình bình hành, suy ra T là trung điểm
1
DJ. Vậy T chuyển động trên ảnh của (P, P Q) qua phép vị tự tâm D tỉ số .
2
Nhận xét. Quay lại bài toán 2. Khi D là trung điểm BC, J ≡ P hay J ∈ (P, 0), suy ra T là trung
điểm DP cố định.
Khai thác thêm bài toán 3 ta thu được kết quả khá đẹp sau.

Bài 286. Cho tam giác ABC nội tiếp đường tròn (O) có B, C cố định, A chuyển động trên (O). Gọi
D là một điểm cố định trên BC. Các điểm K, L lần lượt nằm trên AB và AC sao cho DK ⊥ AC và
DL ⊥ AB. Chứng minh rằng đường tròn (AKL) luôn đi qua một điểm cố định và tâm của (AKL)
chuyển động trên một đường tròn cố định.

253
A

O F

O'
E
D
B C
I
O1
K
R
M L

O2
J
P

Chứng minh. Với các kí hiệu như lời giải bài 3. Gọi M là giao điểm thứ hai của (O1 ) và (O2 ). Ta có
∠KM L = ∠KM D + ∠LM D = ∠ABC + ∠ACB = 180◦ − ∠KAL nên tứ giác AKM L nội tiếp.
Ta sẽ chứng minh M là điểm cố định.
Ta có ∠M O2 P = 2∠M DQ = ∠M O1 P nên M ∈ (P O1 O2 ).
Lại có ∠BM C = ∠BM D+∠DM C = ∠BKD+∠DLC = 180◦ −2∠BAC = ∠BP C nên M ∈ (OP ).
Do (OP ) và (P O1 O2 ) đều là các đường tròn cố định nên giao điểm M là điểm cố định.
Gọi O0 là tâm của (AKL). (O0 ) và (O1 ) giao nhau tại K và M , (O0 ) và (O2 ) giao nhau tại L và
M . Do đó ∠O1 O0 O2 = 180◦ − ∠KM L = ∠BAC = const. Do đó O0 chuyển động trên đường tròn cố
định đi qua O1 và O2 . Ta sẽ xác định chính xác đường tròn này.
Gọi I là trung điểm OP . (I) giao (O1 ) tại B và M , (I) giao (O2 ) tại C và M nên ∠O1 IO2 =
180 −∠BM C = 180◦ −∠BP C, suy ra tứ giác O1 IO2 P nội tiếp. Lại có ∠BP I = ∠CP I nên IO1 = IO2 .

Ta có ∠O1 IO2 = 180◦ − ∠BP C = 2∠BAC = 2∠O1 O0 O2 nên I là tâm ngoại tiếp tam giác O0 O1 O2 .
Vậy O0 chuyển động trên đường tròn (I, IO1 ).

Nhận xét. Cho điểm D bất kì trong mặt phẳng, ta thu được 2 bài toán tương tự như sau.

Bài 287. Cho tam giác ABC nội tiếp đường tròn (O) có B, C cố định, A chuyển động trên (O). Gọi
D là một điểm cố định trong mặt phẳng. Gọi E, F là hình chiếu vuông góc của D trên AC và AB.
Tiếp tuyến tại E và F của đường tròn đường kính AD cắt nhau tại T . Chứng minh rằng T chuyển
động trên một đường tròn cố định. Tâm của đường tròn này là trung điểm của DJ với J là giao của
hai tiếp tuyến tại B và C.
Đồng thời gọi K, L lần lượt là giao của DE và AB, DF và AC, O0 là tâm đường tròn ngoại tiếp
tam giác AKL. Khi đó O0 chuyển động trên đường tròn có tâm là trung điểm OJ.

Bài 288. Cho tam giác ABC nội tiếp đường tròn (O) có B, C cố định, A chuyển động trên (O). Gọi
D là một điểm cố định trong mặt phẳng. Gọi XY Z là tam giác pedal của D ứng với tam giác ABC.
Tiếp tuyến tại Y và Z của (XY Z) giao nhau tại T . Chứng minh rằng T chuyển động trên một đường
tròn cố định.

Qua thảo luận, tác giả Nguyễn Lê Phước có nhận xét rằng nếu cố định tam giác ABC và cho D
chuyển động trên BC thì T chuyển động trên một đường thẳng cố định. Để chứng minh bài toán này
chúng ta sẽ thông qua một bài toán khác như sau.

254
Bài 289. Cho tam giác ABC. D là điểm chuyển động trên BC. Gọi E, F lần lượt là hình chiếu vuông
góc của D trên AC và AB. Chứng minh rằng trung điểm của EF chuyển động trên một đường thẳng
cố định.

Chứng minh. Ta phát biểu một bổ đề.


Bổ đề 33. Cho tam giác ABC. Hai đường cao BB 0 , CC 0 . Các điểm E, F lần lượt nằm trên BC và
B 0 C 0 sao cho AE và AF đẳng giác trong ∠BAC. Khi đó trung điểm EF, BB 0 , CC 0 thẳng hàng.

B'
M G

F
C'
N
I L
J

K B H E C

Chứng minh.
Gọi G, H, K lần lượt là giao của AE với B 0 C 0 , AF với BC, B 0 C 0 với BC. I, J, L, M lần lượt là
trung điểm BB 0 , CC 0 , EF, AK. Phân giác ∠BAC giao IJ tại N , phân giác ∠C 0 KB giao IJ tại N 0 .
NI AI BI BB 0
Ta có AI, AJ đẳng giác trong ∠BAC nên AN là phân giác ∠IAJ. Suy ra = = = .
0 0
NJ AJ CJ CC 0
NI BB
Chứng minh tương tự, 0 = . Do đó N ≡ N 0 . Vậy N nằm trên đường thẳng Gauss của tứ
NJ CC 0
giác toàn phần BCB 0 C 0 .AK hay M, N, I, J thẳng hàng.
Do AE, AF đẳng giác trong góc A nên ta thu được tứ giác EGF H nội tiếp. Chứng minh tương tự
phần trên suy ra N nằm trên đường thẳng Gauss của tứ giác toàn phần EGF H.AK. Suy ra L, N, M
thẳng hàng.
Vậy L, I, J thẳng hàng.
Trở lại bài toán.

B'

C' E
Q
F

B C
D

Gọi B 0 , C 0 lần lượt là hình chiếu vuông góc của B, C trên AC và AB.
Gọi Q là trung điểm EF , J là điểm đối xứng của D qua Q. Ta có DEJF là hình bình hành nên
JE k DF k CC 0 .
JE DF BD B0E
Suy ra = = = . Suy ra B 0 , J, C 0 thẳng hàng.
CC 0 CC 0 BC B0C

255
Ta có AD là đường kính của (AEF ), Q là trung điểm EF nên J là trực tâm tam giác AEF . Do
đó AJ và AD đẳng giác trong ∠BAC. Áp dụng bổ đề 1 suy ra Q chuyển động trên đường thẳng nối
trung điểm của BB 0 và CC 0 .

Bài 290. Cho tam giác ABC. D là điểm chuyển động trên BC. Gọi E, F lần lượt là hình chiếu vuông
góc của D trên AC và AB. Tiếp tuyến tại E và F của đường tròn ngoại tiếp tam giác AEF giao nhau
tại T. Chứng minh rằng T chuyển động trên một đường thẳng cố định.

Q
d1
F
C
B D

L
K T
T'
d2

Chứng minh. Gọi K là giao của DE với AB, L là giao của DF với AC. Q là trung điểm EF , T 0 là
điểm đối xứng với T qua phân giác ∠BAC.
Do D là trực tâm tam giác AKL, T là giao hai tiếp tuyến tại E và F của (AD) nên T là trung
điểm KL.
Suy ra AQ và AT đẳng giác trong ∠KAL. Điều này nghĩa là A, Q, T 0 thẳng hàng.
AQ AF
Do 4AQF ∼ 4AT L nên = = cos ∠BAC.
AT AL
AQ
Suy ra = cos ∠BAC. Theo bài toán 7, Q chuyển động trên một đường thẳng cố định (ta gọi
AT 0
1
là đường thẳng d1 ). Mà T 0 là ảnh của Q qua phép vị tự tâm A tỉ số nên T 0 chuyển động
cos ∠BAC
trên ảnh của d1 qua phép vị tự trên (ta gọi là đường thẳng d2 ).
Lại có T và T 0 đối xứng với nhau qua phân giác ∠BAC nên T chuyển động trên ảnh của đường
thẳng d2 qua phép đối xứng trục trên. Vậy T chuyển động trên một đường thẳng cố định.

Nhận xét. 1. Gọi O0 là tâm đường tròn ngoại tiếp tam giác AKL. Bằng cách xét hai đường đẳng
giác AD và AO0 trong ∠BAC với phương pháp tương tự lời giải bài 8, ta có thể chứng minh O0 chuyển
động trên một đường thẳng cố định.
2. Bài toán 8 phụ thuộc vào hai yếu tố cố định là ∠BAC và đường thẳng BC. Rõ ràng phương
của đường thẳng BC xác định phương của đường thẳng T chuyển động. Tại [4], tác giả Trần Quang
Hùng đưa ra bài toán sau, cũng là mở rộng của ý b) bài 3 VMO 2016.

Bài 291. Cho tam giác ABC có trực tâm H, M là trung điểm BC. P là điểm chuyển động trên HM.
Gọi E, F lần lượt là hình chiếu vuông góc của P trên AB và AC. Tiếp tuyến tại E và F của (AEF )
cắt nhau tại T . Chứng minh rằng T chuyển động trên đường trung trực của BC.

256
A

E
H
F
P
M
B C

Chứng minh. Theo bài 8, để xác định đường thẳng T chuyển động trong bài 9 ta chỉ cần chỉ ra hai
vị trí đặc biệt của T . Rõ ràng khi P ≡ H, T ≡ M và khi P ≡ M , theo câu b) bài 3 VMO 2016 ta
thu được T nằm trên đường trung trực của BC. Vậy T luôn chuyển động trên đường trung trực của
BC.

Các cách giải khác của bài toán này bạn đọc xem tại nguồn trích dẫn trên.
Điều chúng ta quan tâm ở đây là phương của đường thẳng HM xác định phương của đường thẳng
T chuyển động vuông góc với BC. Điều này khá thú vị. Áp dụng tính chất này ta thu được bài toán
sau.

Bài 292. Cho tam giác ABC có trực tâm H, M là trung điểm BC. Qua H kẻ đường thẳng d song
song với AM . P là điểm chuyển động trên d. Gọi E, F lần lượt là hình chiếu vuông góc của P trên
HC và HB. Tiếp tuyến tại E và F của (HEF ) cắt nhau tại T . Chứng minh rằng T chuyển động trên
đường cao AH.

H
E
F

C
B M
P

Chứng minh. Ta có phương của đường thẳng HP trùng với phương của đường thẳng AM nên với mỗi
điểm P chuyển động trên d, phương của AM xác định phương của đường thẳng T chuyển động. Lại
có A là trực tâm của tam giác BHC nên theo nhận xét trên, phương của đường thẳng AM xác định
phương của đường thẳng T chuyển động vuông góc với BC. Nói cách khác T chuyển động trên một
đường thẳng vuông góc với BC. Do d đi qua H nên khi P ≡ H thì T ≡ H. Như vậy T chuyển động
trên đường cao AH.

257
Sau đây chúng ta sẽ mở rộng bài toán 8. Do T là tâm đường tròn ngoại tiếp tứ giác EF KL nên ta
có thể mở rộng theo hướng sau.

Bài 293. Cho tam giác ABC. P là điểm bất kì chuyển động trên BC. Một đường tròn qua B, C cắt
AC, AB lần lượt tại B 0 , C 0 . Qua P kẻ các đường song song với BB 0 , CC 0 , cắt AB, AC tạo thành tứ
giác XY ZT . Chứng minh rằng tứ giác XY ZT nội tiếp đường tròn tâm J và J chuyển động trên một
đường thẳng cố định.

B'
C'
E
X Y

R W
P
B C
U

S F
O2 Z

J
O1

T K

Chứng minh. Do BB 0 và CC 0 là hai đường đối song trong ∠BAC nên XZ và Y T cũng là hai đường
đối song trong ∠BAC. Suy ra tứ giác XY ZT nội tiếp đường tròn tâm J.
Qua B kẻ đường song song với CC 0 cắt AC tại K. Qua C kẻ đường song song với BB 0 cắt AB tại
L. Gọi (O1 ), (O2 ) lần lượt là đường tròn ngoại tiếp các tam giác CC 0 L và BB 0 K. BB 0 giao XZ tại R,
BB 0 giao CC 0 tại E, CC 0 giao Y T tại W , CL giao XZ tại U , CL giao BK tại F , BK giao Y T tại S.
Ta có BB 0 và XZ đối song trong ∠BAC nên tứ giác BXB 0 Z nội tiếp, suy ra RX · RZ = RB · RB 0 .
Điều này nghĩa là R nằm trên trục đẳng phương của (J) và (O2 ). Chứng minh tương tự suy ra RS là
trục đẳng phương của (J) và (O2 ), W U là trục đẳng phương của (J) và (O1 ).
BR FU BP BS
Mặt khác, BECF là hình bình hành nên = = = , suy ra RS k EF . Tương tự,
RE UC PC SF
T U k EF . Ta thu được RS k T U k EF.
Mà JO1 ⊥ T U, JO2 ⊥ RS nên J, O1 , O2 thẳng hàng. Vậy J chuyển động trên đường thẳng O1 O2
cố định.

Cũng với chú ý rằng phương của đường thẳng BC xác định phương của O1 O2 , ta tổng quát bài
toán 9 như sau.

Bài 294. Cho tam giác ABC. Một đường tròn qua B, C cắt AC, AB lần lượt tại B 0 , C 0 . BB 0 giao
CC 0 tại K. M là trung điểm BC. P là một điểm chuyển động trên KM . Qua P kẻ các đường song
song với BB 0 , CC 0 , cắt AB, AC tạo thành tứ giác XY ZT . Chứng minh rằng tứ giác XY ZT nội tiếp
đường tròn tâm J và J chuyển động trên đường trung trực của đoạn thẳng BC.

258
A

Y
F K H
X

G O1
P
B C
M
Z
J L

T O2

Chứng minh. Theo bài toán 11, ta chỉ cần chỉ ra hai vị trí của P nằm trên đường trung trực của đoạn
thẳng BC.
Rõ ràng khi P ≡ K thì J ≡ O1 là tâm đường tròn ngoại tiếp tứ giác BCEF và O1 nằm trên trung
trực của BC.
Khi P ≡ M thì do M B = M C nên áp dụng định lý con bướm suy ra JM ⊥ BC (J ≡ O2 trên
hình vẽ).
Vậy J chuyển động trên trung trực của BC.

Có thể mở rộng theo hướng khác như sau.

Bài 295. Cho tam giác ABC. P là điểm bất kì chuyển động trên BC. Trên hai cạnh AC, AB lần lượt
lấy hai điểm B 0 , C 0 cố định. Qua P kẻ đường song song với BB 0 cắt AC tại E, kẻ đường song song với
CC 0 cắt AB tại F . Tiếp tuyến tại E và F của (AEF ) giao nhau tại T . Chứng minh rằng T chuyển
động trên một đường thẳng cố định.

Chứng minh. Trước tiên ta phát biểu một bổ đề sau.


Bổ đề 34. (ERIQ mở rộng). Cho hai đường thẳng d1 và d2 , trên d1 theo thứ tự lấy các điểm
AB A0 B 0
A, B, C, trên d2 theo thứ tự lấy các điểm A0 , B 0 , C 0 sao cho = 0 0 = k. Xét phép vị tự quay f
BC BC
góc α, tỉ số i có tâm lần lượt là A, B, C: fA (A0 ) = A00 , fB (B 0 ) = B 00 , fC (C 0 ) = C 00 . Khi đó A00 , B 00 , C 00
A00 B 00
thẳng hàng và 00 00 = k.
B C
Chứng minh.

259
A
A''
A'
B''

B C''
B2 B3
B'
B1
C3

C C'
C2

C1

Xét phép tịnh tiến T− −→ : B 7→ B1 , C 7→ C1 , T−−→ : B 7→ B2 , C 7→ C2 , T− −−→ : B 0 7→ B3 , C 0 7→ C3 .


AA0 AA00 A0 A00
Ta có 4BB1 B2 ∼ 4AA0 A00 ∼ 4BB 0 B 00 nên 4BB1 B 0 ∼ 4BB2 B 00 .
B1 B 0 BB1 CC1 C1 C 0
Chứng minh tương tự suy ra 00
= = = .
B2 B BB2 CC2 C2 C 00
B2 B 00 C2 C 00
Suy ra = . Chú ý rằng (B1 B 0 , B2 B 00 ) = (C1 C 0 , C2 C 00 ) nên ∠B 00 B2 B3 = ∠C 00 C2 C3 , tức
B2 B3 C2 C3
A00 B 00 A00 B2 AB
là A00 , B 00 , C 00 thẳng hàng đồng thời 00 00 = = .
B C B2 C2 BC
Trở lại bài toán.

B'

C'
E
F
P
B C

T
L

Gọi K là giao của hai tiếp tuyến tại B và B 0 của (ABB 0 ), L là giao của hai tiếp tuyến tại C và C 0 của
(ACC 0 ). Rõ ràng ∠F T E = ∠BKB 0 = ∠C 0 LC = 180◦ − 2∠BAC nên các tam giác F T E, BKB 0 , C 0 LC
đồng dạng cùng hướng.
BF BP B0E
Chú ý rằng = = nên áp dụng bổ đề 2 suy ra L, T, K thẳng hàng. Vậy T chuyển
F C0 PC EC
động trên đường thẳng LK cố định.

Nhận xét. Ta hoàn toàn có thể sử dụng lời giải trên cho bài toán 11.

260
Ta sẽ xác định cụ thể phương của đường thẳng LK qua bài toán sau.

Bài 296. Cho tam giác ABC. Một đường tròn (J) qua B, C cắt AC, AB lần lượt tại B 0 , C 0 . BB 0 giao
CC 0 tại Q. Gọi K là giao của hai tiếp tuyến tại B và B 0 của (ABB 0 ), L là giao của hai tiếp tuyến tại
C và C 0 của (ACC 0 ). Chứng minh rằng LK ⊥ JQ.

B'

C'
Q

I
B
C
X J
E F

Chứng minh. Dựng đường tròn (L, LC) và (K, KB). Ta sẽ chứng minh JQ là trục đẳng phương của
(L) và (K).
Do QB · QB 0 = QC · QC 0 nên Q nằm trên trục đẳng phương của (L) và (K).
Kẻ đường kính AI của (AB 0 C 0 ). B 0 C 0 giao BC tại X. Ta biết rằng JQ ⊥ AX tại điểm Miquel M
của tứ giác toàn phần BCB 0 C 0 .AX nên JQ đi qua I.
1
Gọi F là giao của C 0 I với AC, E là giao của B 0 I với AB. Ta có ∠C 0 F C = 90◦ − ∠BAC = ∠C 0 LC
2
nên F nằm trên (L). Tương tự E nằm trên (K).
Lại có IF · IC 0 = IE · IB 0 nên I nằm trên trục đẳng phương của (L) và (K). Vậy QI là trục đẳng
phương của (L) và (K) hay QJ ⊥ LK.

Đến đây ta có một hướng mở rộng khác cho bài 12 như sau.

Bài 297. Cho tam giác ABC. Một đường tròn (J) qua B, C cắt AC, AB lần lượt tại B 0 , C 0 . BB 0 giao
CC 0 tại Q. P là một điểm chuyển động trên QJ. Qua P lần lượt kẻ các đường song song với BB 0 , CC 0 ,
cắt AC, AB tại E, F . Tiếp tuyến tại E và F của (AEF ) giao nhau tại T . Chứng minh rằng T chuyển
động trên một đường thẳng vuông góc với BC.

Chứng minh. Ta phát biểu và chứng minh hai bổ đề sau.


Bổ đề 35. Cho tam giác ABC nội tiếp đường tròn (O). T là điểm đối xứng với A qua O. Qua T kẻ
đường thẳng song song với AB cắt AC tại E, kẻ đường thẳng song song với AC cắt AB tại F . Q là
giao của hai tiếp tuyến tại E, F của (AEF ), P là giao của hai tiếp tuyến tại B, C của (O). Khi đó
P Q k AO.

261
A

O
I J

B
C
Q
K L
T

Chứng minh.
Gọi I, L lần lượt là trung điểm AQ, OP . Kẻ QJ ⊥ AC, QK ⊥ AB. Ta có 4F QE ∼ 4BP C và tứ
giác OQEJ nội tiếp nên ∠OJA = ∠OQE = ∠OP C. Do đó J ∈ (OP ). Tương tự K ∈ (OP ).
Suy ra JK là trục đẳng phương của (I) và (L) hay IL ⊥ JK.
Dễ thấy OJ ⊥ AB, OK ⊥ AC nên O là trực tâm tam giác AJK, suy ra AO ⊥ JK. Ta thu được
IL k AO.
Lại có AQ là đường đối trung của tam giác AEF nên AQ và AO đẳng giác trong ∠BAC, suy ra
AQ ⊥ BC hay AQ k OP . Suy ra AOLI là hình bình hành. Mà IA = IQ, LO = LP nên QP k AO.
Bổ đề 36. Cho tam giác ABC nội tiếp (O). D là một điểm chuyển động trên đường trung trực của
BC. Qua D kẻ đường thẳng cắt AC, AB lần lượt tại E, F sao cho D là trung điểm của EF . Gọi X là
giao của hai tiếp tuyến tại E, F của (AEF ). Khi đó X chuyển động trên một đường thẳng song song
với AO.

262
A

L
O

F D K
E
M
B C

Chứng minh.
Gọi M là trung điểm BC. Qua O kẻ đường thẳng cắt AC, AB lần lượt tại K, L sao cho O là trung
điểm KL. Gọi Y là giao của hai tiếp tuyến tại K, L của (AKL), Z là giao của hai tiếp tuyến tại B, C
của (O). Ta có 4LY K ∼ 4F XE ∼ 4BZC.
Lại có O, D, M lần lượt là trung điểm của LK, EF, BC và O, D, M thẳng hàng nên theo định lý
LF KE
ERIQ, = . Áp dụng bổ đề 2 suy ra X, Y, Z thẳng hàng.
FB EC
Theo bổ đề 3, Y Z k AO nên X chuyển động trên đường thẳng Y Z k AO.
Trở lại bài toán.

263
A

B"

O
K
C'
W
S
Q
X
J
Y
C
B

Gọi K, L lần lượt là giao của JQ với (AB 0 C 0 ), (ABC). Do JQ đi qua điểm Miquel của tứ giác toàn
phần BCB 0 C 0 nên K, L lần lượt là điểm đối xứng với A qua tâm ngoại tiếp hai tam giác AB 0 C 0 và
ABC.
Qua L kẻ đường song song với CC 0 cắt AB tại X, kẻ đường song song với BB 0 cắt AC tại Y . C 0 K
cắt BB 0 tại W , BL cắt CC 0 tại U .
C 0K LU C 0X
Ta có C 0 K k BL nên theo định lý Thales, = = . Suy ra KX k BB 0 . Tương tự
KW LB XB
KY k CC 0 . Ta thu được KXLY là hình bình hành. Mà OJ là đường trung bình của tam giác AKL
nên J là trung điểm KL, suy ra J là trung điểm XY.
Gọi S là giao của hai tiếp tuyến tại B 0 và C 0 của (AB 0 C 0 ), R là giao của hai tiếp tuyến tại X và
Y của (AXY ). Do J nằm trên trung trực của B 0 C 0 và J là trung điểm XY nên áp dụng bổ đề 4 suy
ra SR k AK.
Mà AK ⊥ BC nên SR ⊥ BC. Như vậy khi P chuyển động trên JQ, T chuyển động trên đường
thẳng SR ⊥ BC.

Nhận xét. Một điều khá thú vị ta thu được từ hai bài 14 và 15 là nếu P chuyển động trên BC
thì phương của đường thẳng T chuyển động là phương vuông góc với JQ còn nếu P chuyển động trên
JQ thì phương của đường thẳng T chuyển động là phương vuông góc với BC.
Cuối cùng chúng ta sẽ thay đổi giả thiết của bài toán 12.

Bài 298. Cho tam giác ABC và một điểm P bất kì trong mặt phẳng. Một đường tròn (O0 ) chuyển
động đi qua B, C cắt AC, AB tại B 0 , C 0 . Qua P kẻ đường song song với BB 0 , CC 0 cắt AB, AC tạo
thành tứ giác XY ZT. Chứng minh rằng tứ giác XY ZT nội tiếp đường tròn (J) và J chuyển động
trên một đường thẳng cố định.

264
A

M Y1
B'

X1 Y
C' Y2

X
P
X2
B C
O'
J Z

Chứng minh. Ta chọn hai vị trí cố định của X, Y là X1 , X2 và Y1 , Y2 .


Khi đó ∠P X1 B = ∠P Y1 C, ∠P X2 B = ∠P Y2 C. Theo giả thiết, ∠P XB = ∠P Y C.
X1 X Y1 Y
Do đó 4X1 P X2 ∪ X ∼ 4Y1 P Y2 ∪ Y . Suy ra = .
XX2 Y Y2
PX/(AX1 Y1 ) XX1 Y Y1 PY /(AX1 Y1 )
Suy ra = = = hay (AXY ) đi qua giao điểm M của hai đường
PX/(AX2 Y2 ) XX2 Y Y2 PY /(AX2 Y2 )
tròn cố định (AX1 Y1 ) và (AX2 Y2 ). Tương tự (AT Z) đi qua M . Vậy điểm Miquel của tứ giác XY ZT
là điểm cố định. Suy ra J chuyển động trên đường thẳng M P cố định.

Trên đây là những tìm tòi của tác giả về những vấn đề xung quanh bài 3 VMO 2016. Bạn đọc có
thể nhận ra một phương pháp rất hay trong các bài toán chứng minh yếu tố cố định là chọn ra một
số vị trí đặc biệt rồi chứng minh điểm cần tìm nằm trên đường đi qua các vị trí đặc biệt đó. Bạn đọc
cũng có thể phát triển tiếp mô hình và tự tìm ra những bài toán của riêng mình. Bài viết xin được
dừng lại ở đây.

28 Hai bài hình trong kì thi Việt Nam TST 2016


Bài 3. Cho tam giác ABC nội tiếp đường tròn (O) có B, C cố định, A chuyển động trên cung BC
của (O). Các phân giác AD, BE, CF giao nhau tại I. Đường tròn qua D tiếp xúc với OA tại A cắt
(O) tại G. GE, GF giao (O) lần thứ hai tại M, N . BM giao CN tại H.
a) Chứng minh rằng AH đi qua một điểm cố định.
b) BE, CF giao (O) lần lượt tại K, L. AH giao KL tại P . Q là một điểm trên EF sao cho QP = QI.
J là điểm nằm trên (BIC) sao cho IJ ⊥ IQ. Chứng minh rằng trung điểm IJ chuyển động trên một
đường tròn cố định.

265
Z M
R
A K

Y'
N W
P
L E
U H Q
F O M'
I J

V T X B Y D C

G
S

Chứng minh. a) Thực ra câu này cần chứng minh H thuộc phân giác AD và AD luôn đi qua trung
điểm cung BC cố định.
Do đường tròn Apollonius trực giao với (O) nên đường tròn đi qua D và tiếp xúc với (O) tại A
chính là đường tròn A-Apollonius của tam giác ABC. Tâm của đường tròn là X-giao của tiếp tuyến
tại A với BC. Ta có XA2 = XG2 = XB.XC nên tứ giác ABGC điều hòa.
Gọi H 0 là giao của AD với EF . Ta có (AIH 0 D) = −1 suy ra B(AIH 0 D) = −1.
Gọi W là giao của BH 0 với AC suy ra (AEW C) = −1 ⇒ M (AEW C) = −1 = (AGBC). Do đó
BM đi qua W hay BM đi qua H 0 . Tương tự CN đi qua H 0 . Vậy H 0 ≡ H và H ∈ AD.
b) Cách 1.
Gọi Z là trung điểm cung BC chứa A. AZ giao BC tại V.
Ta có A(XLF I) = (ALBS) = C(ALBS) = (AIDS) = B(AIDS) = (AKCS) = A(XKEI).
Suy ra AX, KL, EF đồng quy tại U . Mà KL là trung trực AI nên U A = U I. Suy ra ∠U IA =
∠U AI = 1/2A + C = ∠ADX. Do đó U I k BC.
Gọi Y, Y 0 là giao của đường thẳng qua I vuông góc với OI với BC và KL. Y Y 0 cắt AZ tại R. Y Y 0
giao EF tại Q0 .
Áp dụng định lý con bướm cho (O), điểm I với hai dây BK và CL đi qua I ta có IY = IY 0 . Mà
Y A = Y 0 I, ∠RAI = 90◦ nên Y 0 R = Y 0 I = Y I.
0
TV 1
Gọi T là giao của U Y 0 với BC. Ta có T Y 0 k V R nên = . U I là đường trung bình của tam
TY 2
giác T Y Y 0 nên U T = U Y 0 . Mặt khác (V DBC) = −1 nên EF đi qua V . Áp dụng định lý Menelaus cho
1
tam giác T Y Y 0 với 3 điểm V, U, Q0 suy ra Q0 Y = Q0 Y hay Q0 là trung điểm IY 0 . Suy ra P Q0 = IQ0
3
hay Q0 ≡ Q.
Vậy O ∈ IJ. Gọi M 0 là trung điểm IJ ta có ∠OM 0 S = 90◦ nên M 0 nằm trên (OS) cố định.
Cách 2 (Đào Vũ Quang, HS lớp 12 THPT Hà Nội-Amsterdam).

266
Ib

Ic
K

V P
L
Q E
F R

S I O

T W
G X

B U C

Gọi Ib , Ic lần lượt là tâm bàng tiếp góc B, C. EF cắt (O) tại R, S. RI, SI cắt (O) lần thứ hai
tại T, W . T W cắt BI tại G. Đường thẳng qua I vuông góc với OI cắt LK, BC, T W, SR lần lượt tại
V, U, X, Q0 .
Ta có tứ giác AICIb nội tiếp nên EA · EC = EI · EIb , suy ra E nằm trên trục đẳng phương của (O)
và (Ib IIc ). Tương tự với F. Do đó EF là trục đẳng phương của hai đường tròn, suy ra R, S ∈ (Ib IIc ).
Ta có ∠GT R = ∠W SR = ∠IIb R, suy ra tứ giác GT Ib R nội tiếp. Ta thu được IG · IIb = IT · IR =
1
IB · IK = IB · IIb . Suy ra G là trung điểm IB.
2
Tương tự suy ra T W là đường trung bình của tam giác IBC. Áp dụng định lý con bướm cho
(O), điểm I và hai dây BK, CL đi qua I ta có IV = IU . Lại áp dụng định lý con bướm cho hai dây
SW và RT đi qua I suy ra IQ = IX. Mà X là trung điểm IU nên Q0 là trung điểm IV . Suy ra
Q0 P = Q0 I = Q0 V hay Q0 ≡ Q. Vậy OI ⊥ IQ. Phần còn lại làm tương tự cách 1.

Nhận xét.
- Dựa theo ý tưởng sử dụng tỉ số kép, câu 3a có thể thay I bằng một điểm bất kì trên phân giác
góc A.
- Bản chất của câu 3b là OI ⊥ IQ. Với bài toán chứng minh đường vuông góc đi qua tâm đường
tròn ngoại tiếp, định lý con bướm tỏ ra rất hữu hiệu.
Một số mở rộng cho câu 3b.
Bài 3.1. Cho tam giác ABC nội tiếp (O). (X) là một đường tròn bất kì qua B, C. Phân giác góc
A cắt (X) tại I. BI, CI cắt AC, AB tại E, F. P là điểm chia AI theo tỉ lệ k không đổi. Trung trực P I
cắt EF tại Q. J thuộc (X) sao cho IJ vuông góc IQ. Khi đó trung điểm IJ thuộc một đường tròn cố
định.
Bài 3.2 Với giả thiết như bài 3.1, E, F có thể xác định bằng cách khác: gọi (Ob ), (Oc ) là đường
tròn ngoại tiếp các tam giác AIC, AIB. IOb cắt AC tại E, IOc cắt AB tại F . Kết quả thu được tương
tự.
Bài 3.3. Cho tam giác ABC nội tiếp (O). P là một điểm bất kì trên phân giác. BP, CP cắt
AC, AB tại E, F , cắt (O) tại Y, Z và cắt (AP C), (AP B) lần lượt tại T, R. Gọi Q là điểm liên hợp đẳng
giác của P trong tam giác ABC. (BQC) cắt P C, P B tại Z 0 , Y 0 . Khi đó tồn tại một điểm S trên AP
SA RZ TY
sao cho = = và gọi giao của trung trực P S với EF là J thì P J ⊥ P O.
SP ZZ 0 YY0
Câu 3b có cấu hình tương tự một số bài toán sau.
Bài 299. Cho tam giác ABC nội tiếp đường tròn (O), ngoại tiếp đường tròn (I). Qua I kẻ đường
vuông góc với OI cắt phân giác ngoài góc A tại X, cắt BC tại Y. Chứng minh rằng IX = 2IY.

267
Bài 300. (Chọn đội tuyển Đà Nẵng 2015) Cho tam giác ABC nội tiếp đường tròn (O, R), ngoại tiếp
đường tròn (I, r). Một đường tròn ω đi qua A và tiếp xúc với đường thẳng OI tại I. AO cắt ω lần thứ
hai tại G. Chứng minh rằng AG = 2r.

Bài 301. (Taiwan TST 2014) Cho tam giác ABC nội tiếp đường tròn (O), ngoại tiếp đường tròn (I).
Kẻ tiếp tuyến d của (I) song song với BC. OI giao d tại X, qua I kẻ đường vuông góc với OI giao d
tại Y . Chứng minh rằng tứ giác AXOY nội tiếp.

Bằng cách đổi cấu hình từ tâm nội tiếp thành trực tâm, tác giả Nguyễn Lê Phước trên diễn đàn
Bài toán hay-Lời giải đẹp đã phát biểu lại bài toán theo cách khác như sau.

Bài 302. (Nguyễn Lê Phước). Cho tam giác ABC nội tiếp (O), đường cao AD, trực tâm H. Đường
tròn Euler của tam giác ABC cắt đường tròn (BHC) tại P và Q. Tia HP, HQ cắt (O) lần lượt tại
K, L. Qua H kẻ đường vuông góc với OH cắt KL tại M . Chứng minh rằng M H = M D.
∠BAC
Bài 4. Cho tam giác ABC nhọn có ∠ACB < ∠ABC < ∠ACB + . Lấy điểm D thuộc cạnh
2
∠BAC
BC sao cho ∠ADC = ∠ACB + . Tiếp tuyến với đường tròn ngoại tiếp tam giác ABC tại A
2
cắt BC tại E. Phân giác ∠AEB cắt AD và cắt (ADE) tại G và F , DF giao AE tại H.
a) Chứng minh rằng các đường tròn đường kính AE, DF, GH có một điểm chung.
b) Trên phân giác ngoài ∠BAC và trên tia AC lần lượt lấy các điểm K và M sao cho KB =
KD = KM , trên phân giác ngoài ∠BAC và trên tia AB lần lượt lấy các điểm L và N sao cho
LC = LD = LN. Đường tròn đi qua M, N và trung điểm I của BC cắt BC tại P (P 6= I). Chứng
minh rằng BM, CN, AP đồng quy.

B'
H

L
A
K

F M
G
O

J
E P B Z I C
D S R
Y N

268
Chứng minh. a) Một lần nữa chúng ta lại thấy bóng dáng của định lý Brocard.
Bổ đề. Cho tứ giác ABCD nội tiếp (O), AC giao BD tại P, AB giao CD tại E, AD giao BC tại
F . Gọi M là điểm Miquel của tứ giác toàn phần ABCD.EF. Khi đó O, P, M thẳng hàng và M là hình
chiếu của O trên EF.
Đây là một bổ đề rất quen thuộc xin dành cho bạn đọc tự chứng minh.
1
Quay lại bài toán. Ta có ∠AEB = ∠ABC −∠ACB nên ∠DAF = ∠DEF = (∠ABC −∠ACB) =
2
◦ 1 ◦
90 − (∠ACB + ∠BAC) = 90 − ∠ADC. Do đó AF ⊥ BC tại S.
2
Suy ra (AE), (DF ) lần lượt là (ASE), (DSF ). Gọi J là giao điểm thứ hai của (ASE) và (DSF ).
Khi đó J là điểm Miquel của tứ giác toàn phần AEDF.SH. Theo bổ đề trên ta có J ∈ SH và
∠GJH = 90◦ . Suy ra (AE), (DF ), (GH) đồng quy tại J.
b) Cách 1. Gọi B 0 đối xứng với B qua AK suy ra B 0 ∈ AC và B 0 ∈ (K). Ta có ∠BDM = ∠BB 0 M =
∠BAC/2 = ∠CAR (R là chân phân giác trong). Suy ra ADRM nội tiếp. Tương tự N ∈ (ADR).
Dễ thấy EA = ER nên F A = F R. Mà F A = F D nên F là tâm (ADR). Do tam giác ADR cân
nên AF ⊥ BC. Ta có ∠F RA = ∠F AR = ∠RAO, suy ra F R k AO. Mà AR là phân giác ∠N AM nên
R là điểm chính giữa cung M N , suy ra F R ⊥ M N . Vậy M N ⊥ AO hay M N đối song với BC trong
tam giác ABC. Suy ra tứ giác BN CM nội tiếp.
Gọi Y là giao của BM và CN , Z là giao của M N với BC. Ta có ZP.ZI = ZM.ZN = ZB.ZC,
suy ra (P ZBC) = −1, suy ra P ∈ AY. Ta có đpcm.
Cách 2. (Chỉnh sửa lại cách của Trần Quốc Luật, GV THPT chuyên Hà Tĩnh).

L
A
K

B
D X S R Y C
N

1
AK SX BR BR AB
Gọi X, Y lần lượt là hình chiếu của K, L trên BC. Ta có = = 21 = = .
AL SY 2 CR
CR AC
AK KB
Do đó 4KAB ∼ 4LAC (c.g.c). Ta thu được = hay A là tâm vị tự trong của (K) và
AL LC
(L), ngoài ra A cũng là tâm nghịch đảo của hai đường tròn này.
Lại có hai cát tuyến AM C và AN B đi qua A nên tứ giác BN CM nội tiếp.
Phần còn lại làm tương tự cách 1.

Nhận xét.
- Một tổng quát cho câu 4a.

Bài 303. Cho tam giác ABC. P, Q là hai điểm bất kì trên BC sao cho AP = AQ. Qua A kẻ đường
vuông góc với AQ cắt BC tại K. Gọi E là trung điểm KQ. Phân giác ∠AEP cắt (AP E) tại M và cắt
AP tại N . P M cắt AE tại L. Chứng minh rằng các đường tròn đường kính AE, P M, N L đồng quy.

269
- Câu 4b cũng khá hay và không dễ so với vị trí bài số 4.
Một tổng quát cho câu 4b.

Bài 304. Cho tam giác ABC. Hai điểm P, Q bất kì trên BC sao cho AP = AQ. (AP Q) cắt AC, AB
lần lượt tại E, F . Gọi (O1 ), (O2 ) lần lượt là đường tròn ngoại tiếp các tam giác BP E, CP F . Chứng
minh rằng A là tâm vị tự trong của (O1 ) và (O2 ).

29 Từ một bổ đề về đường thẳng Euler


Chúng ta cùng xem xét bài toán quen thuộc sau.

Bài 305. Cho tam giác ABC không vuông. Gọi D là điểm thỏa mãn ∠DBA = ∠BAC = ∠DCA. Khi
đó D nằm trên đường thẳng Euler của tam giác ABC.

Chứng minh. Cách 1.

E
F
H

K L

A
M N
C
B
O

Gọi E là giao của AB và CD, F là giao của AC và BD. Khi đó hai tam giác F AB và EAC lần
lượt cân tại F và E. Gọi M, N lần lượt là trung điểm của AB, AC suy ra F M giao EN tại tâm ngoại
tiếp O của tam giác ABC.
Gọi K, L lần lượt là hình chiếu của B trên AC, C trên AB. BK giao CL tại trực tâm H của tam
giác ABC.
Xét hai đường tròn đường kính BF và CE. Ta có HK ·HB = HL·HC nên PH /(BF ) = PH /(CE).
Do tứ giác F M N E nội tiếp đường tròn đường kính EF nên OF · OM = OE · ON hay PO /(BF ) =
PO /(CE).
Ta có ∠F BE = ∠F CE nên tứ giác F BCE nội tiếp, suy ra DB · DF = DC · DE hay PD /(BF ) =
PD /(CE).
Vậy H, O, D cùng nằm trên trục đẳng phương của (CE) và (BF ) hay D nằm trên đường thẳng
Euler của tam giác ABC.
Cách 2.

270
E
F

A
M G N

C
B
O

Gọi E, F lần lượt là giao điểm của AB và CD, AC và BD; M, N lần lượt là trung điểm AB, AC.
Do ∠A = ∠B = ∠C nên hai tam giác ABF và ACE lần lượt cân tại F, E.
Suy ra F M giao EN tại O là tâm đường tròn ngoại tiếp của tam giác ABC.
Gọi G là trọng tâm của tam giác ABC thì BN giao CM tại G.
Áp dụng định lý Pappus cho hai bộ ba điểm (E, M, B) và (F, N, C) ta có giao điểm của các cặp
đường thẳng F B và EC, F M và EN , BN và CM lần lượt là D, O, G thẳng hàng hay D nằm trên
đường thẳng Euler của tam giác ABC.

Nhận xét. Bài toán 1 khá nổi tiếng, bạn đọc có thể tìm thấy trong một số tài liệu như [1] hoặc [2].
Qua cách dựng điểm D, ta còn nhận thấy trong tam giác ABC còn hai điểm khác có tính chất giống
như D: điểm E và F lần lượt thỏa mãn ∠EAB = ∠ABC = ∠ECB và ∠F AC = ∠ACB = ∠F BC.
Khi ta viết ba góc bằng nhau, bạn đọc nên hiểu rằng đó là các góc định hướng giữa hai đường
thẳng. Tuy nhiên để bài viết đơn giản và tiện quan sát, tác giả viết theo góc thường.
Tiếp theo chúng ta đến với một số ví dụ.

Bài 306. Cho tam giác ABC nội tiếp đường tròn (O), E là tâm Euler. M, N là hai điểm trên AB, AC
sao cho E là trung điểm M N . Chứng minh rằng O nằm trên đường thẳng Euler của tam giác AM N .

N
H E
M O

B C

Chứng minh. (Nguyễn Tuấn Hải Đăng, HS THPT chuyên KHTN)


Gọi L là điểm đối xứng với A qua E. Suy ra L đối xứng với O qua BC.

271
Ta có AM LN là hình bình hành nên ∠LN C = ∠BAC = ∠LOC. Suy ra tứ giác ON CL nội tiếp,
từ đó ∠AN O = ∠OLC = ∠LOC = ∠BAC. Tương tự ta thu được ∠ON A = ∠N AM = ∠AM O. Theo
bài toán 1, O nằm trên đường thẳng Euler của tam giác AM N.

Bài 307. (Geometry Mathley 2011-2012). Cho tam giác ABC nội tiếp đường tròn (O). E là tâm
Euler. Qua E kẻ các đường thẳng M N song song với BC, P Q song song với AC, KL song song với
AB (K, P ∈ BC, N, L ∈ AC, M, Q ∈ AB). Chứng minh rằng đường thẳng Euler của các tam giác
ALQ, BM K, CP N, ABC đồng quy.

G' L

Q
G
R
H N
M
E
S O
X

B K P C

Chứng minh. Trên AB, AC lấy các điểm S, R sao cho E là trung điểm SR. Theo bài 2, O nằm trên
đường thẳng Euler của tam giác ASR.
Dễ thấy tam giác ELQ là tam giác trung tuyến của tam giác ARS nên đường thẳng Euler của các
tam giác ELQ và ARS trùng nhau. Gọi G là trọng tâm tam giác ELQ thì GO là đường thẳng Euler
của tam giác ELQ.
Gọi G0 là trọng tâm tam giác ALQ thì G0 đối xứng với G qua trung điểm LQ, từ đó G là trung
điểm G0 E.
Do hai tam giác ALQ và ELQ đối xứng nhau qua trung điểm LQ nên đường thẳng Euler của hai
tam giác song song với nhau. Gọi X là điểm đối xứng với E qua O thì G0 X k GO hay G0 X là đường
thẳng Euler của tam giác ALQ.
Chứng minh tương tự suy ra đường thẳng Euler của các tam giác ALQ, BM K, CP N, ABC đồng
quy tại điểm đối xứng với E qua O.

Nhận xét. Thay điều kiện song song với cạnh bằng hình chiếu vuông góc của E trên ba cạnh ta
thu được bài toán mới.

Bài 308. (Vietnam IMO Training Test 2015) Cho tam giác ABC có E là tâm đường tròn Euler. Gọi
X, Y, Z là hình chiếu của E trên BC, CA, AB. Chứng minh rằng đường thẳng Euler của các tam giác
AY Z, BXZ, CXY, ABC đồng quy.

Chứng minh. Cách 1.

272
A

Q
Oa
Y
Mc Mb
J
Z

E L O
H

B
Ma
C

Gọi O là tâm ngoại tiếp tam giác ABC, L là trung điểm EO. ta sẽ chứng minh đường thẳng Euler
của các tam giác AY Z, BXZ, CXY đều đi qua L.
Gọi Oa là tâm ngoại tiếp tam giác AY Z. J là trung điểm Oa L. EJ cắt AO tại Q.
Ta có Oa là trung điểm AE nên Oa L k AO. Từ đó thu được Q là trung điểm AO.
Gọi Ma , Mb , Mc lần lượt là trung điểm BC, CA, AB. Ta có Q và E lần lượt là tâm ngoại tiếp các
tam giác AMb Mc và Ma Mb Mc nên Q và E đối xứng nhau qua Mb Mc . Từ đó J là trung điểm Mb Mc
và EJ ⊥ Mb Mc .
Suy ra tứ giác EJY Mb nội tiếp, ta thu được ∠JY A = ∠JEMb = ∠Mb Ma Mc = ∠BAC.
Tương tự suy ra ∠AY J = ∠AZJ = ∠Y AZ. Áp dụng bài toán 1 suy ra J nằm trên đường thẳng
Euler của tam giác AY Z. Vậy Oa L là đường thẳng Euler của tam giác AY Z. Chứng minh tương tự
ta có đpcm.
Cách 2 (Lê Thị Hải Linh, HS THPT chuyên Bắc Ninh).
A
Hb
T K
R Y
Hc Oa
H Q
Z M
Mc J Mb

L
O

B Ma C

Gọi Q, Qa , J, L lần lượt là trung điểm AO, AE, EQ, EO; Ma , Mb , Mc lần lượt là trung điểm BC, CA, AB.
Ta có (Ma Mb Mc ) và (AMb Mc ) đối xứng qua Mb Mc nên các tâm ngoại tiếp E và Q đối xứng qua Mb Mc ,
suy ra J là trung điểm Mb Mc . Hiển nhiên J là trung điểm Oa L.
Gọi Hb , Hc là chân đường cao kẻ từ B, C. R, M lần lượt là trung điểm Hb Hc , XY .
Hb Y Hc Z
Do = = 1 và R, M, J lần lượt là trung điểm Hb Hc , Y Z, Mb Mc nên theo định lý ERIQ,
Y Mb ZMc
M là trung điểm RJ.
Ta có ER ⊥ Hb Hc , AO ⊥ Hb Hc , Oa L k AO nên ER k Oa L k AO.
Gọi K là giao của M Oa với AO.

273
Kí hiệu dA/l là khoảng cách từ A đến đường thẳng l. Do EL = LO nên dR/AO = 2dJ/AO . M là
3
trung điểm RJ nên dM/AO = dJ/AO . Từ đó Oa K = 2Oa M.
2
Gọi T là trực tâm tam giác AY Z thì AT k= 2Oa M hay AT k= Oa K, suy ra T Oa k AO. Từ đó
T, Oa , L thẳng hàng hay đường thẳng Euler của tam giác AY Z đi qua trung điểm EO. Chứng minh
tương tự ta có đpcm.

Tiếp theo chúng ta đến với một bài toán của tác giả trong đợt tập huấn IMO năm 2015.

Bài 309. Cho tứ giác ABCD có ∠A = ∠C = 120◦ . Phân giác góc A và góc C giao nhau tại P . Chứng
minh rằng đường thẳng Euler của 10 tam giác có đỉnh là 3 trong 5 điểm A, B, C, D, P đồng quy.

A C
P L
K T
O1 G M O2

X Y

Chứng minh. Gọi G, L lần lượt là trọng tâm tam giác BP D, BCD; M là trung điểm BD, O1 , O2 lần
lượt là tâm ngoại tiếp các tam giác BCD, BCA. CP cắt (O1 ) lần thứ hai tại K. Dễ thấy tam giác
M O1 MG ML 1
KBD đều nên O1 là trọng tâm tam giác KBD. Từ đó = = = . Suy ra O1 , G, L
MK MP MC 3
thẳng hàng hay đường thẳng Euler của tam giác BCD đi qua trọng tâm tam giác BP D. Chứng minh
tương tự với các tam giác BAD, ABP, ADP, BCP, DP C.
Như vậy ta cần chứng minh G nằm trên đường thẳng Euler của các tam giác ABC, ADC, AP C.
Gọi Y, X là giao của AG với (O2 ), CG với (O1 ), AP cắt (O2 ) lần thứ hai tại T .
Ta có GO1 k CK nên theo định lý Reim, X, O1 , G, O2 đồng viên.
Tương tự O1 , G, O2 , Y đồng viên. Như vậy 5 điểm X, O1 , G, O2 , Y cùng nằm trên ω.
Mà O1 X = O2 Y nên XY k KT . Từ đó ∠BAY = ∠BCX.
Đặt ∠P AG = ∠P CG = x. Hiển nhiên số đo các cung XO1 , O1 O2 , O2 Y của ω đều bằng 2x. Do đó
∠XGY = 180◦ − 3x.
Mà ∠XGY = 360◦ − ∠ABC − ∠BAG − ∠BCG = 360◦ − 2x − 120◦ − ∠ABC.
Do đó ∠ABC = 60◦ + x = ∠BAG = ∠BCG.
Áp dụng bài toán 1 suy ra G nằm trên đường thẳng Euler của tam giác ABC. Tương tự với tam
giác ADC. Bằng cộng góc cũng suy ra ∠AP C = 180◦ − x = 180◦ − ∠P AG = 180◦ − ∠P CG. Suy ra
G nằm trên đường thẳng Euler của tam giác AP C. Ta có đpcm.

Bài 310. Cho tam giác ABC không cân có l là phân giác góc A. Chứng minh rằng l song song với
đường thẳng Euler của tam giác ABC khi và chỉ khi ∠BAC = 120◦ .

274
H
E
E

F
F
N
A R
Q
A
M

B C
B O C

Chứng minh. - Nếu ∠BAC = 120◦ . Về phía ngoài tam giác dựng các tam giác đều ABF và ACE. Khi
đó d k BF k EC. Theo cách chứng minh bài toán 1, ta biết rằng BF giao CE tại một điểm nằm trên
đường thẳng Euler của tam giác ABC. Do đó các đường thẳng l, BF, CE và đường thẳng Euler của
tam giác ABC đôi một song song.
- Nếu l song song với đường thẳng Euler của tam giác ABC.
Gọi E, F lần lượt là giao của đường trung trực AB với AC, đường trung trực AC với AB. Giả sử
BF và CE không song song. Khi đó BF giao CE tại P . Gọi M, N lần lượt là trung điểm BF, CE.
M N giao AC, AB lần lượt tại Q, R.
BF CE
Theo cách giải 1 của bài toán 1, OP là trục đẳng phương của các đường tròn (M, ) và (N, ).
2 2
Do đó M N ⊥ OP . Mà OP là đường thẳng Euler của tam giác ABC nên l ⊥ M N. Suy ra tam giác
ARQ cân tại A. Mà hai tam giác F BA và ECA đồng dạng nên ∠BM Q = ∠CN R.
Suy ra tam giác P M N cân tại P hay P M = P N . Mặt khác P nằm trên trục đẳng phương của
hai đường tròn (M ), (N ) nên F B = EC. Điều này vô lý do hai tam giác F BA và ECA đồng dạng và
AB 6= AC.
Do đó BF k EC. Suy ra ∠AEC = ∠ABF = ∠EAC = ∠ECA hay AEC và AF B là hai tam giác
đều. Suy ra ∠BAC = 120◦ .

Tiếp theo là một bài toán từ kì thi Tuymaada- Republic of Saka, Russia.

Bài 311. (Tuymaada 2009) Cho tam giác ABC nội tiếp đường tròn (O). Gọi B1 là điểm đối xứng
với B qua AC, C1 là điểm đối xứng với C qua AB, Oa là điểm đối xứng của O qua BC. Chứng minh
rằng tâm đường tròn ngoại tiếp tam giác AB1 C1 nằm trên AOa .

275
J

B1

C1

A
Oc
Ob

B C
Oa

Chứng minh. (Đinh Ngọc Tùng, HS THPT chuyên Hạ Long, Quảng Ninh)
Gọi J, Ob , Oc lần lượt là tâm đường tròn ngoại tiếp các tam giác AB1 C1 , ACB1 và ABC1 . Do phép
đối xứng ta có O và Ob đối xứng qua AC, O và Oc đối xứng qua AB. Do đó AO = AOb = AOc hay A
là tâm đường tròn ngoại tiếp tam giác OOb Oc . Dễ thấy Oa là trực tâm tam giác OOb Oc nên AOa là
đường thẳng Euler của tam giác OOb Oc .
(Oc ) và (J) cắt nhau tại A và C1 nên ∠JOc O = 180◦ − ∠C1 AB = 180◦ − ∠BAC = ∠Oc OOb .
Chứng minh tương tự suy ra ∠JOc O = ∠Oc OOb = ∠JOb O. Áp dụng bài toán 1 suy ra J nằm trên
đường thẳng Euler của tam giác OOb Oc hay J ∈ AOa .

Nhận xét. Thực hiện phép đổi trực tâm thành đỉnh tam giác, ta được một bài toán trong kì thi
APMO 2010.

Bài 312. (APMO 2010). Cho tam giác nhọn ABC nội tiếp đường tròn (O) với H là trực tâm. Đường
tròn ngoại tiếp tam giác AHC cắt AB lần thứ hai tại M , đường tròn ngoại tiếp tam giác AHB cắt
AC lần thứ hai tại N . Chứng minh rằng tâm đường tròn ngoại tiếp tam giác HM N nằm trên OH.

M
Oc Ob
J

H
O

B C

Oa

Chứng minh. Bản chất của bài toán này giống bài 7. Do đó ta hoàn toàn sử dụng được cách tương tự
như sau.

276
Gọi J, Oa , Ob , Oc lần lượt là tâm đường tròn ngoại tiếp các tam giác HM N, BHC, CHA, AHB.
Dễ thấy OH cũng đồng thời là đường thẳng Euler của tam giác Oa Ob Oc .
Do (J) và (Ob ) giao nhau tại H và M , (Ob ) và (Oa ) giao nhau tại H và C nên ∠JOb Oa = 180◦ −
∠M HC = ∠BAC = ∠Ob Oa Oc . Chứng minh tương tự suy ra ∠JOb Oa = ∠Ob Oa Oc = 180◦ − ∠JOc Oa .
Do đó theo bổ đề 1, J nằm trên OH.

Bài 313. Cho tam giác ABC. Ia là tâm đường tròn bàng tiếp góc A. Gọi M là điểm đối xứng với Ia
qua BC. Chứng minh rằng AM song song với đường thẳng Euler của tam giác Ia BC.
A
M

R
O
I
P
E

B K
G C
F

N
J
Q

Ia

Chứng minh. Gọi E, F lần lượt là trung điểm các cung ACB, ABC của đường tròn (O). Suy ra E ∈ CIa
và F ∈ BIa . Gọi G là giao điểm của BE và CF .
Ta có ∠Ia F C = ∠BAC = 180◦ − 2∠BIa C nên F Ia = F C hay ∠GCIa = ∠CIa B. Chứng minh
tương tự suy ra ∠GCIa = ∠CIa B = ∠GBIa . Áp dụng bài toán 1, G nằm trên đường thẳng Euler của
tam giác BCIa .
Gọi K là giao điểm của EF và BC, J là điểm chính giữa cung BC, AK, JK giao (O) lần thứ hai
tại N, R.
Áp dụng định lý Brocard cho tứ giác BF CE, ta có O là trực tâm tam giác GKIa . Lại áp dụng
định lý Brocard lần thứ hai cho tứ giác ARN J, suy ra AR giao JN tại G.
Do EF là đường trung trực của AIa và BC là đường trung trực của Ia M nên K là tâm đường tròn
ngoại tiếp tam giác AIa M .
1
Ta có ∠AN G = ∠ACJ = ∠C + ∠A, ∠N AM = ∠KAM = 90◦ − ∠AIa M = 90◦ − |∠Ia BC −
2
∠Ia CB|.
Bằng một số phép tính góc đơn giản ta thu được ∠AN G = ∠N AM hay AM k GJ.

Nhận xét. Ta có thể suy ra một tính chất khá thú vị là G nằm trên đường thẳng OI với I là tâm
đường tròn nội tiếp tam giác ABC như sau.
Gọi P là giao của Ia K với OG, Q là giao của OK với GIa . Do O là trực tâm tam giác GKIa nên
∠GP Ia = ∠KQIa = 90◦ . Do GIa là đường đối cực của K ứng với (O) nên KP · KIa = KO · KQ =
PK/(O) = KB · KC. Do đó P ∈ (BCIa ). Suy ra GP đi qua điểm đối xứng với Ia qua tâm của (BCIa )
hay GP đi qua I. Vậy G ∈ OI.
Từ nhận xét trên, bằng cách đổi vai trò của các tâm đường tròn bàng tiếp của tam giác ABC về
các đỉnh A, B, C ta thu được hai bài toán sau.

277
Bài 314. Cho tam giác ABC. Các đường cao BB1 , CC1 . Gọi B2 , C2 lần lượt là trung điểm AC, AB.
B1 C2 giao B2 C1 tại P . Chứng minh rằng P là giao điểm của đường thẳng Euler của hai tam giác ABC
và AB1 C1 .

Bài 315. Cho tam giác ABC. Các đường cao BB1 , CC1 . Gọi B2 , C2 lần lượt là trung điểm AC, AB.
Gọi P là giao điểm của B1 C1 và B2 C2 . Chứng minh rằng AP vuông góc với đường thẳng Euler của
tam giác ABC.

Tiếp theo xin giới thiệu với bạn đọc một bài toán khá nổi tiếng về đường thẳng Euler.

Bài 316. Cho tam giác ABC. Một đường thẳng d song song với đường thẳng Euler của tam giác ABC
cắt AB, AC lần lượt tại E, F . Chứng minh rằng đường thẳng Euler của tam giác AEF song song với
BC.

Chứng minh. Ta phát biểu và chứng minh một bổ đề sau.


Bổ đề. Cho hai tam giác ABC, A0 B 0 C 0 và cặp điểm P, Q trong mặt phẳng. Biết rằng A0 B 0 k AB,
A C 0 k P C, A0 Q k AC, B 0 C 0 k BP , A0 C 0 k CP . Khi đó C 0 Q0 k CB.
0

A' C'

B' A T

K
L

P
B C

Chứng minh. Trên AC lấy điểm K sao cho P K k B 0 Q, đường thẳng qua A song song với BC cắt
BP tại T . BT cắt AC tại L.
Theo giả thiết ta thu được hai tam giác A0 B 0 C 0 và AP T đồng dạng với cặp điểm có vị trí tương
đương trong hai tam giác là Q, K. Như vậy ta cần chứng minh T K k BC.
Do AT k BC suy ra LT · LC = LA · LP . Mà P K k AB nên LA · LP = LK · LB. Như vậy
LK LT
LT · LC = LK · LB hay = . Vậy KT k BC hay C 0 Q0 k CB.
LC LB
Trở lại bài toán.
A

O' H' L
F
H O

B C
K

Gọi K, L là điểm thỏa mãn ∠KCA = ∠BAC = ∠KBA, ∠LEA = ∠EAF = ∠LF A.
Theo bài toán 1, K, L lần lượt nằm trên đường thẳng Euler OH, O0 H 0 của các tam giác ABC, AEF .

278
Do cách dựng ta thu được BK k EL, CK k F L. Lại có EH 0 k BH, F H 0 k CH, EF k HK. Áp
dụng bổ đề trên suy ra H 0 L k BC hay O0 H 0 k BC.

Nhận xét. − Theo cách giải trên ta có thể tổng quát bài toán như sau.
Tổng quát. Cho tam giác ABC và hai điểm E, F bất kì nằm trên AB, AC. Gọi H, H 0 lần lượt
là trực tâm tam giác ABC, AEF . P là điểm bất kì trong mặt phẳng. Q là điểm thỏa mãn EQ k BP ,
F Q k CP . Khi đó H 0 Q k BC.
− Ta cũng có thể thay H, H 0 trong bài toán tổng quát thành hai tâm ngoại tiếp O, O0 .
− Bài toán trên là một bổ đề quan trọng trong phép chứng minh định lý Gossard, phát biểu như
sau.
Định lý Gossard. Cho tam giác ABC. Đường thẳng Euler của tam giác ABC cắt các cạnh
BC, CA, AB lần lượt tại X, Y, Z. Chứng minh rằng đường thẳng Euler của các tam giác AY Z, BXZ,
CXY cắt nhau tạo thành một tam giác vị tự với tam giác ABC theo tỉ số −1, tâm vị tự nằm trên
đường thẳng Euler của tam giác ABC.
Để kết thúc bài viết mời bạn đọc thử sức một số bài toán sau.

Bài 317. Cho tam giác ABC. Các đường cao BB1 , CC1 . Chứng minh rằng đường thẳng Euler của
hai tam giác AB1 C1 và ABC song song khi và chỉ khi ∠BAC = 60◦ .

Bài 318. Chứng minh rằng tiếp tuyến chung của đường tròn nội tiếp và đường tròn Euler của một
tam giác song song với đường thẳng Euler của tam giác đó khi và chỉ khi có một trong ba góc của tam
giác bằng 60◦ .

Bài 319. Cho tam giác ABC có ∠A = 30◦ . Gọi E là tâm đường tròn Euler của tam giác ABC, P, Q
là hình chiếu của E trên AC, AB, M là trung điểm BC. Chứng minh rằng M P = M Q.

30 Đường tròn tiếp xúc trong tứ giác ngoại tiếp


30.1 Giới thiệu.
Trong đợt tập huấn đội tuyển chuẩn bị cho kì thi toán quốc tế năm 2014, bạn Nguyễn Huy Tùng,
HS THPT chuyên Trần Phú, Hải Phòng, đã phát hiện ra một bổ đề khá thú vị.

Bài 320. Cho tứ giác ngoại tiếp ABCD. Gọi E là giao điểm của AB và CD, F là giao điểm của
AD và BC. Khi đó tồn tại một đường tròn tiếp xúc với 4 đường tròn ngoại tiếp các tam giác
EAD, EBC, F AB, F CD.

Bài viết này sẽ đưa ra một số cách chứng minh và phát triển bài toán lạ trên.

30.2 Chứng minh.


Cách 1.

279
F

M B
X H

E I J
Y
D

Gọi M là điểm Miquel của tứ giác toàn phần ABCDEF . Từ M kẻ hai tiếp tuyến M H, M K tới
(I) sao cho H, K ∈ (EBC). Theo định lý Poncelet suy ra HK tiếp xúc với (I).
Qua I kẻ đường vuông góc với M I cắt M H, M K tại X, Y . Gọi (J) là đường tròn tiếp xúc với
M H, M K lần lượt tại X, Y thì theo bổ đề Sawayama, (J) là đường tròn M-mixtilinear của tam giác
M HK hay (J) tiếp xúc với (EBC). Chứng minh tương tự suy ra (J) tiếp xúc với các đường tròn
(EAD), (EBC), (F AB), (F CD).
Cách 2 (Ngô Quang Dương, HS 11A2 toán, THPT chuyên KHTN, Hà Nội).

B
X
A
Y
T
I
E

D
Z

Gọi X, Y, Z, T lần lượt là tiếp điểm của AB, BC, CD, DA với (I).
2
Xét phép nghịch đảo IIr biến (EAD), (EBC), (F AB), (F CD) lần lượt thành đường tròn Euler
của các tam giác T XZ, Y XZ, T XY, T ZY.

280
Do (EAD), (EBC), (F AB), (F CD) đồng quy tại điểm Miquel của tứ giác toàn phần ABCDEF
nên các đường tròn Euler của các tam giác T XZ, Y XZ, T XT, T ZY đồng quy tại J. Mặt khác các
đường tròn này đều có bán kính bằng r/2 nên đường tròn (J, r) tiếp xúc với cả 4 đường tròn Euler.
Nghịch đảo ngược lại ta có đpcm.

30.3 Khai thác.


Trước tiên chúng ta thử cho tứ giác ABCD vừa ngoại tiếp vừa nội tiếp xem có gì đặc biệt, kết quả
thu được khá thú vị.

Bài 321. Cho tứ giác lưỡng tâm ABCD có tâm đường tròn ngoại tiếp là O. Gọi E, F lần lượt là giao
điểm của AB và CD, AD và BC. Khi đó tồn tại một đường tròn tâm O tiếp xúc với bốn đường tròn
ngoại tiếp các tam giác EAD, EBC, F AB, F CD.

Chứng minh. Trước tiên ta phát biểu một bổ đề.


Bổ đề 37. Cho tứ giác ABCD nội tiếp đường tròn (O), ngoại tiếp đường tròn (I). AC giao BD tại
P . Khi đó O, I, P thẳng hàng.
Chứng minh.

B
C1
D1
A

P
I
O
L

B1

A1
C

Gọi A1 , B1 , C1 , D1 lần lượt là giao điểm của AI, BI, CI, DI với (O). Bằng một số phép cộng góc
đơn giản dễ thấy A1 C1 và B1 D1 là các đường kính của (O).
Gọi L là giao điểm của A1 B và D1 C.
Áp dụng định lý Pascal cho 6 điểm A1 , B1 , C1 , D1 , B, C suy ra I, O, L thẳng hàng.
Lại áp dụng định lý Pascal cho 6 điểm A, B, C, D, A1 , D1 suy ra I, P, L thẳng hàng. Như vậy P
nằm trên OI.
Trở lại bài toán.

281
E G

H X
K

T O
I B
P
D
Z
Y
M
N L J
C

Gọi M là điểm Miquel của tứ giác toàn phần ABCDEF . Khi đó dễ thấy M nằm trên EF và
M, P, O thẳng hàng. Áp dụng bổ đề trên suy ra M, P, O, I thẳng hàng.
Từ M kẻ tiếp tuyến M K, M N tới (I). Từ O kẻ OH ⊥ M K, OL ⊥ M N . Gọi X, Y, Z, T lần lượt là
tiếp điểm của AB, BC, CD, DA với (I).
Theo kết quả quen thuộc, AC, BD, XZ, Y T đồng quy tại P.
Do đó EF là đường đối cực của P với đường tròn (I). Suy ra P nằm trên đường đối cực của M
với (I) hay P nằm trên KN .
Theo định lý Brocard, P là trực tâm của tam giác OEF suy ra M P.M O = M E.M F .
Mặt khác tứ giác ABCD nội tiếp nên bằng một số phép cộng góc đơn giản, phân giác các góc
DEA và DF C vuông góc với nhau tại I. Tức là tam giác EIF vuông tại I có IM là đường cao. Ta
thu được M E.M F = M I 2 .
MP MI
Như vậy M P.M O = M I 2 hay = .
MI MO
MP MI MK MP
Gọi I 0 là giao của M O với HL. Ta có = = = . Suy ra I 0 ≡ I, tức là I là trung
MI MO MH M I0
điểm HL.
Gọi G, J là giao điểm thứ hai của M K, M N với (ECB). Ta có (I) là đường tròn nội tiếp tam giác
ECB nên theo định lý Poncelet, (I) đồng thời là đường tròn nội tiếp tam giác M GJ.
Mà I là trung điểm đoạn nối hai tiếp điểm của đường tròn ω(O, OH) với M G, M J nên theo bổ
đề Sawayama, ω là đường tròn mixtilinear nội tiếp ứng với đỉnh M của tam giác M GJ. Tức là ω tiếp
xúc với (ECB).
Chứng minh tương tự ta cũng có ω tiếp xúc với (EAD), (F AB), (F CD).

282
Tiếp theo chú ý rằng các đường tròn (EAD), (EBC), (F AB), (F CD) đồng quy tại điểm Miquel
M của tứ giác toàn phần ABCDEF , điều đó khiến chúng ta nghĩ đến phép nghịch đảo tâm M . Rất
bất ngờ qua phép nghịch đảo, bài toán ban đầu trở thành một bài toán khác đã có trước đó của tác
giả Trần Quang Hùng, GV THPT chuyên KHTN, Hà Nội.
Bài 322. Cho tứ giác ngoại tiếp ABCD, AB giao CD tại E, AD giao BC tại F . Gọi M là điểm Miquel
của tứ giác toàn phần ABCDEF . Xét phép nghịch đảo tâm M phương tích bất kì biến A, B, C, D lần
lượt thành A0 , B 0 , C 0 , D0 . Khi đó A0 B 0 C 0 D0 cũng là một tứ giác ngoại tiếp.
Hiển nhiên theo bài toán 1, tồn tại một đường tròn ω tiếp xúc với 4 đường tròn (M AB), (M BC),
(M CD), (M DA) nên ảnh của ω qua phép nghịch đảo sẽ tiếp xúc với các đường thẳng A0 B 0 , B 0 C 0 ,
C 0 D 0 , D 0 A0 .
Ở đây chúng ta giới thiệu một lời giải khác cho bài toán 3.
MB MC BC
Chứng minh. Ta có 4M BC ∼ 4M AD nên = = .
MA MD AD
MC MD BC
4M CD ∼ 4M BA nên = = .
MB MA AD
AB BC CD DA
Mặt khác, A0 B 0 = k. , B 0 C 0 = k. , C 0 D0 = k. , D0 A0 = k. .
M A.M B M B.M C M C.M
 D  M D.M A
AB CD DC DC
Suy ra A0 B 0 + C 0 D0 = k + = k. 1+
M A.M B M C.M D M D.M C AB
AB 2 DC 2
(Do = )
M A.M B M D.M C  
0 0 0 0 BC BC
Tương tự, B C + D A = k. 1+
M B.M C AD
DC AB BC AD
Ta cũng có = và = nên
MD MA MB
 MA  
0 0 0 0 AB DC AB + DC 0 0 0 0 AD BC
A B + C D = k. 1+ = , B C + D A = k. 1+ =
M A.M C AB M A.M C M A.M C AD
AD + BC
.
M A.M C
Mà tứ giác ABCD ngoại tiếp nên AB + DC = AD + BC, suy ra A0 B 0 + C 0 D0 = B 0 C 0 + D0 A0 . Tức
là tứ giác A0 B 0 C 0 D0 cũng ngoại tiếp.
Bài toán 3 khiến tác giả nhớ tới một bài toán tương tự được tác giả tìm ra từ khá lâu.
Bài 323. Cho tứ giác ngoại tiếp ABCD. Đường tròn ngoại tiếp tam giác ABD giao AC lần thứ hai
tại X, đường tròn ngoại tiếp tam giác BCD giao AC lần thứ hai tại Y . Khi đó tứ giác BXDY ngoại
tiếp.
Bản chất của bài toán 4 là trường hợp đặc biệt của bài toán sau.
Bài 324. Cho tứ giác ngoại tiếp ABCD. AC giao BD tại P . Phép nghịch đảo tâm P phương tích bất
kì biến các điểm A, B, C, D lần lượt thành A0 , B 0 , C 0 , D0 . Khi đó A0 B 0 C 0 D0 cũng là một tứ giác ngoại
tiếp.
B
A x X
J y
T P
Y
t
I

D
Z
z
C

283
AB BC
Chứng minh. Theo tính chất của phép nghịch đảo, A0 B 0 = k. , B 0 C 0 = k. , C 0 D0 =
P A.P B P B.P C
CD DA
k. , D0 A0 = k. .
P C.P D P D.P A 
0 0 0 0 AB CD
Suy ra A B + C D = k. +
 P A.P B P C.P D 
k AB CD
= sin ∠AP B. +
2  1/2P A.P B. sin ∠AP B 1/2P C.P D. sin ∠DP C
k AB CD
= sin ∠AP B. +
2 SAP B SDP C
Gọi tiếp điểm của (I) với AB, BC, CD, DA lần lượt là X, Y, Z, T , AX = AT = x, BX = BY = y,
CY = CZ = z, DZ = DT = t.
Từ A kẻ đường song song với DC cắt XZ tại J. Dễ thấy ∠AXJ = ∠DZX = ∠AJX nên AX =
AJ = x.
SAP B AP AJ x
Suy ra = = = .
SBP C PC CZ z
SAP B SBP C SCP D SDP A
Chứng minh tương tự suy ra = = = = q.
xy yz  zt tx
AB CD x+y z+t 1 1 1 1 1
Từ đó + = + = + + + .
SAP B SDP C qxy qzt  q x y z t
k 1 1 1 1
Như vậy A0 B 0 + C 0 D0 = sin ∠AP B. + + + .
2q x y z t
Tương tự suy ra A0 B 0 + C 0 D0 = A0 D0 + B 0 C 0 . Tức là tứ giác A0 B 0 C 0 D0 ngoại tiếp.

Do tồn tại một đường tròn γ 0 tiếp xúc với các đường thẳng A0 B 0 , B 0 C 0 , C 0 D0 , D0 A0 nên ảnh của γ 0
qua phép nghịch đảo tâm P sẽ tiếp xúc với đường tròn ngoại tiếp các tam giác P AB, P BC, P CD, P DA.
Chúng ta có bài toán mới.

Bài 325. Cho tứ giác ngoại tiếp ABCD. Gọi P là giao của AC và BD. Chứng minh rằng tồn tại một
đường tròn tiếp xúc với đường tròn ngoại tiếp các tam giác P AB, P BC, P CD, P DA.

P
I

D
C

284
K

F
M
J

E
A X B

T A' P
B'
D' Y
I C'
D
Z

Bây giờ gọi X, Y, Z, T lần lượt là tiếp điểm của (I) với AB, BC, CD, DA. xét phép nghịch đảo tâm
2
I, phương tích R2 , IIR : A 7→ A0 , B 7→ B 0 , C 7→ C 0 , D 7→ D0 , P 7→ M .
Dễ thấy A0 , B 0 , C 0 , D0 lần lượt là trung điểm T X, XY, Y Z, ZT.
Gọi T X giao Y Z tại K, XY giao ZT tại J. Theo định lý Brocard, dễ thấy M là hình chiếu của I
trên JK đồng thời M là điểm Miquel của tứ giác toàn phần XY ZT KJ. Chúng ta có bài toán mới.

Bài 326. Cho tứ giác nội tiếp ABCD. AB giao CD tại E, AD giao BC tại F . Gọi M là điểm Miquel
của tứ giác toàn phần ABCDEF , X, Y, Z, T lần lượt là trung điểm AB, BC, CD, DA. Khi đó tồn tại
một đường tròn tiếp xúc với đường tròn ngoại tiếp của các tam giác M XY, M Y Z, M ZT, M T X.

Bài 327. Với kí hiệu như bài toán 7, phép nghịch đảo tâm M phương tích bất kì biến X, Y, Z, T thành
X 0 , Y 0 , Z 0 , T 0 . Khi đó X 0 Y 0 Z 0 T 0 là một tứ giác ngoại tiếp.

Như vậy trong tứ giác ngoại tiếp ABCD, điểm Miquel và giao của hai đường chéo AC và BD biến
các đỉnh của tứ giác ABCD thành các đỉnh của một tứ giác ngoại tiếp khác. Không khó để nhận ra
rằng giao điểm E và F cũng có tính chất này. Chúng ta chú ý rằng (I) là đường tròn bàng tiếp của
tam giác F AB và là đường tròn nội tiếp của tam giác F CD. Do đó đường tròn mixtilinear nội tiếp ứng
với đỉnh F của tam giác F CD đồng thời là đường tròn mixtilinear bàng tiếp ứng với đỉnh F của tam
giác F AB. Như vậy tồn tại một đường tròn ω tiếp xúc với AD, BC và tiếp xúc với (F AB), (F CD).
Khi đó phép nghịch đảo tâm F biến A, B, C, D lần lượt thành A0 , B 0 , C 0 , D0 , ω thành ω 0 thì ω 0 tiếp
xúc với các đường thẳng A0 B 0 , B 0 C 0 , C 0 D0 , D0 A0 .
Trước khi kết thúc bài viết, một câu hỏi đặt ra cho bạn đọc là trên mặt phẳng liệu còn những điểm
nào ngoài 4 điểm trên mà phép nghịch đảo có tâm là điểm đó biến 4 đỉnh của một tứ giác ngoại tiếp
thành 4 đỉnh của một tứ giác ngoại tiếp khác?

31 Chuỗi bài toán về họ đường tròn đi qua điểm cố định


Chúng ta bắt đầu từ bài toán sau.

Bài 328. (USA TST 2012) Cho tam giác ABC. P là một điểm chuyển động trên BC. Gọi Y, Z lần
lượt là các điểm trên AC, AB sao cho P Y = P C, P Z = P B. Chứng minh rằng (AY Z) luôn đi qua
trực tâm tam giác ABC.

Đây là một bài toán khá thú vị và có nhiều cách để tiếp cận bài toán này. Trong bài viết này xin
đưa ra 5 cách giải.

Chứng minh. Cách 1.

285
A

Y
Hb

Hc
H
Z

B P M C

Gọi Hb , Hc là chân đường cao kẻ từ B, C. M là trung điểm BC. Dễ thấy các tam giác BM Hc và
ZHc PM PM Y Hb
CM Hb cân tại M nên M Hc k P Z, M Hb k P Y . Từ đó = = = .
BHc BM CM CHb
Mà 4BHHc ∼ 4CHHb nên 4ZHc H ∼ 4Y Hb H.
Suy ra ∠ZHHc = ∠Y HHb hay ∠ZHY = ∠Hc HHb = 180◦ − ∠BAC. Vậy H ∈ (AY Z).
Cách 2.
A

Y
Hb

F Hc
E T
H
Z

B
P C

Gọi E là giao của P Y với BHb , F là giao của P Z với CHc . Kẻ P T ⊥ AC.
Dễ thấy P T là phân giác ∠Y P C và P T k BHb nên ∠BEP = ∠EBP = ∠HAY , suy ra A, H, Y, E
cùng thuộc một đường tròn.
Chứng minh tương tự, A, H, Z, F cùng thuộc một đường tròn.
Mặt khác, P E = P B = P Z, P F = P C = P Y nên F Y EZ là hình thang cân hay F, Y, E, Z cùng
thuộc một đường tròn.
Vậy 6 điểm A, H, Y, Z, E, F cùng thuộc một đường tròn hay H ∈ (AY Z).
Cách 3.

286
A

Y
Hb

Hc

L H
Z

E B P F C

Qua Y kẻ đường vuông góc với AC, cắt BC tại E; qua Z kẻ đường vuông góc với AB, cắt BC tại
F . Y E cắt ZF tại L.
Do tam giác BZF vuông và P Z = P B nên P B = P Z = P F . Tương tự P C = P E.
Suy ra EF = P E + P F = P B + P C = BC. Lại có LF k HC, LE k HB nên 4BHC = 4ELF .
Suy ra đường cao kẻ từ L và H tới BC bằng nhau hay LH k BC. Suy ra LH ⊥ AH.
Như vậy Z, Y, H ∈ (AL) hay H ∈ (AEF ).
Cách 4.

Z T
H
S

B G P C

Gọi T, S lần lượt là hình chiếu của P trên AC, AB. Kẻ đường cao AG cắt (AEF ) tại H, cắt (ABC)
tại L.
PT /(AY Z) TY · TA PS/(AY Z)
Dễ thấy T Y = T C nên = = −1, tương tự = −1. Suy ra (AY Z),
PT /(ABC) TC · TA PS/(ABC)
(AT S), (ABC) đồng trục (xem [3]).
PG/(AY Z) GH · GA
Do G, T, S ∈ (AT ) nên = −1 = hay G là trung điểm HL.
PG/(ABC) GL · GA
Suy ra H là trực tâm tam giác ABC.
Cách 5.

287
A

O
Y
Z

B P C

Gọi O là tâm đường tròn ngoại tiếp tam giác AY Z. Dễ thấy ∠Y OZ = 2∠BAC, ∠Y P Z = 180◦ −
2∠BAC nên O ∈ (Y P Z).
Áp dụng định lý Ptolemy ta có OP.Y Z = OZ.P Y + OY.P Z = OY.(P Y + P Z) = OY.BC.
OY
Suy ra OP = BC · = const.
YZ −−→ −→ −−→
Lại có hướng của các vector P Y , P Z không đổi và P O là phân giác ∠Y P X nên P O không đổi. Do
P chuyển động trên BC nên O chuyển động trên một đường thẳng song song với BC. Suy ra (AY Z)
đi qua một điểm cố định nằm trên đường cao kẻ từ A tới BC.
Khi P trùng trung điểm BC, Y, Z trở thành chân đường cao kẻ từ B và C, khi đó (AY Z) đi qua
trực tâm tam giác ABC. Vậy (AY Z) luôn đi qua trực tâm tam giác ABC.

Thay đổi giả thiết của bài toán bằng cách cho các tam giác BZP, CY P lần lượt cân tại Z, Y, ta
thu được bài toán sau.
Bài 329. Cho tam giác ABC. P là một điểm chuyển động trên BC. Gọi Y, Z lần lượt là các điểm trên
AC, AB sao cho Y P = Y C, ZP = ZB. Chứng minh rằng (AY Z) luôn đi qua tâm đường tròn ngoại
tiếp tam giác ABC.

A Q

Y
O

B P C

Chứng minh. Gọi (O) là đường tròn ngoại tiếp tam giác ABC, Q là điểm đối xứng với P qua Y Z.
Theo phép đối xứng, ∠Y QZ = ∠Y P Z = ∠BAC.
Ta thu được A, Q, Y, Z cùng thuộc một đường tròn. Suy ra ∠AY Q = ∠AZQ.
1
Cũng theo phép đối xứng, Y Q = Y P = Y C, ZQ = ZP = ZB suy ra ∠ABQ = ∠AZQ =
2
1
∠AY Q = ∠ACQ. Từ đó Q cũng nằm trên (O).
2

288
Suy ra ∠AOQ = 2∠ACQ = ∠AY Q hay Q,A,O,Y cùng thuộc một đường tròn. Ta thu được
O ∈ (AY Q).

Tiếp tục cho các tam giác BZP, CY P lần lượt cân tại B, C ta thu được bài toán mới.

Bài 330. Cho tam giác ABC. P là một điểm chuyển động trên BC. Gọi Y, Z lần lượt là các điểm
trên AC, AB sao cho CP = CY, BP = BZ. Chứng minh rằng (AY Z) luôn đi qua tâm đường tròn nội
tiếp tam giác ABC.

Z
E
F
I Y

B D P C

Chứng minh. Gọi D, E, F là tiếp điểm của đường tròn (I) nội tiếp tam giác ABC với các cạnh
BC, CA, AB. Không mất tổng quát giả sử Z nằm giữa A và F.
Ta có BD = BE, BP = BZ suy ra ZF = P D, tương tự EY = P D.
Ta thu được F Z = EY , từ đó 4ZF I = 4Y EI(c.g.c)
Suy ra ∠ZIF = ∠Y IE.
Do Z nằm giữa A và F nên D nằm giữa B và P hay P nằm giữa D và C, suy ra E nằm giữa A
và Y . Suy ra ∠ZIY = ∠F IE = 180◦ − ∠BAC. Vậy (AY Z) đi qua I.

Ta có thể phát biểu lại bài toán 3 theo cách khác.

Bài 331. Cho tam giác ABC. P là một điểm chuyển động trên BC. Gọi Y, Z lần lượt là các điểm
trên AC, AB sao cho BZ + BP = AC và CY + CP = AB. Gọi Y 0 , Z 0 lần lượt đối xứng với Y, Z qua
trung điểm AC, AB. Chứng minh rằng (AY 0 Z 0 ) luôn đi qua tâm đường tròn nội tiếp tam giác ABC.

Chứng minh. Ta có CY 0 + BZ 0 = AY + AZ = AB + AC − CY − BZ = BP + CP = BC nên tồn tại


một điểm P 0 trên BC sao cho BP 0 = BZ 0 , CP 0 = CY 0 . Áp dụng bài 3 suy ra đpcm.

Có nhiều cách xác định hai điểm Y, Z trên AC, AB. Bài toán tiếp theo là một kết quả khá quen
thuộc.

Bài 332. Cho tam giác ABC. P là một điểm chuyển động trên BC. Gọi Y, Z lần lượt là các điểm
trên AC, AB sao cho P Y k AB, P Z k AC. Chứng minh rằng (AY Z) luôn đi qua một điểm cố định
khác A.

289
A

Z Y

O
L

B P C

Chứng minh. Dễ thấy khi P ≡ C, đường tròn (AY Z) đi qua A, A, C, tức là đường tròn ω1 qua A, C
và tiếp xúc với AB. Tương tự khi P ≡ B, đường tròn (AY Z) biến thành đường tròn ω2 qua A, B và
tiếp xúc với AC.
Gọi L là giao điểm thứ hai của ω1 và ω2 suy ra L cố định. Ta chứng minh (AY Z) đi qua L.
Ta có ∠LAB = ∠LCA, ∠LAC = ∠LBA. Suy ra ∠ALB = ∠ALC = 180◦ − ∠BAC và 4ALB ∼
4CLA.
AZ CP CY
Do = = , ta thu được 4ALZ ∼ 4CLY.
ZB PB YA
Suy ra ∠ALZ = ∠CLY hay ∠ZLY = ∠ALC = 180◦ − ∠BAC. Vậy (AY Z) luôn đi qua L cố
định.

Nhận xét. Bạn đọc có thể chứng minh L là hình chiếu của tâm ngoại tiếp tam giác ABC trên
đường đối trung ứng với đỉnh A.
Thay giả thiết song song bằng vuông góc ta thu được bài toán mới.

Bài 333. Cho tam giác ABC. P là một điểm chuyển động trên BC. Gọi Y, Z lần lượt là các điểm
trên AC, AB sao cho P Y ⊥ AB, P Z ⊥ AC. Chứng minh rằng (AY Z) luôn đi qua một điểm cố định
khác A.

O
O'
B C
P

Z
M Y

A'

290
Chứng minh. Gọi M là trung điểm Y Z. Do P là trực tâm tam giác AY Z nên AP = 2OM , suy ra
AP 2OM
= = 2 cos ∠BAC = const.
AO OY
AO0 1
Trên AP lấy điểm O0 sao cho AO0 = AO. Suy ra = .
AP 2 cos ∠BAC
Do P chuyển động trên BC nên O0 chuyển động trên đường thẳng d là ảnh của BC qua phép vị
1
tự tâm A tỉ số .
2 cos ∠BAC
Mặt khác, AP và AO đẳng giác trong ∠BAC nên O và O0 đối xứng qua phân giác ∠BAC. Suy ra
O chuyển động trên l là ảnh của d qua phép đối xứng trục là phân giác ∠BAC.
Như vậy (AY Z) luôn đi qua điểm đối xứng với A qua l.

Sau đây là một số bài toán được biến đổi giả thiết khác, mỗi bài toán đều có phát biểu khá đơn
giản nhưng lại không dễ, mời bạn đọc thử tự chứng minh trước khi xem lời giải.

Bài 334. (ELMO Shortlist 2013) Cho tam giác ABC. P là một điểm chuyển động trên BC. (ABP )
giao AC tại Y , (ACP ) giao AB tại Z. Chứng minh rằng (AY Z) luôn đi qua một điểm cố định khác
A.

L K Y
H

T
B M P C

J
A'

Chứng minh. Gọi T là hình chiếu của A trên BC, H là trực tâm tam giác ABC. M là trung điểm
BC, AM cắt (BHC) tại J sao cho A và J khác phía với BC. AM cắt (AY Z) tại L. Gọi K là giao của
BY và CZ. A0 đối xứng với A qua BC.
Ta có ∠BY A = ∠BP A = 180◦ − ∠AP C = 180◦ − ∠AZC. Suy ra K ∈ (AY Z).
Suy ra ∠Y KC = ∠BAC = ∠Y P C hay K ∈ (CY P ).
Từ đó ∠KP C = ∠KY A = ∠AP B = ∠A0 P B hay A0 , P, K thẳng hàng.
Do K ∈ (AY Z) nên ∠BKC = 180◦ − ∠BAC = ∠BHC, suy ra K ∈ (BHC).
Lại có (ABC) và (BHC) đối xứng nhau qua BC nên A0 ∈ (BHC), AM = M J.
Từ đó T M k A0 J. Ta thu được ∠KLJ = ∠AY B = ∠AP B = ∠BP A0 = ∠KA0 L hay L ∈ (BHC).
Vậy (AY Z) luôn đi qua giao điểm của trung tuyến ứng với đỉnh A với (BHC) hay hình chiếu của
trực tâm H trên AM.

291
Bài 335. Cho tam giác ABC. P là một điểm chuyển động trên BC. Gọi Y, Z lần lượt là các điểm
trên AC, AB sao cho Y B = Y P, ZC = ZP . Chứng minh rằng (AY Z) luôn đi qua một điểm cố định
khác A.
O'
Z
Y

Z' Y'

B H K' P P' K H' C

Chứng minh. Gọi O là tâm ngoại tiếp tam giác ABC, H, K lần lượt là hình chiếu của Y, Z trên BC.
1
Do Y P = Y P, ZC = ZP ta thu được HP = HB, KP = KC, từ đó HK = BC.
2
Gọi Y 0 , Z 0 lần lượt đối xứng với Y, Z qua A. Suy ra Y 0 Z 0 k= Y Z. Gọi H 0 , K 0 là hình chiếu của
1
Y 0 , Z 0 trên BC. Ta có H 0 K 0 = HK = BC. Trên BC lấy P 0 sao cho H 0 C = H 0 P 0 , suy ra K 0 P 0 = K 0 B.
2
Áp dụng bài 2 ta thu được O ∈ (AY 0 Z 0 ).
Theo phép đối xứng tâm A, (AY Z) đi qua điểm đối xứng với O qua A và đó là điểm cố định.

Bài 336. Cho tam giác ABC. P là điểm chuyển động trên BC. Kẻ P Y, P Z lần lượt vuông góc với
AC, AB. Đường tròn (BP Y ) giao AB lần thứ hai tại M , đường tròn (CP Z) giao AC lần thứ hai tại
N . Chứng minh rằng (AM N ) luôn đi qua một điểm cố định.

N
E M

L
A

Y
Z

B H P C

292
Chứng minh. Qua B, C lần lượt kẻ đường vuông góc với BC cắt AC, AB tại E, F . Khi đó E ∈ (BP Y )
và F ∈ (CP Z).
Gọi L là giao của EF với (BP Y ) thì ∠F LP = 90◦ , suy ra L ∈ (CP Z) hay L là giao điểm của hai
đường tròn (BP Y ) và (CP Z).
Ta có ∠N LM = ∠N LF + ∠ELM = ∠ACF + ∠ABE = ∠BAC = ∠N AM hay L ∈ (N AM ).
Kẻ AH ⊥ BC, AH giao EF tại T . Ta có ∠N AT = ∠HAC = ∠ACF = ∠N LT , do đó T ∈ (N AM ).
Mà BEF C là hình thang có A là giao điểm 2 đường chéo nên AH = AT . Suy ra T cố định. Vậy
(AM N ) luôn đi qua điểm đối xứng với chân đường cao kẻ từ A qua A.

32 Tứ giác ngoại tiếp thần kì


Bài toán sau đây được đề xuất bởi tác giả Fazakas Tünde, một nữ giáo viên chuyên toán Hungary.

Bài 337. Cho tứ giác ABCD, AB giao CD tại P , AD giao BC tại Q. Từ mỗi điểm P và Q kẻ n − 1
đường thẳng chia tứ giác ABCD thành một ma trận có n hàng và n cột. Từ n2 tứ giác con ta có thể
chọn được n tứ giác ngoại tiếp sao cho mỗi hàng và mỗi cột có đúng một tứ giác. Chứng minh rằng tứ
giác ABCD ngoại tiếp.

B
A

Bài toán này thực sự gây không ít khó khăn cho người tiếp cận. Ban đầu tác giả thử chứng minh
trường hợp n = 2, kết quả khá đơn giản (xem bổ đề 1). Trường hợp n = 3 lại có thể quy về n = 2
bằng cách sử dụng hai lần trường hợp n = 2. Tuy nhiên khi n = 4 vấn đề không còn đơn giản như
vậy nữa. Ta không thể quy nạp về n = 3 theo cách trên, nguyên nhân là do khi n = 4, có thể có khả
năng 4 tứ giác ngoại tiếp được chọn đều không chung đỉnh như hình vẽ trên. Khi đó không thể đưa về
trường hợp nhỏ hơn theo cách chứng minh n = 3. Mặc dù vậy, ta đã có thêm một nhận xét là nếu tồn
tại hai tứ giác ngoại tiếp chung 1 đỉnh, ta có thể rút bớt hai đường thẳng giao nhau tại đỉnh chung ấy
và đưa về trường hợp n − 1. Ngoài ra nếu tồn tại một tứ giác ngoại tiếp ở một trong bốn góc của tứ
giác ABCD, ta cũng có thể rút hai đường thẳng là cạnh của tứ giác ngoại tiếp đó và đưa về trường
hợp n − 1. Một ý tưởng nảy sinh là có cách nào biến các tứ giác n · n về một trong hai dạng trên hay
không?

293
Q

Y
X B
A

D Z

Trước tiên chúng ta hãy thử giải quyết trong trường hợp n = 4. Tác giả đã dựng thêm đường tròn
thứ 5 như hình vẽ. Sau đó từ P kẻ tiếp tuyến thứ hai tới đường tròn này. Rõ ràng tứ giác XY ZT
thỏa mãn yêu cầu của đề bài và có một tứ giác ngoại tiếp nằm ở đỉnh T . Do đó có thể chứng minh tứ
giác XY ZT ngoại tiếp nhờ phép quy nạp. Ta cũng có thể thực hiện bằng phương pháp tương tự cho
trường hợp tổng quát như sau.

Chứng minh. Trước tiên ta phát biểu hai bổ đề.


Bổ đề 1. Cho tứ giác ABCD ngoại tiếp đường tròn (I). AB giao CD tại E, AD giao BC tại F .
Các điểm M, N, P, Q lần lượt nằm trên AB, BC, CD, DA sao cho M P đi qua F , N Q đi qua E. M P
giao N Q tại S. Khi đó tứ giác AM SQ ngoại tiếp khi và chỉ khi tứ giác CN SP ngoại tiếp.
F

M B
A

E
Q
S N

Chứng minh. Ta xét trường hợp A nằm giữa B và E, D và F , các trường hợp còn lại chứng minh
tương tự.
Tứ giác AM SQ ngoại tiếp khi và chỉ khi tứ giác lõm EAF S ngoại tiếp. Theo định lý Pythot suy
ra điều này tương đương EA − F A = ES − F S. (1)
Do tứ giác ABCD ngoại tiếp nên EA − F A = EC − F C. Suy ra (1) tương đương ES − F S =
EC − F S, khi và chỉ khi tứ giác lõm F CES ngoại tiếp hay tứ giác CN SP ngoại tiếp.
Bổ đề 2. Cho tứ giác ABCD ngoại tiếp. Gọi P, Q lần lượt là giao của AD và BC, AB và CD,
d1 , d2 là 2 đường thẳng bất kì qua P . Dựng 2 đường tròn (I1 ), (I2 ) lần lượt nội tiếp các tam giác tạo bởi

294
AB, d1 , d2 và CD, d1 , d2 . Từ Q kẻ 2 tiếp tuyến l1 , l2 khác AB, CD tới (I1 ), (I2 ). Khi đó d1 , d2 , AD, BC
cắt nhau tạo thành một tứ giác ngoại tiếp.
Chứng minh.
P

I3 B

X I2

A
Z

T I4
I1

Q D M N C

Kí hiệu (d1 , d2 , d3 , d4 ) là tứ giác tạo bởi giao điểm của 4 đường thẳng d1 , d2 , d3 , d4 .
Dựng đường tròn (I3 ) và (I4 ) lần lượt nội tiếp tam giác QY B và QZC.
Từ P kẻ đường thẳng d3 khác BC tiếp xúc với (I4 ).
Áp dụng định lý Monge-D’Alembert cho 3 đường tròn (I1 ), (I2 ), (I4 ) suy ra tâm vị tự ngoài của
(I2 ) và (I4 ) nằm trên P Q.
Lại áp dụng định lý Monge-D’Alembert cho 3 đường tròn (I2 ), (I3 ), (I4 ) ta có tâm vị tự ngoài của
(I2 ) và (I3 ) là Q, tâm vị tự ngoài của (I2 ) và (I4 ) nằm trên P Q nên tâm vị tự ngoài của (I3 ) và (I4 )
cũng nằm trên P Q - cũng chính là điểm P .
Vậy tứ giác (d3 , QY, QB, P B) ngoại tiếp.
Do các tứ giác ABCD và (QZ, P N, QC, P C) ngoại tiếp nên áp dụng bổ đề 1 suy ra tứ giác
(P N, QZ, QA, P A) ngoại tiếp.
Mà tứ giác (P N, QB, QY, P B) ngoại tiếp nên lại áp dụng bổ đề 1 suy ra tứ giác XY ZT ngoại tiếp.
Trở lại bài toán.
-Trường hợp n = 2, theo bổ đề 1 bài toán hiển nhiên đúng.
-Xét trường hợp n = k. Nếu một tứ giác chứa tứ giác nhỏ ngoại tiếp nằm ở 1 trong 4 góc của tứ
giác thì ta có thể quy nạp về trường hợp n = k − 1.
Vì vậy ta xét bài toán trong trường hợp không có tứ giác ngoại tiếp nằm ở 1 trong 4 góc.

295
Xn+2

Xn+1

Xk

X2

X1

Ta đơn giản hóa bài toán bằng một bảng ô vuông n · n, trong đó các đường thẳng thuộc các hàng
đồng quy (tại P ) và các đường thẳng thuộc các cột đồng quy (tại Q). Tính từ hàng dưới cùng, kí hiệu
Xi là tứ giác ngoại tiếp thuộc hàng thứ i. Gọi Xk là tứ giác ngoại tiếp ngoài cùng bên phải.
Thực hiện liên tiếp phép dựng tứ giác ngoại tiếp Xn+1 trên cột có tứ giác X1 , tứ giác Xn+2 trên
cột có tứ giác X2 , ... đến tứ giác ngoại tiếp Xn+k−1 như hình vẽ.
Khi đó ta có một bảng n · n chứa n tứ giác ngoại tiếp Xk , Xk+1 , ..., Xn+k−1 , với Xk là tứ giác ngoại
tiếp nằm ở 1 trong 4 góc. Kí hiệu Ck là tứ giác bao ngoài bảng trên. Theo trường hợp thứ nhất, tứ
giác Ck ngoại tiếp.
Áp dụng bổ đề 2, tứ giác Ck−1 chứa các tứ giác ngoại tiếp Xk−1 , Xk , ..., Xn+k−2 cũng là một tứ
giác ngoại tiếp.
Lại tiếp tục áp dụng bổ đề 2 suy ra Ck−2 , ..., C1 là các tứ giác ngoại tiếp. Ta có đpcm.

Bài toán 1 có hình thức khá giống một bài toán từng được tác giả đề xuất cách đây 3 năm trên
cuộc thi Mathley Geometry Contest.

Bài 338. Cho tứ giác ngoại tiếp ABCD. AB giao CD tại E, AD giao BC tại F . Hai đường thẳng
bất kì qua E lần lượt cắt AD, BC tại M, N, P, Q (M, N ∈ AD, P, Q ∈ BC). Hai đường thẳng bất kì
qua F lần lượt cắt AB, CD tại X, Y, Z, T (X, Y ∈ AB, Z, T ∈ CD). Gọi d1 , d2 là tiếp tuyến thứ hai
kẻ từ E tới đường tròn nội tiếp các tam giác F XY, F ZT ; d3 , d4 là các tiếp tuyến thứ hai kẻ từ F tới
đường tròn nội tiếp các tam giác EM N, EP Q. Chứng minh rằng d1 , d2 , d3 , d4 cắt nhau tạo thành một
tứ giác ngoại tiếp.

Để kết thúc bài viết xin giới thiệu tới bạn đọc một dạng tứ giác ngoại tiếp khác khá thú vị do tác
giả tìm ra, mời bạn đọc thử sức.
Cho tứ giác ABCD được chia thành một bảng ô vuông 5 · 5 như hình vẽ. Chứng minh rằng nếu
có 12 trong 13 tứ giác màu xanh là tứ giác ngoại tiếp thì tứ giác còn lại cũng ngoại tiếp, đồng thời
ABCD cũng là một tứ giác ngoại tiếp.
Không khó nhận ra rằng bài toán vẫn đúng nếu ta thay 5 · 5 bằng (2n + 1) · (2n + 1), với điều kiện
có (n + 1)2 + n2 − 1 tứ giác ngoại tiếp.

296
A B

D C

33 Chứng minh định lý Sondat dựa theo ý tưởng của Jean-Louis


Ayme
33.1 Giới thiệu
Tại [1], tác giả người Pháp Jean-Louis Ayme đưa ra một chứng minh sơ cấp khá thú vị cho định
lý Sondat về hai tam giác trực giao có tâm thấu xạ. Hướng chứng minh của tác giả rất đặc sắc khi sử
dụng phép vị tự để đưa về hai tam giác có chung tâm trực giao. Tuy nhiên phần sau của tác giả khá
dài và có một đoạn lí luận chưa chặt chẽ. Trong bài viết này tôi xin chứng minh lại định lý Sondat
dựa theo ý tưởng của tác giả cùng phần đã chỉnh sửa.

33.2 Chứng minh


Trước tiên xin giới thiệu tới bạn đọc khái niệm về hai tam giác trực giao.
Định lý 1. Cho hai tam giác ABC và A0 B 0 C 0 . Khi đó các đường vuông góc kẻ từ A, B, C tới
B C 0 , C 0 A0 , A0 B 0 đồng quy tại P khi và chỉ khi các đường vuông góc kẻ từ A0 , B 0 , C 0 tới BC, CA, AB
0

đồng quy tại Q. Ta gọi ABC và A0 B 0 C 0 là hai tam giác trực giao, P là tâm trực giao của tam giác
A0 B 0 C 0 ứng với bộ điểm A, B, C, Q là tâm trực giao của tam giác ABC ứng với bộ điểm A0 , B 0 , C 0 .

B'

C'
P

B C

A'

Chứng minh. Áp dụng định lý Carnot, các đường vuông góc kẻ từ A, B, C tới B 0 C 0 , C 0 A0 , A0 B 0 đồng
quy khi và chỉ khi (AB 02 − AC 02 ) + (BC 02 − BA02 ) + (CA02 − CB 02 ) = 0.
Hay (B 0 A2 − B 0 C 2 ) + (A0 C 2 − A0 B 2 ) + (C 0 B 2 − C 0 A2 ) = 0 tương đương các đường vuông góc kẻ
từ A0 , B 0 , C 0 tới BC, CA, AB đồng quy. Ta có đpcm.

297
Định lý 2 (Pierre Sondat). Cho hai tam giác ABC và A1 B1 C1 trực giao có tâm trực giao là P
và Q. Giả sử hai tam giác ABC và A1 B1 C1 thấu xạ theo tâm O. Khi đó O, P, Q thẳng hàng.

Chứng minh. Trước tiên ta cần hai bổ đề sau.


Bổ đề 38. (Định lý Dergiades). Cho tam giác ABC. 3 đường tròn ωa , ωb , ωc lần lượt đi qua các
cặp đỉnh B, C; C, A; A, B. Gọi D, E, F là giao điểm thứ hai của 3 đường tròn này. Đường thẳng Qua
D vuông góc với AD cắt BC tại X. Tương tự xác định Y, Z. Khi đó X, Y, Z thẳng hàng.
Chứng minh.

E
F J

D C
X B

Đặt ∠BEC = ∠BF C = α, ∠ADC = ∠AF C = β, ∠AEB = ∠ADB = γ, bán kính của ωa , ωb , ωc
lần lượt tại Ra , Rb , Rc .
XB BD · sin ∠XDB BD · (− cos ∠ADB) BD · cos γ
Ta có = = = .
XC CD · sin ∠XDC CD · (− cos ∠ADC) CD · cos β
XB Y C ZA BD CE AF
Chứng minh tương tự suy ra · · = · · .
XC Y A ZB CD AE BF
BD 2Rc sin ∠BAD
Ta lại có = . Tương tự và áp dụng định lý Céva sin cho tam giác ABC với các
CD 2Rb sin ∠CAD
BD CE AF
đường AD, BE, CF đồng quy tại tâm đẳng phương của ωa , ωb , ωc ta thu được · · = 1.
CD AE BF
Vậy X, Y, Z thẳng hàng.
Bổ đề 39. Cho hai tam giác ABC và XY Z thỏa mãn các đường vuông góc kẻ từ A, B, C tới
Y Z, ZX, XY và các đường vuông góc kẻ từ X, Y, Z tới BC, CA, AB cùng đồng quy tại O. Khi đó
hai tam giác ABC và XY Z thấu xạ.
Chứng minh.

298
F

X
A

E
Y'
Z'

O
P

t Z C
B D

Y
K

Gọi X 0 , Y 0 , Z 0 lần lượt là hình chiếu của X, Y, Z trên BC, CA, AB. D, E, F lần lượt là giao của BC
và Y Z, AC và XZ, AB và XY . Gọi H, K lần lượt là giao của AB và OY 0 , AC và OZ 0 .
Do O là trực tâm của tam giác AHK nên AO ⊥ HK. Mà AO ⊥ Y Z nên Y Z k HK. Lại có
HZ 0 Y 0 K là tứ giác nội tiếp nên áp dụng định lý Reim suy ra Y, Z, Y 0 , Z 0 cùng nằm trên đường tròn
ωx . Tương tự có ωy , ωz .
Áp dụng định lý Dergiades cho tam giác XY Z và 3 đường tròn ωx , ωy , ωz suy ra D, E, F thẳng
hàng. Theo định lý Desargues ta có hai tam giác ABC và XY Z thấu xạ.
Trở lại định lý Sondat.

A A1
A2

Q P

P' C1
C'2
C2

B
C
B1 B'2
B2

Gọi A2 là điểm nằm trên AA1 sao cho P A2 ⊥ BC, B2 , C2 là hai điểm trên BB1 , CC1 sao cho
A2 B2 k A1 B1 , A2 C2 k A1 C1 .
Do A1 A2 , B1 B2 , C1 C2 đồng quy tại O nên hai tam giác A1 B1 C1 và A2 B2 C2 vị tự theo tâm O. Suy
ra B1 C1 k B2 C2 . P là tâm trực giao của tam giác A2 B2 C2 ứng với tam giác ABC.
Gọi D, E, F là giao điểm của B2 C2 với BC, A2 C2 với AC, A2 B2 với AB. Do hai tam giác A2 B2 C2
và ABC thấu xạ nên theo định lý Desargues, D, E, F thẳng hàng.

299
Qua P kẻ đường thẳng vuông góc với AC, AB cắt A2 B2 , A2 C2 lần lượt tại B20 , C20 . Do các đường
vuông góc kẻ từ B2 tới AC, C2 tới AB cắt nhau tại một điểm P 0 trên A2 P nên hai tam giác P B20 C20 và
P 0 B2 C2 vị tự theo tâm A2 . Suy ra B20 C20 k B2 C2 và do đó P là tâm trực giao của hai tam giác A2 B20 C20
và ABC. Theo bổ đề trên suy ra A2 B20 C20 và ABC thấu xạ. Theo định lý Desargues, giao điểm D0 của
B20 C20 với BC nằm trên EF . Mà D và D0 cùng nằm trên BC nên D ≡ D0 hay hai tam giác ABC và
A2 B2 C2 có chung tâm trực giao P.
Ta có hai tam giác A2 P B2 và A1 QB1 có cạnh tương ứng song song nên vị tự theo tâm O. Suy ra
O, P, Q thẳng hàng.

Nhận xét. Tại [1], kết luận A2 B2 C2 và ABC có chung tâm trực giao được đưa ra sau khi chứng
minh P là tâm trực giao của A2 B2 C2 ứng với ABC. Điều này có vẻ không hiển nhiên khi tôi nhận
thấy việc chứng minh nó khá khó khăn.

34 Sáng tạo trong hình học


34.1 Mở đầu
Hình học là một mảng rất đặc biệt trong toán học. Vẻ đẹp của phân môn này nằm trong hình vẽ
mà muốn cảm nhận được chúng đòi hỏi người đọc phải có một cái nhìn tinh tế, một trí tưởng tượng
phong phú. Điều đó thuộc về năng khiếu mà không phải ai cũng có được. Vì vậy nói về câu chuyện
sáng tạo một bài toán hình học như thế nào thực sự rất khó, không thể giải thích tường tận cho người
đọc rằng bài toán đã được tìm ra như thế nào, tại sao lại biết cách để dựng ra một mô hình như vậy.
Tuy nhiên vẫn có thể có lời giải thích cho nhiều bài toán. Bài viết này hi vọng sẽ giúp bạn đọc thay
đổi cách nhìn trong hình học và hỗ trợ phần nào trong việc sáng tác ra một bài toán mới.

34.2 Sáng tạo ra bài toán mới như thế nào?


34.2.1 Lấy cảm hứng từ một mô hình đã biết
Rất nhiều bài toán khác nhau được sinh ra từ một dạng mô hình giống nhau. Lấy cảm hứng từ
một mô hình đã biết tức là tìm ra những tính chất khác xung quanh hình vẽ của một bài toán, hoặc
một dạng kết luận tương tự với bài toán đã biết.
Chúng ta hãy cùng xem xét một số ví dụ sau.

Bài 339. (Đường tròn Hagge) Cho tam giác ABC nội tiếp đường tròn (O) với trực tâm H. Gọi P là
điểm bất kì trên mặt phẳng. Các đường thẳng AP, BP, CP giao (O) lần thứ hai tại A1 , B1 , C1 . Gọi
A2 , B2 , C2 lần lượt là các điểm đối xứng với A1 , B1 , C1 qua các cạnh BC, CA, AB. Chứng minh rằng
H, A2 , B2 , C2 cùng thuộc một đường tròn.

300
A

B1
A2
C1 H O
P
C2

B C
B2

A1

Sau đây là 3 bài toán gần giống với đường tròn Hagge.

Bài 340. Cho tam giác ABC nội tiếp đường tròn (O) với trực tâm H. Gọi A1 , B1 , C1 lần lượt là các
điểm đối xứng với A, B, C qua O, P là điểm bất kì trên mặt phẳng. Gọi A2 B2 C2 là tam giác pedal
của P ứng với 4ABC, A3 , B3 , C3 lần lượt là các điểm đối xứng với A1 , B1 , C1 qua A2 , B2 , C2 . Chứng
minh rằng H, A3 , B3 , C3 cùng thuộc một đường tròn.

C1 B1

A3 H O B2
C2

B3
P

B A2 C
C3

A1

Bài 341. Cho tam giác ABC nội tiếp đường tròn (O) với trực tâm H. Gọi P là điểm bất kì trên mặt
phẳng, A1 , B1 , C1 lần lượt là giao của AP, BP, CP qua O, . Gọi A2 B2 C2 là tam giác pedal của P ứng
với 4ABC, A3 , B3 , C3 lần lượt là các điểm đối xứng với A1 , B1 , C1 qua A2 , B2 , C2 . Chứng minh rằng
H, A3 , B3 , C3 cùng thuộc một đường tròn.

301
A

B3

C2 C3 B2
O
C1 B1
H A3
P

B A2 C

A1

Bài 342. Cho tam giác ABC nội tiếp đường tròn (O) với trực tâm H. Gọi P là điểm bất kì nằm trên
OH, A1 , B1 , C1 lần lượt là giao của AP, BP, CP qua O, . Gọi A2 , B2 , C2 lần lượt là chân đường cao hạ
từ A, B, C của 4ABC, A3 , B3 , C3 lần lượt là các điểm đối xứng với A1 , B1 , C1 qua A2 , B2 , C2 . Chứng
minh rằng H, A3 , B3 , C3 cùng thuộc một đường tròn có tâm nằm trên OH.

B1
B2
B3

C1
A3 C2 H
P O
C3

B A2 C

A1

Đến với một ví dụ khác.

Bài 343. Cho hai đường tròn (O1 ) và (O2 ) ngoài nhau. Gọi d1 và d2 là tiếp tuyến chung ngoài của
hai đường tròn, P là điểm bất kì trên d1 , Q là điểm bất kì trên d2 . Từ P kẻ hai tiếp tuyến l1 , l2 khác
d1 tới (O1 ), (O2 ), từ Q kẻ hai tiếp tuyến l3 , l4 khác d2 tới (O1 ), (O2 ). Chứng minh rằng l1 , l2 , l3 , l4 cắt
nhau tạo thành một tứ giác ngoại tiếp.

Bài toán này không khó, chỉ cần một số phép cộng đoạn thẳng rồi sử dụng định lý Pythot. Tuy
nhiên sau một hồi biến đổi hình vẽ tôi tìm ra bài toán khá hay và lạ sau.

Bài 344. Cho hai đường tròn (O1 ) và (O2 ) cùng tiếp xúc trong với đường tròn (O) tại A và B. Từ A
kẻ hai tiếp tuyến d1 , d2 tới (O2 ). Từ B kẻ hai tiếp tuyến d3 , d4 tới (O1 ). Chứng minh rằng d1 , d2 , d3 , d4
cắt nhau tạo thành một tứ giác ngoại tiếp.

302
X
A B

O1 O2

Y
O

Bài 345. (Tổng quát IMO 2009). Cho tam giác ABC có tâm ngoại tiếp O. Gọi P, Q là hai điểm bất
kì trên AC, AB. Gọi M, N, J lần lượt là trung điểm của BP, CQ, P Q; R là hình chiếu của O trên P Q.
Chứng minh rằng M, N, R, J cùng thuộc một đường tròn.
Quan sát hình vẽ thấy P Q là một đường thẳng đi qua AB, AC. Tôi cho đường thẳng P Q cắt BC
và tìm các tính chất xung quanh mô hình. Kết quả thu được khá đặc sắc mà sau này trở thành đề thi
IMO Shortlist 2012. Có thể bạn đọc sẽ thấy hai bài toán này không có gì liên quan đến nhau nhưng
đó chính là cách tôi đã tìm ra bài toán thứ 2.
Bài 346. Cho tam giác ABC có tâm ngoại tiếp O. Một đường thẳng d bất kì cắt BC, CA, AB lần
lượt tại X, Y, Z. Gọi P là hình chiếu của O trên d. Chứng minh rằng các đường tròn ngoại tiếp các
tam giác AXP, BY P, CZP đồng trục.

Y
P
Z
O

X B C

Bài 347. Cho tứ giác ABCD ngoại tiếp đường tròn (O). Tia OC cắt đường tròn ngoại tiếp tam giác
ABD tại E, tia OA cắt đường tròn ngoại tiếp tam giác BCD tại F . Chứng minh rằng tứ giác BEDF
ngoại tiếp.
Bài toán trên được gửi bởi một thành viên trên diễn đàn AoPS, đáng tiếc bài toán này sau khi
kiểm tra thì không đúng. Tuy nhiên dựa vào mô hình trên lại thu được bài toán mới khá thú vị.
Bài 348. Cho tứ giác ABCD ngoại tiếp đường tròn (O). Đường thẳng AC lần lượt cắt đường tròn
ngoại tiếp tam giác ABD và BCD lần thứ hai tại E, F . Chứng minh rằng tứ giác BEDF ngoại tiếp.

303
B

E
O
C

Chúng ta cùng đến với một tính chất quen thuộc liên quan đến định lý Sawayama-Thebault.
Bài 349. Cho tam giác ABC nội tiếp đường tròn (O), ngoại tiếp đường tròn (I). D là một điểm bất
kì trên BC. Gọi (O1 ) là đường tròn tiếp xúc với tia DB, DA và tiếp xúc trong với (O), (O2 ) là đường
tròn tiếp xúc với tia DC, DA và tiếp xúc trong với (O). Chứng minh rằng (O1 ), (O2 ), (I) có chung
một tiếp tuyến khác BC.
Bài toán trên là hệ quả trực tiếp của định lý Sawayama-Thebault, phát biểu rằng O1 , I, O2 thẳng
hàng.
Bây giờ thay vì chọn điểm D bất kì trên BC ta chọn điểm P bất kì trên (O), kết quả thu được
khá thú vị.
Bài 350. Cho tam giác ABC nội tiếp đường tròn (O). P là điểm bất kì trên cung BC không chứa A.
Chứng minh rằng đường tròn A-mixtilinear nội tiếp của tam giác ABC, các đường tròn P -mixtilinear
nội tiếp của các tam giác P AB, P AC có chung một tiếp tuyến.

B C

34.2.2 Đặc biệt hóa


Tư tưởng chính của phép đặc biệt hóa là từ một bài toán đã biết, lấy một trường hợp đặc biệt của
hình vẽ rồi tạo ra bài toán mới.
Chúng ta cùng xem xét một số ví dụ sau.

304
Bài 351. Cho tứ giác ABCD nội tiếp đường tròn (O). Gọi I1 , I2 , I3 , I4 lần lượt là tâm đường tròn nội
tiếp các tam giác ABC, BCD, CDA, DAB. Chứng minh rằng I1 I2 I3 I4 là hình chữ nhật.
Bài 352. Cho tứ giác ABCD ngoại tiếp đường tròn (O). Gọi P là giao điểm của AC và BD; I1 , I2 , I3 , I4
lần lượt là tâm đường tròn nội tiếp các tam giác AP B, BP C, CP D, DP A. Chứng minh rằng I1 , I2 , I3 , I4
cùng thuộc một đường tròn.
Hai bài toán trên khá giống nhau, kết hợp chúng lại làm một ta thu được bài toán sau.
Bài 353. Cho tứ giác lưỡng tâm ABCD. Gọi P là giao điểm của hai đường chéo AC và BD. Chứng
minh rằng tâm nội tiếp của 8 tam giác ABC, BCD, CDA, DAB, AP B, BP C, CP D, DP A cùng
thuộc một đường tròn.

Bài 354. Cho tam giác ABC. Một đường thẳng d vuông góc với đường thẳng Euler của tam giác
ABC cắt 3 cạnh BC, CA, AB lần lượt tại X, Y, Z. Chứng minh rằng tâm đường tròn Euler của các
tam giác AY Z, BXZ, CXY lần lượt nằm trên các đường cao tương ứng của tam giác ABC.
Ta cho đường thẳng d chạy và thử xét bài toán trong trường hợp d đi qua đỉnh A. Khi đó tâm
đường tròn Euler của các tam giác ABX, ACX lần lượt nằm trên đường cao hạ từ B và C. Từ đây ta
lại thử liên hệ với đường tròn Euler của tam giác ABC xem có gì đặc biệt và thu được bài toán mới.
Bài 355. Cho tam giác ABC nội tiếp đường tròn (O) với trực tâm H. Đường thẳng qua A vuông góc với
OH cắt BC tại X. Gọi E, Eb , Ec lần lượt là tâm đường tròn Euler của các tam giác ABC, ABX, ACX.
Chứng minh rằng E, Eb , Ec , H cùng thuộc một đường tròn.

H E O
Ec
Eb

B X C

Bài 356. Cho 5 điểm A1 , A2 , A3 , A4 , A5 nằm trên mặt phẳng và một điểm P bất kì sao cho P không
nằm trên đường tròn ngoại tiếp của mỗi 3 điểm trong 5 điểm trên. Khi đó đường tròn pedal của P
ứng với các tam giác A1 A2 A3 , A2 A3 A4 , A1 A3 A4 , A1 A2 A4 đồng quy tại X5 . Tương tự ta cũng có các
điểm X1 , X2 , X3 , X4 . Khi đó X1 , X2 , X3 , X4 , X5 cùng thuộc một đường tròn.

305
Bài toán trên là một trường hợp riêng của một dạng phát biểu cho chuỗi đường tròn Clifford ứng
với 5 điểm. Bây giờ thay vì 5 điểm bất kì trên mặt phẳng ta cho 5 điểm A1 , A2 , A3 , A4 , A5 cùng thuộc
một đường tròn có tâm O và cho P trùng O. Từ đó thu được một bài toán thú vị.

Bài 357. Cho ngũ giác ABCDE nội tiếp đường tròn (O). Gọi A0 , B 0 , C 0 , D0 , E 0 lần lượt là giao
điểm thứ hai không nằm trên cạnh ngũ giác ABCDE của đường tròn Euler của các tam giác
ABC, BCD, CDE, DEA, EAB. Chứng minh rằng A0 , B 0 , C 0 , D0 , E 0 cùng thuộc một đường tròn.

A
D' E'

C'
A'

B'

E D

34.2.3 Tổng quát hóa


Ngược lại với đặc biệt hóa, tổng quát là việc làm cho bài toán trở nên mở rộng hơn, bao hàm bài
toán cũ. Chúng ta hãy thử tìm hiểu phương pháp này qua một số ví dụ sau.

Bài 358. (Đường thẳng Newton của tứ giác ngoại tiếp). Cho tứ giác ABCD ngoại tiếp đường tròn
(O). Khi đó O nằm trên đường thẳng nối hai trung điểm của AC và BD.

Gọi các tiếp điểm của AB, BC, CD, DA lần lượt là X, Y, Z, T . Ta có OX, OY, OZ, OT lần lượt
vuông góc với 4 cạnh của tứ giác ABCD. Thay vì chọn O là tâm của đường tròn chúng ta sẽ chọn
một điểm P bất kì nằm trong đường tròn và thu được bài toán sau.

Bài 359. Cho tứ giác ABCD nội tiếp đường tròn (O), P là điểm bất kì nằm trong (O). Gọi XY ZT
là tứ giác tạo bởi giao điểm của các đường thẳng qua A và vuông góc với P A, qua B và vuông góc với
P B, qua C và vuông góc với P C, qua D và vuông góc với P D. Khi đó O nằm trên đường thẳng nối
hai trung điểm XZ và Y T.

306
X
B
A

Y
N
T
O
P
M

D C

Hoàn toàn tương tự ta cũng có thể mở rộng định lý Brianchon cho lục giác ngoại tiếp.
Bài 360. (Định lý Brianchon). Cho lục giác ABCDEF ngoại tiếp. Khi đó AD, BE, CF đồng quy.
Mở rộng.
Bài 361. Cho lục giác ABCDEF nội tiếp đường tròn (O). P là điểm bất kì nằm trong (O). Gọi
XY ZT RS là lục giác tạo bởi giao điểm của các đường thẳng qua A vuông góc với P A, qua B vuông
góc với P B, ..., qua F vuông góc với P F . Khi đó XT, Y R, ZS đồng quy.

X B Y
A

S
C

Z
F

R
D

E
T

Bài 362. (IMO 2011). Cho tam giác nhọn ABC nội tiếp đường tròn ω. Gọi t là tiếp tuyến bất kì của
ω, ta , tb , tc lần lượt là các đường thẳng đối xứng với t qua BC, CA, AB. Chứng minh rằng đường
tròn ngoại tiếp tam giác được tạo bởi giao điểm của các đường thẳng ta , tb , tc tiếp xúc với ω.

Gọi A0 B 0 C 0 là tam giác tạo bởi giao điểm của ta , tb , tc . Ý tưởng quan trọng có trong lời giải bài
toán là chứng minh AA0 , BB 0 , CC 0 đồng quy tại một điểm I trên (O). Xuất phát từ ý tưởng này ta có
thể mở rộng bài toán như sau.
Bài 363. Cho tam giác ABC nội tiếp đường tròn (O). Một đường tròn (O0 , R0 ) tiếp xúc trong với
(O) tại I sao cho R < R0 . P là một điểm bất kì trên (O). Các tia P A, P B, P C lần lượt cắt (O0 ) tại
A1 , B1 , C1 . Gọi A2 B2 C2 là tam giác tạo bởi giao điểm của các đường thẳng đối xứng với B1 C1 qua
BC, C1 A1 qua CA, A1 B1 qua AB. Chứng minh rằng đường tròn ngoại tiếp tam giác A2 B2 C2 cũng
tiếp xúc với (O).

307
A1 A A2

I
C2 P

B2
B C
B1

C1

Tiện đây chúng ta cũng xem xét một bài toán và ý tưởng chứng minh khá giống với bài toán IMO
2011, có thể bài toán được lấy cảm hứng từ chính mô hình của IMO 2011.

Bài 364. (APMO 2014). Cho hai đường tròn (O1 ) và (O2 ) giao nhau tại hai điểm A, B. Gọi M là
điểm chính giữa cung AB của (O1 ) sao cho M nằm trong (O2 ). Dây cung M P của (O1 ) cắt (O2 ) tại Q
sao cho Q nằm trong (O1 ). Gọi l1 , l2 lần lượt là tiếp tuyến của (O1 ) tại P và (O2 ) tại Q. Chứng minh
rằng tam giác tạo bởi giao điểm của các đường thẳng l1 , l2 , AB tiếp xúc với (O2 ).

O1 M O2
Q
N

Y
P

X
Z

Hãy cùng nhìn lại bài toán 4. Khi mới được tìm ra, thực sự đó là một bài toán rất khó và không
có lời giải. Một thời gian khá dài sau trên diễn đàn AoPS có một lời giải sơ cấp của tác giả Kostas
Vittas sử dụng tới 6 bổ đề. Khi đó trong lúc bế tắc, tôi đã thử tổng quát bài toán và hóa ra trong
trường hợp tổng quát nó trở nên khá đơn giản.

Bài 365. Cho tam giác ABC nội tiếp đường tròn (O). Gọi X, Y là hai điểm bất kì nằm trong (O)
sao cho O nằm trên XY . AX, BX, CX lần lượt cắt (O) lần thứ hai tại A1 , B1 , C1 , AY, BY, CY cắt
(O) lần thứ hai tại A2 , B2 , C2 . Gọi A3 , B3 , C3 lần lượt là điểm đối xứng với A2 , B2 , C2 qua trung điểm
A1 X, B1 Y, C1 Z. Khi đó A3 , B3 , C3 , X cùng nằm trên một đường tròn có tâm nằm trên XY .

308
Bây giờ chúng ta cùng nhìn bài toán 15. Chúng ta đã có 8 tâm đường tròn nội tiếp đồng viên. Một
câu hỏi đặt ra là có thể tổng quát cho những đa giác lưỡng tâm nhiều hơn 4 cạnh không? Sau một hồi
biến đổi tôi đã chứng minh bài toán đúng với ngũ giác lưỡng tâm và nhận ra rằng lời giải cho trường
hợp ngũ giác hoàn toàn áp dụng được cho trường hợp đa giác n cạnh bất kì.

Bài 366. Cho n−giác lưỡng tâm A1 A2 A3 ...An (≥ 3). Kí hiệu Ii là tâm đường tròn nội tiếp của tam
giác Ai−1 Ai Ai+1 ; Ai(i+1) là giao điểm của Ai Ai+2 và Ai−1 Ai+1 ; Ii(i+1) là tâm đường tròn nội tiếp của
tam giác Ai Ai(i+1) Ai+1 (i = 1, n). Khi đó 2n điểm I1 , I2 , ..., In , I12 , I23 , ..., In1 cùng thuộc một đường
tròn.

34.2.4 Nghịch đảo


Phép nghịch đảo là một trong những công cụ rất mạnh của hình học. Nó biến một mô hình đã biết
thành một mô hình mới mà thoạt nhìn có thể không hề liên quan đến bài toán cũ. Như vậy có thể tìm
ra bài toán mới bằng cách nghịch đảo từ bài toán đã biết, sau đó lại sử dụng các phép đặc biệt hóa,
tổng quát hóa để "chế biến". Có nhiều bài toán rất đẹp được tạo ra từ phép nghịch đảo, tuy nhiên
theo quan điểm của tác giả bài viết thì sử dụng phương pháp nghịch đảo để chế bài không hay bằng
các phương pháp trên do nó đã một phần che dấu đi bản chất vấn đề.
Một ví dụ điển hình cho phép nghịch đảo là định lý về điểm Miquel của tam giác và định lý Miquel
về 6 đường tròn.

Bài 367. Cho tam giác ABC. A1 , B1 , C1 lần lượt là các điểm bất kì trên các cạnh BC, CA, AB. Khi
đó đường tròn ngoại tiếp các tam giác AB1 C1 , A1 BC1 , A1 B1 C đồng quy tại một điểm gọi là điểm
Miquel của tam giác ABC ứng với bộ 3 điểm A1 , B1 , C1 .

Bây giờ ta chọn một điểm P bất kì trên mặt phẳng sao cho P không nằm trên các cạnh của tam
giác ABC và nằm ngoài các đường tròn (AB1 C1 ), (A1 BC1 ), (A1 B1 C). Sử dụng phép nghịch đảo cực
P phương tích bất kì, bài toán trên biến thành bài toán sau:

Bài 368. (Bài toán 6 đường tròn). Cho 4 điểm A, B, C, D theo thứ tự nằm trên đường tròn (O).
Gọi (O1 ), (O2 ), (O3 ), (O4 ) lần lượt là các đường tròn bất kì qua các cặp điểm (A, B), (B, C), (C, D),
(D, A); B 0 , C 0 , D0 , A0 là giao điểm thứ hai của các cặp đường tròn (O1 ) và (O2 ), (O2 ) và (O3 ), (O3 ) và
(O4 ), (O4 ) và (O1 ). Khi đó A0 , B 0 , C 0 , D0 cùng thuộc một đường tròn.

B
A

A' B'

D' C'

D C

309
34.3 Thế nào là một bài toán đẹp?
Chúng ta đã điểm qua 4 phương pháp chính trong việc sáng tạo ra bài toán mới. Chế một bài toán
không khó nhưng để tìm ra một kết quả đẹp, một bài toán hay thì không đơn giản. Cần một chút kinh
nghiệm, một chút tinh tế trong quan sát và tất nhiên là một chút may mắn. Bản thân tác giả thích
sử dụng phương pháp 1 nhất khi nó có thể tạo ra những kết quả đẹp và lạ đến bất ngờ. Tuy nhiên
phương pháp 1 khá khó áp dụng và hay dẫn đến ngõ cụt.
Để kết thúc bài viết chúng ta sẽ bàn luận một chút về bài toán hình học đẹp. Theo quan điểm
cá nhân thì tôi cho rằng một bài toán đẹp là bài toán có cách phát biểu ngắn gọn, dễ hiểu, dễ tưởng
tượng. Kết luận cần chứng minh cũng độc đáo lạ mắt. Tuy nhiên nó phải đảm bảo hai yếu tố. Một là
kết luận không quá hiển nhiên vì khi đó người ra đề đã thất bại trong việc sáng tác. Có nhiều bài toán
nhìn qua thì phát biểu khá đẹp, cách giải của người ra đề cũng khá phức tạp, tưởng như thành công
trong phân loại người giải bài. Tuy nhiên đến khi xem lời giải của người làm thì mới nhận ra mình đã
bị hớ vì bài toán của họ quá hiển nhiên. Hai là các dữ liệu của bài toán không được kết hợp cơ học,
có nghĩa là ghép nối một cách không khéo léo các tính chất của bài toán nọ với bài toán kia để tạo ra
bài toán mới. Những bài toán sinh ra kiểu này nhìn rất khó chịu vì trong quá trình giải phải chia nhỏ
ra mấy bài toán. Vấn đề thực chất bị che giấu nên giải những bài toán như vậy rất mất thời gian và
không thu được nhiều kinh nghiệm.
Vì vậy một lời khuyên tới bạn đọc là không nên mất thời gian cho những bài toán quá "trâu bò"
và rắc rối trong cách phát biểu mà nên làm những bài toán đẹp, có ý nghĩa và mang tính chìa khóa
cho những bài toán khác.
Chúc các bạn thành công!

310
PHẦN IV: BÀI TẬP TỔNG HỢP

35 Bài tập tổng hợp


Bài 369. Cho tam giác ABC nội tiếp đường tròn (O), ngoại tiếp đường tròn (I). Gọi P là tâm vị tự
ngoài của (O) và (I). Chứng minh rằng AP ⊥ OI khi và chỉ khi OI k BC.
Chứng minh. Cách 1.
A
F

P I O

J
B E C

Gọi D là điểm chính giữa cung BC không chứa A. E là tiếp điểm của (I) với BC. F là giao điểm
thứ hai khác A của (AI) với (O). AF cắt BC tại J.
Do JF.JA = JB.JC nên J nằm trên trục đẳng phương của (AI) và (BIC), mà hai đường tròn
này tiếp xúc nhau tại I nên JI ⊥ AI.
Gọi F 0 là giao của DE và (O). Suy ra DE.DF 0 = DB 2 = DI 2 , từ đó ∠DIE = ∠IF 0 E.
Mà ∠DIE = IJE (cùng bằng 90◦ − ∠JIE) nên ∠IF 0 E = ∠IJE hay ∠JF 0 I = 90◦ . Từ đó F 0 ≡ F .
Dễ thấy DE đi qua P nên F, P, E, D thẳng hàng.
Ta có ∠AP I = 90◦ khi và chỉ khi P ∈ (AI). Tương đương ∠P IJ = ∠P F I = ∠IJE hay OI k BC.
Cách 2.
L
A

P I
O

B C

K
D

Gọi K là giao của AP với (O) thì K là tiếp điểm của đường tròn A-mixtilinear với (O). Suy ra KI
đi qua L là điểm chính giữa cung BAC.
Ta có AP ⊥ OI khi và chỉ khi P là trung điểm AK. Mà O là trung điểm DL nên điều này tương
đương 4AP I ∼ 4LOI hay ∠IOL = 90◦ . Tương đương OI k BC.

311
Bài 370. Cho tứ giác ABCD nội tiếp đường tròn (O) có DA, BC không song song. P là giao điểm
của các đường chéo AC, BD. AD giao BC tại K. Các điểm M, N theo thứ tự chạy trên các đoạn
DM BN
DA, BC sao cho = . M N theo thứ tự cắt AC, BD tại Q, R. Chứng minh rằng đường tròn
DA BC
(P QR) tiếp xúc với (KM N ).
K

P
R
M Q

N
O
T

D
C

Chứng minh. Gọi T là giao điểm thứ hai của (ADP ) và (BCP ). Dễ thấy 4AT D ∼ 4CT B và
DM BN
= nên 4DT M ∼ 4BT N.
DA BC
Từ đó 4DT B ∼ 4M T N , suy ra ∠RDT = ∠RM T hay T ∈ (RM D). Như vậy T là điểm Miquel
của tứ giác toàn phần AM RP DQ, suy ra (RP Q) đi qua T.
Mặt khác do T là điểm Miquel của tứ giác toàn phần KAP BDC nên ∠M T N = ∠DT B =
180◦ − ∠K. Suy ra T ∈ (KM N ).
Mà ∠M T R = ∠ADP = ∠BCP = ∠QT N nên T R, T Q đẳng giác trong ∠M T N . Từ đó kẻ tiếp
tuyến t của (KM N ) tại T , bằng cộng góc dễ dàng chứng minh t cũng là tiếp tuyến của (P QR). Ta có
đpcm.

Bài 371. (Chọn đội tuyển THPT chuyên Sư Phạm 2014) Cho tam giác ABC. D là một điểm chuyển
động trên BC. Gọi (I1 ), (I2 ) lần lượt là đường tròn nội tiếp các tam giác ADB, ADC; EF là tiếp
tuyến chung ngoài khác BC của (I1 ) và (I2 ) (E ∈ (I1 ), F ∈ (I2 )). Chứng minh rằng khi D chuyển
động, giao điểm của BE và CF thuộc một đường tròn cố định.

J
F

I
E
I2

I1

B D C

312
Chứng minh. Gọi (I) là đường tròn nội tiếp tam giác ABC. r, r1 , r2 lần lượt là bán kính của (I), (I1 ), (I2 ).
r r
Xét phép vị tự HB1 : E 7→ J, HC2 : F 7→ J 0 .
r r

Do đó tiếp tuyến tại J của (I) song song với tiếp tuyến tại E của (I1 ), tiếp tuyến tại J 0 của (I)
song song với tiếp tuyến tại F của (I2 ). Nghĩa là J ≡ J 0 . Vậy giao điểm của BE và CF nằm trên (I)
cố định.

Bài 372. (Nguyễn Văn Linh) Cho tam giác ABC ngoại tiếp đường tròn (I). D, E là hai điểm nằm
trên cạnh BC. Gọi (I1 ), (I2 ) là đường tròn nội tiếp các tam giác ABD, ACE. (I) tiếp xúc với AB, AC
tại X, Y . P là một điểm chuyển động trên cung nhỏ XY . P B, P C lần lượt giao (I1 ), (I2 ) tại L, K sao
cho giao điểm còn lại của P B với (I1 ), P C với (I2 ) nằm giữa B, L và nằm giữa C, K. Chứng minh
rằng khi P chuyển động, giao điểm của LD và KE nằm trên một đường tròn cố định.

P
Y
X
I K
L

I1 I2

B E C
D
J

Chứng minh. Gọi (J) là đường tròn bàng tiếp góc A của tam giác ADE. r, r1 , r2 , ra lần lượt là bán
kính của (I), (I1 ), (I2 ), (J).
r1 r2
Xét phép vị tự HBr : P 7→ L, HCr : P 7→ K.
Suy ra P I k I1 L k I2 K.
− rra − rra
Lại xét HD 1 : L 7→ Q, HE 2 : L 7→ Q0 .
Suy ra LI1 k JQ, KI2 k JQ0 hay Q0 ≡ Q. Như vậy giao điểm của LD và KE nằm trên (J) cố
định.

Bài 373. (Nguyễn Văn Linh) Cho tam giác ABC nội tiếp đường tròn (O) với trực tâm H. Đường
thẳng qua A vuông góc với OH cắt BC tại D. Gọi E, Eb , Ec lần lượt là tâm đường tròn Euler của các
tam giác ABC, ABD, ACD. Chứng minh rằng E, Eb , Ec , H cùng thuộc một đường tròn.

313
A

T Hb

Mc S Mb

Hc E
H
Eb Ec O

B Ha D M Ma C

Chứng minh. Gọi Ma , Mb , Mc lần lượt là trung điểm BC, CA, AB; Ha , Hb , Hc là chân các đường cao
của tam giác ABC. AD giao Mb Mc tại S, T là trung điểm AHa .
Qua E kẻ đường song song với AD cắt BC tại M . Từ M kẻ M Eb0 ⊥ BHb , M Ec0 ⊥ BHc . Ta sẽ
chứng minh Eb0 ≡ Eb , Ec0 ≡ Ec .
Do Eb0 , E, Ha cùng nằm trên đường tròn đường kính HM nên ∠Eb0 EHa = ∠Eb0 HHa = ∠Mc T Ha =
1
∠Mc EHa .
2
Do đó EEb0 là phân giác ∠Mc EHa . Mà EMc = EHa nên Eb0 Mc = Eb0 Ha .
Ta có ∠Mc Eb0 Ha = 360◦ − 2∠EEb0 Ha = 2∠EM Ha = 2∠EOMa .
Do AD ⊥ EO, Mb Mc ⊥ OMa nên ∠EOMa = ∠ASMc , suy ra ∠Mc Eb0 Ha = 2∠ASMc = 2∠Mc SD.
Điều này nghĩa là Eb0 là tâm của đường tròn Euler của tam giác ADB hay Eb0 ≡ Eb .
Chứng minh tương tự, Ec0 ≡ Ec . Từ đó H, E, Eb , Ec cùng nằm trên đường tròn đường kính M H.

Bài 374. (Nguyễn Văn Linh) Cho tam giác ABC. D là một điểm bất kì trên BC. Gọi (O1 ) là đường
tròn tiếp xúc với AB, BC và tiếp xúc ngoài với đường tròn ngoại tiếp tam giác ADC, (O2 ) là đường
tròn tiếp xúc với AC, BC và tiếp xúc ngoài với đường tròn ngoại tiếp tam giác ADB. Gọi E, F lần
lượt là tiếp điểm của (O1 ) và (O2 ) với BC, G là tiếp điểm của (O1 ) với (ADC), H là tiếp điểm của
(O2 ) với (ADB). Chứng minh rằng đường tròn ngoại tiếp các tam giác BHF, CGE và đường tròn
đường kính BC đồng quy.

Chứng minh. Trước tiên ta phát biểu một bổ đề.


Bổ đề 1. Cho 4 điểm A, B, C, D theo thứ tự cùng nằm trên một đường thẳng. Gọi ω1 , ω2 , ω3 , ω4
lần lượt là đường tròn bất kì qua các cặp điểm (A, B), (B, C), (C, D), (D, A). X, Y, Z, T lần lượt là giao
điểm thứ hai của các cặp đường tròn ω1 và ω2 , ω2 và ω3 , ω3 và ω4 , ω4 và ω1 . Khi đó X, Y, Z, T cùng
thuộc một đường tròn.
Chứng minh.

314
Z

T Y
X

A B C D

Bổ đề 1 có thế chứng minh bằng một số phép cộng góc đơn giản hoặc xét phép nghịch đảo cực A
phương tích bất kì biến bài toán thành định lý điểm Miquel của tam giác.
Trở lại bài toán.

K H O2
O1 G

B E
D F C

Gọi K là tiếp điểm của (O1 ) với AB. (AKG) cắt (CEG) lần thứ hai tại I. AI, CI lần lượt cắt EK
tại N, M.
Áp dụng định lý điểm Miquel cho tam giác AKN với bộ 3 điểm K, E, I ta có (AKI) cắt (KKE)
tại G nên G là điểm Miquel của tam giác AKN ứng với 3 điểm K, E, I, hay N ∈ (CEG). Tương tự,
M ∈ (AKG).

315
Gọi C 0 là giao của (AM N ) và (IEN ). Áp dụng bổ đề 1 cho 4 điểm M, K, E, N và 4 đường tròn
(M KI), (O1 ), (N EI), (M AN ) ta thu được (AGC 0 ) tiếp xúc với (O1 ). Từ đó suy ra C 0 ≡ C hay tứ giác
A, M, N, C nội tiếp.
Suy ra ∠IAB = ∠IM K = ∠IAC, ∠ICB = ∠IEN = ∠ICA hay I là tâm đường tròn nội tiếp của
tam giác ABC.
Ta có ∠AGI = ∠AM I = ∠CN I = ∠IGC hay GI là phân giác ∠AGC.
Tương tự, (BHF ) cũng đi qua I và HI là phân giác ∠BHA.
Gọi L là giao điểm thứ hai của (BHF ) và (CGE).
Ta có
1 1
∠BLC = ∠BLI + ∠CLI = ∠BHI + ∠CGI = (∠BHA + ∠CGA) = (∠ADB + ∠ADC) = 90◦ .
2 2
Suy ra đpcm.

Bài 375. (Nguyễn Minh Hà) Cho tam giác ABC nội tiếp đường tròn (O), trực tâm H. P là điểm bất
kì trên OH. Gọi B 0 , C 0 lần lượt là điểm đối xứng của B, C qua OH. B 0 P cắt AC tại X, C 0 P cắt AB
tại Y . Chứng minh rằng điểm đối xứng với H qua XY nằm trên (O).

Chứng minh. Ta phát biểu một số bổ đề sau.


Bổ đề 40. Cho tam giác ABC nội tiếp đường tròn (O), trực tâm H. Gọi X, Y là các điểm nằm trên
AC, AB sao cho ∠XHY = ∠BHC. Khi đó điểm đối xứng với H qua XY nằm trên (O).
Chứng minh.

H
O

Y
J
M C
B

P
K X

Gọi K là điểm nằm trên (O) sao cho K không đối xứng với H qua AB và thỏa mãn Y K = Y H.
Trung trực KH cắt AC tại X 0 .
Gọi J là trung điểm KH, M, N, P lần lượt là hình chiếu của K trên BC, CA, AB. Khi đó M, N, P, J
cùng nằm trên đường thẳng Simson của K ứng với tam giác ABC.
Ta có ∠Y KX 0 = ∠Y KJ + ∠JKX 0 = ∠AP N + ∠AN P = 180◦ − ∠BAC. Do đó ∠Y HX 0 =
∠Y KX 0 = 180◦ − ∠BAC = ∠BHC. Suy ra X 0 ≡ X. Suy ra đpcm.
Bổ đề 41. Cho tam giác ABC và hai điểm P, Q liên hợp đẳng giác. X, Y là hai điểm lần lượt nằm
trên AC, AB sao cho ∠XP Y = ∠BP C. Khi đó ∠XQY = ∠BQC.
Chứng minh.

316
A

F
P
Q

K C
B D

Gọi D, E, F lần lượt là hình chiếu của P trên BC, CA, AB. K là giao điểm thứ hai khác F của
đường tròn đường kính P Y với đường tròn pedal của P ứng với tam giác ABC. Y K cắt AC tại X 0 .
Ta có ∠F KE = ∠F DE nên ∠F KP + ∠EKP = ∠F DP + ∠EDP hay ∠AY P + ∠AX 0 P =
∠ABP + ∠ACP.
Suy ra ∠BAC + ∠AY P + ∠AX 0 P = ∠BAC + ∠ABP + ∠ACP hay ∠Y P X 0 = ∠BP C, do đó
0
X ≡ X.
Tương tự cũng suy ra ∠XQY = ∠BQC.
Bổ đề 42. Cho tam giác ABC và hai điểm P, Q liên hợp đẳng giác. L là điểm bất kì trên P Q. B 0 , C 0
lần lượt là đối xứng của B, C qua P Q. B 0 L cắt AC tại X, C 0 L cắt AB tại Y . Khi đó ∠XP Y =
∠BP C, ∠XQY = ∠BQC.
Chứng minh.

C'

B'

Q
P R E
L

M Y

B
C
X

D N

317
Dễ thấy BC 0 cắt CB 0 tại một điểm R trên P Q.
Gọi C 0 P, C 0 Q lần lượt cắt AB tại M, D; B 0 P, B 0 Q lần lượt cắt AC tại N, E.
Ta có ∠M P N = ∠B 0 P C 0 = ∠BP C.
∠DQE = ∠B 0 QC 0 = ∠BQC nên theo bổ đề 2, ∠DP E = ∠BP C.
Mặt khác, (M XBD) = C 0 (M XBD) = (P LRQ) = B 0 (P LRQ) = (N Y CE).
−−→ −−→ −−→ −−→ −−→ −−→ −−→ −−→
sin(P M , P B) sin(P X, P B) sin(P N , P C) sin(P Y , P C)
Do đó −−→ −−→ : −−→ −−→ = −−→ −−→ : −−→ −−→ (1)
sin(P M , P D) sin(P X, P D) sin(P N , P E) sin(P Y , P E)
Do các góc M P N, BP C, DP E bằng nhau nên kết hợp với (1) suy ra ∠XP B = ∠Y P C. Từ đó
∠XP Y = ∠BP C. Chứng minh tương tự suy ra ∠XQY = ∠BQC.
Trở lại bài toán.

B'

C'
H
O P

Y
C
B

H' X

Áp dụng bổ đề 3 cho hai điểm H và O liên hợp đẳng giác với P nằm trên HO. Ta có ∠XHY =
∠BHC. Do đó theo bổ đề 1 suy ra điểm đối xứng với H qua XY nằm trên (O).

Bài 376. (Trần Quang Hùng) Cho tam giác ABC nội tiếp đường tròn (O), trực tâm H. E, F thuộc
(O) sao cho EF k BC. D là trung điểm HE. Đường thẳng qua O song song với AF cắt AB tại G.
Chứng minh rằng DG ⊥ DC.

318
G

J
P
L N
Q
O

C' H
T
J
B C
D

E F

Chứng minh. Gọi C 0 là giao của CH với AB, D0 là giao điểm thứ hai khác C 0 của đường tròn đường
kính CG và đường tròn Euler. D0 H cắt (O) tại hai điểm E 0 và L sao cho D0 là trung điểm HE 0 .
Phép nghịch đảo tâm H phương tích k = HC.HC 0 biến (ABC) thành đường tròn Euler, do đó
HL.HD = HC.HC 0 hay L ∈ (GC).
Gọi N là giao của JE 0 với AB, J là điểm đối xứng với C qua O thì ∠CLG = ∠CLJ = 90◦ nên
L, J, G thẳng hàng.
Gọi Q, P, T lần lượt là trung điểm AB, LJ, E 0 J. Dễ thấy Q đồng thời là trung điểm HJ nên Q, P, T
thẳng hàng.
Mà Q, P, T lần lượt là hình chiếu của O trên N G, JG, JN nên theo định lý đảo về đường thẳng
Simson, O ∈ (GJN ).
Suy ra ∠E 0 AC = ∠E 0 JC = ∠BGO = ∠BAF hay E 0 F k BC. Tức là E 0 ≡ E. Ta có đpcm.

Bài 377. Cho tứ giác ABCD nội tiếp đường tròn (O). AC giao BD tại E. Gọi (I) là đường tròn tiếp
xúc với tia EA, ED và tiếp xúc trong với (O) tại L. M là điểm bất kì trên cung AD không chứa B, C.
Gọi I1 , I2 là tâm đường tròn nội tiếp của các tam giác M AC, M BD. Chứng minh rằng I1 , I2 , M, L
cùng thuộc một đường tròn.

319
C
N

P E

O
K I1

I2
D
I

M
L

Chứng minh. Gọi K, H lần lượt là tiếp điểm của (I) với AC, BD. KL, HL giao (O) lần thứ hai tại
P, N thì P, N là điểm chính giữa các cung AC, BD. Do đó M, I1 , P và M, I2 , N .
Bằng phép cộng góc đơn giản suy ra P N vuông góc với phân giác ∠AED hay P N k KH.
PK NH P K.P L N H.N L
Từ đó = hay 2
=
PL NL PL N L2
P A2 N D2 P I1 N I2
Suy ra = hay = .
P L2 N L2 PL NL
Ta thu được 4LP I1 ∼ 4LN I2 (c.g.c), do đó ∠LI1 P = ∠LI2 N hay L, I1 , I2 , M cùng thuộc một
đường tròn.

Bài 378. (Nguyễn Văn Linh) Cho tam giác ABC nội tiếp đường tròn (O). L là điểm bất kì trên cung
BC không chứa A. Chứng minh rằng đường tròn A-mixtilinear nội tiếp của tam giác ABC, các đường
tròn L-mixtilinear nội tiếp của các tam giác LAB, LAC có chung một tiếp tuyến.

Chứng minh. Cách 1 (Luis González).

F
N
M
E

R
O
J1 I
J2
Q
I1
J I2
B C

K
L

320
Gọi (J), (J1 ), (J2 ) lần lượt là các đường tròn A-mixtilinear nội tiếp của tam giác ABC, các đường
tròn L-mixtilinear nội tiếp của các tam giác LAB, LAC; l là tiếp tuyến chung của (J1 ) và (J2 ), cắt
(O) tại E và F. Gọi I, I1 , I2 lần lượt là tâm đường tròn nội tiếp của các tam giác LEF, LAB, LAC.
Gọi M, N là tiếp điểm của (J1 ), (J2 ) với l, R là tiếp điểm của (J1 ) với AL.
Theo định lý Sawayama-Thebault, I nằm trên J1 J2 và M R, I1 R ⊥ J1 L. Dễ thấy ∠M IN = 90◦ .
Gọi Q là giao điểm thứ hai của đường tròn đường kính M N với J1 J2 .
Do J1 M là tiếp tuyến của (M N ) nên J1 I1 .J1 L = J1 R2 = J1 M 2 = J1 Q.J1 I.
Suy ra I1 nằm trên (QIL).
Tương tự I2 cũng nằm trên (QIL). Tức là I, I1 , I2 , L cùng thuộc một đường tròn.
Gọi K là tiếp điểm của (J) với (O). Áp dụng bài toán trên cho tứ giác AABC suy ra I1 , I2 , L, K
cùng thuộc một đường tròn.
Do đó lại áp dụng bài toán trên cho tứ giác AF CE ta có (LII2 ) đi qua tiếp điểm K của đường
tròn tiếp xúc với EF, AC và (O). Mà qua điểm K chỉ có duy nhất một đường tròn tiếp xúc với AC và
tiếp xúc trong với (O), suy ra (J) tiếp xúc với EF . Ta có đpcm.
Cách 2 (Ngô Quang Dương, lớp 11A2 toán THPT chuyên KHTN)

K
I C'
O
H
J1
J2
B'
J
B F C
E

Gọi (J), (J1 ), (J2 ) lần lượt là đường tròn A− mixtilinear của tam giác ABC, P − mixtilinear của
các tam giác P AB, P AC.
(J1 ) tiếp xúc với P A, P B tại C 0 , E; (J2 ) tiếp xúc P A, P C tại B 0 , F ; (J) tiếp xúc AB, AC tại H, K.
Gọi t1 , t2 , t lần lượt là độ dài tiếp tuyến chung ngoài của (J1 ) và (J), (J2 ) và (J), (J1 ) và (J2 ).
Áp dụng định lý Casey cho 4 đường tròn (A, 0), (J1 ), (B, 0), (J) ta có:
t1 .AB = AH.BE + BH.AC 0 = AH.BE + BH.(P A − P C 0 ) = AH.BE + BH.(P A − P E) (1)
Tương tự ta có:
t2 .AC = AK.CF + CK.(P A − P F ) = AH.CF + CK.(P A − P F ) (2)
Lại áp dụng định lý Casey cho (A, 0), (J1 ), (P, 0), (J2 ) ta có:
t.AP = P E.AB 0 + P F.AC 0 = P E.(P A − P F ) + P F.(P A − P E) (3)
2AB.AC
Ta chứng minh công thức sau : AH = .
AB + BC + CA
Gọi (I, r) là đường tròn nội tiếp tam giác ABC. Theo bổ đề Sawayama suy ra I là trung điểm
r
HK. Ở đây ta dùng công thức tính bán kính ra của đường tròn A−mixtilinear: ra = 2
.
cos A/2
r ra r2 4r2 16R2 r2 4a2 b2 c2
Từ đó AH 2 = AI.AJ = · = = = = =
sin A/2 sin A/2 sin2 A/2. cos2 A/2 sin2 A a2 a2 (a + b + c)2
4b2 c2
.
(a + b + c)2

321
2AB.AC
Như vậy AH = .
AB + BC + CA
2P A.P B 2P C.P A
Tương tự ta cũng có P E = , PF = .
P A + P B + AB P C + P A + CA
Ta sẽ chứng minh t1 + t2 = t bằng biến đổi tương đương, chú ý rằng BE = P B − P E, CF =
P C − P F , BH = AB − AH, CK = AC − AH, kết hợp với (1), (2), (3) để đẳng thức cần chứng minh
chỉ còn P A, P B, P C, AB, BC, CA.
Cuối cùng sau một số phép biến đổi ta đưa về đẳng thức
BC.P B.P C + AB.BC.CA = P A.P B.AB + P A.P C.CA.
⇔ 4R.S(BP C) + 4R.S(ABC) = 4R.S(P AB) + 4R.S(P AC)
⇔ S(BP C) + S(ABC) = S(P AB) + S(P AC) = S(ABP C).
Đẳng thức cuối hiển nhiên đúng.
Do (J1 ), (J2 ) không cắt nhau nên phương tích của T đến ba đường tròn là không âm và bằng p2
đồng thời vẽ được tiếp tuyến từ T đến ba đường tròn mixtilinear.
Ta có: p.t1 + p.t2 = p.t nên theo phần đảo của định lý Casey, tồn tại đường tròn (C) đi qua T và
tiếp xúc với (J), (J1 ), (J2 ).
2
Xét phép nghịch đảo ITp : (J) 7→ (J), (J1 ) 7→ (J1 ), (J2 ) 7→ (J2 ), (C) biến thành đường
thẳng tiếp xúc với (J), (J1 ), (J2 ).
Vậy ba đường tròn có một tiếp tuyến chung

Bài 379. (Nguyễn Văn Linh) Cho hai đường tròn (O1 ) và (O2 ) cùng tiếp xúc trong với đường tròn
(O) lần lượt tại A, B. Từ A kẻ hai tiếp tuyến t1 , t2 tới (O2 ), từ B kẻ hai tiếp tuyến l1 , l2 tới (O1 ) sao
cho t1 và l1 nằm cùng một phía với đường thẳng AB. Gọi X, Y lần lượt là giao điểm của t1 và l1 , t2
và l2 . Chứng minh rằng tứ giác AXBY ngoại tiếp.

X
B
A L

I O2
O1

Y
O

Chứng minh. (Lê Thị Hải Linh-HS THPT chuyên Bắc Ninh)
Gọi O1 , O2 , O lần lượt là tâm của γ, δ, ω. AO2 giao BO1 tại I. Gọi α1 là đường tròn tâm I và tiếp
xúc với AX, AY ; α2 là đường tròn tâm I và tiếp xúc với BX, BY . OI giao AB tại L.
Áp dụng định lý Monge-D’Alembert cho 3 đường tròn δ, ω, α1 ta có A là tâm vị tự ngoài của α1
và δ, B là tâm vị tự ngoài của δ và ω, suy ra tâm vị tự ngoài của α1 và ω nằm trên AB hay L là tâm
vị tự ngoài của α1 và ω.
Chứng minh tương tự L cũng là tâm vị tự ngoài của α2 và ω. Từ đó α1 ≡ α2 hay tứ giác AXBY
ngoại tiếp.

322
Bài 380. Với kí hiệu như bài toán trên, gọi M, N lần lượt là giao của AX, AY với (O1 ), P, Q lần lượt
là giao của BX, BY với (O2 ). Gọi (I1 ), (I2 ) lần lượt là đường tròn mixtilinear incircle ứng với đỉnh A
và B của các tam giác AM N và BP Q. Chứng minh rằng (I1 ) và (I2 ) có cùng bán kính.

X P
M
B T
A K H
I2
I1 I
O2
O1 N Q
Y
O

Chứng minh. Cách 1. Gọi K, H lần lượt là tiếp điểm của (I1 ) với (O1 ), (I2 ) với (O2 ); T là giao của
O1 O2 với AB . Áp dụng định lý Monge-D’Alembert cho 3 đường tròn (O1 ), (I1 ), (I) suy ra A, K, B
thẳng hàng. Tương tự suy ra A, K, H, B thẳng hàng.
Theo bài 1 thì tứ giác AXBY ngoại tiếp đường tròn (I). Do đó R(I1 ) = R(I2 ) khi và chỉ khi
I1 I2 k AB.
II2 II1
Điều này tương đương = . (1)
I2 B I1 A
II2 HB O2 A II2 HA O2 I OO2 O2 I
Theo định lý Menelaus, · · = 1 suy ra = · = · .
I2 B HA O2 I I2 B HB O2 A O2 B O2 A
II2 OO1 O1 I
Tương tự, = · .
I1 A O1 A O1 B
OO2 O2 I OO1 O1 I
Như vậy (1) tương đương · = · .
O2 B O2 A O1 A O1 B
OO2 O1 A O1 I O2 A TA
Hay · = · , đúng vì cùng bằng .
O2 B OO1 O1 B O2 I TB
Bài toán được chứng minh.
Cách 2. Gọi R, R1 , R2 , r1 , r2 lần lượt là bán kính (O), (O1 ), (O2 ), (I1 ), (I2 ).
R
R R1 .R2
Xét phép vị tự HA 1 : (I1 ) 7→ (O2 ) suy ra r1 = . Chứng minh tương tự suy ra r1 = r2 .
R
Bài 381. (Đào Thanh Oai) Cho hình thang ABCD. E, F là hai điểm trên CD sao cho BE k AC, AF k
BD. Dựng ra ngoài hình thang các tam giác đều EF H, ADP, BCQ. Gọi Oa , Oc0 , Ob0 lần lượt là tâm
của các tam giác EF H, ADP, BCQ. Chứng minh rằng tam giác Oa Ob0 Oc0 đều.

Chứng minh.
Bổ đề 43. (ERIQ mở rộng). Cho hai đường thẳng d1 và d2 , trên d1 theo thứ tự lấy các điểm
AB A0 B 0
A, B, C, trên d2 theo thứ tự lấy các điểm A0 , B 0 , C 0 sao cho = 0 0 = k. Xét phép vị tự quay f
BC BC

323
góc α, tỉ số i có tâm lần lượt là A, B, C: fA (A0 ) = A00 , fB (B 0 ) = B 00 , fC (C 0 ) = C 00 . Khi đó A00 , B 00 , C 00
A00 B 00
thẳng hàng và 00 00 = k.
B C
Chứng minh.

A
A''
A'
B''

B C''
B2 B3
B'
B1
C3

C C'
C2

C1

Xét phép tịnh tiến T− −→ : B 7→ B1 , C 7→ C1 , T−−→ : B 7→ B2 , C 7→ C2 , T− −−→ : B 0 7→ B3 , C 0 7→ C3 .


AA0 AA00 A0 A00
Ta có 4BB1 B2 ∼ 4AA0 A00 ∼ 4BB 0 B 00 nên 4BB1 B 0 ∼ 4BB2 B 00 .
B1 B 0 BB1 CC1 C1 C 0
Chứng minh tương tự suy ra = = = .
B2 B 00 BB2 CC2 C2 C 00
B2 B 00 C2 C 00
Suy ra = . Chú ý rằng (B1 B 0 , B2 B 00 ) = (C1 C 0 , C2 C 00 ) nên ∠B 00 B2 B3 = ∠C 00 C2 C3 , tức
B2 B3 C2 C3
A00 B 00 A00 B2 AB
là A00 , B 00 , C 00 thẳng hàng đồng thời 00 00 = = .
B C B2 C2 BC
Trở lại bài toán.

J
X Ob
Oc
B
A
M
T O'b Q
P
O'c

F D N C E

Oa

324
Gọi X là giao của AF và BE. Dựng ra ngoài các tam giác đều F XJ, EXI với Oc , Ob lần lượt là
tâm của chúng.
Gọi M, N, T lần lượt là trung điểm XF, EF, BF.
Do ABDF là hình bình hành nên T là trung điểm AD.
XA XB MT 2
Ta có = = = k. Xét phép vị tự quay f góc quay 30◦ , tỉ số √ :
AF BE TN 3
Oc O0
fX (M ) = Oc , fA (T ) = Oc , fF (N ) = Oa nên theo bổ đề trên, Oa , Oc , Oc thẳng hàng và 0 c = k.
0 0
Oc Oa
Ob Ob0
Tương tự 0 = k. Áp dụng định lý Napoleon cho tam giác XEF suy ra tam giác Oa Ob Oc đều,
Ob Oa
do đó tam giác Oa Ob0 Oc0 đều.

Bài 382. (Trần Quang Hùng) Cho tam giác ABC có trọng tâm G. Gọi A0 , B 0 , C 0 lần lượt là trung
điểm của các cạnh BC, CA, AB. Trên AA0 , BB 0 , CC 0 lần lượt lấy hai cặp điểm A1 , A2 ; B1 , B2 ; C1 , C2
−−−→ −−→ −−−→ −−→ −−−→ −−→
sao cho A1 A2 = k AA0 , B1 B2 = k BB 0 , C1 C2 = k CC 0 . Qua A1 , A2 kẻ các đường vuông góc với AA0 ,
tương tự với B1 , B2 , C1 , C2 , ta thu được 6 đường thẳng. Khi đó 6 đường thẳng này theo thứ tự xoay
vòng cắt nhau tạo thành 6 điểm cùng thuộc một đường tròn.

O1
A1 O6

T B'
C' C2
G B2 O4

B1 C1
O3 V
A2

B A' C
O2 O5

Chứng minh. Gọi O1 , O2 , ..., O6 lần lượt là giao điểm của 6 đường thẳng như hình vẽ. O1 O6 cắt O2 O3
tại T , O5 O6 cắt O3 O4 tại V .
Rõ ràng T O6 k V O3 và T O3 k V O6 nên T O6 V O3 là hình bình hành.
T O6 B1 B2 BB 0
Suy ra ST O6 V O3 = T O6 · A1 A2 = T O3 · B1 B2 hay = = .
T O3 A1 A2 AA0
Mặt khác, gọi U là điểm đối xứng với G qua A0 .
Ta có ∠T O1 O2 = ∠U GC, ∠T O2 O1 = ∠C 0 GB = ∠GCU nên 4T O1 O2 ∼ 4U GC.
T O2 UC BG BB 0 T O6 T O2
Suy ra = = = 0
. Từ đó = hay T O1 · T O6 = T O2 · T O3 . Suy ra
T O1 UG AG AA T O3 T O1
O1 , O2 , O3 , O6 cùng thuộc một đường tròn. Chứng minh tương tự ta có đpcm.

Nhận xét. Bài toán trên là mở rộng của đường tròn Van Lamoen. Một cách tương tự có thể chứng
minh bài toán sau.
Bài toán. (Đào Thanh Oai) Cho hai tam giác ABC và A0 B 0 C 0 có chung trọng tâm G sao cho
AA , BB 0 , CC 0 đồng quy tại P . Khi đó tâm đường tròn ngoại tiếp của 6 tam giác AP C 0 , C 0 P B, BP A0 ,
0

A0 P C, CP B 0 , B 0 P A cùng thuộc một đường tròn.

Bài 383. (Trần Quang Hùng) Cho hai tam giác ABC và A0 B 0 C 0 có chung trọng tâm G. Trên
−−−→ −−→ −−−→ −−→
AA0 , BB 0 , CC 0 lần lượt lấy hai cặp điểm A1 , A2 ; B1 , B2 ; C1 , C2 sao cho A1 A2 = k AA0 , B1 B2 = k BB 0 ,

325
−−−→ −−→
C1 C2 = k CC 0 . Qua A1 , A2 kẻ các đường vuông góc với AA0 , tương tự với B1 , B2 , C1 , C2 , ta thu được
6 đường thẳng. Khi đó 6 đường thẳng này theo thứ tự xoay vòng cắt nhau tạo thành 6 điểm cùng
thuộc một đường tròn.

O1 A1 O6 Y
C' C2
B'
O2 B3 O5

K A3 B2
C1
C3
B1
X A2 O4
O3
B C

A'

Chứng minh. Kí hiệu O1 , O2 , ..., O6 là giao điểm của 6 đường thẳng như hình vẽ.
Gọi X, Y lần lượt là giao điểm của O1 O2 và O3 O4 , O1 O6 và O4 O5 . Dễ dàng thu được XO1 Y O4 là
hình bình hành.
XO1 A1 A2 AA0
Suy ra SXO1 Y O4 = XO1 · C1 C2 = XO4 · A1 A2 , từ đó = = .
XO4 C1 C2 CC 0
Giả sử 3 đường thẳng AA0 , BB 0 , CC 0 cắt nhau tạo thành tam giác A3 B3 C3 .
Ta có ∠XO2 O3 = ∠B3 A3 C3 , ∠XO3 O2 = ∠B3 C3 A3 . Do đó 4XO2 O3 ∼ 4B3 A3 C3 .
Mặt khác, do hai tam giác ABC và A0 B 0 C 0 có chung trọng tâm G nên
−−→0 −−→0 −−→0 −→ −−→0 −−→ −−→0 −−→ −−→0 −→ −−→ −−→ −−→ −−→ −−→
AA + BB + CC = AG+ GA + BG+ GB + CG+ GC = (AG+ BG+ CG)+(GA0 + GB 0 + GC 0 ) = 0.
Dựng hình bình hành AB 0 BK. Ta có
−−→0 −−→ −−→ −−→0 −−→0 −−→ −−→0 −−→0 −−→0 −−0→
KA = KB + BA + AA = AB + BA + AA = BB + AA = C C.
Suy ra KA0 CC 0 là hình bình hành.
Ta thu được ∠KA0 A = ∠C3 B3 A3 , ∠KAA0 = ∠B3 C3 A3 nên 4KAA0 ∼ 4A3 C3 B3 .
XO2 A0 K CC 0
Như vậy 4XO2 O3 ∼ 4A0 KA. Từ đó = 0 = .
XO3 AA AA0
XO3 XO1
Suy ra = hay XO1 · XO2 = XO3 · XO4 . Suy ra O1 , O2 , O3 , O4 cùng thuộc một đường
XO2 XO4
tròn. Chứng minh tương tự ta có đpcm.

Bài 384. Cho tam giác ABC nội tiếp đường tròn (O), trực tâm H. Chứng minh rằng đường tròn
Euler của các tam giác AOH, BOH, COH, ABC đồng quy.

Chứng minh. Cách 1.

326
A

J
Q T
L
Mc Mb

K
E O
H

B Ha Ma C

Gọi Q, J, E lần lượt là trung điểm AH, AO, HO; Ma , Mb , Mc lần lượt là trung điểm BC, CA, AB.
Ma O cắt (Ma Mb Mc ) lần thứ hai tại T . Kẻ AK ⊥ OH. L đối xứng với K qua Mb Mc .
1 R
Ta có QJ k OH và JK = AO = = QE nên QJKE là hình thang cân hay K ∈ (QJE).
2 2
Ta có JO k Ma E và JO = Ma E nên EJ = OMa , từ đó suy ra J đối xứng với E qua Mb Mc . Suy
ra EJLK là hình thang cân.
Nghĩa là L ∈ (QJE).
Mặt khác do O là trực tâm tam giác Ma Mb Mc nên T và O đối xứng nhau qua Mb Mc .
Kẻ AHa ⊥ BC thì A đối xứng với Ha qua Mb Mc . Từ đó ∠Ha LT = ∠AKO = 90◦ , mà Ha T là
đường kính của (E) nên L ∈ (E).
Như vậy L là điểm Anti-Steiner của OH ứng với tam giác Ma Mb Mc .
Chứng minh tương tự ta cũng có đường tròn Euler của các tam giác BHO, CHO đi qua L.
Cách 2.
Bổ đề. Cho tam giác ABC, các đường cao AHa , BHb , CHc . Khi đó đường thẳng Euler của các
tam giác AHb Hc , BHa Hc , CHa Hb đồng quy tại một điểm nằm trên đường tròn Euler của tam giác
ABC.
Chứng minh.
Gọi Oa , Ob lần lượt là tâm ngoại tiếp các tam giác AHb Hc , BHa Hc .
Ta có hai tam giác 4AHb Hc và 4Ha BHc đồng dạng cùng hướng nên phép vị tự quay tâm Hc góc
quay α = ∠(Hc B, Hc Ha ) = ∠C biến tam giác AHb Hc thành tam giác Ha BHc . Khi đó góc giữa hai
đường thẳng Euler của hai tam giác này cũng bằng ∠C.
Mà Oa , Ob lần lượt là trung điểm AH, BH nên chúng nằm trên đường tròn Euler của tam giác
ABC và dễ thấy số đo cung Oa Ob bằng 2∠C. Điều đó nghĩa là giao điểm của hai đường thẳng Euler
của các tam giác AHb Hc và BHa Hc nằm trên đường tròn Euler của tam giác ABC. Chứng minh tương
tự ta có đpcm.
Trở lại bài toán.

327
A

T Hb
X
Oa

J O
Hc
K E
H

B C

Gọi da , db , dc lần lượt là đường thẳng Euler của các tam giác AHb Hc , BHa Hc , CHa Hb , ωa , ωb , ωc , ω
lần lượt là đường tròn Euler của các tam giác AHO, BHO, CHO, ABC.
Ta có ωa và ω giao nhau tại trung điểm X của AH. Gọi J là tâm của ωa , Oa là tâm của (AHO).
Kẻ AK ⊥ OH. JE cắt AK tại T .
Ta có AH, AO đẳng giác trong ∠BAC, AK, AOa đẳng giác trong ∠HAO nên AK, AOa đẳng giác
trong ∠BAC.
Mặt khác, Hb Hc đối song với BC ứng với góc BAC nên da và OH cũng đối song ứng với góc BAC.
Suy ra các đường thẳng qua A tương ứng vuông góc với da , OH đẳng giác trong ∠BAC.
Từ đó AOa ⊥ da . Mà AT EOa là hình bình hành nên EJ k AOa hay EJ ⊥ da . Điều này nghĩa là
da là trục đẳng phương của ωa và ω.
Chứng minh tương tự db là trục đẳng phương của ωb và ω.
Theo bổ đề trên da , db , dc đồng quy tại tâm đẳng phương L của ωa , ωb , ωc , ω và L nằm trên ω.
Do phương tích của L ứng với ω bằng 0 nên 4 đường tròn ωa , ωb , ωc , ω đồng quy tại L.

Nhận xét. Điểm đồng quy trên được gọi là điểm Jerabek của tam giác ABC.

Bài 385. (Nguyễn Văn Linh) Cho 4 cung tròn ω1 , ω2 , ω3 , ω4 (được sắp xếp theo thứ tự) cùng dựng
trên dây cung AB sao cho chúng cùng nằm trên một nửa mặt phẳng bờ AB. Giả sử tồn tại đường
tròn (O1 ) tiếp xúc với ω1 và ω2 , đường tròn (O2 ) tiếp xúc với ω3 và ω4 sao cho giao điểm của hai tiếp
tuyến chung ngoài của (O1 ) và (O2 ) nằm trên AB. Gọi (O3 ) là đường tròn bất kì tiếp xúc với ω1 và
ω2 . Từ điểm C trên AB kẻ hai tiếp tuyến tới (O3 ), cắt ω3 và ω4 lần lượt tại (X, Y ); (T, Z). Chứng
minh rằng tứ giác cong XY ZT (tạo bởi các cạnh XY, ZT và các cung Y Z, XT ) ngoại tiếp.

Chứng minh. Trước tiên ta phát biểu hai bổ đề.


Bổ đề 1 (Định lý Monge-D’Alembert). Cho ba đường tròn C1 (O1 , R1 ), C2 (O2 , R2 ), C3 (O3 , R3 ) phân
biệt trên mặt phẳng. Khi đó tâm vị tự ngoài của các cặp đường tròn (C1 , C2 ), (C2 , C3 ), (C3 , C1 ) cùng
thuộc một đường thẳng. Hai tâm vị tự trong của hai trong ba cặp đường tròn trên và tâm vị tự ngoài
của cặp đường tròn còn lại cùng thuộc một đường thẳng.
Bổ đề 1 tương đối đơn giản xin phép không chứng minh lại.
Bổ đề 2. Cho hai dây cung γ1 và γ2 cùng dựng trên dây AC sao cho chúng cùng nằm trên một
nửa mặt phẳng bờ AC. Gọi C1 , C2 là hai đường tròn tiếp xúc với γ1 và γ2 . Khi đó tâm vị tự ngoài của
C1 và C2 nằm trên AC.

328
T

I2

Z
I1
Y

X O2

I
A C

O1

Chứng minh. Gọi (O1 , R1 ) và (O2 , R2 ) lần lượt là đường tròn chứa các cung γ1 , γ2 ; I là tâm vị tự ngoài
của C1 và C2 . X, Y là tiếp điểm của C1 , C2 với γ1 , Z, T là tiếp điểm của C1 , C2 với γ2 .
Áp dụng định Monge-D’Alembert cho bộ ba đường tròn ((O1 ), γ1 , γ2 ) và ((O2 ), γ1 , γ2 ) suy ra I, X, Y
thẳng hàng và I, Z, T thẳng hàng.
Gọi IA1 A2 là tiếp tuyến chung ngoài của γ1 , γ2 .
Phép nghịch đảo IIIA1 .IA2 : γ1 7→ γ2 , đồng thời các cặp đường thẳng I1 Z và I2 T , I1 X và I2 Y không
song song nên IZ.IT = IA1 .IA2 = IX.IY . Suy ra X, Y, Z, T đồng viên.
Xét ba đường tròn (O1 ), (O2 ), (XY T Z) có trục đẳng phương lần lượt là AC, XY, ZT nên AC, XY, ZT
đồng quy. Từ đó I ∈ AC.

Trở lại bài toán.

O2

O4
Z
X
O1
T
O3

A D C B E F

Gọi (O40 ) là đường tròn tiếp xúc với XY , cung Y Z, cung XT.
Áp dụng bổ đề 2 suy ra tâm vị tự ngoài của các cặp đường tròn (O1 ) và (O3 ); (O2 ) và (O40 ) nằm
trên AB.
Áp dụng bổ đề 1 cho 3 đường tròn (O1 ), (O2 ), (O3 ) ta có tâm vị tự ngoài của các cặp đường tròn
(O1 ) và (O2 ), (O1 ) và (O3 ) cùng nằm trên AB nên tâm vị tự ngoài của (O2 ) và (O3 ) nằm trên AB.
Lại áp dụng bổ đề 1 cho 3 đường tròn (O2 ), (O3 ), (O40 ) suy ra tâm vị tự ngoài của (O3 ) và (O40 )
nằm trên AB. Ta có tiếp tuyến chung ngoài của (O3 ) và (O40 ) cắt AB tại C. Do đó C là tâm vị tự
ngoài của (O3 ) và (O40 ), nghĩa là (O40 ) tiếp xúc với ZT hay (O40 ) ≡ (O4 ). Ta có đpcm.

329
Bài 386. (APMO 2014) Cho hai đường tròn (O1 ) và (O2 ) giao nhau tại hai điểm A, B. Gọi M là
điểm chính giữa cung AB của (O1 ) sao cho M nằm trong (O2 ). Dây cung M P của (O1 ) cắt (O2 ) tại Q
sao cho Q nằm trong (O1 ). Gọi l1 , l2 lần lượt là tiếp tuyến của (O1 ) tại P và (O2 ) tại Q. Chứng minh
rằng tam giác tạo bởi giao điểm của các đường thẳng l1 , l2 , AB tiếp xúc với (O2 ).

Chứng minh. Cách 1.

S
A

O1 M O2
T
Q

X
N
P

Z
Y

Gọi T là giao của P M và AB, S là giao điểm thứ hai của P M với (O2 ). X, Y, Z lần lượt là giao
của các cặp đường thẳng (AB, l2 ), (l1 , l2 ), (AB, l1 ). ZS cắt (O2 ) tại N.
Ta có PY là tiếp tuyến của (O1 ) nên ∠Y P M = ∠P AM = ∠P T Z. Suy ra ZP = ZT.
Do Z thuộc trục đẳng phương của (O1 ) và (O2 ) nên ZT 2 = ZP 2 = ZN.ZS hay ZT là tiếp tuyến
của (T N S).
Ta thu được ∠XT N = ∠QSN = ∠XQN hay N ∈ (XQT ).
Lại có ∠T N S = ∠AT S = ∠T P Z nên N ∈ (T P Z). Suy ra N là điểm Miquel của tứ giác toàn phần
P T ZQXY hay N ∈ (XY Z).
Gọi I là tâm của (XY Z). Ta có ∠ZN I = 90◦ − ∠N Y Z = 90◦ − ∠N T S = ∠O2 N S, suy ra O2 , N, I
thẳng hàng. Từ đó có đpcm.
Cách 2.

330
A

O1 M O2
Q
N

Y
P

X
Z

Gọi N là giao của P M và AB, X, Y, Z được xác định như hình vẽ.
Áp dụng định lý Casey cho 4 đường tròn (O2 ), (X, 0), (Y, 0), (Z, 0) ta có (XY Z) tiếp xúc với (O2 )
khi và chỉ khi
XP.Y Z = XY.QZ + QY.XZ. (1)
Chú ý rằng XP = XN . Áp dụng định lý Menelaus cho tam giác XY Z với đường thẳng (P, Q, N )
P X QZ N Y
suy ra · · = 1.
P Z QY N X
QZ N X − XY
Hay · = 1, tương đương với (1). Ta có đpcm.
QY P X + XZ

Bài 387. Cho đường tròn (O) ngoại tiếp tam giác ABC. Một đường tròn ω tiếp xúc với các cạnh
AB, AC lần lượt tại L, K và tiếp xúc ngoài với (BOC). Chứng minh rằng LK chia đôi AI với I là
tâm đường tròn nội tiếp tam giác ABC.

A N

L
M
O
E
I

B C

Chứng minh. Gọi E là giao điểm thứ hai của AC với (BOC). Ta có ∠BEA = 180◦ − ∠BEC =
180◦ − ∠BOC = 180◦ − 2∠BAC.
Do đó tam giác AEB cân tại E. Suy ra OE là trung trực của đoạn thẳng AB và OE giao CI tại
điểm chính giữa M của cung AB.

331
Do ω là đường tròn Thebault của tam giác BEC ứng với đường thẳng BA và M là tâm đường
tròn bàng tiếp góc C của tam giác BEC nên M, L, K thẳng hàng.
Tương tự gọi N là điểm chính giữa cung AC thì N, L, K thẳng hàng. Mà M N là trung trực của
đoạn thẳng AI nên LK chia đôi AI.

Bài 388. (All-Russian MO 2013). Cho tam giác ABC nội tiếp đường tròn (O), ngoại tiếp đường tròn
(I). Gọi X, Y là giao điểm của (BIC) và (I), Z là tâm vị tự ngoài của (I) và (BIC). Chứng minh rằng
(XY Z) tiếp xúc với (O).

Chứng minh. Cách 1.

Z
O
K H
Y
I
L
N X
T
B C

Gọi F là giao của AI với (O), T là tâm vị tự trong của (O) và (I), K là tâm của (XY Z).
ZI TI r YI XI
Ta có = = = = nên X, Y, Z, T cùng nằm trên đường tròn Apollonius của
ZF TF R YF XF
r
đoạn thẳng EF ứng với tỉ số , nói cách khác là đường tròn Apollonius của tam giác IY F.
R
Tâm K của (XY ZT ) là giao của tiếp tuyến tại Y của (IY F ) với IF . Suy ra ∠KY I = ∠IF Y.
Ta thu được ∠KY F + ∠IY F = ∠IF Y + 2∠IY F = 180◦ , suy ra Y F là phân giác ngoài ∠KY I.
FI IY
Nghĩa là = hay F là tâm vị tự ngoài của (I) và (XY Z).
FK KI
Kẻ tiếp tuyến F N, F H tới (XY Z), F N, F H lần lượt cắt (O) tại L, M.
Do (ZT IF ) = −1 nên theo hệ thức Newton ta có KN 2 = KT 2 = KI.KF , suy ra I là hình chiếu
của N trên KF , nghĩa là I là trung điểm N H.
Theo định lý Poncelet, (I) là đường tròn nội tiếp tam giác LF M nên áp dụng bổ đề Sawayama
suy ra (XY Z) là đường tròn mixtilinear nội tiếp của tam giác F LM , hay (XY Z) tiếp xúc với (O).
Cách 2.

332
A
Z

I Y

X O

B T C

Gọi T là giao của đoạn thẳng IF với (I), J là điểm đối xứng với I qua F.
ZT r ZI
Ta có = = .
ZJ R ZF
Suy ra ZT.ZF = ZJ.ZI. Theo hệ thức Maclaurin ta thu được (ZT IJ) = −1.
Từ đó theo hệ thức Newton, F I 2 = F T.F Z.
2
Xét phép nghịch đảo IFR : (ABC) 7→ BC, X 7→ X, Y 7→ Y, Z 7→ T.
Do đó (XY Z) 7→ (I) và do (I) tiếp xúc với BC nên (XY Z) tiếp xúc với (O).

Bài 389. Cho tam giác ABC nội tiếp đường tròn (O), ngoại tiếp đường tròn (I). Một đường tròn ω
đi qua I và tiếp xúc với (O) cắt các đoạn thẳng IB, IC lần lượt tại J1 , J2 . Gọi (J1 ), (J2 ) là các đường
tròn tâm J1 , J2 và tiếp xúc với các cặp cạnh BA, BC và CA, CB. Chứng minh rằng ω, (J1 ), (J2 ) có
chung một tiếp tuyến.

I
J2

Z J1

B C

J3

Chứng minh. Bài toán là hệ quả của Mathley 3-P4.


Gọi giao điểm của tiếp tuyến chung ngoài khác BC của (J1 ) và (J2 ) với AC, AB lần lượt là Y, Z.

333
ZJ1 giao Y J2 tại J3 , ta có J3 là tâm đường tròn bàng tiếp góc A của tam giác AY Z, suy ra
1
∠J2 J3 J1 = 90◦ − ∠BAC = 180◦ − ∠J2 IJ1 .
2
Do đó J3 ∈ (J1 J2 I).
Từ đó áp dụng bài toán Mathley 3-P4 cho tam giác J1 J2 J3 ta có J3 A, J1 B, J2 C giao nhau tại I
nằm trên (J1 J2 J3 ), (ABC) tiếp xúc với (J1 J2 J3 ), đường thẳng đối xứng với AB qua J3 J1 , với AC qua
J3 J2 đều là Y Z nên Y Z là tiếp tuyến của (J1 J2 J3 ). Ta có đpcm.

Bài 390. Cho tam giác ABC nội tiếp đường tròn (O). P là điểm bất kì chuyển động trên (O). Gọi
R, S là giao điểm của hai tiếp tuyến kẻ từ P tới (I) với BC. Khi đó (P RS) luôn đi qua tiếp điểm X
của đường tròn A-mixtilinear với (O).

N
A
K

I
M

L
R S
B
C

X
P

Chứng minh. Gọi X 0 là giao của (P RS) với (O). Ta chỉ cần chứng minh X 0 I là phân giác ∠BX 0 C thì
sẽ có X 0 ≡ X.
Gọi M, N là giao của P R, P S với (O), L, K lần lượt là giao của P I với (P RS), (O).
Xét hai tam giác M BX 0 và CSX 0 có ∠BM X 0 = ∠SCX 0 , ∠M BX 0 = 180◦ − ∠X 0 P R = 180◦ −
∠RSX 0 = ∠X 0 SC.
Suy ra ∠BX 0 M = ∠SX 0 C.
Lại có ∠M X 0 K = ∠M P K = ∠LP S = ∠LX 0 S.
X 0K KI
Do đó X 0 I là phân giác ∠BX 0 C khi và chỉ khi X 0 I là phân giác ∠KX 0 L, tương đương 0 = .
XL LI
(1)
Do I là tâm bàng tiếp của tam giác P RS và L là điểm chính giữa cung RS nên LI = LR.
Theo định lý Poncelet, M N tiếp xúc với (I), suy ra K là tâm ngoại tiếp tam giác M IN hay
KI = KM.
KI KM
Như vậy = .
LI LR
Xét hai tam giác KM X 0 và LRX 0 có ∠M X 0 K = ∠RP L = ∠RX 0 L, ∠M KX 0 = ∠M P X 0 =
∠RLX 0 . Do đó 4KM X 0 ∼ 4LRX 0 .
KM KX 0
Suy ra = , nghĩa là (1) đúng. Ta có đpcm.
LR LX 0
Nhận xét. Bài toán trên có thể được mở rộng như sau: (Vladimir Zajic). Cho tam giác ABC nội
tiếp đường tròn (O), ngoại tiếp đường tròn (I). P là một điểm bất kì nằm ngoài (I). Các tiếp tuyến kẻ

334
từ P tới (I) giao BC tại E, F . P I giao (O) tại P 0 . Khi đó (P 0 EF ) luôn đi qua tiếp điểm của đường
tròn A-mixtilinear nội tiếp với (O).

Bài 391. Với kí hiệu như bài trên, chứng minh rằng (P RS) luôn tiếp xúc với một đường tròn cố định.

K
L

I O

Z S

B R C
Y

X
P

Chứng minh. Từ X kẻ tiếp tuyến tới (I), cắt (O) tại L, K. (P RS) cắt XL, XK lần lượt tại Y, Z.
Do (I) là đường tròn bàng tiếp của tam giác PRS nên theo định lý Poncelet suy ra (I) cũng là
đường tròn bàng tiếp của tam giác XY Z.
Cũng theo định lý Poncelet, (I) là đường tròn nội tiếp của tam giác XLK nên đường tròn ω
X-mixtilinear incircle của tam giác XLK đồng thời là đường tròn X-mixtilinear excircle của tam giác
XY Z. Điều này nghĩa là (XY Z) luôn tiếp xúc với đường tròn ω cố định.

Nhận xét. Có thể phát biểu bài toán theo cách khác như sau.
Cho tam giác ABC nội tiếp đường tròn (O), ngoại tiếp đường tròn (I). P là một điểm bất kì trên
(O). Từ P kẻ hai tiếp tuyến tới (I), cắt tiếp tuyến của (I) song song với BC tại R, S. Khi đó (P RS)
luôn tiếp xúc với đường tròn A-mixtilinear của tam giác ABC.

Bài 392. (Nguyễn Thanh Trà) Cho một điểm O bất kì nằm trong tam giác ABC. Gọi P, Q, R lần
lượt là hình chiếu vuông góc của O trên BC, CA, AB, A1 , B1 , C1 lần lượt là các điểm bất kì nằm trên
BC, CA, AB và A2 , B2 , C2 lần lượt là đối xứng của A1 , B1 , C1 qua P, Q, R. Z1 , Z2 lần lượt là các điểm
Miquel của tam giác ABC ứng với bộ điểm A1 , B1 , C1 và A2 , B2 , C2 . Chứng minh rằng OZ1 = OZ2 .

B2
C1

Z2 Q
R
Z X
B1
C2 O
Z1
Y

B A1 P A2 C

335
Chứng minh. Gọi X = A1 Z1 ∩ OP, Y = OQ ∩ B1 Z1 , Z = OR ∩ C1 Z1 .
Dễ thấy ∠Z1 ZO = ∠Z1 XO = ∠Z1 Y O nên Z1 , O, X, Y, Z cùng thuộc một đường tròn.
Gọi XA2 ∩ Y B2 = {Z20 }.
Ta có ∠B2 Y Q = ∠B1 Y Q = ∠A1 XP = ∠Z2 XP ⇒ Z20 nằm trên (OXY ).
⇒ ∠Z20 Y O = ∠Z20 ZO = ∠OXA2 = ∠RZC1 = ∠RZC2 .
⇒ C2 , Z, Z20 thẳng hàng.
⇒ ∠Z20 C2 A = ∠Z20 B2 C = ∠Z20 Z2 B ⇒ Z20 là điểm Miquel của tam giác ABC ứng với (A2 , B2 , C2 ).
⇒ Z20 ≡ Z2 .
Mà ∠Z2 ZO = ∠Z1 ZO nên OZ1 = OZ2

Bài 393. Cho tam giác ABC, các đường tròn bàng tiếp (Ia ), (Ib ), (Ic ). Các tiếp tuyến chung ngoài
khác các cạnh tam giác ABC của 2 trong 3 đường tròn trên cắt nhau và tạo thành tam giác Ta Tb Tc .
Gọi D là điểm bất kì nằm trên Tb Tc . Tiếp tuyến của (Ic ) qua D cắt Ta Tc tại E, tiếp tuyến của (Ib )
qua D cắt Ta Tb tại F . Chứng minh rằng EF tiếp xúc với (Ia ).

Tb

D
Tc Ib
Ic A

B D' C

N
M
Ia

F
E

Ta

Chứng minh. Cách 1.


Gọi M, N lần lượt là điểm đối xứng với F qua Ia Ib , E qua Ia Ib . D0 là điểm đối xứng với D qua
Ib Ic . Hiển nhiên D0 nằm trên BC.
1
Ta có ∠DIb F = 90◦ + ∠Tc Tb Ta = 2∠Ic Ib Ia .
2
Do đó Ib Ic là phân giác ∠DIb M hay M, D0 , Ib thẳng hàng. Tương tự N, D0 , Ic thẳng hàng.
Áp dụng định lý Pappus cho hai bộ 3 điểm (Ic , A, Ib ) và (B, D0 , C) ta có M, Ia , N thẳng hàng.
Như vậy ∠Tc EIa + ∠Tb F Ia = ∠Ia N A + ∠Ia M A = 180◦ − ∠BAC.
1
Từ đó dễ dàng tính được ∠EIa F = 90◦ − ∠Tb Ta Tc , tức là (Ia ) là đường tròn bàng tiếp của tam
2
giác Ta EF . Ta có đpcm.
Cách 2 (Vũ Tiến Sinh)

336
Tb

D X
Y
Tc Ib
Ic A

B C

M
N Ia

E F

R
Ta
S

Gọi X, Y lần lượt là giao của AB, AC với Tb Tc , M là giao của AC với Ta Tb , N là giao của AB với
Ta Tc .
Áp dụng định lý Brianchon cho đường gấp khúc DXBCM F ngoại tiếp đường tròn (Ib ) suy ra
XM, DC, BF đồng quy tại S.
Chứng minh tương tự, Y N, DB, CE đồng quy tại R.
Xét hai tam giác BN R và SM C có X = BN ∩ SM, Y = N R ∩ CM, D = BR ∩ CS thẳng hàng
nên theo định lý Desargues, BS, CR, M N đồng quy.
Từ đó theo định lý Brianchon, lục giác BCM F EN ngoại tiếp hay EF tiếp xúc với (Ia ).

Bài 394. (Nguyễn Văn Linh) Cho tam giác ABC nội tiếp đường tròn (O) với trực tâm H. Gọi
A1 , B1 , C1 lần lượt là các điểm đối xứng với A, B, C qua O, P là điểm bất kì trên mặt phẳng. Gọi
A2 B2 C2 là tam giác pedal của P ứng với 4ABC, A3 , B3 , C3 lần lượt là các điểm đối xứng với A1 , B1 , C1
qua A2 , B2 , C2 . Chứng minh rằng H, A3 , B3 , C3 cùng thuộc một đường tròn.

B1
C1 B2
B3
B4
B5
C4
A3 C2 H A5 O

P
C5

B C3 A A4 C
2

A1

Chứng minh. Gọi A4 , B4 , C4 lần lượt là trung điểm của BC, CA, AB, suy ra A4 là trung điểm A3 H.
Tương tự với B4 , C4 .

337
Ta thu được HA3 k= 2A2 A4 , HB3 k= 2B2 B4 , HC3 k= 2C2 C4 .(1)
Dựng các hình chữ nhật OA4 A2 A5 , OB4 B2 B5 , OC4 C2 C5 suy ra A5 , B5 , C5 nằm trên đường tròn
đường kính OP.(2)
Lại có OA5 k= A4 A2 , OB5 k= B4 B2 , OC5 k= C4 C2 (3)
Từ (1), (2), (3) suy ra H, A3 , B3 , C3 cùng thuộc một đường tròn. Hơn nữa ta có thể chứng minh
4A3 B3 C3 ∼ 4ABC.

Bài 395. (Nguyễn Văn Linh) Cho tứ giác ABCD nội tiếp đường tròn (O) thỏa mãn AC ⊥ BD. Tiếp
tuyến tại (O) lần lượt qua A, B, C, D cắt nhau tạo thành tứ giác XY ZT . XZ cắt Y T tại P . Chứng
minh rằng tâm đường tròn nội tiếp của 8 tam giác XP Y , Y P Z, ZP T , T P X, XY Z, Y ZT , ZT X,
T XY cùng thuộc một đường tròn.

A
I1 I5
X B
I2
P

I3

T C Z

Chứng minh. Gọi I1 , I2 , I3 , I4 , I5 , I6 , I7 , I8 lần lượt là tâm nội tiếp của các tam giác XP Y , Y P Z, ZP T ,
T P X, XY Z, Y ZT , ZT X, T XY . Dễ thấy AC, BD, XZ, Y T đồng quy tại P .
Do AC ⊥ BD nên ∠T XY + ∠T ZY = 180o hay tứ giác XY ZT nội tiếp.
PY AY
Suy ra ∠XY P = ∠T ZP . Nhưng ∠P AY = ∠P CZ nên 4P AY ∼ 4P CZ. Suy ra = =
PZ CZ
YB
.
ZB
Ta thu được P B là phân giác ∠Y P Z hay I2 ∈ P B. Tương tự, I3 ∈ P C.
P I3 PZ PZ P I2
Ta có = = = . Do đó I2 I3 k BC.
CI3 CZ BZ BI2
1
Suy ra ∠I1 I3 I2 = ∠ACB = 90o − ∠XY Z.
2
1 1
Vậy ∠I1 I3 I2 + ∠I1 I5 I2 = 90 − ∠XY Z + 90o + ∠XY Z = 180o .
o
2 2
Tương đương I1 , I2 , I3 , I5 cùng thuộc một đường tròn. Tương tự, I1 , I2 , I4 , I5 cùng thuộc một đường
tròn hay I5 ∈ (I1 I2 I3 I4 ). Tương tự với I6 , I7 , I8 ta có đpcm.

Bài 396. Cho 5 điểm A1 , A2 , A3 , A4 , A5 cùng thuộc một đường tròn. Với 1 ≤ i < j ≤ 5, kí hiệu Xij là
giao điểm của đường thẳng Simson của Ai , Aj ứng với tam giác tạo bởi 3 điểm còn lại. Chứng minh
rằng 10 điểm Xij cùng thuộc một đường tròn.

Chứng minh. Trước tiên ta phát biểu 2 bổ đề.


Bổ đề 1. Cho 4 điểm A1 , A2 , A3 , A4 cùng nằm trên đường tròn (O). Khi đó đường thẳng Simson
d1 , d2 , d3 , d4 của A1 , A2 , A3 , A4 lần lượt ứng với 4A2 A3 A4 , A1 A3 A4 , A1 A2 A4 , A1 A2 A4 đồng quy tại
trực tâm X của tứ giác A1 A2 A3 A4 . Gọi G là trọng tâm của A1 A2 A3 A4 thì OG ~ = 1 OX. ~
2
Chứng minh.

338
A3
A2

X
H2
N M H3
G
O

A4
A1

Gọi H2 , H3 lần lượt là trực tâm các tam giác A1 A3 A4 và A1 A2 A4 . Khi đó A2 H3 k= A3 H2 .


Suy ra A2 H3 H2 A3 là hình bình hành. Từ đó A2 H2 , A3 H3 đồng quy tại trung điểm X của A2 H2 và
A3 H3 . Dễ thấy X nằm trên d2 và d3 . Chứng minh tương tự ta thu được d1 , d2 , d3 , d4 đồng quy tại X.
Gọi M, N lần lượt là trung điểm A1 A3 và A2 A4 suy ra G là trung điểm M N.
1
Ta có XN k= A4 H2 k= OM , suy ra XN OM là hình bình hành. Suy ra G là trung điểm OX.
2
Bổ đề 2. Cho 5 điểm A1 , A2 , A3 , A4 , A5 cùng nằm trên (O). Gọi H1 , H2 , H3 , H4 , H5 lần lượt là trực
tâm của các tứ giác A2 A3 A4 A5 , A1 A3 A4 A5 , A1 A2 A4 A5 , A1 A2 A3 A5 , A1 A2 A3 A4 ,. Khi đó H1 , H2 , H3 , H4 , H5
đồng viên.
Chứng minh.

A2
P
A1

G4

A3
N
G3 O G1

A5

A4

Gọi G1 , G3 , G4 lần lượt là trọng tâm của A2 A3 A4 A5 , A1 A2 A4 A5 , A1 A2 A3 A5 . Theo bổ đề 1, G1 G4 k


H1 H4 .
Gọi M, N, P lần lượt là trung điểm của A3 A5 , A2 A4 , A1 A2 suy ra G1 G4 là đường trung bình của
tam giác M N P . Suy ra G1 G4 k N P k A1 A4 . Tương tự, G3 G4 k A3 A4 .
Từ đó ∠H1 H4 H3 = ∠A1 A4 A3 .
Tương tự, ∠H1 H5 H3 = ∠A1 A5 A3 = ∠A1 A4 A3 = ∠H1 H4 H3 . Vậy H1 , H3 , H4 , H5 đồng viên. Chứng
minh tương tự ta có đpcm.
Trở lại bài toán.

339
Gọi H1 , H2 , H3 , H4 , H5 lần lượt là trực tâm các tứ giác Ai Ai+1 Ai+2 Ai+3 (1 ≤ i ≤ 5). Theo bổ đề 2
ta có H1 H2 H3 H4 H5 đồng viên.
Theo bổ đề 1, đường thẳng Simson d, l của A3 , A1 ứng với các tam giác A2 A4 A5 , A2 A4 A5 lần lượt
đi qua H1 , H3 .
Gọi F là giao của d và l thì theo tính chất góc tạo bởi hai đường thẳng Simson, ∠H1 F H3 =
∠A1 A4 A3 = ∠H1 H4 H3 hay F nằm trên (H1 H2 H3 H4 H5 ). Chứng minh tương tự ta có đpcm.

Bài 397. Cho tam giác ABC, AB > AC. M là trung điểm BC. P là điểm nằm trong tam giác AM C
sao cho ∠M AB = ∠P AC. Gọi O, O1 , O2 lần lượt là tâm ngoại tiếp các tam giác ABC, ABP, ACP .
Chứng minh rằng AO đi qua trung điểm của O1 O2 .

O2

O1

J
P
O
I

B C

E
D

Chứng minh. Gọi I, J, E lần lượt là ảnh của O1 , O2 , O qua phép vị tự tâm A tỉ số 2. D là giao của AP
với (O).
Do AP là đường đối trung của tam giác ABC nên tứ giác ABDC điều hòa, suy ra E(DBAC) = −1.
Lại có IJ ⊥ AD, ED ⊥ AD nên ED k IJ. Điều này nghĩa là EA đi qua trung điểm của IJ, suy
ra AO đi qua trung điểm của O1 O2 .

Bài 398. Cho hình chữ nhật ABCD nội tiếp đường tròn (O, R). M là điểm bất kì trên cung nhỏ AB.
Trung trực của đoạn thẳng OD cắt M A tại X, trung trực của đoạn thẳng OC cắt M B tại Y . Chứng
minh rằng XY tiếp xúc với một đường tròn cố định.

340
D C

A B

X Y

I M

J K

Chứng minh. Gọi I là giao điểm thứ hai của (X, XO) và (Y, Y O); J, K lần lượt là điểm đối xứng của
O qua M A, M B.
Bằng một số phép cộng góc đơn giản suy ra hai tam giác ADJ và BCK có cạnh tương ứng song
song.
Ta có ∠AM B = 180◦ −∠JOK = 180◦ −(∠JOI+∠KOI) = 180◦ −∠JDI−∠KCI = 180◦ −∠DIC =
∠AIB.
R
Do đó I ∈ (O). Ta thu được XY tiếp xúc với (O, ).
2
Bài 399. Cho tứ giác ABCD nội tiếp đường tròn (O). Chứng minh rằng nếu đường thẳng Simson
của A ứng với tam giác BCD vuông góc với đường thẳng Euler của tam giác BCD thì đường thẳng
Simson của B ứng với tam giác ACD vuông góc với đường thẳng Euler của tam giác ACD.
B

O
Hb

Ha

C
X

Chứng minh. Gọi Ha , Hb lần lượt là trực tâm các tam giác ACD, BCD. X, Y là giao của AHa , BHb
với (O).
Ta có đường thẳng Simson của A ứng với tam giác BCD vuông góc với BX, do đó BX ⊥ OHb .
Tức là OHb là trung trực của đoạn thẳng BX. Suy ra ∠OXHb = ∠OBHb = ∠OY Hb hay tứ giác
OXY Hb nội tiếp.

341
Mà Ha XY Hb là hình thang cân suy ra Ha XY O nội tiếp. Ta thu được ∠Ha Y O = ∠Ha XO =
∠Ha AO.
Suy ra A và Y nằm trên hai cung đối xứng nhau qua Ha O, mà OA = OY nên A và Y đối xứng
nhau qua Ha O.
Từ đó Ha O ⊥ AY hay đường thẳng Simson của B ứng với tam giác ACD vuông góc với đường
thẳng Euler của tam giác ACD.

Bài 400. Cho lục giác ABCDEF nội tiếp và điểm P bất kì nằm trong lục giác. Gọi A0 , B 0 , C 0 , D0 , E 0 , F 0
lần lượt là tâm ngoại tiếp của các tam giác P AB, P BC, P CD, P DE, P EF, P F A. Chứng minh rằng
A0 D0 , B 0 E 0 , C 0 F 0 đông quy.
A1

F1

A'' B''

A
F''
E1 F F' A' B
B1
E' P B'
E

E'' D' C
C'

D
C''
D1 D''

C1

Chứng minh. Xét phép nghịch đảo I cực P , phương tích ω dương.
I: A 7→ A00 , B 7→ B 00 , C 7→ C 00 , D 7→ D00 , E 7→ E 00 , F 7→ F 00 . (P AB) 7→ A00 B 00 , (P BC) 7→
B C 00 , (P CD) 7→ C 00 D00 , (P DE) 7→ D00 E 00 , (P EF ) 7→ E 00 F 00 , (P F A) 7→ F 00 A00 .
00

Gọi A1 , B1 , C1 , D1 , E1 , F1 lần lượt là điểm đối xứng với P qua A00 B 00 , B 00 C 00 , C 00 D00 , D00 E 00 , E 00 F 00 , F 00 A00 .
Khi đó qua phép nghịch đảo I, A0 7→ A1 , B 0 7→ B1 , C 0 7→ C1 , D0 7→ D1 , E 0 7→ E1 , F 0 7→ F1 .
Như vậy A0 D0 , B 0 E 0 , C 0 F 0 đồng quy khi và chỉ khi (P A1 D1 ), (P B1 E1 ), (P C1 F1 ) đồng trục.
Do A, B, C, D, E, F cùng thuộc một đường tròn nên A00 , B 00 , C 00 , D00 , E 00 , F 00 cùng thuộc một đường
tròn. Áp dụng định lý Pascal ta suy ra giao điểm của các cặp đường thẳng A00 B 00 và D00 E 00 ; B 00 C 00
và E 00 F 00 ; A00 F 00 và C 00 D00 thẳng hàng, nghĩa là tâm của các đường tròn (P A1 D1 ), (P B1 E1 ), (P C1 F1 )
thẳng hàng. Từ đó suy ra đpcm.

Bài 401. Cho tam giác ABC có ∠C = 2∠A, phân giác CD. Gọi S là tâm của đường tròn nằm cùng
phía với B bờ AC sao cho tiếp xúc với AC và tiếp xúc ngoài với đường tròn ngoại tiếp 4ACD và
4BCD,. Chứng minh rằng AB ⊥ CS.

Chứng minh. Cách 1.

342
A' D'

D
B
C B'

Gọi ω là đường tròn tâm S tiếp xúc với AC và tiếp xúc ngoài với (ACD) và (BCD).
Xét phép nghịch đảo I cực C, phương tích bằng phương tích từ C tới ω.
I: A 7→ A0 , B 7→ B 0 , D 7→ D0 . Do đó (ACD) biến thành đường thẳng qua A0 tiếp xúc với ω, (BCD)
biến thành đường thẳng qua B 0 và tiếp xúc với ω. Hai đường thẳng này giao nhau tại D0 .
Dễ thấy CB tiếp xúc với (ACD) nên A0 D0 k CB 0 . Do A, D, B thẳng hàng nên C, A0 , B 0 , D0 đồng
viên, nghĩa là CA0 D0 B 0 là hình thang cân.
Ta có ∠A0 CD0 = ∠D0 CB 0 = ∠CD0 A0 nên A0 C = A0 D0 , suy ra SC = SD0 . Tương tự SB 0 = SA0 .
Mà SA0 = SD0 nên S là tâm của (CA0 D0 B 0 ). Do hai đường AD và A0 D0 đối song nên CS và đường
thẳng nối C với tâm của (ACD) đẳng giác trong ∠ACD. Nghĩa là CS ⊥ AD.
Cách 2.

Q K
A
I R
D

C J B

Gọi P, Q, R lần lượt là tiếp điểm của ω với AC, (ACD), (BCD). Gọi I, J lần lượt là tâm của
(ACD), (BCD).
Ta có ∠CAD = ∠DCA nên D là điểm chính giữa cung AC. Suy ra ID ⊥ AC và ID k SP . Do Q
là tâm vị tự trong của (S) và (I) nên P, Q, D thẳng hàng.
Gọi K là giao điểm thứ hai của AQ với (S). Ta có P K k AD nên ∠DAQ = ∠QKP = ∠AP D, suy
ra DP.DQ = DA2 .
Mặt khác, gọi E là giao điểm thứ hai của AC với (J). Dễ thấy B là điểm chính giữa cung AC nên
chứng minh tương tự suy ra B, R, P thẳng hàng và BP.BR = BC 2 .
Điều này có nghĩa là B, D cùng nằm trên trục đẳng phương của (S) và (C, 0). Vậy CS ⊥ AB.

343
Bài 402. Cho ba đường tròn (O1 ), (O2 ), (O3 ) đôi một giao nhau. (O1 ) giao (O2 ) tại A, B; (O2 ) giao
(O3 ) tại C, D; (O3 ) giao (O1 ) tại E, F . Với mỗi điểm P1 nằm trên (O1 ), kí hiệu P2 là giao của P1 A
với (O2 ), P3 là giao của P2 C với (O3 ), P4 là giao của EP3 với (O1 ), P5 là giao của P4 B với (O2 ), P6
là giao của P5 D với (O3 ), P7 là giao của P6 F với (O1 ). Chứng minh rằng P7 ≡ P1 .

P2

P1
O2

O1
E
C

P3 P5

B
P4
F O3 D

P6

Chứng minh. Do tứ giác P1 ABP4 , ABP5 P2 nội tiếp nên theo định lý Reim, P1 P4 k P2 P5 .
Chứng minh tương tự suy ra P2 P5 k P3 P6 , P3 P6 k P4 P7 .
Vậy P2 P5 k P1 P4 k P4 P7 hay P7 ≡ P1 .

Bài 403. Từ các đỉnh A, B, C của tam giác ABC lần lượt kẻ ba đường thẳng song song da , db , dc . Gọi
la , lb , lc lần lượt là đường thẳng đối xứng với da , db , dc qua BC, CA, AB; XY Z là tam giác tạo bởi giao
điểm của la , lb , lc . Chứng minh rằng tâm đường tròn nội tiếp tam giác XY Z thuộc một đường tròn cố
định.

C3 I
A1
X
B3 Z

O
H
C
B
D
M

B1 E A3 C1

A2

344
Chứng minh. Xét phép vị tự HO 2 : A 7→ A , B 7→ B , C 7→ C .
1 1 1
Gọi H là trực tâm tam giác ABC, AH ∩ BC = {D}, AH ∩ B1 C1 = {E}, A2 là điểm đối xứng với A
qua BC, M là trung điểm B1 C1 . Đường thẳng l qua H song song với la , lb , lc giao B1 C1 , C1 A1 , A1 B1
lần lượt tại A3 , B3 , C3 .
Do EA2 = DA2 − DE = AD − DE = AH + DH − DE = OM + DH − DE = HE nên A2 đối
xứng với H qua B1 C1 . Suy ra A3 , B3 , C3 lần lượt nằm trên Y Z , XZ, XY . Từ đó XY Z là tam giác
tạo bởi giao điểm của các đường thẳng đối xứng với l qua B1 C1 , C1 A1 , A1 B1 .
Mặt khác, A1 là tâm đường tròn nội tiếp tam giác XB3 C3 , tương tự với B1 , C1 suy ra A1 X, B1 Y, C1 Z
đồng quy tại tâm đường tròn nội tiếp I của tam giác XY Z. Hơn nữa ∠C3 A1 B3 = 90◦ + 21 ∠Y XZ hay
∠B1 A1 C1 = 90◦ − 21 ∠Y XZ = ∠B1 IC1 . Ta thu được I ∈ (A1 B1 C1 ) hay I ∈ (O, 2R).

Bài 404. (Nguyễn Văn Linh). Cho tam giác ABC nội tiếp đường tròn (O). P là một điểm chuyển
động trên (O). Gọi la , lb , lc lần lượt là các đường thẳng đối xứng với AP, BP, CP qua BC, CA, AB.
la , lb , lc cắt nhau tạo thành tam giác XY Z. Chứng minh rằng khi P chuyển động, tâm đường tròn nội
tiếp của tam giác XY Z luôn nằm trên một đường tròn cố định.

Z'
B'
X Z
I
C' X'
Y A
P

Y'
B
C

A'

Chứng minh. (TelvCohl). Gọi A0 , B 0 , C 0 lần lượt là các điểm đối xứng với A, B, C qua BC, CA, AB.
Do la đối xứng với P A qua BC nên A0 ∈ la . Tương tự B 0 ∈ lb , C 0 ∈ lc .
Ta có (XZ, XY ) ≡ (XZ, AC) + (AC, AB) + (AB, XY ) ≡ (AC, BP ) + (AC, AB) + (P C, AB) ≡
(AC, P C) + (P C, BP ) + (P C, AB) + (AC, AB) ≡ 3(AC, AB) (mod π).
Mặt khác (AB 0 , AC 0 ) ≡ (AB 0 , AC)+(AC, AB)+(AB, AC 0 ) ≡ 3(AC, AB) (mod π) nên (XZ, XY ) ≡
(AB 0 , AC 0 ) (mod π).
Suy ra X ∈ (AB 0 C 0 ). Tương tự Y ∈ (BA0 C 0 ), Z ∈ (A0 B 0 C).
Từ đó XI đi qua trung điểm X 0 của cung B 0 C 0 của (AB 0 C 0 ). Tương tự Y I đi qua Y 0 , ZI đi qua
0
Z.
Do tam giác XY Z có dạng không đổi khi P chuyển động nên ∠Y 0 IX 0 , ∠Z 0 IX 0 luôn không đổi. Từ
đó I là giao của hai cung tròn cố định dựng trên dây Y 0 X 0 , Z 0 X 0 . Do I chuyển động nên I, X 0 , Y 0 , Z 0
đồng viên. Vậy I ∈ (X 0 Y 0 Z 0 ) cố định.

Bài 405. Cho hai đường tròn (O1 ) và (O2 ) cùng tiếp xúc trong với đường tròn ngoại tiếp tam giác
ABC và lần lượt tiếp xúc với AB, AC tại D, E sao cho (O1 ), (O2 ) nằm ngoài tam giác ABC. Gọi l
là tiếp tuyến chung ngoài của (O1 ) và (O2 ) sao cho (O1 ), (O2 ) nằm khác phía với BC bờ là l. Chứng
minh rằng l k BC khi và chỉ khi DE k BC.

345
A

J
O2
O1
E
D
U Y I Z V

B C

Chứng minh. Gọi U, V là giao của l với các cung AB, AC. (O1 ), (O2 ) tiếp xúc với l lần lượt tại Y, Z.
Áp dụng định lý Sawayama-Thebault cho tam giác AU V suy ra DY giao EZ tại tâm nội tiếp J của
4AU V .
+Nếu ` k BC.
Ta có ∠BAU = ∠CAV nên AJ là phân giác ∠BAC. Gọi I là tâm nội tiếp 4ABC, dễ thấy
JDY k IB và JEZ k IC. Ta thu được 4JDE và 4IBC vị tự theo tâm A. Từ đó DE k BC.
+Nếu DE k BC.
Kẻ tiếp tuyến l0 của (O1 ) song song với BC. Dựng đường tròn (O20 ) tiếp xúc với l0 , (O) và tiếp xúc
với AC tại E 0 . Theo phần thuận ta thu được DE 0 k BC, suy ra E 0 ≡ E. Từ đó (O20 ) ≡ (O2 ) và l0 ≡ l.
Suy ra l k BC.

Bài 406. Cho tam giác ABC nội tiếp đường tròn (O). Gọi A1 , B1 , C1 lần lượt là trung điểm BC, CA, AB;
A2 , B2 , C2 lần lượt là điểm chính giữa cung BC, CA, AB. Gọi (Oa ), (Ob ), (Oc ) lần lượt là đường tròn
đường kính A1 A2 , B1 B2 , C1 C2 . da là tiếp tuyến chung ngoài của (Ob ) và (Oc ) sao cho (Ob ), (Oc ) nằm
khác phía với BC bờ da . Tương tự xác định db , dc . Gọi XY Z là tam giác tạo bởi giao điểm của da , db , dc .
Chứng minh rằng tam giác XY Z vị tự với tam giác ABC theo tỉ số 1/4.

Chứng minh. Cách 1.

346
A G
B2
E

C2 Ob
Oc
C1 Z B1
Y
I

B A1 C

D Oa
F
A2

Từ I kẻ tiếp tuyến IG với (B2 , B2 B1 ). Dễ thấy 4IGB2 = 4CB1 B2 (c.c.c) nên ∠GIB2 = ∠B2 CA =
∠ABI, suy ra IG k AB. Tương tự kẻ tiếp tuyến IF tới (A2 , A2 A1 ) thì IF k AB nên G, I, F thẳng
hàng.
1 1
Phép vị tự HB2 1 : G 7→ E, HA2 1 : F 7→ D.
Suy ra ED là đường trung bình của hình thang GF A1 B1 . Ta có Oa D k A2 F nên Oa D ⊥ DE.
Chứng minh tương tự suy ra DE là tiếp tuyến của (Oa ) và (Ob ).
Gọi DE giao IA1 , IB1 lần lượt tại X 0 , Y 0 suy ra X 0 , Y 0 là trung điểm IA1 , IB1 .
Chứng minh tương tự cho da , db suy ra X 0 ∈ db hay X 0 ≡ X. Như vậy X, Y, Z lần lượt là trung
1
điểm IA1 , IB1 , IC1 . Vậy tam giác XY Z vị tự với tam giác ABC theo tỉ số .
4
Cách 2.

347
Ib

B2
A

Ic Hb

Y
I B1

X
A1
B C

Ha
A2

Ia

Gọi Ia , Ib , Ic lần lượt là tâm bàng tiếp góc A, B, C của tam giác ABC. Ha , Hb , Hc lần lượt là trực
tâm tam giác BCIa , CAIB , ABIc .
Dễ thấy BICHa là hình bình hành nên A1 là trung điểm IHa .
Do đó HI2 : (Oa ) 7→ (Ia Ha ). Tương tự (Ob ) 7→ (Ib Hb ).
Gọi M là giao của CHa và BIa , N là giao của CHb và AIb .
Cộng góc dễ thấy M A1 k AB, N B1 k AB, suy ra M, N, A1 , B1 thẳng hàng. Do ∠B1 N Hb =
∠B1 CN = ∠N Ib Hb nên N B1 là tiếp tuyến của (Ib Hb ). Chứng minh tương tự suy ra B1 C1 là tiếp
tuyến chung của (Ia Ha ) và (Ib Hb ).
1
Phép vị tự HI2 : B1 7→ Y 0 , A1 7→ X 0 thì Y 0 X 0 là tiếp tuyến chung của (Oa ), (Ob ). Đến đây làm tiếp
theo cách 1.
Cách 3.

348
A B2

C2 Ob
Oc
Z B1
C1
Y
I

B A1 E C
D Oa

A2

Theo bài 2 ta thu được hai tam giác ABC và XY Z có cạnh tương ứng song song.
Gọi D, E là tiếp điểm của dc , db với (Oa ). Theo bài 2 ta có dc k AB. Suy ra A2 D đi qua C2 , tương
tự A2 E đi qua B2 .
Do A2 , B2 , C2 là điểm chính giữa cung BC, CA, AB nên theo kết quả quen thuộc, I là trực tâm
tam giác A2 B2 C2 .
Lại có A2 là tâm vị tự ngoài của (Oa ) và (O) nên phép vị tự tâm A2 biến D thành C2 , E thành
B2 . Từ đó DE k C2 B2 . Mà XOa ⊥ DE nên XOa k A2 I.
Vậy các đường thẳng lần lượt qua A2 , B2 , C2 và song song với XOa , Y Ob , ZOc đồng quy tại I.
1 BA2
Mặt khác, ∠DXOa = ∠A2 BA1 = ∠BAC nên 4XDOa ∼ 4BA1 A2 . Ta thu được =
2 XOa
A2 A1
= 2. Từ đó A2 I = A2 B = 2Oa X. Suy ra X là trung điểm IA1 . Tương tự ta thu được đpcm.
Oa D
Bài 407. Cho tứ giác ABCD ngoại tiếp. Chứng minh rằng trung trực của các cạnh AB, BC, CD, DA
cắt nhau tạo thành một tứ giác ngoại tiếp.

M N

J B

C1 B1
I
D1 A1

D
C

349
Chứng minh. Gọi A1 B1 C1 D1 là tứ giác tạo bởi giao điểm các trung trực của AB, BC, CD, DA.
Gọi E là giao của AD và BC, (J) là đường tròn nội tiếp tam giác ABE. Kẻ tiếp tuyến của (J)
song song với CD, cắt EA, EB lần lượt tại M, N.
Ta có tứ giác AJBI nội tiếp nên ∠EAJ = ∠JAB = ∠JIB. Suy ra 4EAJ ∼ 4EIB.
EA EJ
Suy ra = hay EI · EJ = EA · EB.
EI EB
1 1
Mặt khác, ∠EID = 90◦ + ∠BCD = 90◦ + ∠EN M = ∠EN J.
2 2
EI ED
Suy ra 4EID ∼ 4EN J. Ta thu được = hay EI · EJ = EN · ED.
EN EJ
EA EN
Suy ra EA · EB = EN · ED hay = . Vậy AN k BD.
ED EB
Do A1 , C1 lần lượt là tâm ngoại tiếp các tam giác ADB, CDB nên A1 C1 ⊥ BD hay A1 C1 ⊥ DN .
Mà C1 D1 ⊥ DC nên C1 D1 ⊥ M N và lại có D1 A1 ⊥ AM nên 4D1 A1 C1 ∼ 4M AN.
Chứng minh tương tự suy ra tứ giác A1 B1 C1 D1 đồng dạng tứ giác ABN M . Mà tứ giác ABN M
ngoại tiếp nên A1 B1 C1 D1 ngoại tiếp.

Bài 408. Cho tứ giác nội tiếp ABCD. Một đường tròn bất kì qua C, D cắt AD, AC tại A1 , A2 , cắt
BC, BD tại B1 , B2 . Một đường tròn bất kì qua A, B cắt CB, CA tại C1 , C2 , cắt DA, DB tại D1 , D2 .
Chứng minh rằng các đường thẳng A1 A2 , B1 B2 , C1 C2 , D1 D2 cắt nhau tạo thành một tứ giác ngoại
tiếp.

A
M B2
A1 B1
N
D2 Q
C2
A2
D1
C1

P C

Chứng minh. Gọi M N P Q là tứ giác tạo bởi giao điểm của A1 A2 , B1 B2 , C1 C2 , D1 D2 .


Do các tứ giác ABCD và ABC2 D2 nội tiếp nên C2 D2 k CD, tương tự D1 C1 k DC.
Từ đó suy ra D2 C2 C1 D1 là hình thang cân, ta thu được P C1 D1 là tam giác cân.
Chứng minh tương tự suy ra các tam giác QA1 D1 , M A1 B2 , N C1 B2 cân.
Suy ra M N + P Q = M B2 + B2 N + P D1 + D1 Q = M A1 + N C1 + P C1 + QA1 = M Q + N P . Vậy
tứ giác M N P Q ngoại tiếp.

Bài 409. (Trần Minh Ngọc). Cho n-giác lưỡng tâm A1 A2 A3 ...An . Gọi Ai(i+1) là giao của Ai−1 Ai và
Ai+1 Ai+2 (i = 1, n). Chứng minh rằng tồn tại hai đường tròn tiếp xúc với đường tròn ngoại tiếp các
tam giác Ai Ai(i+1) Ai+1 .

350
A23

A12

O23
A2
O12
D12 D23
B2
A1
X12
X23

A3
O'
Ak+1
O P
I

Ak

Ak-2
Ak-1
A(k-1)k

Chứng minh. Ta chứng minh kết quả mạnh hơn:


Nếu A1 A2 , A2 A3 , ..., Ak Ak+1 lần lượt là các tiếp tuyến kẻ từ các điểm A1 , A2 , ..., Ak nằm trên (O) tới
đường tròn (I) bất kì chứa trong (O) thì tồn tại hai đường tròn tiếp xúc với (A1 A12 A2 ), (A2 A23 A3 ),...,
(Ak−1 A(k−1)k Ak ).
Thật vậy, gọi Di(i+1) là tiếp điểm của (I) với Ai Ai+1 , Xi(i+1) là giao điểm thứ hai của Ai(i+1) I với
(Ai Ai(i+1) Ai+1 ), Oi(i+1) là tâm của (Ai Ai(i+1) Ai+1 ).
Do Xi(i+1) là điểm chính giữa cung Ai Ai+1 của (Oi(i+1) ) nên Oi(i+1) Xi(i+1) đồng quy tại O.
Gọi Bi là giao của tiếp tuyến tại X(i−1)i của (O(i−1)i ) và tiếp tuyến tại X(i(i+1) của (O(i(i+1) ).
Do I là tâm đường tròn bàng tiếp của các tam giác Ai Ai(i+1) Ai+1 nên Xi(i+1) là tâm ngoại tiếp
tam giác Ai Ai+1 I. Điều này nghĩa là IAi vuông góc với X(i−1)i Xi(i+1) hay X(i−1)i Xi(i+1) song song với
D(i−1)i Di(i+1) . Từ đó hai tam giác Bi X(i−1)i Xi(i+1) và Ai D(i−1)i Di(i+1) có cạnh tương ứng song song
hay đồng dạng với nhau. Suy ra Bi X(i−1)i = Bi Xi(i+1) .
Từ đó hai tam giác vuông OBi X(i−1)i và OBi Xi(i+1) bằng nhau, suy ra OX(i−1)i = OXi(i+1) . Như
vậy tồn tại một đường tròn tâm O tiếp xúc với tất cả các đường tròn (Oi(i+1) ) (i = 1, k − 1).
Dễ thấy hai đường gấp khúc A1 A2 ...Ak−1 và B1 B2 ...Bk−1 đều có đường tròn tiếp xúc với các cạnh
và có cạnh tương ứng song song nên chúng vị tự nhau theo tâm P . Do Bi X(i−1)i = Bi Xi(i+1) nên Bi
nằm trên trục đẳng phương của (O(i−1)i ) và (Oi(i+1) ). Từ đó P là tâm đẳng phương của các đường
tròn (Oi(i+1) ).
PP /(O )
Xét phép nghịch đảo IP i(i+1)
: (Oi(i+1) ) 7→ (Oi(i+1) ), (O, OXi(i+1) ) 7→ (O0 ). Như vậy (O0 ) là
đường tròn thứ hai tiếp xúc với các đường tròn (Oi(i+1) ).

Bài 410. (Nguyễn Văn Linh). Cho tứ giác A0 B 0 C 0 D0 . Gọi A, C lần lượt là hình chiếu của A0 , C 0 trên
B 0 D0 ; B, D lần lượt là hình chiếu của B 0 , D0 trên A0 C 0 . Chứng minh rằng tứ giác A0 B 0 C 0 D0 ngoại tiếp
khi và chỉ khi tứ giác ABCD ngoại tiếp.

351
D'

C'

A
B
E
A'
D
B'

Chứng minh. Dễ thấy các tứ giác AA0 DD0 , ABB 0 A0 , BB 0 CC 0 , DCC 0 D0 nội tiếp đường tròn đường kính
A0 D0 , A0 B 0 , B 0 C 0 , C 0 D0 .
Đặt ∠AA0 B = ∠AB 0 B = ∠AD0 D = ∠BC 0 C = α. Theo định lý hàm số sin, AD = A0 D0 sin ∠AD0 D =
A D0 sin α, BC = B 0 C 0 sin ∠BC 0 C = B 0 C 0 sin α. Suy ra AD + BC = sin α(A0 D0 + B 0 C 0 ). Tương tự
0

AB + CD = sin α(A0 B 0 + C 0 D0 ). Từ đó tứ giác A0 B 0 C 0 D0 ngoại tiếp khi và chỉ khi A0 B 0 + C 0 D0 =


A0 D0 + B 0 C 0 khi và chỉ khi AB + CD = AD + BC hay tứ giác ABCD ngoại tiếp.

Bài 411. (Đào Thanh Oai). Cho tứ giác ngoại tiếp ABCD. AC giao BD tại E. Một đường tròn ω có
tâm E bất kì. Các đường đối cực của A, C ứng với ω cắt BD tại A0 , C 0 . Các đường đối cực của B, D
ứng với ω cắt AC tại B 0 , D0 . Chứng minh rằng tứ giác A0 B 0 C 0 D0 ngoại tiếp.
Chứng minh. Cách 1.
M B
A D'
U
C'
X Y
E
Q Z
T

B'

V
D A'
P
C

Gọi X, Y, Z, T là các điểm liên hợp của A, B, C, D ứng với ω. Suy ra B 0 C 0 Y Z nội tiếp đường tròn
đường kính B 0 C 0 .

352
Ta có EY.EB = EZ.EC nên BY ZC nội tiếp. Từ đó B 0 C 0 k BC. Tương tự suy ra hai tứ giác
ABCD và A0 B 0 C 0 D0 có cạnh tương ứng song song. Từ đó ta có thể chọn ω sao cho A ≡ D0 , D ≡ A0 .
Gọi M, P, Q là tiếp điểm của đường tròn (I) nội tiếp tứ giác ABCD với AB, CD, DA. M P cắt
AC 0 , DB 0 lần lượt tại U, V.
Do AC 0 k DC nên ∠AU M = ∠M P D = ∠AM U , suy ra AM = AU = AQ. Tương tự, DP =
AR + DR − AD
DV = DQ. Gọi R là giao của AC 0 và DB 0 suy ra RU = RV = . Vậy U, V là tiếp
2
điểm của đường tròn (J) nội tiếp tam giác ARD với AR, DR. Như vậy (J) tiếp xúc với AC 0 , DB 0 , AD
lần lượt tại U, V, Q. Mặt khác, AC, BD, M P đồng quy nên AB 0 , DC 0 , U V đồng quy. Áp dụng định lý
Brianchon đảo cho lục giác AU C 0 B 0 V D suy ra C 0 B 0 tiếp xúc với (J). Vậy tứ giác AC 0 B 0 D ngoại tiếp
hay A0 B 0 C 0 D0 ngoại tiếp.
Cách 2.
B
A
D'
C'
X Y
E
T
Z

B'

A'

Gọi X, Y, Z, T lần lượt là điểm liên hợp với A, B, C, D ứng với ω. Gọi r là bán kính của ω. Ta có
2
EA · EX = EB · EY = EC · EZ = ED · ET = r2 nên phép nghịch đảo IEr : A 7→ A0 , B 7→ B 0 , C 7→
C 0 , D 7→ D0 . Do đó A0 B 0 C 0 D0 là tứ giác ngoại tiếp (xem []). Áp dụng bài toán trên suy ra đpcm.

Bài 412. Cho tứ giác ABCD ngoại tiếp đường tròn (O). AD giao BC tại E, AC giao BD tại L.
Đường thẳng qua L vuông góc với OL cắt AD, BC lần lượt tại X, Y . Đường thẳng qua X song song
với CD cắt đường thẳng qua Y song song với AB tại Z. Chứng minh rằng XZ + Y Z = XE + Y E.

353
W

S
M B
A
K Y
Q

F Z X L N
G T
O
J
R
D
P
C

Chứng minh. Gọi M, N, P, Q lần lượt là tiếp điểm của (O) với AB, BC, CD, DA. QM cắt P N tại W ,
QP cắt M N tại V . XY cắt AB, CD tại S, R.
Ta có L nằm trên đường đối cực của E, F, V, W nên E, F, V, W nằm trên đường đối cực của L, suy
ra E, F, V, W thẳng hàng. Theo định lý Brocard, OL ⊥ EF nên EF k XY.
Lại có E(F LDC) = −1 nên LX = LY . Tương tự LS = LR.
Qua Y kẻ đường song song với ED cắt QN tại T , suy ra Y T = T N . Mà LX = LY nên Y T = QX.
Từ đó QX = T N . Vậy XE + Y E = EQ + EN = 2EN.
Gọi J, K là giao của ZX, ZY với M P . Qua X kẻ đường song song với AB cắt M P tại G. Suy ra
XG = XJ. Mà LX = LY nên XL = KY . Suy ra ZX + ZY = ZJ + ZK = 2ZJ.
LJ LX LY LK
Ta có = = = nên L, Z, F thẳng hàng. Dễ thấy V, A, C thẳng hàng. Suy ra
LP LR LS LM
ZJ LX LX VE
= = = .
FP LR LS VF
VE EN VE VF sin ∠M N B sin ∠V M F
Như vậy ta cần chứng minh = hay = , khi và chỉ khi = ,
VF FM EN FM sin ∠EV N sin ∠F V M
hiển nhiên đúng.
Vậy EN = ZJ hay XE + Y E = XZ + Y Z.

Bài 413. (Lym) Cho tam giác ABC nội tiếp đường tròn (O), ngoại tiếp đường tròn (I). G là điểm
chính giữa cung BC không chứa A. D là tiếp điểm của (I) với BC. J là điểm nằm trên BC sao cho
JG cắt ID tại K thỏa mãn IK = IJ. KJ cắt (O) lần thứ hai tại H. HI cắt (O) tại L. Chứng minh
rằng đường thẳng Simson của L ứng với tam giác ABC tiếp xúc với (I).

354
A

L
P O
I

H
B J
C
D

G
K

Chứng minh. Kẻ đường kính LQ của đường tròn ngoại tiếp tam giác LIG.
Do GI 2 = GJ.GH nên ∠GIH = ∠IJG = ∠IKJ = 90◦ − ∠GJB = 90◦ − ∠GLH.
Mà ∠GIH + ∠GIQ = 90◦ nên ∠QIG = ∠ILG = ∠IQG, suy ra GI = GQ hay Q ∈ (BIC).
Kẻ LM ⊥ AC, LP ⊥ AB. Ta có ∠LQI = ∠LGI = ∠LBP = ∠LCM nên phép vị tự quay góc
LP
quay bằng ∠BLP , tỉ số lần lượt biến B 7→ P, Q 7→ I, C 7→ M, suy ra 4BQC 7→ 4P IM . Như vậy
LB
1
4BQC ∼ 4P IM . Suy ra ∠P IM = ∠BQC = 180◦ − ∠BIC = 90◦ − ∠M AP . Vậy (I) là đường tròn
2
bàng tiếp góc A của tam giác AP M hay P M tiếp xúc với (I). Ta có đpcm.

Bài 414. Cho tam giác nhọn ABC nội tiếp đường tròn (O) có ∠B > ∠C, trực tâm H. P là điểm nằm
2
trên (O) sao cho H và P khác phía với AO và ∠AOP = ∠HAO. Gọi Q là điểm đối xứng với P qua
3
O. Chứng minh rằng đường thẳng Simson của Q tiếp xúc với đường tròn Euler của tam giác ABC.
P
A

H
E O

B C
N

K Q

355
Chứng minh. Qua P kẻ P K ⊥ BC(K ∈ (O)). P K cắt AO tại L. Ta có ∠LP O = ∠ALP − ∠AOP =
1
∠HAO − ∠AOP = ∠AOP = ∠AKP . Suy ra AK k OP.
2
Mà AK song song với đường thẳng Simson của P ứng với tam giác ABC nên đường thẳng Simson
của P song song với OP.
Gọi E là trung điểm HO, M là trung điểm HP suy ra M nằm trên đường thẳng Simson của P ,
mà M E k OP nên M E là đường thẳng Simson của P . Gọi N đối xứng với M qua E. Chúng ta biết
rằng đường thẳng Simson của hai điểm là hai đầu của đường kính thì vuông góc với nhau tại một điểm
trên đường tròn Euler nên đường thẳng Simson của Q là đường thẳng qua N vuông góc với M N hay
tiếp tuyến tại N của (E). Ta có đpcm.

Bài 415. (Trần Quang Hùng- Luis González). Cho tam giác ABC. P là điểm bất kì trong mặt phẳng.
Gọi Pa , Pb , Pc lần lượt đối xứng với P qua BC, CA, AB. Chứng minh rằng (BPa C), (CPb A), (APc B)
đồng quy tại Q và đường tròn Euler của các tam giác AP Q, BP Q, CP Q tiếp xúc với nhau.
A

Pc Pb

M
P
Q

B C

Pa

Chứng minh. Gọi Q là giao của (APb C) và (BPa C). Ta có (QB, QA) ≡ (QB, QC) + (QC, QA) ≡
(Pa B, Pa C) + (Pb C, Pb A) ≡ (P C, P B) + (P A, P C) ≡ (P A, P B) ≡ (Pc B, Pc A) (mod π)
Suy ra Q ∈ (APc B).
Gọi M là trung điểm P Q. Tiếp tuyến tại M của các đường tròn Euler của các tam giác AP Q,
BP Q, CP Q hiển nhiên lần lượt song song với tiếp tuyến da tại A của (AP Q), db tại B của (BP Q),
dc tại C của (CP Q). Do đó ta cần chứng minh da k db k dc .
Ta có (db , P Q) = ∠BQP − ∠BP Q, (da , P Q) = ∠AP Q − ∠AQP.
Do ∠AQB = ∠AP B nên ∠BQP + ∠AQP = ∠BP Q + ∠AP Q. Suy ra ∠BQP − ∠BP Q = ∠AP Q −
∠AQP . Vậy (db , P Q) = (da , P Q) hay da k db . Chứng minh tương tự ta có đpcm.

Bài 416. Chứng minh rằng trung điểm của P Q nằm trên đường tròn Euler của tam giác ABC.
Chứng minh. Do đường tròn Euler của tam giác AP Q và BP Q tiếp xúc nhau tại M nên M là điểm
Euler-Poncelet của 4 điểm A, Q, B, P. Suy ra M nằm trên đường tròn Euler của tam giác AP B. Tương
tự M cũng nằm trên đường tròn Euler của tam giác BP C, CP A. Do đó M là điểm Euler-Poncelet của
4 điểm A, B, C, P . Từ đó M nằm trên đường tròn Euler của tam giác ABC.

Hệ quả. Trung điểm của đoạn nối hai điểm Fermat nằm trên đường tròn Euler. Q được gọi là
điểm Antigonal conjugate của P.

356
Bài 417. Cho tam giác ABC nội tiếp đường tròn (O), I là tâm nội tiếp. AI, BI, CI giao (O) lần thứ
hai tại D, E, F . A1 , B1 , C1 lần lượt là điểm đối xứng với D, E, F qua BC, CA, AB. X, Y, Z lần lượt là
giao của B1 C1 với EF , A1 C1 với DF , A1 B1 với DE. Chứng minh rằng X, Y, Z thẳng hàng và đường
thẳng này tiếp xúc với đường tròn Euler của tam giác ABC.
B
D

C1

A1
Y
C
I

M
B1

E X F
J

Z A

Chứng minh. Gọi J là điểm Antigonal conjugate của I ứng với tam giác ABC. M là trung điểm IJ.
Khi đó M là điểm Euler-Poncelet của 4 điểm A, B, C, I. Mà đường tròn pedal của I ứng với tam giác
ABC đi qua M nên M là điểm Feuerbach của tam giác ABC.
Do E là tâm ngoại tiếp tam giác AIC nên B1 là tâm ngoại tiếp tam giác AJC, tương tự C1 là tâm
ngoại tiếp tam giác AJB. Suy ra C1 B1 là trung trực AJ.
Lại có EF là trung trực IA nên X là tâm ngoại tiếp tam giác AIJ. Tương tự suy ra X, Y, Z cùng
nằm trên trung trực của IJ hay đường thẳng qua M vuông góc với IM. Vậy đường thẳng đi qua
X, Y, Z là tiếp tuyến tại điểm Feuerbach của đường tròn Euler của tam giác ABC.

Bài 418. (Nguyễn Văn Linh) Cho tam giác ABC nội tiếp đường tròn (O). Tiếp tuyến tại A, B, C
cắt cạnh đối diện lần lượt tại Pa , Pb , Pc ; Ma , Mb , Mc lần lượt là trung điểm BC, CA, AB. Chứng minh
rằng đường tròn Euler của các tam giác APa Ma , BPb Mb , CPc Mc có trung trục đẳng phương là đường
thẳng Euler của tam giác ABC.

Xa
T

Mc Mb

O
E
H

Pa B Ha Ma C

357
Chứng minh. Gọi ωa , ωb , ωc lần lượt là đường tròn Euler của các tam giác APa Ma , BPb Mb , CPc Mc .
Gọi E là tâm của ωa , Ha , Hb , Hc là hình chiếu của A, B, C trên BC, CA, AB.
Ta có ∠EMa Ha = 90◦ − ∠EMa O = 90◦ − |∠Ma Mb Mc − ∠Ma Mc Mb | = 90◦ − |∠B − ∠C|.
Mà ∠APa Ma = |∠B − ∠Pa AB| = |∠B − ∠C| nên ∠EMa Ha + ∠APa Ma = 90◦ .
Suy ra Ma E ⊥ APa . Gọi Ta = Ma E ∩ AHa thì Ta là trực tâm của tam giác APa Ma . Do E là trung
điểm Ma Ta nên E nằm trên wa . Chứng minh tương tự suy ra ωa , ωb , ωc đồng quy tại E.
Mặt khác, gọi Xa là giao của ωa với AHa . Ta thu được Xa là trung điểm ATa . Mà Ta là trung
3 3
điểm AH nên HXa · HHa = HA · HHa = HB · HHb = HXb · HHb . Do đó H nằm trên trục đẳng
4 4
phương của ωa và ωb .
Chứng minh tương tự suy ra đường thẳng Euler của tam giác ABC là trục đẳng phương của
ωa , ωb , ωc .

Bài 419. (Nguyễn Văn Linh). Cho tam giác ABC nội tiếp đường tròn (O) có I là tâm nội tiếp. AI
cắt (O) lần thứ hai tại J. Gọi ω là đường tròn tâm J và tiếp xúc với AB, AC. Hai tiếp tuyến chung
ngoài của (O) và ω tiếp xúc với (O) tại hai điểm E, F . Chứng minh rằng E, I, F thẳng hàng.

I T
E F
K
C
B

Chứng minh. Gọi P là tâm vị tự ngoài của (O) và ω. Hiển nhiên J nằm trên phân giác ∠EP F. Mà
OE = OF = OJ, O nằm trên (EP F ) nên J là tâm nội tiếp tam giác P EF . Từ đó EF tiếp xúc với ω.
1
Ta có EF ⊥ OJ nên EF k BC. Gọi T là giao của OJ với ω. Ta có ∠OJC = 90◦ − ∠BAC = ∠AJK
2
nên ∠IJT = ∠CJK. Từ đó 4IJT = 4CJK. Suy ra ∠IT J = 90◦ hay tiếp tuyến kẻ từ I tới ω song
song với BC. Như vậy I ∈ EF.

Bài 420. (Trần Quang Hùng - Luis González) Cho tứ giác ABCD nội tiếp đường tròn (O). P là giao
của AC và BD. Gọi (O1 ) là đường tròn tiếp xúc với tia P C, P D và tiếp xúc trong với (O), (O2 ) là
đường tròn tiếp xúc với tia AD, BC và tiếp xúc ngoài với (O). Hai tiếp tuyến chung ngoài của (O) và
(O2 ) tiếp xúc với (O) lần lượt tại E, F . Chứng minh rằng EF tiếp xúc với (O1 ).

Chứng minh.

358
Bổ đề 44. Cho tứ giác ABCD nội tiếp đường tròn (O), AC cắt BD tại E. Gọi ω là đường tròn tiếp
xúc với tia EC, ED lần lượt tại M, N và tiếp xúc trong với (O) tại P . I, J lần lượt là tâm đường tròn
nội tiếp các tam giác ECD, ACD. Khi đó P, I, J, N, D cùng thuộc một đường tròn.

K
A
L
E

J M
N
O
I

D C

Chứng minh.
Gọi K, L lần lượt là giao của P M, P N với (O).
Theo định lý Sawayama-Thebault ta có J nằm trên M N.
1
Suy ra ∠EN J = 90◦ − ∠DEC = ∠JID. Từ đó N, J, I, D cùng thuộc một đường tròn.
2
Mặt khác, P là tâm vị tự ngoài của ω và (O) nên LK k M N. Suy ra ∠N P D = ∠LKD = ∠N JD
hay N, J, P, D cùng thuộc một đường tròn.
Vậy P, I, J, N, D cùng thuộc một đường tròn.
Bổ đề 45. Cho tứ giác ABCD nội tiếp đường tròn (O). AC giao BD tại E. Gọi (I) là đường tròn
tiếp xúc với tia EA, EB và tiếp xúc trong với (O); (J) là đường tròn tiếp xúc với AD, BC và tiếp xúc
trong với (O) tại một điểm nằm trên cung AB không chứa C, D. Khi đó (I), (J), (O) có chung một
tiếp điểm.

T
P
I
Z
A X
Q N
Y M
E
J

O
D C

Chứng minh.

359
Gọi T là tiếp điểm của (I) với (O), X, Y, Z lần lượt là tâm nội tiếp các tam giác ABC, ABD, ABE.
M, N là tiếp điểm của (I) với EA, EB; M N cắt BC, AD lần lượt tại P, Q.
Hiển nhiên các bộ 3 điểm A, Z, X; B, Z, Y thẳng hàng. Theo định lý Sawayama-Thebault, X, Y
nằm trên M N.
Áp dụng bổ đề 2 suy ra T ZN XB, T ZM Y A nội tiếp.
Suy ra ∠T Y M = ∠T AC = 180◦ − ∠T BC. Ta thu được Y T BP nội tiếp. Tương tự T AQX nội tiếp.
Từ đó ∠T P B = ∠T Y B = ∠T AX = ∠T QP hay BC tiếp xúc với (P QT ). Tương tự AD tiếp xúc
với (P QT ).
1 1
Đồng thời ∠BT P = ∠BY P = ∠M AZ = ∠BAC = ∠BT C nên T P là phân giác ∠BT C. Suy
2 2
ra (P T Q) tiếp xúc với (O) tại T . Vậy (P T Q) ≡ (J). Ta có đpcm.
Trở lại bài toán.

Q
B U
Y
A O4 E
P O6

F Z
O5 O C

D O1

X
T

O2

O3

Gọi X, T lần lượt là tiếp điểm của (O1 ), (O2 ) với (O). (O3 ) là đường tròn tiếp xúc với tia AC, BD
và tiếp xúc ngoài với (O).
Áp dụng bổ đề 2 suy ra T là tiếp điểm chung của (O), (O2 ), (O3 ).
Gọi (O4 ) là đường tròn tiếp xúc với tia P A, P B và tiếp xúc trong với (O) tại Y.
Áp dụng định lý Monge-D’Alembert cho 3 đường tròn (O3 ), (O4 ), (O) suy ra Y, P, T thẳng hàng.
Gọi Q là giao của AD và BC, (O5 ), (O6 ) là hai đường tròn tiếp xúc với AD, BC và tiếp xúc trong
với (O). Lại áp dụng bổ đề 1 suy ra (O5 ), (O6 ) lần lượt tiếp xúc với (O) tại X, Y . Gọi R là tâm vị tự
ngoài của (O) và (O2 ).
Áp dụng định lý Monge-D’Alembert cho 3 đường tròn (O), (O5 ), (O6 ) suy ra Q, X, Y thẳng hàng.
Lại áp dụng cho 3 đường tròn (O), (O2 ), (O5 ) suy ra R, X, Q thẳng hàng. Vậy R, X, Y, Q thẳng hàng.
Gọi U là giao điểm thứ hai của RT với (O).
XU giao (O1 ) tại Z. Do X là tâm vị tự ngoài của (O) và (O1 ) nên tiếp tuyến tại U của (O) song
song với tiếp tuyến tại Z của (O1 ) hay song song với EF.

360
Do P là tâm vị tự ngoài của (O1 ) và (O3 ), tiếp tuyến tại Z của (O1 ) song song với EF nên song
song với tiếp tuyến tại T của (O3 ), suy ra P, Z, T thẳng hàng. Như vậy Y T giao U X tại Z, suy ra Z
nằm trên đường đối cực của R với (O) hay Z ∈ EF . Từ đó EF tiếp xúc với (O1 ) tại Z.

Bài 421. (Nguyễn Văn Linh) Cho tứ giác ABCD nội tiếp đường tròn (O). AC giao BD tại E, AD
giao BC tại F . Gọi (O1 ) là đường tròn tiếp xúc với tia EA, EB và tiếp xúc trong với (O). (O2 ) là
đường tròn tiếp xúc với tia F A, F B và tiếp xúc ngoài với (O) tại một điểm trên cung AB không chứa
C, D. Chứng minh rằng giao của hai tiếp tuyến chung ngoài của (O1 ) và (O2 ) nằm trên (O).

J B
P

T
I
A

D L C

Chứng minh.
Gọi (L) là đường tròn tiếp xúc với tia EC, ED và tiếp xúc trong với (O) tại K. T, P lần lượt là
tiếp điểm của (I), (J) với (O).
Theo bổ đề trong bài toán trước, tồn tại đường tròn ω1 và ω2 tiếp xúc với AD, BC và lần lượt tiếp
xúc trong với (O) tại T, K. Tương tự ta cũng thu được tồn tại đường tròn ω3 tiếp xúc với AC, BD và
tiếp xúc ngoài với (O) tại P.
Áp dụng định lý Monge D’Alembert cho 3 đường tròn (I), ω2 , (O) suy ra F, T, K thẳng hàng; cho
3 đường tròn ω3 , (L), (O) suy ra P, E, K thẳng hàng.
Lại áp dụng định lý Monge D’Alembert cho 3 đường tròn (I), (J), ω1 suy ra tâm vị tự ngoài của
(I) và (J) nằm trên T F ; cho 3 đường tròn (I), (J), ω3 suy ra tâm vị tự ngoài của (I) và (J) nằm trên
P E. Mà F T giao P E tại K nên K là tâm vị tự ngoài của (I) và (J). Ta có đpcm.

Bài 422. (Sharygin Geometry Olympiad) Cho hình vuông ABCD. P là một điểm bất kì trên AB.
Gọi I1 , I2 , I3 lần lượt là tâm đường tròn nội tiếp các tam giác AP D, DP C, CP B; H1 , H2 , H3 lần lượt
là trực tâm các tam giác AI1 D, DI2 C, CI3 B. Chứng minh rằng H1 , H2 , H3 thẳng hàng.

361
A P B
N
H3 M
I3

I1 H2 H1

I2

D C

Chứng minh. Trên P D, P C lấy hai điểm M, N sao cho DM = DC = CN . Ta có DA = DM nên DI1
là trung trực của AM . Suy ra AM đi qua H1 . Tương tự BN đi qua H3 , DN cắt CM tại H2 . Áp dụng
định lý Desargues cho 2 tam giác DN B và tam giác AM C ta có DM, CN, AB đồng quy tại P nên
H1 = AM ∩ DB, H2 = DN ∩ CM, H3 = AC ∩ BN thẳng hàng.

Bài 423. (Nguyễn Văn Linh). Cho hình vuông ABCD. P là điểm bất kì trên AB. Gọi (I1 ), (I2 ) lần
lượt là đường tròn nội tiếp các tam giác ADP, CBP . DI1 , CI2 cắt AB lần lượt tại E, F . Đường thẳng
qua E song song với AC cắt BD tại M , đường thẳng qua F song song với BD cắt AC tại N . Chứng
minh rằng M N là tiếp tuyến chung của (I1 ) và (I2 ).

Chứng minh. Cách 1.


Ta phát biểu một bổ đề.
Bổ đề 46. Cho tứ giác ABCD. Gọi (I1 , r1 ) và (I2 , r2 ) lần lượt là đường tròn nội tiếp 4ABC và
BP cot ∠ABI1
4ADC. Tiếp tuyến chung trong ` khác AC của (I1 ) và (I2 ) cắt BD tại P. Khi đó = .
DP cot ∠ADI2
M

P
A I1
F

I2 S
K E
D Z N
C

Chứng minh. Gọi ` cắt CB, CD, CA lần lượt tại M, N, S. (I1 ) tiếp xúc với CA, CB lần lượt tại
E, Y , (I2 ) tiếp xúc với CA, CD lần lượt tại F, Z. Do tứ giác ADN S ngoại tiếp nên AN, DS, F Z đồng
quy tại K. Áp dụng định lý Menelaus cho 4DSC, với cát tuyến AN K, ta có:

362
CN AC KS AC SF
= · = · .
ND AS DK AS DZ
MB AS BY
Chứng minh tương tự, = ·
CM AC SE
Lại áp dụng định lý Menelaus cho 4BCD với cát tuyến `, ta thu được:
BP M B CN SF BY r2 BY cot ∠ABI1
= · = · = · = .
DP CM N D SE DZ r1 DZ cot ∠ADI2
Trở lại bài toán.
A S E P F V B

I2 U
R I1 Z
T M
L Y
G
N
X

I3

D K C

Gọi (I3 ) là đường tròn nội tiếp tam giác P CD.


Ta sẽ chứng minh 3 đường tròn (I1 ), (I2 ), (I3 ) có chung một tiếp tuyến.
Gọi XY là tiếp tuyến chung ngoài khác AB của (I1 ) và (I2 ), XY giao P D, P C lần lượt tại G, L.
(I1 ) tiếp xúc với AD, DP, P A lần lượt tại R, T, S; (I2 ) tiếp xúc với BC, CP, P B lần lượt tại U, Z, V .
Tứ giác GLCD ngoại tiếp khi và chỉ khi GL + CD = DG + CL.
⇔ XY − GX − LY + CD = DT − GT + CZ − LZ
⇔ SV + CD = DT + CZ = DR + CU = AD − AR + BC − BU.
⇔ AB − AS − BV + CD = AD − AR + BC − BU.
⇔ AB + CD = AD + BC, luôn đúng.
1
Kẻ I3 K ⊥ CD. Ta có ∠I1 DI3 = ∠ADC = ∠BDC nên ∠EDM = ∠I3 DK.
2
DM DM DK cot ∠I3 DC cot ∠I3 DC
Do đó 4EDM ∼ 4I3 DK. Suy ra = = = cot ∠I3 DC = ◦
= .
MB ME KI3 cot 45 cot ∠I2 BC
Áp dụng bổ đề trên suy ra M nằm trên tiếp tuyến chung trong của (I2 ) và (I3 ). Chứng minh tương
tự suy ra M N là tiếp tuyến chung của (I1 ) và (I2 ).
Cách 2 (Jean-Louis Ayme).
Ta phát biểu một bổ đề.
Bổ đề 47. Các đường tròn nội tiếp các tam giác AP D, BP C, DP C có chung một tiếp tuyến.
Chứng minh. Xem cách 1.
Trở lại bài toán.

363
A E P B

I2
Y

I1
M L

K X

T S

I3
Q

D C

Gọi K, L lần lượt là giao của tiếp tuyến chung ngoài thứ hai của (I1 ) và (I2 ) với P D, P C, I3 là tâm
đường tròn nội tiếp tam giác P DC. Gọi Q là giao của EK và BL.
Do 3 đường tròn (I1 ), (I2 ), (I3 ) có chung một tiếp tuyến nên tứ giác DKLC ngoại tiếp đường tròn
(I3 ). Suy ra I1 K giao I2 L tại I3 .
Ta có EB, I1 I2 , KL đồng quy tại tâm vị tự ngoài của hai đường tròn (I1 ) và (I2 ) nên hai tam giác
EI1 K và BI2 L thấu xạ. Theo định lý Desargues, giao điểm của các cặp đường thẳng EI1 và BI2 , EK
và BL, I1 K và I2 L thẳng hàng hay Q ∈ DI3 .
Gọi Y là hình chiếu của E trên DP, EY giao BC tại X. Ta có DE là phân giác ∠ADY nên
DY = DA = DC. Suy ra 4XY D = 4XCD. Từ đó X ∈ DI3 .
1
Gọi T là giao của DX với AC, ta có ∠EDX = ∠ADC = 45◦ = ∠T AE, suy ra tứ giác AET D
2
nội tiếp. Suy ra ∠ET D = 90◦ .
Do EM k AC nên EM ⊥ BD. Suy ra các điểm A, E, Y, M, T, D cùng thuộc đường tròn đường
kính DE.
Suy ra ∠M Y X = ∠M DE = ∠BDE = ∠XDC = ∠CY X. Ta thu được Y, M, C thẳng hàng.
Áp dụng định lý Pascal cho 6 điểm A, E, Y, M, T, D ta có AE ∩ DY = {P }, EM ∩ DT = {S},
Y M ∩ AT = {C} thẳng hàng.
Áp dụng định lý Pappus cho 2 bộ 3 điểm (E, P, B) và (D, Q, S) ta có EQ∩P D = {K}, ES ∩BD =
{M }, P S ∩ BQ = {L} thẳng hàng. Vậy M nằm trên tiếp tuyến chung ngoài của (I1 ) và (I2 ).
Chứng minh tương tự suy ra M N là tiếp tuyến chung ngoài của (I1 ) và (I2 ).

Bài 424. Cho tam giác ABC nội tiếp đường tròn (O). P là điểm bất kì khác O. Đường tròn (P, P A)
cắt AB, AC tại Ab , Ac , tương tự xác định Ba , Bc , Ca , Cb . Chứng minh rằng nếu Ab , Ac , Ba , Bc , Ca , Cb
cùng thuộc một đường tròn thì P nằm trên (O).

364
A

O
Ca
Cb X M' Y Bc
B
Z M
Ba C

Ac

Ab O'

Chứng minh. Gọi O0 là tâm của đường tròn đi qua 6 điểm Ab , Ac , Ba , Bc , Ca , Cb , X, Y, Z lần lượt là
hình chiếu của P trên BC, CA, AB; M, N lần lượt là hình chiếu của O và O0 trên BC.
Ta có B đối xứng với Bc qua X, C đối xứng với Cb qua X do đó M đối xứng với N qua X. Suy
ra P M là đường trung bình của hình thang OM O0 N , suy ra P O = P O0 .
Lại có P Ab = P Ac , O0 Ab = O0 Ac nên O0 P là trung trực của Ab Ac , suy ra Ab Ac ⊥ O0 P . Tương tự
suy ra Ab Ac k Ba Bc k Ca Cb . Mà Y, Z lần lượt là trung điểm của AAc , AAb nên Y Z k Ab Ac . Tương tự
XY k Ca Cb . Từ đó X, Y, Z thẳng hàng. Theo định lý về đường thẳng Simson suy ra P ∈ (O).

Bài 425. (Nguyễn Văn Linh- Trần Quang Hùng). Cho tam giác ABC có N là điểm Nagel. Gọi X, Y, Z
lần lượt là điểm chính giữa các cung BAC, ABC, ACB. Chứng minh rằng N là tâm đẳng phương của
các đường tròn (X, XA), (Y, Y B), (Z, ZC).

Chứng minh. Ta phát biểu và không chứng minh một bổ đề quen thuộc: Cho tam giác ABC có tâm
nội tiếp I. Gọi H, K, L lần lượt là hình chiếu của A trên BC, của B, C trên AI, M là trung điểm BC.
Khi đó H, K, L, M cùng thuộc một đường tròn có tâm là điểm chính giữa cung M H của đường tròn
Euler.
Trở lại bài toán.

365
Ac

X
C' A Ha B'

Ab

N Z
Ba
Cb
B
C Ca
Y
Bc

A'

Qua A, B, C lần lượt kẻ đường thẳng song song với BC, CA, AB cắt nhau tạo thành tam giác
A0 B 0 C 0 . Khi đó N là tâm nội tiếp của tam giác A0 B 0 C 0 . Gọi Ac , Ab lần lượt là hình chiếu của C 0 , B 0
trên A0 N , tương tự xác định Ba , Bc , Ca , Cb , Ha là hình chiếu của A0 trên BC. Khi đó Ac , Ab , A, Ha
cùng nằm trên (X, XA). Dễ thấy Ab , Ac , Ba , Bc , Ca , Cb nằm trên 3 cạnh của tam giác ABC.
Ta có ∠C 0 Cb Bc = ∠C 0 B 0 N = ∠B 0 Ba C nên tứ giác Bc Ba Cb Ca nội tiếp. Suy ra N nằm trên trục
đẳng phương của (Y, Y B) và (Z, ZC). Chứng minh tương tự ta có đpcm.

Bài 426. Cho tam giác ABC nội tiếp đường tròn (O). Giả sử rằng trung trực của AB cắt AC
tại A1 , trung trực của AC cắt AB tại A2 . Tương tự xác định B1 , B2 , C1 , C2 . Các đường thẳng
A1 A2 , B1 B2 , C1 C2 cắt nhau tạo thành tam giác DEF . Chứng minh rằng (DEF ) tiếp xúc với (O).

366
B2 B'

C1
C"

O F
A1

B
C
C2
B1
A2 E

A'

Chứng minh. Dễ thấy A1 , A2 ∈ (BOC), B1 , B2 ∈ (AOC), C1 , C2 ∈ (AOB).


Ta có ∠DC2 B1 = ∠BAC = ∠DB1 C2 nên ∠OC1 C2 = 90◦ −∠C1 C2 C = 90◦ −∠C2 B1 B2 = ∠C1 B2 D.
Suy ra DB2 = DC1 .
Lại có ∠B2 DC1 = 2∠DC2 B1 = 360◦ − 2∠B2 AC1 nên D là tâm ngoại tiếp tam giác B2 AC1 . Nghĩa
là ∠DAC1 = ∠DC1 A = ∠ABC hay DA là tiếp tuyến của (O). Gọi A0 B 0 C 0 là tam giác tạo bởi giao
điểm của các đường thẳng AD, BE, CF . Ta thu được (O) là đường tròn nội tiếp tam giác A0 B 0 C 0 .
Mặt khác, A1 A2 k AD, B1 B2 k BE, C1 C2 k CF nên DEF là tam giác trung tuyến của tam giác
A0 B 0 C 0 . Vậy (DEF ) tiếp xúc với (O) tại điểm Feuerbach của tam giác A0 B 0 C 0 .

Bài 427. (Iran TST 2007). Cho tam giác ABC. Một điểm O nằm trong tam giác thỏa mãn OA =
OB + OC. Gọi Y, Z lần lượt là điểm chính giữa các cung AOC và AOB của đường tròn ngoại tiếp các
tam giác AOC và AOB. Chứng minh rằng (BOY ) tiếp xúc với (COZ).

Chứng minh. Cách 1.

367
A

Y Z

C
B
E F

A'

Dựng hai điểm E, F trên tia OB, OC sao cho OA = OE = OF . Hai điểm P, Q thỏa mãn các cặp
tam giác AOC và AEP, AOB và AF Q đồng dạng cùng hướng. Gọi A0 đối xứng với A qua BC.
Ta có ∠AEP = ∠AOC = 2∠AEF = ∠AEA0 nên P ∈ EA0 . Tương tự Q ∈ F A0 .
Theo giả thiết OA = OB + OC nên BE = OC. Do đó từ cặp tam giác đồng dạng AOC và AEP
AO AE EO EA0
ta thu được = hay = . Suy ra P B k A0 O. Tương tự QC k A0 O.
OC EP EB EP
1
Mặt khác, ∠AZO = ∠ABE, ∠AOZ = 90◦ − ∠AOB = ∠AEB nên 4AZO ∼ 4ABE. Mà
2
4ACO ∼ 4AP E nên 4OZC ∼ 4EBP.
Chứng minh tương tự 4BY O ∼ 4QCF.
Vậy ∠BY O + ∠CZO = ∠EBP + ∠F CQ = ∠EOA0 + ∠F OA0 = ∠BOC.
Kẻ tiếp tuyến Ot của (BOY ) ta có ∠BOt = ∠BY O, do đó ∠COt = ∠CZO hay Ot là tiếp tuyến
của (CZO). Vậy hai đường tròn (BOY ) và (CZO) tiếp xúc nhau.
Cách 2.

368
A

Ob
M
Oc
Y
Z

Ic
Ib N O P
C

Oa

Gọi Oa , Ob , Oc lần lượt là tâm đường tròn ngoại tiếp các tam giác BOC, COA, AOB, Ib , Ic lần lượt
là tâm đường tròn ngoại tiếp các tam giác BOY, COZ.
Gọi M, N, P lần lượt là trung điểm OA, OB, OC.
Do Ib Ob ⊥ OY, Ob Oc ⊥ OA nên ∠Ib Ob Oc = ∠Y OM.
Tương tự ∠Ib Ob Oa = 180◦ − ∠Y OC.
Theo giả thiết Y là điểm chính giữa cung AOC nên OY là phân giác ngoài ∠AOC. Từ đó suy ra
∠Ib Ob Oc = ∠Ib Ob Oa hay Ib là chân phân giác kẻ từ Ob của tam giác Oa Ob Oc .
Tương tự Ic là chân phân giác kẻ từ Oc của tam giác Oa Ob Oc .
Lại có OA = OB + OC nên OM = ON + OP , tức là khoảng cách từ O đến Ob Oc bằng tổng khoảng
cách từ O đến Oa Ob và Oa Oc . Theo kết quả quen thuộc O nằm trên Ib Ic . Vậy (Ib ) tiếp xúc với (Ic )
tại O.

Nhận xét. Bạn đọc có thể sử dụng phép nghịch đảo tâm O và định lý Thales để chứng minh bài
toán.

Bài 428. (Le Viet An). Cho tam giác ABC nội tiếp đường tròn (O). D là điểm bất kì trên cạnh BC.
Trung trực của đoạn AD cắt OA tại T . T D cắt đường thẳng OB, OC lần lượt tại E, F . Gọi ω là đường
tròn tiếp xúc với T A, T D lần lượt tại A, D. Chứng minh rằng đường tròn ngoại tiếp tam giác OEF
tiếp xúc với ω.

369
A

L T
F

I O

E
K

B D C

Chứng minh. (Telv Cohl).


Qua O kẻ đường song song với BC cắt AD tại I, OE, OF lần lượt cắt AD tại K, L.
Áp dụng định lý Menelaus cho tam giác F T O với 3 điểm A, L, D thẳng hàng ta có:
AT LO DF
· · = 1.
AO LF DT
DF AO OC LI
Do T A = T D nên = = = .
LF LO LO ID
Suy ra F I là phân giác ∠LF D.
Lại có tam giác BOC cân nên OI là phân giác ∠EOF . Vậy I là tâm bàng tiếp góc O của tam giác
OEF.
Do ω tiếp xúc với OA và EF nên áp dụng định lý Sawayama-Thebault suy ra ω là đường tròn
Thebault của tam giác OEF ứng với cát tuyến OT . Từ đó ω tiếp xúc với (OEF ).

Bài 429. (Le Viet An). Cho tứ giác ABCD nội tiếp đường tròn (O). Gọi A0 , B 0 , C 0 , D0 lần lượt là
điểm đối xứng với A, B, C, D qua O. Gọi da , db , dc , dd lần lượt là đường thẳng Simson của A0 , B 0 , C 0 , D0
ứng với các tam giác BCD, ACD, ABD, ABC. Chứng minh rằng da , db , dc , dd cắt nhau tạo thành một
tứ giác ngoại tiếp.

370
A B

C'
P

D'
Hb
da O Ha
db
S

D Y

X
P'
C

B' A'

Chứng minh. Gọi X, Y lần lượt là hình chiếu của A0 , B 0 trên CD. S là giao của da và db . Gọi
Ha , Hb , Hc , Hd lần lượt là trực tâm các tam giác BCD, ACD, ABD, ABC. Chúng ta biết rằng AHa ,
BHb , CHc , DHd đồng quy tại P là trung điểm mỗi đoạn.
Gọi P 0 đối xứng với P qua O. Kí hiệu dP /l là khoảng cách từ điểm P đến đường thẳng l.
Theo phép đối xứng tâm O, d0P /A0 X = dP /AHa = dP /BHb = d0P /B 0 Y.
Do đó khoảng cách từ P 0 đến hai đường thẳng A0 X và B 0 Y bằng nhau, tức là P 0 X = P 0 Y.
1
Mặt khác, DC là đường thẳng Simson của B 0 ứng với tam giác BCD nên (DC, da ) = ∠A0 OB 0 .
2
1
Tương tự DC cũng là đường thẳng Simson của A ứng với tam giác ADC nên (DC, db ) = ∠A0 OB 0 .
0
2
Vậy tam giác SXY cân tại S. Mà P 0 nằm trên trung trực của XY nên SP 0 là phân giác ∠XSY.
Chứng minh tương tự suy ra P 0 cách đều các đường thẳng da , db , dc , dd .

Bài 430. (Jean-Louis Ayme) Cho tam giác ABC ngoại tiếp đường tròn (I). (I) tiếp xúc với CA, AB
lần lượt tại E, F . Kẻ AH ⊥ BC. Đường tròn (A, AE) giao đoạn thẳng AH tại M . M I cắt BC tại T .
Chứng minh rằng đường tròn đường kính AT tiếp xúc với (I).

371
N

J E
F
t M
I

B H D T C

Chứng minh. Gọi X là giao điểm thứ hai của M I với (A). (I) tiếp xúc với BC tại D. DM cắt (A) lần
thứ hai tại N.
Kẻ tiếp tuyến M t của (A) suy ra M t k BC.
Ta có ∠XN M = ∠XM t = ∠M T D suy ra tứ giác XN T D nội tiếp.
Suy ra ∠DN T = ∠DXT.
Lại có IM · IX = IE 2 = ID2 nên ∠DXI = ∠IDM = ∠DM H = ∠AM N = ∠AN M.
Do đó ∠AN M = ∠DXT = ∠DN T , suy ra N, A, T thẳng hàng.
Gọi R là giao điểm của (AM N ) với (I) (R nằm trên cung EDF ). AR cắt M N tại U .
Ta có AN = AM nên AU · AR = AM 2 = AF 2 nên U ∈ (I). Suy ra ∠RDH = ∠RU D = ∠RU M =
∠RN A, suy ra RDT N nội tiếp. Từ đó ∠RAH = ∠RN M = ∠RT D, suy ra tứ giác RAT H nội tiếp.
Do đó ∠HRT = ∠HAT = 2∠AN M = 2∠DRT , suy ra RD là phân giác ∠HRT . Suy ra (I) tiếp
xúc với (AT ) tại R.

Bài 431. (Nguyễn Văn Linh-mở rộng bài toán của Trần Quang Hùng) Cho tam giác ABC nội tiếp
(O). P là điểm bất kì nằm trên phân giác góc A. BP giao (AP C) lần thứ hai tại K, CP giao (AP B)
lần thứ hai tại L. J là điểm bất kì nằm trên AP sao cho đường tròn tâm J tiếp xúc với BC không
chứa trong (O). Hai tiếp tuyến chung ngoài của (O) và (J) tiếp xúc với (O) tại M và N . Chứng minh
rằng B, C, M, N đồng viên.

372
K

S
A
L
M N

P
O
T X RC Y
B

E V

Chứng minh. Gọi S là điểm chính giữa cung BAC. Ta có SB = SC và ∠BSC = ∠BAC = 2∠P AC =
2∠BKC = 2∠BLC nên S là tâm ngoại tiếp của tứ giác BLKC.
Mặt khác, ∠LAB = ∠LP B = ∠CP K = ∠CAK nên AB, AC đẳng giác trong ∠LAK hay AP là
phân giác của ∠LAK, suy ra AS là phân giác ngoài ∠LAK. Mà SL = SK nên L, A, S, K đồng viên.
Áp dụng định lý về tâm đẳng phương cho 3 đường tròn (LASK), (O), (S) suy ra LK, AS, BC đồng
quy tại T .
Kéo dài SM, SN giao BC tại X, Y . Gọi U, V lần lượt là tiếp điểm của tiếp tuyến chung ngoài với
(J), R là tiếp điểm của (J) với BC. Ta có OS k JR suy ra M S k RU, N S k RV.
Ta có ∠SM · SX = SB 2 = SN · SY nên tứ giác XM N Y nội tiếp, suy ra ∠M XY = ∠M N S =
∠XM U , suy ra XM RU là hình thang cân có J nằm trên trục đối xứng, suy ra JM = JX. Tương tự
JN = JY , mà JM = JN nên tứ giác XM N Y nội tiếp đường tròn tâm J. Gọi E là điểm chính giữa
cung BC suy ra A là giao của JE với (SM N ). Từ đó A là điểm Miquel của tứ giác toàn phần nội tiếp
XM N Y ST , suy ra AS, M N, XY đồng quy tại T.
Vậy T L · T K = T A · T S = T M · T N hay tứ giác LM N K nội tiếp.

Bài 432. (THTT tháng 1/2016) Cho tam giác ABC nội tiếp (O). Gọi D, E, F lần lượt là trung điểm
BC, CA, AB. P là điểm bất kì trên BC. AP cắt EF tại Q. QO cắt (OP D) lần thứ hai tại L. Dựng
−→ −−→
điểm T sao cho LT = P D. Chứng minh rằng T nằm trên đường tròn Euler của tam giác ABC.

373
A

F R E
Q
O
H N
L
T

B K P D C

Chứng minh. Lấy các điểm R, N sao cho phép tịnh tiến T− → : R 7→ Q, O 7→ N. Gọi H là trực tâm tam
TL
giác ABC, M là trung điểm AH.
−−→ −−→ −−→ −−→ −−→
Ta có AH = 2OD = 2N P suy ra AM = N P . Suy ra QM = N Q = OR. Ta thu được M R k QO.
Mà QL k RT nên M, R, T thẳng hàng.
Do Q, L, P lần lượt là ảnh của R, T, D qua phép tịnh tiến T−→ nên ∠M T D = ∠RT D = ∠QLP =
TL

90 hay T thuộc đường tròn Euler của tam giác ABC.

Bài 433. (Nguyễn Văn Linh) Cho tam giác ABC nội tiếp đường tròn (O). Các đường cao AA1 , BB1 , CC1
đồng quy tại H. P là điểm bất kì trên OH. AP, BP, CP cắt (O) lần thứ hai lần lượt tại A2 , B2 , C2 . Gọi
A3 , B3 , C3 là các điểm đối xứng với A2 , B2 , C2 qua A1 , B1 , C1 . Chứng minh rằng H, A3 , B3 , C3 cùng
thuộc một đường tròn có tâm nằm trên OH.

T
B1
C2

Z
B2
B3
C1 V

S PL O
O'
X Y Q

A3
C3 C
B W

U
A2

A1

Chứng minh. Ta chứng minh bằng cách mở rộng bài toán như sau.
Mở rộng. Cho tam giác ABC nội tiếp đường tròn (O). Gọi X, Y là hai điểm bất kì sao cho X, O, Y
thẳng hàng. Gọi A1 B1 C1 , A2 B2 C2 lần lượt là các tam giác circumcevian của X và Y ứng với tam giác
ABC. Gọi A3 , B3 , C3 lần lượt là điểm đối xứng của A2 , B2 , C2 qua trung điểm A1 X, B1 X, C1 X. Khi
đó A3 , B3 , C3 , X cùng nằm trên một đường tròn có tâm O0 nằm trên XY.
Chứng minh.

374
Gọi S là giao điểm của A1 C2 và A2 C1 . Áp dụng định lý Pascal cho 6 điểm C1 , C2 , A1 , A2 , A, C ta
thu được S ∈ XY .
Gọi T, U lần lượt là giao điểm thứ hai của đường thẳng qua A1 , C1 lần lượt vuông góc với A1 A2 , C1 C2
với (O). A1 T cắt C1 U tại P . Áp dụng định lý Pascal lần thứ hai cho 6 điểm A1 , A2 , C1 , C2 , T, U suy
ra P ∈ XY . Tương tự ta thu được đường thẳng qua B1 vuông góc với B1 B2 cũng đi qua P , các đường
thẳng lần lượt qua A2 , B2 , C2 và vuông góc với A1 A2 , B1 B2 , C1 C2 đồng quy tại Q ∈ XY .
−−→ −−−→ −−→ −−−→ −→ −−−→ −−→ −−−→ −−→
Dựng các điểm V, W, Z sao cho P V = A1 A2 , P W = B1 B2 , P Z = C1 C2 . Suy ra P V = A1 A2 = A3 X.
Gọi L là trung điểm XP thì V đối xứng với A3 qua L.
Tương tự suy ra (A3 B3 C3 ) là đối xứng của (V W Z) qua L. Mà P, V, W, Z nằm trên đường tròn
tâm O đường kính P Q và X đối xứng với P qua L nên X nằm trên (A3 B3 C3 ). Hơn nữa, O0 đối xứng
với O qua L, suy ra O0 ∈ XY .

Bài 434. (Nguyễn Văn Linh) Cho tam giác ABC nội tiếp đường tròn (O), ngoại tiếp đường tròn (I).
(I) tiếp xúc với BC tại D. Gọi L là điểm đối xứng với D qua I; X, Y, Z lần lượt là tiếp điểm của
các đường tròn bàng tiếp góc A, B, C với cạnh BC, CA, AB. Giả sử ∠AIO = 90◦ . Chứng minh rằng
X, Y, Z, L cùng thuộc một đường tròn.

L
Z
K
T Y
F O
G
I N

Q X C
B D P

Chứng minh. Gọi M là điểm chính giữa cung BC, G là trọng tâm tam giác ABC, Q là trung điểm
BC, P là giao của AI với BC, F là tiếp điểm của (I) với AB.
Do ∠AIO = 90◦ nên AI = IM = M B = M C. Lại có ∠IAF = ∠M BQ nên 4IF A =
1 1
4M QB(g.c.g). Ta thu được M Q = IF = ID. Suy ra P I = P M = IM = IA.
2 2
AI AG
Suy ra = hay IG k BC. Gọi N là giao của IG với OM ta thu được QN = ID = QM . Gọi T
IP GQ
1 1
là giao của AQ với ID suy ra IT là đường trung bình của tam giác AM Q, suy ra IT = M Q = N Q.
2 2
IG 1
Do đó = . Ta thu được N là điểm Nagel của tam giác ABC, tức là AX, BY, CZ đồng quy tại
IN 2
N.
Gọi K là giao của Y Z và AX suy ra (AN KX) = −1. Mà IL là đường trung bình của tam giác
AM N nên LA = LN . Áp dụng hệ thức Maclaurin suy ra KL · KX = KN · KA.
Mặt khác, do N đối xứng với M qua BC nên ∠Y N Z = ∠BN C = ∠BM C = 180◦ − ∠BAC, suy
ra tứ giác AZN Y nội tiếp. Do đó KN · KA = KY · KZ.
Vậy KY · KZ = KL · KX hay tứ giác LZXY nội tiếp.

375
Bài 435. (Nguyễn Văn Linh) Cho tam giác ABC nội tiếp đường tròn (O), ngoại tiếp đường tròn (I).
Các đường tròn (Ia ), (Ib ), (Ic ) bàng tiếp góc A, B, C lần lượt tiếp xúc với BC, CA, AB tại X, Y, Z. Giả
sử OI k BC. Chứng minh rằng tâm đường tròn ngoại tiếp của tam giác XY Z nằm trên phân giác
∠BAC.

J
Ic N E
K
Z
I O
F Y

U C V
M
B D X

T
Q
L

Ia

Chứng minh. Gọi D, E, F lần lượt là tiếp điểm của (I) với BC, CA, AB; K đối xứng với D qua I, M
là trung điểm BC.
Do AK đi qua X và M là trung điểm DX nên IM k AK. Mà IO k BC nên OM = ID, suy ra
OM k= IK hay IKOM là hình bình hành. Suy ra OK k IM . Ta thu được A, K, O, X thẳng hàng.
Qua A kẻ đường vuông góc với BC cắt IM tại N . Gọi U, V lần lượt là tiếp điểm của (Ic ), (Ib ) với
BC. L là điểm chính giữa cung BC không chứa A.
NI AI AI
Do AN k M L nên = = .
MI LI LB
AI AF BZ BU NI BU
Lại có 4AF I ∼ 4BM L nên = = = . Suy ra = hay BI k U N . Mà
LB BM BM BM MI BM
BI k U Z nên U Z đi qua N . Tương tự V Y đi qua N .
1
Gọi J, Q lần lượt đối xứng với Z, Y qua phân giác AI. Ta có ∠ZN Y = ∠BIC = 90◦ + ∠BAC,
2
◦ 1 ◦
∠ZQY = ∠AQY = 90 − ∠BAC nên ∠ZN Y + ∠ZQY = 180 , suy ra tứ giác ZN Y Q nội tiếp. Suy
2
ra 5 điểm Z, J, Y, Q, N cùng thuộc một đường tròn. Lại có AKIN là hình bình hành và AN, AO đẳng
giác trong ∠BAC nên AI là phân giác ∠N AK, suy ra AKIN là hình thoi. Nghĩa là N đối xứng với
K qua AI. Suy ra 6 điểm K, J, Y, Q, N, X thuộc một đường tròn.
Kéo dài AX cắt (O) tại T . Ta có O là trung điểm KX nên AK = XT .
Suy ra AK · AX = AX · XT = XB · XC = DB · DC = BF · CE = AZ · AY = AJ · AY. Suy ra tứ
giác KJY X nội tiếp. Vậy X ∈ (ZJY Q). Mà ZJY Q là hình thang cân có trục đối xứng AI nên tâm
của (XY Z) nằm trên AI.

Bài 436. (Nguyễn Văn Linh) Cho tứ giác ABCD nội tiếp đường tròn (O). AC giao BD tại P . Đường
tròn ngoại tiếp các tam giác AP D và BP C lần lượt giao CD tại E và F sao cho E, F thuộc đoạn
thẳng CD. AE, BF giao (O) lần lượt tại K, L. Gọi I1 , I2 , J1 , J2 lần lượt là tâm đường tròn nội tiếp
các tam giác ADE, CEK, BCF, DF L. Chứng minh rằng I1 , I2 , J1 , J2 đồng viên.

376
Chứng minh. Cách 1.

B
M
A

Y
P
X

I
J1
I1
E F
D
I2 C
K J2

Gọi T là giao của AD và BC. I là tâm đường tròn nội tiếp tam giác T CD, (X) và (Y ) lần lượt
là đường tròn ngoại tiếp các tam giác AP D và BP C. Ta có X và Y lần lượt là điểm chính giữa cung
AD và BC của (AP D) và (BP C) nên XY là phân giác ∠AP D.
Dễ thấy T I song song với phân giác ∠DP C nên T I ⊥ XY . Mà T A · T D = T B · T C nên T nằm
trên trục đẳng phương của (X) và (Y ). Suy ra T I là trục đẳng phương của (X) và (Y ).
Do đó ID · II1 = IC · IJ1 hay tứ giác DI1 J1 C nội tiếp.
1 1 1 1
Suy ra ∠EI1 J1 = ∠DI1 J1 − ∠DI1 E = 180◦ − ∠BCD − (90◦ + ∠DAE) = ∠DAB − ∠DAE =
2 2 2 2
∠EAP = ∠EXP . Do đó I1 J1 k XY.
Gọi M, N lần lượt là điểm đối xứng với I1 qua X, J1 qua Y suy ra M N k XY k I1 J1 .
Suy ra ∠M N C = ∠M N J1 + ∠J1 N C = ∠I1 J1 F + ∠J1 BC = ∠I1 J1 F + ∠F J1 C − 90◦ = ∠I1 J1 C −
90◦ = 270◦ − ∠I1 DC = 180◦ − (90◦ + ∠I1 DC) = 180◦ − ∠M DC. Do đó tứ giác M N CD nội tiếp.
1 1
Ta có ∠M I2 C = 90◦ + ∠EKC = 90◦ + ∠ADC = ∠M DC nên I2 ∈ (M N CD), tương tự với J2 .
2 2
Suy ra tứ giác M N J2 I2 nội tiếp. Mà I1 J1 k M N nên theo định lý Reim, tứ giác I1 J1 J2 I2 nội tiếp.

Cách 2 (Cho phần chứng minh I1 , J1 , C, D đồng viên).

377
B J5
I5
A

T S
P Y
X
I3 J3

I4 J4 J1
I1

E F
D
C
I2 J2

K
M L

Gọi I3 , I4 , J3 , J4 lần lượt là tâm đường tròn nội tiếp các tam giác AP D, ADC, BP C, BDC.
AI4 giao BJ4 tại M là trung điểm cung CD của (O). Dễ thấy tứ giác DCJ4 I4 nội tiếp đường tròn
(M, M B) nên M I4 = M J4 .
1 1
Lại có I3 J3 đi qua P và ∠P I3 I4 = 90◦ − ∠ADP = 90◦ − ∠ACB = ∠P J3 J4 nên tam giác M I3 J3
2 2
cân tại M . Ta thu được I3 J3 J4 I4 là hình thang cân.
Mặt khác, dễ thấy tứ giác AI3 I1 D và BJ3 J1 C lần lượt nội tiếp đường tròn có tâm X và Y là trung
điểm cung AD và BC của (AP D) và (BP C). Gọi T là giao của I1 I3 và J1 J4 , S là giao của I1 I4 và
J1 J3 .
Ta có ∠I1 I3 I4 = ∠ADI1 = ∠CDI1 = ∠I4 J4 T nên tứ giác T I3 I4 J4 nội tiếp.
Tương tự tứ giác SJ3 J4 I4 nội tiếp. Vậy S, T ∈ (I4 J4 J3 I3 ).
Suy ra ∠I1 T J1 = ∠I3 T J4 = ∠J3 SI4 = ∠J1 SI1 hay tứ giác T SJ1 I1 nội tiếp.
Do các tứ giác T SJ4 I4 và DCJ4 I4 nội tiếp nên áp dụng định lý Reim, T S k DC. Lại có T SJ1 I1
nội tiếp nên áp dụng định lý Reim ta có DCJ1 I1 nội tiếp và I1 J1 k I4 J4 k I3 J3 .

Bài 437. (Nguyễn Văn Linh) Cho tam giác ABC nội tiếp (O), trực tâm H. P là điểm bất kì trên
cung BC. P 0 đối xứng với P qua BC. Đường tròn ngoại tiếp tam giác OP P 0 cắt AP tại G. Chứng
minh rằng trực tâm tam giác AGO nằm trên HP 0 .

378
A

L G
O
P'
H

B C

M A'
P
Oa

Chứng minh. Gọi J là giao của OP 0 với AH. Ta có ∠JAG = ∠GP P 0 = ∠GOJ nên tứ giác AJGO
nội tiếp. (AGH) giao HP 0 lần thứ hai tại K suy ra G là điểm Miquel của tam giác JHP ứng với bộ
3 điểm (A, O, K). Ta thu được tứ giác KGOP 0 nội tiếp.
Gọi Oa là tâm ngoại tiếp của tam giác BHC, GH giao (Oa ) lần thứ hai tại M . Gọi A0 là giao
điểm của AH với (O). Do hai đường tròn (O) và (Oa ) đối xứng nhau qua BC nên A0 P = HP 0 . Ta có
∠GM P 0 = ∠HM P 0 = ∠HAP = ∠HKG nên M ∈ (OP P 0 ).
Gọi L là giao của Oa H với (OP P 0 ). Do Oa H = Oa M = Oa P 0 , ta thu được H là tâm nội tiếp của
tam giác LM P 0 . Suy ra GL = GH = GP 0 .
Lại có ∠HLP 0 = ∠Oa OP 0 = ∠HJP 0 nên tứ giác HLJP 0 nội tiếp đường tròn tâm G. Do hai
đường tròn (AGO) và (AGH) cắt nhau tại A và G, đồng thời ∠AOG = ∠GJH = ∠GHJ nên
R(AGO) = R(AGH). Mặt khác, ∠GAO = ∠GP O nên R(AGO) = R(P GO).
Vậy 3 đường tròn (AGO), (AGK), (KGO) có bán kính bằng nhau và đồng quy tại G nên K là trực
tâm tam giác AGO.

Bài 438. (Nguyễn Văn Linh) Cho tam giác ABC nội tiếp (O). Hai tiếp tuyến tại B, C giao nhau tại
P . Phân giác góc A cắt (P, P B) tại điểm E nằm trong tam giác ABC. Gọi M, N là điểm chính giữa
cung BC và cung BAC. Đường tròn đường kính BC cắt đoạn thẳng EN tại F . Chứng minh rằng trực
tâm tam giác EF M nằm trên BC.

379
N

K E
C
B I

J
M

Chứng minh. Gọi I là trung điểm BC. Ta có ∠ICM = ∠M AC = ∠M CP nên CM là phân giác
∠ICP , suy ra M là tâm vị tự trong của (I) và (P ). Do ∠M CN = 90◦ nên N là tâm vị tự ngoài của
(I) và (P ).
−P C
Gọi L, T là giao của F M với (P ) (L nằm giữa F và M ). Xét phép vị tự HMIC : F 7→ T , suy ra
IF k P T.
PC
Lại xét phép vị tự HNIC : F 7→ E, suy ra IF k P E.
Vậy E, P, T thẳng hàng hay ET là đường kính của (P ), suy ra ∠ELP = 90◦ .
Gọi J là giao điểm thứ hai của F N với (O). Xét 3 đường tròn (O), (P ), (ELM J) có trục đẳng
phương lần lượt là EL, BC, JM nên EL cắt JM tại K nằm trên BC. Vậy trực tâm K của tam giác
EF M nằm trên BC.

Bài 439. (Nguyễn Văn Linh) Cho tam giác ABC nội tiếp (O). Đường cao AH. M là trung điểm BC.
AM giao OH tại G. Chứng minh rằng G nằm trên trục đẳng phương của (BOC) và đường tròn Euler
của tam giác ABC.

380
A

K
E
L N

O
F
G

C
T B H M

Chứng minh. Gọi P, Q lần lượt là giao của OM, OH với (BOC); E, F là chân đường cao kẻ từ B, C.
Gọi (Eu ) là đường tròn Euler của tam giác ABC, d là trục đẳng phương của (Eu ) và (BOC).
Xét 3 đường tròn (BOC), (Eu ), (BC) có EF, BC, d là các trục đẳng phương nên EF, BC, d đồng
quy tại T.
Ta có OP là đường kính của (BOC) nên ∠HQP = 90◦ . Xét 3 đường tròn (HM P Q), (BOC), (Eu )
có P Q, HM, d là các trục đẳng phương nên P Q đi qua T .
Gọi K là giao điểm thứ hai của AM với (Eu ), N là trung điểm BC. Ta có AO vuông góc với EF
tại L, suy ra tứ giác LON E nội tiếp.
Suy ra AL · AO = AE · AN = AK · AM hay tứ giác KM OL nội tiếp. Mà các điểm L, M, Q cùng
nằm trên đường tròn đường kính OT nên tứ giác KOM Q nội tiếp. Vậy GK · GM = GO · GQ hay G
thuộc trục đẳng phương của (BOC) và (Eu ).

Bài 440. (Nguyễn Văn Linh) Cho tam giác ABC nội tiếp đường tròn (O), trực tâm H. Gọi M là
điểm chính giữa cung BHC của đường tròn ngoại tiếp tam giác BHC. BM giao AC tại E, CM giao
AB tại F . Kẻ phân giác AD của tam giác ABC. Chứng minh rằng bán kính đường tròn ngoại tiếp
tam giác AEF bằng OD.

381
J
A

F T

E
M
H

C
B D

1
Chứng minh. Ta có ∠F AD = ∠BAC = ∠M CB nên tứ giác F ACD nội tiếp.
2
Suy ra ∠DF C = ∠DAC = ∠M BC, ta thu được tứ giác F M DB nội tiếp, tương tự M ECD nội
tiếp.
Dễ thấy M ∈ (AEF ), gọi T là tâm của (AEF ). Ta có
T B 2 − T C 2 = PB/(T ) − PC/(T ) = BM · BE − CM · CF = BD · BC − CD · CB = BC(BD − CD) =
(BD + CD)(BD − CD) = BD2 − CD2 . Suy ra T D ⊥ BC.
Lại có ∠ADB = ∠AEB = ∠M DC nên DA và DM đối xứng nhau qua DT.
Do BM = CM và ∠BF M + ∠CEM = 180◦ nên R(BM F ) = R(CM E) . Suy ra BF = CE.
Ta có (AEF ) giao (O) tại J là tâm của phép vị tự quay biến BF thành CE nên JB = JC. Do
OT ⊥ AJ và AJ ⊥ AD nên OT k AD.
Ta thu được ∠T OM = ∠ADT = ∠T DM , suy ra tứ giác T M OD nội tiếp. Mà OM k T D nên
T M OD là hình thang cân. Vậy T M = OD hay R(AEF ) = OD.

Bài 441. (Nguyễn Văn Linh) Cho tam giác ABC nội tiếp đường tròn (O). Phân giác góc A cắt BC
tại D và cắt (O) tại E. Gọi A0 là điểm đối xứng với A qua O, K, L lần lượt là tâm đường tròn ngoại
tiếp các tam giác ABD, ACD. Chứng minh rằng K, L, E, A0 đồng viên.

M
A

J
L
K
O

D
C
B

A'
E

382
Chứng minh. Ta có EB 2 = ED · EA = PE/(K) nên ∠KBE = 90◦ . Tương tự, ∠LCE = 90◦ . Gọi M là
giao của BK và CL suy ra M là điểm chính giữa cung BAC.
Ta có ∠AOM = 2∠ABM = ∠AKM nên tứ giác AM OK nội tiếp. Chứng minh tương tự suy ra
A, M, K, L, O cùng thuộc một đường tròn.
Lại có M O là phân giác ∠KM L nên OK = OL. Gọi J là tâm của (AKL) thì OJ là trung trực
của KL và AM . Mà A0 , E lần lượt đối xứng với A, M qua O nên phép đối xứng trục OJ:
SOJ : K 7→ L, A 7→ M, A0 7→ E, suy ra KLA0 E là hình thang cân. Vậy KLA0 E là tứ giác nội
tiếp.

Bài 442. (Nguyễn Văn Linh) Cho tam giác ABC nội tiếp đường tròn (O). Đường tròn (Ia ) bàng tiếp
góc A tiếp xúc với BC tại D. AD cắt (O) lần thứ hai tại P . Gọi J là điểm chính giữa cung BAC
của (O). JP cắt Ia D tại Q. Gọi I1 , I2 lần lượt là tâm đường tròn nội tiếp các tam giác BP D, CP D.
Chứng minh rằng (QI1 I2 ) tiếp xúc với (J, JB).

J
A

Q
O

D
C
B
I1 I2
I

P
M

Ia

Chứng minh. Gọi I là tâm đường tròn nội tiếp tam giác BP C, M là điểm chính giữa cung BC.
Ta có Ia Q k JM nên ∠Ia QP = ∠M JP = ∠Ia AP , suy ra tứ giác AQP Ia nội tiếp. Từ đó DQ·DIa =
DA · DP = DB · DC hay BQCIa nội tiếp.
1 1
Suy ra ∠BQD = ∠BCIa = 90◦ − ∠BAC = 90◦ − ∠BP A = 180◦ − ∠BI1 D. Do đó tứ giác
2 2
BQDI1 nội tiếp hay ∠QI1 B = ∠QDB = 90◦ . Tương tự, ∠QI2 C = 90◦ . Ta thu được tứ giác QI1 II2
nội tiếp.
Mặt khác, gọi r1 , r2 lần lượt là bán kính của (I1 ), (I2 ), kí hiệu s là nửa chu vi tam giác.
SBP D
r1 s BD sDP C
Ta có = SBP D = · . (1)
r2 s
CP D CD sBP C
CP D
Do các cặp tam giác BDP và ADC, P DC và BDA đồng dạng nên
BD BD DA sBDP sADB sBDP sADB
= · = · = · . Lại có D là tiếp điểm của đường tròn bàng
CD DA CD sADC sCDP sCDP sADC
BD sBDP
tiếp góc A với BC nên sADB = sADC . Suy ra = . (2)
CD sCDP

383
r1
Từ (1) và (2) suy ra = 1 hay r1 = r2 . Suy ra I1 I2 k BC. Do đó (II1 I2 ) tiếp xúc với (IBC). Mà
r2
J là tâm đường tròn ngoại tiếp của tam giác BIC nên (QI1 I2 ) tiếp xúc với (J, JB).

Bài 443. (Nguyễn Văn Linh) Cho tam giác ABC nội tiếp đường tròn (O), trực tâm H. Trung tuyến
AM cắt (O) lần thứ hai tại N . AH cắt (O) tại K. Các đường thẳng KN, BC và đường thẳng qua H
vuông góc với AN cắt nhau tạo thành tam giác XY Z. Chứng minh rằng (XY Z) tiếp xúc với (O).

J
T
O

H P

Z C
Y B M
X
x
K Q
N

Chứng minh. Giả sử vị trí các điểm X, Y, Z như hình vẽ. Gọi T là giao điểm thứ hai của (AH) và (O),
P là hình chiếu của H trên AM , Q là điểm đối xứng với A qua O.
Theo kết quả quen thuộc, T, H, M, Q thẳng hàng.
Ta có ∠HXK = 90◦ − ∠P N X = 90◦ − ∠AN K = 90◦ − ∠AQK = ∠KAQ = ∠KT H. Suy ra tứ
giác XKHT nội tiếp.
Do HP ⊥ AM , Y M ⊥ AH nên Y là trực tâm tam giác AHM , suy ra Y, T, A thẳng hàng.
Từ đó ∠T XK = ∠T HA = ∠T Y B hay tứ giác ZXY T nội tiếp.
Kẻ tiếp tuyến T x của (O). Ta có ∠XT x = ∠XT K − ∠xT K = ∠XHK − ∠T AK = ∠T Y H =
∠T ZX. Suy ra T x đồng thời là tiếp tuyến của (XY Z). Vậy hai đường tròn (XY Z) và (O) tiếp xúc
nhau tại T.

Bài 444. (Nguyễn Văn Linh) Cho tam giác ABC. Một đường tròn (Oa ) đi qua B, C cắt AC, AB lần
lượt tại E, F. BE giao CF tại P . Gọi M là trung điểm BC. Kẻ P K vuông góc với AOa . Q đối xứng
với P qua BC, L đối xứng với K qua M . Các đường thẳng P K, QL, BC cắt nhau tạo thành tam giác
XY Z. Chứng minh rằng (XY Z) tiếp xúc với (ABC).

384
V

Hc Hb
H'
J E
T

H O
F K
P

Z M
S C
Y B
X Oa U
Q

L
R W

Chứng minh. Giả sử vị trí của ba điểm X, Y, Z như hình vẽ.


Gọi T là giao điểm thứ hai của đường tròn đường kính AP với đường tròn (O) ngoại tiếp tam giác
ABC. Do P K ⊥ AOa nên theo định lý Brocard, Z là giao của EF và AT.
Gọi H, H 0 lần lượt là trực tâm các tam giác ABC, AEF . Hb , Hc lần lượt là hình chiếu của F, E
trên AC, AB.
Ta có H 0 F · H 0 Hb = H 0 E · H 0 Hc suy ra H 0 thuộc trục đẳng phương của (BE) và (CF ). Chứng
minh tương tự suy ra HH 0 là trục đẳng phương của (BE) và (CF ). Mà P E · P B = P F · P C nên
P, H, H 0 thẳng hàng.
Do Oa T ⊥ AZ nên T là điểm Miquel của tứ giác toàn phần BF EC.AZ. Suy ra T có chung đường
thẳng Simson với hai tam giác AEF, ABC. Do đường thẳng Steiner là ảnh của đường thẳng Simson
qua phép vị tự tâm T tỉ số 2 nên HH 0 là đường thẳng Steiner của T ứng với hai tam giác AEF, ABC
hay P H là đường thẳng Steiner của P ứng với tam giác ABC.
Kéo dài AH cắt (O) tại R suy ra T R đi qua Q.
Ta có ZP · ZK = PZ/(Oa ) = ZB · ZC. Suy ra tứ giác BP KC nội tiếp.
Gọi U là đối xứng của P qua M . Phép đối xứng tâm M biến đường tròn (BP KC) thành (BLU C).
Lại có Q đối xứng với P qua BC nên QU k BC và QB = QP = U C hay tứ giác BQU C là hình thang
cân. Vậy 5 điểm B, Q, L, U, C đồng viên.
Ta có P K k U L nên ∠P XL = 180◦ − ∠QLU . (1)
Gọi V là giao điểm thứ hai của P Q với (BQC). W là điểm đối xứng với A qua O. Do hai đường
tròn (BP C) và (BQC) đối xứng với nhau qua BC nên P là trực tâm tam giác BV C. Đồng thời V U
đi qua tâm ngoại tiếp tam giác V BC nên ∠QV U = |∠V BC − ∠V CB|.
Lại có ∠ABP = ∠ACP nên ∠V BA = ∠V CA. Suy ra |∠V BC − ∠V CB| = |∠ABC − ∠ACB| =
∠RAW = ∠RT W = ∠QT P.
Vậy ∠QV U = ∠QT P . (2)
Từ (1) và (2) suy ra ∠QT P = ∠QXP hay tứ giác XT P Q nội tiếp đường tròn (S).

385
Ta có ∠ST P = 90◦ − ∠T QP = 90◦ − ∠T RA = 90◦ − ∠T W A = ∠T AW . Suy ra ST là tiếp tuyến
của (O).
Mặt khác, ∠ST X = 90◦ − ∠T P X = ∠T ZP = ∠T Y X. Suy ra ST là tiếp tuyến của (XY Z).
Vậy hai đường tròn (O) và (XY Z) tiếp xúc nhau tại T.

Bài 445. (ARMO 2016) Cho tam giác nhọn ABC, AC < BC, M là trung điểm AB và Ω là đường
tròn ngoại tiếp tam giác ABC. Gọi C 0 là điểm đối xứng của C qua tâm của Ω. AC 0 , BC 0 giao CM
lần lượt tại K, L. Đường thẳng qua K vuông góc với AC 0 giao đường thẳng qua L vuông góc với BC 0
giao AB và cắt nhau tạo thành tam giác ∆. Chứng minh rằng đường tròn ngoại tiếp tam giác ∆ và Ω
tiếp xúc nhau.

Chứng minh. (Cách 1)

O S

Z M Y
A B
T
J K

X C'
J1
Q
L J2

Gọi Q là giao điểm của đường đối trung ứng với đỉnh C với (O). AC 0 giao AQ tại T ,BC 0 giao CP
tại R. Gọi S là hình chiếu vuông góc của T trên CB.
Do T nằm trên đường đối trung ứng với đỉnh C của tam giác ABC và 4CAT ∼ 4CBL, nên
TA CA TA
T S = CB = LB . Suy ra T S = LB hay T L ⊥ RB hay Z, T, L thẳng hàng.
Chứng minh tương tự, R, Y, X thẳng hàng. Ta thu được X là trực tâm của tam giác T RC 0 . Nhưng
∠CQC 0 = 90◦ nên Q, X, C 0 thẳng hàng.
Mặt khác, ZT k CB nên áp dụng định lý Reim, tứ giác AZT Q nội tiếp. Chứng minh tương tự,
BY QR là tứ giác nội tiếp.
Suy ra ∠ZQY = ∠ZQT + ∠T QY = ∠C 0 AB + ∠C 0 BA = 180◦ − ∠AC 0 B = ∠ZXY . Ta thu được
Q ∈ (XY Z).
Gọi Qt là tiếp tuyến của (O). Ta có ∠tQC 0 = ∠QAC 0 = ∠QZX nên Qt cũng là tiếp tuyến của
(J). Suy ra đpcm.
(Cách 2).
Ta sử dụng các tính chất sau của hai tam giác paralogic (xem chương tam giác paralogic).

386
Cho hai tam giác ABC và A2 B2 C2 là hai tam giác paralogic ứng với bộ 3 thẳng hàng (A1 , B1 , C1 ).
Khi đó (ABC) và (A2 B2 C2 ) trực giao, hai đường tròn này giao nhau tại hai điểm, một điểm là điểm
Miquel của tứ giác toàn phần ABCA1 B1 C1 , một điểm là giao của AA2 , BB2 , CC2 .
Trở lại bài toán.

Z M Y
A B
J
K
P
X C'
S
Q L

Gọi J, T lần lượt là giao điểm của OM với AC 0 , BC 0 .


Do hai tam giác XY Z và T JC 0 là hai tam giác paralogic ứng với bộ (M, K, L) ta thu được (XY Z)
trực giao với (JT C 0 ). Gọi S là giao điểm thứ hai của (T JC 0 ) và (O). Do T JC 0 và ABC là hai
tam giác paralogic ứng với (A, M, B) nên S là điểm Miquel của tứ giác toàn phần ABCAM B hay
là giao điểm của đường tròn qua M, A và tiếp xúc với CA, qua M, B và tiếp xúc với CB. Suy ra
∠M SA = ∠CAM = ∠CSB hay CABS là tứ giác điều hòa.
Gọi P, Q lần lượt là giao của M O với ZL, Y K suy ra XP Q và AC 0 B là hai tam giác paralogic ứng
với bộ (M, K, L). Gọi S 0 là giao điểm của (XP Q) và (AC 0 B) sao cho C 0 X, BP, AQ đồng quy tại S 0 .
Suy ra ∠S 0 CA = ∠S 0 C 0 A = ∠XLK = ∠LCB hay CS 0 là đường đối trung của tam giác ABC. Nghĩa
là S 0 ≡ S hay S, X, C 0 thẳng hàng. Ta biết rằng XC 0 , ZT, Y J đồng quy tại giao điểm của (XY Z) và
(JT C 0 ). Do đó (XY Z), (JT C 0 ), (O) đồng quy tại S. Mà (XY Z) và (O) đều trực giao với (JT C 0 ) nên
(XY Z) tiếp xúc với (O) tại S.

Bài 446. (Nguyễn Văn Linh) Cho tam giác ABC nội tiếp đường tròn (O). Gọi A0 là điểm đối xứng
với A qua O. Trung tuyến AM của tam giác ABC cắt BA0 , CA0 lần lượt tại L, K. Các đường thẳng
qua L vuông góc với BA0 , qua K vuông góc với CA0 và đường thẳng OM cắt nhau tạo thành tam
giác XY Z. Gọi P là giao của hai tiếp tuyến tại B và C của (O). Chứng minh rằng (AM P ) tiếp xúc
với (XY Z).

387
W
A

B M C
L
E
Y A'
Z U
J S
V T
K
X

Chứng minh. (Cách 1) Giả sử vị trí của các điểm X, Y, Z như hình vẽ.
Gọi U, V lần lượt là giao điểm thứ hai của AM, AP với (O). BA0 giao OM tại E.
Ta có ∠M EC = ∠M EB = ∠ABC = ∠M U C nên tứ giác M EU C nội tiếp.
Suy ra ∠EU M = ∠ECM = ∠EBM = ∠EXL hay tứ giác ELU X nội tiếp. Từ đó ∠EU X =
∠ELX = 90◦ = ∠EU C. Suy ra X, U, C thẳng hàng.
Do V, U đối xứng nhau qua OM nên B, V, X thẳng hàng. Chứng minh tương tự ta cũng có B, Y, U
và C, Y, V lần lượt thẳng hàng.
Ta có ∠Y V X = ∠BA0 K = ∠Y ZL nên tứ giác XZY V nội tiếp đường tròn (J).
Do đó ∠JV C = 90◦ − ∠V XY = ∠V BC hay JV là tiếp tuyến của (O). Từ đó (XY Z) trực giao
với (O). (1)
Mặt khác, kéo dài P M cắt (O) tại hai điểm W và S thì (T M SW ) = −1. Suy ra PO/(AM P ) =
OM · OP = OB 2 nên (AM P ) trực giao với (O). (2)
Gọi T là giao điểm thứ hai của (XY Z) và (O) thì T là điểm Miquel của tứ giác toàn phần
AM Y V.CX. Suy ra ∠T M P = ∠T BX = ∠T AP hay T ∈ (AM P ). (3)
Từ (1), (2), (3) suy ra đường nối tâm các đường tròn (AM P ) và (XY Z) nằm trên tiếp tuyến tại T
của (O) hay (AM P ) tiếp xúc với (XY Z) tại T .
(Cách 2)

388
F

M
B L C

Y Z A'
E
R T
K
X

Dễ thấy XY Z và BCA0 là hai tam giác paralogic ứng với (L, X, M ). Suy ra (XY Z) trực giao với
(O). Gọi E, F lần lượt là trung điểm cung BC và cung BAC ta thu được OE 2 = OM · OP suy ra
(AM P ) cũng trực giao với (O). Vậy ta chỉ cần chứng minh 3 đường tròn (XY Z), (AM P ) và (O) đồng
quy.
Gọi T và R là giao điểm của (XY Z) và (O) (T là điểm Miquel của tứ giác toàn phần BCA0 M LK,
R là giao điểm của BX, CY, A0 Z).
Ta có ∠RAB = ∠RCB = ∠M KY = ∠M AC nên R ∈ AP . Suy ra ∠P M T = ∠RCT = ∠P AT hay
T ∈ (AM P ). Suy ra đpcm.

Bài 447. (Trịnh Huy Vũ). Cho tam giác ABC. Đường cao AD (D ∈ BC), G là trung điểm AD. Gọi
X, Y lần lượt là hình chiếu của D trên GC, GB. BX giao CY tại Z. Chứng minh rằng (XY Z) tiếp
xúc với (AD).

389
A

L
G

J
Y
Z
T
B C
D

Chứng minh. Gọi T là giao điểm của XY và BC. Ta có tứ giác GY DX nội tiếp nên ∠GY X =
∠GDX = ∠GCB, suy ra tứ giác BY XC nội tiếp đường tròn (O). Theo định lý Brocard, OZ vuông
góc với T G tại J là điểm Miquel của tứ giác toàn phần BY XCGT . Do (GY XD) tiếp xúc với BC tại
D nên T D2 = T Y · T X = T J · T G, suy ra DJ ⊥ T G. Suy ra O, D, Z, J thẳng hàng. Gọi L là điểm
đối xứng với D qua J, GZ giao OT tại E.
Ta có T L2 = T D2 = T Y · T Z nên (LY X) tiếp xúc với G tại L. Do J là điểm Miquel của tứ giác
toàn phần BCY XGT nên J ∈ (Y OC) và (BOX).
Ta có ZY · ZC = ZX · ZB = ZL · ZO = ZG · ZE = ZD · ZL. Suy ra LY DC và LXBD nội tiếp.
Do đó ∠LY Z + ∠LXZ = ∠LDC + ∠LDB = 180◦ hay L ∈ (XY Z). Vậy (XY Z) tiếp xúc với (G)
tại L.

Bài 448. (Ba Lan 2013) Cho tứ giác ngoại tiếp ABCD. Chứng minh rằng tồn tại đường tròn tiếp
xúc với các đường tròn đường kính AB, BC, CD, DA.

390
B
M
A

J
Q

E N

K
D

Chứng minh. Không mất tổng quát giả sử BC > AB.


Gọi E là trung điểm AC, M, N, P, Q lần lượt là trung điểm AB, BC, CD, DA. Tia N E cắt (BC)
1 1 1
tại K. Ta có EK = N K − N E = BC − AB. Do đó đường tròn tâm E, bán kính (BC − AB) tiếp
2 2 2
xúc với (BC).
1 1
Chứng minh tương tự, (E, (BC − AB)) tiếp xúc với (AB), (E, (CD − AD)) tiếp xúc với
2 2
(CD), (AD). Mà tứ giác ABCD ngoại tiếp nên AB + CD = AD + BC hay BC − AB = CD − AD.
1
Suy ra (E, (BC − AB)) tiếp xúc với cả 4 đường tròn đường kính AB, BC, CD, DA.
2
1
Tương tự ta cũng có đường tròn có tâm là trung điểm BD, bán kính bằng |AB − AD| tiếp xúc
2
với 4 đường tròn trên.

Bài 449. (Nguyễn Văn Linh) Cho tam giác ABC nội tiếp đường tròn (O), trực tâm H, đường đối
trung AD (D ∈ BC). Qua D kẻ đường thẳng cắt AC, AB lần lượt tại E, F sao cho D là trung điểm
EF . Gọi K là trực tâm tam giác AEF . Chứng minh rằng đường tròn đường kính AK tiếp xúc với (O)
và (BHC).

391
A

N
K E
X T
O
Q H
V
Y
L M
D C
B

W Z
J
F

R G

Chứng minh. Gọi M là trung điểm BC. Do AM và AD đẳng giác trong ∠BAC và D là trung điểm
EF nên tứ giác F BEC nội tiếp và 4AF E ∼ 4ACB. Do AH và AO đẳng giác trong góc A nên
K ∈ AO. Suy ra (AK) tiếp xúc với (O) tại A.
Mặt khác, gọi T là hình chiếu của H trên AM . Ta chứng minh (AK) và (BHC) tiếp xúc nhau tại
T.
Gọi X, Y, Z lần lượt là trung điểm AD, BE, CF . AD cắt BE, CF lần lượt tại V, W ; BE cắt CF
tại R.
Ta có (RW F C) = (RV BF ) = −1 nên theo hệ thức Maclaurin, RV · RY = RB · RE = RF · RC =
RW · RZ. Suy ra tứ giác V W ZY nội tiếp.
2
Theo hệ thức Newton, XY · XZ = XV · XW = XA . Do AY, AZ cũng đẳng giác trong ∠BAC
nên ta thu được ∠AZX = ∠XAY = ∠ZAM hay AM k XZ.
Do XZ là đường thẳng Gauss-Newton của tứ giác toàn phần ABDE.F C nên XZ vuông góc với
đường thẳng Steiner HK. Suy ra AM ⊥ HK hay HK đi qua T .
Gọi Q là giao điểm thứ hai của (AH) với (O). HT cắt BC tại L. Dễ thấy Q, H, M thẳng hàng và
H là trực tâm tam giác ALM nên L, A, Q thẳng hàng. Suy ra LH · LT = LA · LQ = LB · LC nên
T ∈ (BHC).
Gọi G là điểm đối xứng với A qua M, J là tâm của (BHC). Ta có J và O đối xứng với nhau qua
BC nên G ∈ (BHC) và JG k AO. Vậy T là tâm vị tự của hai đường tròn (AK) và (BHC) hay hai
đường tròn tiếp xúc nhau tại T.

392

You might also like